Boardvitals (PMHNP) Flashcards
Paranoid personality disorder involves a number of key characteristics. Which of the following is a key characteristic of paranoid personality disorder?
A. It is characterized by mistrust of others.
B. It is seen with a 30% prevalence among the general population.
C. Long-term treatment with medication is not needed.
D. A decreased incidence in families with a history of schizophrenia and delusional disorder.
Answer: A
It is characterized by mistrust of others.
Paranoid personality disorder involves pervasive distrust and suspiciousness of others. Patients with paranoid personality disorder often suspect that others are exploiting or deceiving them, and are preoccupied with doubts about the loyalty of friends, associates, and romantic partners. They persistently bear grudges or read hidden meanings or threats into benign events or circumstances (e.g. when asked to fill out basic questionnaires prior to starting their clinical work). They likely have intact reality testing and do not typically have any other perceptual disturbances, and they otherwise function well.
Incorrect Answers:
B. Paranoid personality disorder’s prevalence in the general population is 2.3-4.4%
C. Long-term, lifelong medication is needed for paranoid personality disorder
D. Patients with paranoid personality disorder have higher, not lower, the incidence of family members with schizophrenia and delusional disorder.
Patients with histrionic personality disorder may be treated with therapy to address provocative and attention-seeking behavior. What is that therapy?
A. Cognitive-behavioral therapy
B. Problem-solving approach therapy
C. Interpersonal therapy
D. Group therapy
Correct Answer: D.
Group therapy can be useful for addressing these behaviors, as the patient may be unaware of his or her behaviors and can benefit from having others point them out.
Incorrect Answers:
A, B, and C. Patients with histrionic personality disorder are often the most difficult to establish a therapeutic relationship with to avoid crossing boundaries. A group setting can be helpful to highlight a patient’s behaviors in a setting with other patients present.
A 35-year-old woman has been working for the past 8 years at a financial company. She believes she is better than the other workers and follows a strict routine of discipline at work. She gets to work at 6:00am and is frustrated if she is more than 5 minutes late. She follows the company’s rules and pays extreme attention to details when drafting contracts. The patient spends multiple days reviewing the same report or drafts, often submitting her work late because of excessive reviews. Although her superiors admire her work dedication, she is often “difficult to work with,” especially because she adapts poorly to new situations. The patient has also been storing her clothes since she was 20 years old, even though she has not worn most of them for years, and feels frustrated when she is told to get rid of her old and unused clothes. What is the most likely diagnosis?
A. Obsessive-compulsive disorder
B. Obsessive-compulsive personality disorder
C. Histrionic personality disorder
D. Narcissistic personality disorder
Correct Answer: B
This patient displays typical features of obsessive-compulsive personality disorder (OCPD). Patients with OCPD tend to be restrained, conscientious, respectful, and rigid in their daily routine and activities. They are easily upset if their routine is affected, maintain a rule-bound lifestyle, and see the world in terms of regulations and hierarchies. They see themselves as devoted, reliable, efficient, and productive. However, due do their rigidness, their functioning may be affected. Patients are also sometimes uneasy about discarding old items, even though they may no longer require them.
Incorrect Answers:
A. Avoid confusing obsessive-compulsive personality disorder with obsessive-compulsive disorder (OCD). The latter is an anxiety disorder characterized by intrusive thoughts that produce uneasiness, fear or worry (obsessions), and repetitive behaviors aimed at reducing the associated anxiety (compulsions). The prototypical example is fear of “germs” and constantly hand-washing or repeatedly checking the gas stove before leaving the house. Patients with OCD often know that their behavior is irrational but cannot stop it.
C. Histrionic personality disorder refers to individuals who are dramatic, seductive, shallow, stimulus-seeking, and vain. They often tend to overreact to minor events but quickly forget about them. They constantly seek attention. This personality’s tendency to over-dramatize may impair relationships and lead to depression, but sufferers are often high-functioning.
D. Individuals with narcissistic personality disorder are egoistic, arrogant, and grandiose. They are often preoccupied with fantasies of success, beauty, or achievement. They see themselves as admirable, superior, and entitled to special treatment.
Anorexia nervosa patients who restrict, rather than binge-purge, often have a certain personality disorder. What is that personality disorder?
A. Borderline
B. Narcissistic
C. Obsessive-compulsive
D. Dependent
Correct Answer: C
A psychological profile for a patient with anorexia nervosa often demonstrates premorbid anxiety disorders and more severe affective disorders, such as major depression and dysthymic disorder. Patients may also have symptoms of obsessive-compulsive personality disorder with rigid and ritualistic eating behaviors. Obsessive-compulsive personality defines certain character traits (being a perfectionist, morally rigid, or preoccupied with rules and order). This personality disorder has been strongly associated with a higher risk for anorexia. These traits should not be confused with the anxiety disorder called obsessive-compulsive disorder (OCD), although they may increase the risk for this disorder. Impulsivity in individuals with anorexia nervosa correlates with a lower lifetime BMI, reflecting poorer long-term outcomes, and can be measured with the Yale-Brown-Cornell Eating Disorder Scale (YBC-EDS).
Incorrect Answers:
A, B, and D. Thse personality disorders don’t overlap with anorexia nervosa patients who restrict rather than binge-purge
Which treatment is typically well-tolerated and most effective for patients with paranoid personality disorder?
A. Individual psychotherapy
B. Group therapy
C. Cognitive-behavioral therapy
D. Benzodiazepines
Correct Answer: A
Consistent suspicion and a lack of trust are the cardinal characteristics of this cluster A disorder. Given these patients’ suspicion towards people, individual psychotherapy is typically tolerated best because it involves a single, consistent therapist with whom the patient can develop trust. However, the therapist must be aware of the patient’s suspicion and should tailor their treatment style accordingly.
Incorrect Answers:
B. Group therapy is typically not well-tolerated due to the higher number of people.
C. CBT is typically difficult for the patient, as the depth of the invasive emotional exploration can trigger paranoia in these individuals.
D. Benzodiazepines can help with some symptomatic management (e.g., decreasing anxiety), but ultimately it does not treat the underlying pathology. Medications are adjunctive therapy for personality disorders, not the primary treatment.
Vital Concept:
Psychotherapy is the treatment of choice for all personality disorders. Individual psychotherapy tends to be the treatment of choice for individuals with paranoid personality disorder.
Which personality disorder’s criteria include magical thinking?
A. Schizoaffective
B. Paranoid
C. Schizotypal
D. Schizoid
Correct Answer: C
Schizotypal personality disorder consists of a pattern of interpersonal deficits, cognitive/perceptual distortions, and eccentric behavior with 5 or more of the following:
· fantastical thinking or strange ideas that are not mainstream and affect the patient’s actions and decision-making
· abnormal patterns of talking or mental processing (e.g., analogies/metaphors, non-specific, indirect)
· displays of emotion that are limited or unsuitable
· has very few (or no) people that they are emotionally close to outside of family
· a false belief that random events in the world are directly related to them
· atypical sensations, including physical perceptions that are false or not objectively observable by others
· wariness, doubt, and lack of trust regarding people’s underlying motivation
· abnormal or bizarre appearance or actions
· an extreme, consistent, and unabating nervousness or concern regarding social interactions and events that is directly related to wariness and distrust regarding other’s underlying motivation in place of poor self-regard
Incorrect Answers:
A. Schizoaffective is not a personality disorder. Schizoaffective disorder is under the schizophrenia spectrum and other psychotic disorders in the DSM-5-TR.
B. Cluster A personality disorders include schizotypal, schizoid, and paranoid personality disorders. Common features in this group of disorders include social isolation and restricted affectivity. Schizotypal is the only personality disorder in cluster A that has magical thinking as a criterion.
D. Cluster A personality disorders include schizotypal, schizoid, and paranoid personality disorders. Common features in this group of disorders include social isolation and restricted affectivity. Schizotypal is the only personality disorder in cluster A that has magical thinking as a criterion.
Vital Concept:
Cluster A personality disorders (schizotypal, schizoid, and paranoid) have common features of social isolation and restricted affectivity. Schizotypal is the only one that has magical thinking as a criterion
Which of the following personality disorder is most common in clinical settings?
A. Narcissistic personality disorder
B. Antisocial personality disorder
C. Borderline personality disorder
D. Histrionic personality disorder
Correct Answer: C
Borderline personality disorder has a prevalence of 12-15% in clinical settings and occurs in 2-3% of the general population.
Incorrect Answers:
A. Narcissistic personality disorder has a prevalence of 2-16% in clinical populations and <1% in the general population.
B. Antisocial personality disorder has a prevalence of 3% in males and 1% in females in the general population.
D. Histrionic personality disorder has a prevalence of 2-3% in the general population.
Vital Concept:
BPD has a prevalence of 12-15% in clinical settings.
Some health conditions can cause personality changes. Which of the following factors is thought to be the most common cause of personality changes due to another medical condition?
A. Renal carcinoma
B. AIDS
C. Diabetes
D. Traumatic brain injury
Correct Answer: D
Structural changes in the brain are believed to underlie the mechanism leading to personality changes. Among the causes of brain structure change, head trauma is the most likely cause.
Incorrect Answers:
A. C. These are not common causes of significant permanent personality changes.
B. These are among the top 10 causes of personality change due to another medical condition, but they are not the most common causes.
Which of the following terms describes a stable and realistic sense of self?
A. Social system
B. Self-in-relation system
C. Self-system
D. Relation system
Correct Answer: C
Self-system describes a stable and realistic sense of self. The social system is a means of interpreting social situations and understanding the relational motives and actions of others. A self-in-relation system describes the capacity to observe the self as it relates to others.
Incorrect Answers:
A. This is a means of interpreting social situations and understanding the relational motives and actions of others
B. This describes the capacity to observe the self as it relates to others
D. Not a term
Vital Concept:
Self-system describes a stable and realistic sense of self.
Assertiveness training can benefit people with some personality disorders. Of the following, which personality disorder can benefit from assertiveness training?
A. Dependent personality disorder
B. Narcissistic personality disorder
C. Schizoid personality disorder
D. Histrionic disorder
Correct Answer: A
Dependent personality disorder and avoidant personality disorder can benefit from assertiveness training, as self-confidence is absent in both disorders.
Incorrect Answers:
B. Narcissistic personality disorder does not have a clear preferred modality, but group therapy has been postulated as useful.
C. Schizoid personality disorder does not have a clear preferred modality, but individual therapy with clear boundaries is the current recommendation.
D. Histrionic personality disorder seems to respond to psychoanalytic psychotherapy.
Mahler theorizes that the disruption of a certain stage of infant development leads children to develop narcissistic personality disorder. What stage is this?
A. Symbiotic phase
B. Autistic phase
C. Rapprochement
D. Practicing
Correct Answer: A
Mahler’s theory is based on psychoanalytic observation of children ages 6 months to 3 years. Mahler’s work has been expanded by other theorists to understand the basis of personality disorder. The symbiotic phase occurs until about 5 months of age; the infant recognizes his or her mother but lacks a sense of individuality.
Incorrect Answers:
B. The autistic phase occurs during the first weeks of life and is characterized by a total detachment and self-absorption, as the infant spends most of his or her time sleeping.
C. D. Separation-individuation has 3 phases: hatching (increased interest in the outside world), practicing (9-16 months, developing the physical ability to separate from mother), and rapprochement (15-24 months, exploring the outside world but requiring mother to be present for emotional support in completing the task). Disturbance of the rapprochement subphase is associated with persistent longing for and dread of fusion with the object that is thought to be secondary to aggression or withdrawal in the mother. Disruption of the rapprochement subphase is thought to contribute to the development of borderline personality disorder. Object constancy describes the child’s understanding that the mother is a separate individual and that the child is also separate. Object constancy leads to the formation of internalization that will allow the child to have an internal representation of the mother, allowing healthy separation, exploration, and self-esteem development.
Vital concept:
Narcissistic personality disorders are likely due to inadequate soothing during the symbiotic phase and inadequate refueling during separation-individuation.
Which of the following is the current prevalence of schizotypal personality disorder?
A. <1%
B. 3%
C. 5%
D. 10%
Correct Answer: B.
3%
About 3% of the population has schizotypal personality disorder. The criteria for schizotypal personality disorder include social and interpersonal impairment associated with discomfort with close relationships and eccentric behavior, including at least 5 of the following: ideas of reference, odd/magical beliefs, strange perceptual experiences/bodily illusions, odd speech or thought process, suspicion or paranoia, constricted affect, odd behavior or appearance, lack of close friends other than first-degree family, and social anxiety that does not improve with familiarity. The diagnostic criteria include:
A consistent lack of close connections with people due to a lack of desire and/or decreased capability to foster these relationships, as well as mental misrepresentations and oddities of conduct. This personality disorder typically presents by the patient’s ‘20s and is evidenced by at least five of the following:
· fantastical thinking or strange ideas that are not mainstream and affect the patient’s actions and decision-making
· abnormal patterns of talking or mental processing (e.g., analogies/metaphors, non-specific, indirect)
· displays of emotion that are limited or unsuitable
· has very few (or no) people that they are emotionally close to outside of family
· a false belief that random events in the world are directly related to them
· atypical sensations, including physical perceptions that are false or not objectively observable by others
· wariness, doubt, and lack of trust regarding people’s underlying motivation
· abnormal or bizarre appearance or actions
· an extreme, consistent, and unabating nervousness or concern regarding social interactions and events that is directly related to wariness and distrust regarding other’s underlying motivation in place of poor self-regard
The symptoms must be present outside of and notwithstanding a diagnosis of bipolar disorder, schizophrenia, or some other psychotic disorder, autism spectrum disorder, or other medical condition.
Incorrect Answers:
A. The prevalence is approximately 3%, not <1%.
C. The prevalence is approximately 3%, not 5%.
D. The prevalence is approximately 3%, not 10%.
Vital Concept:
The prevalence of schizotypal personality disorder is approximately 3%.
Patients with a history of childhood separation anxiety or chronic illness may be predisposed to a certain personality disorder. What is that disorder?
A. Obsessive-compulsive
B. Schizoid
C. Avoidant
D. Dependent
Correct Answer: D.
Dependent
Patients with a history of childhood separation anxiety or chronic illness may be predisposed to dependent personality disorder. Patients with dependent personality disorder commonly experienced over-involvement and intrusive behavior by their primary caretaker as a child. They were often socially humiliated during childhood, which leads them to doubt in their own ability to function independently of others.
Incorrect Answers:
A. Patients with obsessive-compulsive personality disorder (OCPD) often had parents who were emotionally withholding and overprotective or over-controlling. Children who develop OCPD were often punished by their parents and were rarely rewarded. They develop their OCPD symptoms as a strategy to avoid punishment.
B. Patients with schizoid personality disorder typically come from families that are emotionally reserved, highly formal, aloof, and impersonal. Their parents likely provided inadequate affection, prompting their disinterest in forming close relationships later in life.
C. Patients with avoidant personality disorder displayed excessive shyness and fear when confronted with new people and situations as children. Many of these patients have a history of painful early experiences and chronic parental criticism.
Vital Concept:
Someone with a history of separation anxiety or significant illness may develop a dependent personality disorder, a consistent and extreme desire to be cared for, resulting in actions that are passive, docile, and insecure and concerns of estrangement.
Narcissistic personality disorder has a number of criteria. Which of the following is one of them?
A. Exploitation of others to achieve goals
B. High empathy
C. Low self-esteem
D. Indifference towards others
Correct Answer: A.
Exploitation of others to achieve goals
Narcissistic personality criteria include a consistent desire for veneration from others, grandness, and a lack of understanding and compassion for the feelings of others. This personality disorder typically presents by the patient’s ‘20s and is evidenced by at least five of the following:
- is myopically focused on daydreams regarding intelligence, magnificence, achievement, true love, and authority/control
- craves constant veneration from others
- will use others to accomplish their goals
- is somewhat preoccupied with jealousy and envy- of others or suspicion of others directed at them
- an inflated self-regard, with a false belief that they are more crucial than they are, giving the impression that they are more successful and expert than their past accomplishments would suggest
- an impression that they are superior and one-of-kind, and therefore can only be truly appreciated or wants to work with other top-notch or exclusive groups or people
- believes that others owe them special treatment or expects immediate submission to their wishes
-mlack of understanding and compassion for others’ feelings or emotions
- actions and manners are proud, conceited, and egotistical
Incorrect Answers:
B. Lack of empathy is a criteria for narcissistic personality disorder
C. Belief that others are envious of oneself is a criteria for narcissistic personality disorder
D. People with narcisstic personality are very interested in how they stand relative to others
Vital Concept:
Patients with narcissistic personality disorder often display a consistent desire for veneration from others, grandness, and a lack of understanding and compassion for the feelings of others.
Obsessive-compulsive personality disorder is relatively difficult to treat. Which treatment can enable patients to understand that the world is not divided into clearly defined, “black and white” lines of rigid beliefs?
A. Psychodynamic therapy
B. Cognitive-behavioral therapy
C. Fluoxetine
D. Imipramine
Therapy is used for Cluster C personality disorders. Of the following types of psychotherapy, which is the most studied and used for Cluster C personality disorders?
A. Group psychotherapy
B. Family therapy
C. Couples therapy
D. Individual psychotherapy
Correct Answer: D.
Individual psychotherapy
Individual psychotherapy is the most studied and used therapy for treating Cluster C personality disorders. Individual psychotherapy is effective for people with various personality disorders, including dependent, avoidant, passive aggressive and obsessive-compulsive personality disorders. It can help people understand how their personality disorder is connected to their problems and learn new ways of interacting and coping.
Incorrect Answers:
A, B, and C. These aren’t used for Cluster C personality disorders
Paranoid personality disorder involves a number of key characteristics. Which of the following accurately describes paranoid personality disorder?
A. Displays cognitive, perceptual, and behavioral eccentricities and has pervasive discomfort with close relationships
B. Displays pervasive distrust and suspiciousness of others
C. Displays pervasive disregard for rights of others and engages in repetitive unlawful acts
D. Displays impaired capacity to form stable interpersonal relationships, affective instability, impulsivity, and identity disturbance
Correct Answer: B.
Displays pervasive distrust and suspiciousness of others
Paranoid personality disorder involves pervasive distrust and suspiciousness of others. Patients with paranoid personality disorder often suspect that others are exploiting or deceiving them, and are preoccupied with doubts about the loyalty of friends, associates, and romantic partners. They persistently bear grudges or read hidden meanings or threats into benign events or circumstances (e.g. when asked to fill out basic questionnaires prior to starting their clinical work). They likely have intact reality testing and do not typically have any other perceptual disturbances, and they otherwise function well.
Incorrect Answers:
A. This describes schizotypal personality disorder.
C. This describes antisocial personality disorder.
D. This describes borderline personality disorder.
Vital Concept:
A defining characteristic of paranoid PD is distrust and suspicion.
The DSM defines a tendency towards frequent and extreme negative emotion using what personality trait domain?
A. Detachment
B. Negative affectivity
C. Psychoticism
D. Disinhibition
Correct Answer: B.
Negative affectivity
Negative affectivity (or neuroticism) is excessive worry and fear of the worst in every life experience. It is defined as extreme negative emotions (depression, guilt/shame) that are experienced often. This is often associated with external repercussions, such as dependency or self-harm. Neuroticism is a predictor for the development of psychiatric illness and is associated with low self-esteem, anxiety, and depression. The facets of this domain include hostility, anxiousness, perseveration, depression, suspiciousness, emotional lability, separation insecurity, and submissiveness.
Incorrect Answers:
A. Detachment is defined as limited interaction, both emotional and/or social, with others. This also involves limited emotional expression.
C. Psychoticism is defined by the DSM as a patient who displays strange, bizarre, or different thoughts, behaviors, beliefs, and opinions.
D. Disinhibition is defined by the DSM as a tendency towards behaving and acting impulsively, without much self-control or forethought. Patients who exhibit this trait domain often fail to learn from prior experiences or fail to conceive of potential future ramifications.
Vital Concept:
The personality trait domains defined in the DSM include negative affectivity (versus emotional stability), psychoticism (versus lucidity), disinhibition (versus conscientiousness), antagonism (versus agreeableness), and detachment (versus extraversion).
Which type of therapy has been the most efficacious in the treatment of borderline personality disorder?
A. Aversion therapy
B. Group therapy
C. Biofeedback therapy
D. Dialectical behavior therapy
Correct Answer: D.
Dialectical behavior therapy
Dialectical behavior therapy (DBT) seeks to uncouple a patient’s harmful coping mechanisms from a stressor. DBT helps the patient to tolerate emotional stress and find nonharmful coping mechanisms to reduce emotional actions in response to a stressful situation. Other therapeutic techniques unique to borderline personality disorder include mentalization-based treatment and transference-focused psychotherapy.
Incorrect Answers:
A, B, and C. These aren’t useful for BPD
Affective aggression can be treated effectively with which medication?
A. Sertraline
B. Venlafaxine
C. Imipramine
D. Lithium
Correct Answer: D.
Lithium
Multiple studies have shown lithium carbonate to be effective in the treatment of affective aggression.
Incorrect Answers:
A. Used to treat depression, panic, anxiety, or obsessive-compulsive symptoms, is an SSRI
B. Used to treat major depressive disorder, anxiety and panic disorder, is an SNRI
C. Used to treat depression, is a TCA
Which of the following is a standard therapy used for the treatment of borderline personality disorder (BPD)?
A. Aversion therapy
B. Psychoanalytic therapy
C. Eye movement desensitization and reprocessing (EMDR) therapy
D. Dialectical behavioral therapy
Correct Answer: D.
Dialectical behavioral therapy
Dialectical behavioral therapy (DBT) is the standard for therapeutic evidence in borderline personality disorder. DBT was developed by Marsha Linehan and focuses on improving interpersonal skills and decreasing self-destructive behavior using techniques involving advice, metaphor, storytelling, confrontation, and mindfulness. DBT assumes that all behavior is learned and that patients with borderline personality disorder behave in a way that reinforces their own maladaptive behavior.
Incorrect Answers:
A. Aversion therapy is a generally effective treatment for addictions.
B. Psychoanalytic and supportive therapies have not been widely studied in the treatment of personality disorders.
C. EMDR is a generally effective treatment for PTSD.
Vital Concept:
DBT is a common therapy utilized for BPD.
Personality disorders have certain manifestations, symptoms, and patient behaviors. Which of the following is true about personality disorders?
A. Personality disorders not due to another medical condition typically first manifest in later life.
B. Patients often accept responsibility for their symptoms and seek to change their maladaptive patterns.
C. Patients are eager to seek treatment.
D. Symptoms are usually consistent with patient’s internal sense of self, which is called ego-syntonic.
Correct Answer: D.
Symptoms are usually consistent with patient’s internal sense of self, which is called ego-syntonic.
The symptoms of a personality disorder are usually consistent with the patient’s internal sense of self, which is referred to as ego-syntonic.
Incorrect Answers:
A. Personality disorders typically first present in adolescence or early adulthood.
B. C. Patients usually are unable to identify their maladaptive behaviors and have difficulty identifying their role in the problems their disorder causes. They usually do not feel like they need treatment.
Histrionic personality disorder has a number of criteria. Which is one of those criteria?
A. Not comfortable when they are in the background and others are in the spotlight
B. Belief that relationships are less serious than they are
C. Flat affect
D. Lack of emotion
Correct Answer: A.
Not comfortable when they are in the background and others are in the spotlight
The diagnostic criteria for histrionic personality disorder include a consistent finding of emotional lability and pursuing notice and attention from others. This personality disorder typically presents by the patient’s ‘20s and is evidenced by at least five of the following:
- when socializing, the patient is often overly sensual or erotic
- dresses and grooms themselves to attract attention
- displays of emotion are vivid, melodramatic, and dynamic
- perceives interpersonal connections to be closer and more intense than they are in reality
- is not comfortable when they are in the background, and others are in the spotlight
- emotional communication is superficial and changes quickly
- communication is exceptionally imprecise and vague
- is impressionable or pliable
Incorrect Answers:
B. People with histrionic personality disorder perceive interpersonal connections to be closer and more intense than they are in reality, not less serious.
C. Histrionic personality disorder is characterized by displays of emotion that are vivid, melodramatic, and dynamic.
D. Histrionic personality disorder is characterized by displays of emotion that are vivid, melodramatic, and dynamic.
Vital Concept:
Patients with histrionic personality disorder are not comfortable when they are in the background, and others are in the spotlight.
A 29-year-old man with dependent personality disorder presents for a refill of his antidepressant. To encourage him to do something that will be helpful for his personality disorder, what should be suggested?
A. Exercising more frequently
B. Seeking out more personal contacts
C. Attending a support group at a community-based organization
D. Switching antidepressants to bupropion
Correct Answer: C.
Attending a support group at a community-based organization
Patients with personality disorders can benefit greatly from attending support groups. It is essential to refer them to a community-based organization for this treatment.
Incorrect Answers:
A, B, and D. These are unlikely to help with dependent personality disorder.
What is the prevalence of borderline personality disorder among patients in psychiatric inpatient facilities?
A. 1.6%
B. 6%
C. 10%
D. 20%
Correct Answer: D.
20%
The prevalence of borderline personality disorder in the population is estimated to be from 1.6% to 5.9%. The prevalence of borderline personality disorder among patients in psychiatric inpatient facilities is 20%. The prevalence of borderline personality disorder at primary care clinics is estimated to be about 6%, and at outpatient psychiatric clinics is estimated to be about 10%.
Borderline personality disorder has been associated with depressive and bipolar disorders, posttraumatic stress disorder, substance use disorders, attention-deficit hyperactivity disorder, and eating disorders. The primary treatment for borderline personality disorder is psychotherapy, but medications can be utilized to target symptoms. For impulsive aggression can utilize fluoxetine, fluvoxamine, olanzapine, and divalproex. For affective dysregulation can use an SSRI or venlafaxine.
Incorrect Answers:
A. The prevalence of borderline personality disorder in the population is estimated to be from 1.6% to 5.9%.
B. The prevalence of borderline personality disorder at primary care clinics is estimated to be about 6%
C. The prevalence of borderline personality disorder at outpatient psychiatric clinics is estimated to be about 10%.
Vital Concept:
Estimates of borderline personality disorder prevalence include 6% at primary care clinics, 10% at outpatient psychiatric clinics, and 20% at psychiatric inpatient facilities.
Abnormalities in the temporal lobe, stratal nuclei, and thalamic nuclei on MRI have been identified on MRI studies of patients with which condition?
A. Borderline personality disorder
B. Schizotypal personality disorder
C. Narcissistic personality disorder
D. Antisocial personality disorder
Correct Answer: B.
Schizotypal personality disorder
There is evidence that patients with schizotypal personality disorder have abnormalities in the temporal lobe, striatal nuclei, and thalamic nuclei on MRI. In studies on patients with schizotypal personality disorder MRIs have shown the superior temporal gyrus to have decreased volume. The thalamic nucleus, pulvinar, had decreased volume, which plays a role in visual processing. Also found on MRI smaller putamen volume. The putamen is a part of the dorsal striatum (putamen, globus pallidus and caudate nucleus), which participates, in cognitive and motor activity.
Incorrect Answers:
A. In patients with borderline personality disorder MRIs have shown decreased limbic volumes, particularly in the hippocampus and amygdala.
C. In patients with narcissistic personality disorder there has been some evidence showing right prefrontal and bilateral medial prefrontal decrease in grey matter.
D. In patients with antisocial personality disorder there has been insufficient evidence of brain structural changes.
Vital Concept:
There is evidence that patients with schizotypal personality disorder have abnormalities in the temporal lobe, striatal nuclei, and thalamic nuclei on MRI.
A recently divorced 49-year-old woman with borderline personality disorder presents for psychotherapy. In sessions for individuals with personality disorders, as the therapist, which action blurs the line between doctor and patient boundaries?
A. Having a positive attitude
B. Never relating your personal problems to a patient’s problems
C. Establishing ground rules for therapy, especially regarding therapy schedule and emergency situations
D. Giving a personal home number in case of a crisis
Correct Answer: D.
Giving a personal home number in case of a crisis
It is important to maintain a positive attitude, find support from peers and supervisors for advice or consultation, and establish rules for therapy concerning what the patient should do in a crisis situation and how frequently he or she should receive therapy. Therapists must also respect personal boundaries and avoid relating personal problems to the patient or giving the patient their home phone number.
Incorrect Answers:
A, B, and C. These are appropriate and effective therapy provider techniques and boundaries
There are multiple criteria for antisocial personality disorder. Which of the following is included in these criteria?
A. Challenges starting tasks due to a lack of self-assurance in skill or decision-making
B. When socializing, the patient is often overly sensual or erotic
C. Repeated discussions, attempts, or contemplations of suicide or intentional self-harm
D. Quick decision-making without consideration of consequences or preparation
Correct Answer: D.
Quick decision-making without consideration of consequences or preparation
Antisocial personality disorder is characterized by:
consistent ignorance and abuse of surrounding people’s rights that begins by age 15 as evidenced by at least three of the following:
- dishonesty in the form of defrauding, deceiving, or misleading others for individual gain
- an agitated and sometimes hostile nature with a history of numerous altercations
- a lack of personal accountability as evidenced by a failure to fulfill financial responsibilities or deliver professionally when promised
- lack of concern regarding social norms or laws, with repeated arrests or undetected breaches of the law
- quick decision-making without consideration of consequences or preparation
- lack of concern about the well-being or security of others or oneself
- an inability to feel guilty or personally responsible following the harm, deception, or abuse of another person
The symptoms must be present outside of and notwithstanding a diagnosis of bipolar disorder, schizophrenia, or some other psychotic disorder.
Conduct disorder must be diagnosed or evident prior to age 15, but antisocial PD is not to be confirmed in an individual under 18.
Incorrect Answers:
A. Challenges starting tasks due to a lack of self-assurance in skill or decision-making is a characteristic of dependent personality disorder, not antisocial.
B. Overly sensual or erotic communication is characteristic of histrionic personality disorder, not antisocial.
C. Repeated discussions, attempts, or contemplations of suicide or intentional self-harm are characteristic of borderline personality disorder, not antisocial.
Vital Concept:
Quick decision-making without regard for consequences or proper preparation is characteristic of antisocial personality disorder, as well as borderline personality disorder.
The PMHNP is evaluating Brody, 19 year old Marine who is currently in the brig. The NP asks Brody to tell her what happened that lead to his confinement to the brig. Brody states “Eh, I’ve been a bad, bad dude since I was 10 when my old man left. I ditched school for good when I was 15 because it wasn’t my scene, but I got my GED so I could be a Marine. I’ve always loved to crack heads and get that high when my fists hit flesh. I got some practice with cracking heads tonight at the club. The local boys got what they deserved, a few less teeth and a free ambulance ride to the hospital. My sergeant was there. He’s proud of the job I did on them fellas, even though he didn’t tell me outright, I know he is. He said you’d be coming by. You’re a pretty girl, I’m glad you’re here. Do you have a smoke I can bum?” For which disorder does Brody meet criteria?
A. Antisocial Personality Disorder
B. Histrionic Personality Disorder
C. Oppositional Defiant Disorder
D. Intermittent Explosive Disorder
Correct Answer: A.
Antisocial Personality Disorder
The patient described meets the criteria for APD by: 1. Pervasive pattern for disregard for others since age 15 as indicated by (at least 3 of 7 criteria): - breaking the law - impulsivity - aggressiveness - lack of remorse 2. Individual is at least 18 3. There is evidence of conduct disorder with onset before age 15.
Incorrect Answers:
B. Histrionic Personality Disorder (HPD) is a pervasive pattern of excessive emotionality and attention seeking beginning in early adulthood and present in a variety of contexts.
C. Oppositional Defiant Disorder (ODD). ODD is a pattern of angry/irritable mood, argumentative /defiant behavior, or vindictiveness lasting at least 6 months. ODD does not include aggression towards people or animals, destruction of property, or a pattern of theft or deceit.
D. Intermittent Explosive Disorder is recurrent behavioral outbursts representing a failure to control aggressive impulses. Physical aggression may result in damage or destruction of property and/or physical injury to animals or other individuals.
While psychotherapy can be effective for some types of patients, it doesn’t work well for others. Which of the following patients is least likely to achieve meaningful improvement with psychotherapy?
A. A patient with antisocial personality disorder
B. A patient with borderline personality disorder
C. A patient with narcissistic personality disorder
D. A patient with histrionic personality disorder
Correct Answer: A.
A patient with antisocial personality disorder
Patients with antisocial personality disorder who do not have depression are least likely to achieve meaningful improvement with psychotherapy alone. Treatment for aggression may include the use of antipsychotic medication. Treatment for impulsivity may include the use of anticonvulsants. Those with comorbid ADHD may be managed with bupropion or atomoxetine.
Incorrect Answers:
B. Patients with borderline personality disorder generally respond to psychotherapy, potentially in combination with other treatments.
C. Patients with narcissistic personality disorder generally respond to psychotherapy, potentially in combination with other treatments.
D. Patients with histrionic personality disorder generally respond to psychotherapy, potentially in combination with other treatments.
Vital Concept:
Antisocial personality disorder does not have a clearly identified treatment algorithm. It is least likely to improve with psychotherapy.
A 27-year-old female patient is admitted for 2 weeks for depressive symptoms. Her history is significant for an inflexible and pervasive history of interpersonal turmoil, poor self-image, affective lability, sexual indiscretions, and spending sprees starting in early adolescence and continuing on this admission. The patient does not currently admit to a risk of self-harm. Which of the following therapeutic modalities is indispensable in treating this patient?
A. Antipsychotics
B. Anticonvulsants
C. Antidepressants
D. Psychotherapy
Correct Answer: D.
Psychotherapy
Psychotherapy, specifically dialectical-behavioral therapy (DBT), has been shown to cause lasting changes in a person’s character structure. Medications are an essential part of treating the symptoms of borderline personality disorder, but psychotherapy will target the root cause of the disorder. Borderline PD is likely secondary to many factors (i.e. genetic, neurobiological, and psychosocial). However, specific genetic risk factors have not been identified. Some studies have indicated that impulsive-aggressive behavior is related to low CNS serotonin levels. Specific brain abnormalities include reduced volume in the amygdala and hippocampus, altered amygdala activation, and hippocampal hypometabolisim. Other findings include abnormalities in the prefrontal, corticostriatal, and limbic networks, which could be related to low serotonin neurotransmission and behavioral disinhibition.
The primary treatment for borderline PD is psychotherapy and medications to target symptoms. Most patients will need long-term therapy to see benefits in interpersonal relationships, changes in personality, and an improvement in overall functioning. Psychodynamic, dialectical-behavioral, transference-focused therapy (TFP), interpersonal therapy, and CBT have been shown to be effective. Family therapy or couples therapy alone is not effective. Fluoxetine, fluvoxamine, olanzapine, and divalproex reduce impulsive aggression and depressive symptoms. Affective dysregulation is treated with an SSRI or venlafaxine, and TCAs have had inconsistent results. Benzodiazepines should be used sparingly, as research on their use with BPD is limited, and they carry more substantial risks in this population. MAOIs show strong empirical support, but their side effect profiles make them second-line. Mood stabilizers are second-line as well since few studies support their efficacy in this population. Patients with borderline PD are unlikely to respond to ECT.
Impulsive-behavioral dyscontrol symptoms that present serious risk to the patient (e.g. aggression, self-mutilation, or self-damaging behaviors like reckless spending, promiscuous sex, and substance abuse) are best treated by adding an antipsychotic to the SSRI. Other alternatives include adding lithium or switching to an MAOI. A few studies support the use of valproate or carbamazepine to treat impulsive aggression. Cognitive-perceptual symptoms (paranoid ideation, derealization, depersonalization, or hallucinations) respond best to low-dose antipsychotics.
Incorrect Answers:
A. No need for antipsychotics as there are no psychotic symptoms
B. No need for anticonvulsants as there are no symptoms
C. Mood stabilizers are second line
Narcissistic personality disorder has a number of symptoms. Which of the following is one of them?
A. Affective instability
B. Chronic feelings of emptiness
C. Inappropriate anger
D. Grandiose sense of self-importance
Correct Answer: D.
Grandiose sense of self-importance
Narcissistic personality disorder involves a grandiose sense of self-importance.
Incorrect Answers:
A. Affective lability or emotional instability is a symptom of borderline personality disorder (BLPD).
B. Feelings of emptiness occur in BLPD.
C. Inappropriate anger, including recurrent fights, is a symptom of BLPD.
During an outpatient follow-up visit, a woman with a history of a personality disorder discloses that she is happy because she and her son have “never been closer.” She describes how she recently pointed out to her son that he develops intense relationships too quickly. The patient says that this originally caused stress because her son disagreed with this assertion; however, after much effort, she was able to get him to agree that he does have intense, rapid attachments. She reports that once he became able to accept this part of himself, they became tremendously close. Although the patient feels closer to her son, this may be a defense mechanism common to her personality disorder. What is this defense mechanism called?
A. Countertransference
B. Splitting
C. Dissociation
D. Projective identification
Correct Answer: D.
Projective identification
Projective identification is seen in this patient and is common in borderline personality disorder. Projective identification has three steps:
1) patient assigns a component of himself or herself to another individual
2) patient manipulates this individual into integrating this component into his or her consciousness
3) this process leads to an increased feeling of closeness between the parties
Incorrect Answers:
A. Countertransference is the analyst’s emotional response to a patient.
B. Splitting is when a patient tends to identify aspects as either completely good or completely bad.
C. Dissociation is a mental process that gives rise to a lack of connection in the patient’s thoughts, memory, and sense of identity.
An adult patient presents to the emergency department. The emergency provider states that the patient, who presented for treatment of a broken arm, seems “off” and should be seen before being discharged. They have no other medical history and have not seen a physician since they were an adolescent. During the interview, the patient states that they live alone in a farmhouse 30 miles from the hospital. They claim no family or friends and state that they have never been married or had children. They spend their time reading or building onto their house, which is how they injured their arm. Their affect is cold and flat. The patient denies other mood or psychotic symptoms.
Which of the following is the most likely diagnosis?
A. Schizotypal personality disorder
B. Paranoid personality disorder
C. Schizoid personality disorder
D. Autistic spectrum disorder
Correct Answer: C.
Schizoid personality disorder
Schizoid personality disorder is defined as a pattern of detachment from social relationships and restricted expression of emotions in interpersonal settings. This personality disorder typically presents by the patient’s ‘20s and is evidenced by at least four of the following:
· consistently picks actions or events that are performed alone
· does not enjoy most (or any) events, actions, or hobbies
· appears unaffected by people’s positive or negative opinions of them
· does not want or like to be emotionally close to people, including family members
· minimal (or no) desire to have a sexual relationship with someone else
· has very few (or no) people that they are emotionally close to outside of family
· appears to display minimal attachment, emotion, or responsiveness
The symptoms must be present outside of and notwithstanding a diagnosis of bipolar disorder, schizophrenia, or some other psychotic disorder, autism spectrum disorder, or other medical condition.
Incorrect Answers:
A. Schizotypal personality disorder is often confused with schizoid personality disorder. Schizotypal personality disorder is defined as a pervasive pattern of social and interpersonal deficits marked by acute discomfort with and reduced capacity for close relationships, as well as by cognitive or perceptual distortions and eccentricities of behavior. Those with schizotypal personality disorder try and want to be “typical” but are odd and awkward and are unsuccessful. In contrast, patients with schizoid personality disorder do not want any relationships and are content loners.
B. Paranoid personality disorder is defined as a pervasive pattern of distrust and suspiciousness of others so that their motives are interpreted as malevolent. This condition begins in early adulthood and presents in a variety of contexts. James is not described as being suspicious or paranoid but as a content loner.
D. Autism spectrum disorder has some unique characteristics to distinguish it from a schizoid personality disorder, including stereotypies, repetitive self-soothing, highly perseverative or unusual interests, concrete thought processes, and difficulties with social pragmatics. Patients with autism spectrum disorder are generally interested in social interactions, compared to patients with schizoid personality disorder who are aloof and indifferent to social interactions. Typically, patients with autism spectrum disorder want relationships but have trouble initiating and maintaining them. Patients with autism spectrum disorder also will have a much stronger reaction to praise or criticism. Autism spectrum traits are observed much earlier, typically in early childhood, and can be associated with some developmental delays.
Vital Concept:
Schizoid personality disorder is defined as a consistent lack of attachment to others and a limited display of emotions when interacting with others.
Obsessive-compulsive personality disorder is relatively difficult to treat. Which medication can reduce the need for perfectionism and the development of unnecessary rituals?
A. Lorazepam
B. Lithium
C. Fluoxetine
D. Imipramine
Correct Answer: C.
Fluoxetine
SSRIs like fluoxetine may be helpful in this regard.
Incorrect Answers:
A, B, and C. These aren’t useful for reducing the need for perfectionism and the development of unnecessary rituals
A 42-year-old man is referred for therapy. He believes that his neighbors are in collusion with the local police officers and neighborhood watch to spy on him and collect his personal information. He states that he “doesn’t even trust anyone anymore.” The patient broke up with his girlfriend recently because he was sure she had been cheating on him with a coworker, although she denies it. On further evaluation, he does not meet criteria for a mood or psychotic disorder. In the psychotherapeutic treatment of this patient, which of the following would be effective?
A. Agree that there may be some truth to his claims.
B. Bluntly tell him that his claims are unfounded.
C. Gently discuss why his claims are untrue.
D. Try to disprove his claims.
Correct Answer: A.
Agree that there may be some truth to his claims.
It is to the benefit of the patient to enter his world of paranoia by agreeing that there may be some truth to his claims. The therapist should not outright agree with or affirm these paranoid ideas, however, as this may seem patronizing and may alienate the patient.
Incorrect Answers:
B. It benefits the patient to enter his world of paranoia; so denying the claims is a poor move by the therapist
C. It benefits the patient to enter his world of paranoia; so explaining why they’re wrong is a poor move by the therapist
D. It benefits the patient to enter his world of paranoia; so disproving the claims is a poor move by the therapist
Women with antisocial personality disorder are a fairly small proportion of the general population. What is the prevalence of antisocial personality disorder in women in the community?
A. less than 0.5%
B. .5-2%
C. 3-5%
D. 6%
Correct Answer: B.
.5-2%
Women with antisocial personality disorder (ASPD) comprise 0.5-2% of individuals in community settings. In clinical settings, this percentage is much higher (ranging from 3-30%). By contrast, the prevalence of ASPD in the general population is roughly 6% in males. The Epidemiologic Catchment Area (ECA) survey of five United States cities found that 2 to 4 percent of men and 0.5 to 1 percent of women met these criteria. Two nationally representative surveys of US adults, the National Comorbidity Study and the NESARC study, estimated prevalence rates of 3.5 and 3.6 percent, respectively; the latter survey found a rate of 5.5 percent for men and 1.9 percent for women.
Incorrect Answers:
A. This figure is too low — the prevalence of antisocial personality disorder in women in the community is 0.5-2%.
C. The prevalence of ASPD is less than 3-5% amongst females in the US.
D. The estimated prevalence of ASPD amongst men in the US is 6%.
Vital Concept:
The prevalence of antisocial personality disorder (ASPD) is roughly 3-5 times higher in males than females.
Borderline personality disorder (BPD) may occur in psychiatric patients. What is the prevalence of BPD in psychiatric outpatients?
A. <1%
B. 2%
C. 5%
D. 10%
Correct Answer: D.
10%
The prevalence of borderline personality disorder is 1.6-5.9% in the general population, 6% in primary care patients, 10% in psychiatric outpatients, and 20% in psychiatric inpatients. Diagnosis of borderline personality disorder includes 5 or more of the following criteria: frantic avoidance of abandonment, unstable relationships, disturbances of identity, recurrent suicidality, unstable mood/affect, a chronic feeling of emptiness, intense bouts of difficult-to-control anger, and transient paranoia or dissociation (see table below).
Incorrect Answers:
A, B, C. These prevalence percentages are all below the actual prevalence
Vital Concept:
Estimates of borderline personality disorder prevalence include 6% at primary care clinics, 10% at outpatient psychiatric clinics, and 20% at psychiatric inpatient facilities.
The parents of an 18-year-old boy bring him in for evaluation. They are worried about his concern with rules and orderliness to the point of resisting and being incapable of adapting to change. They report that throughout his whole life, the patient has demonstrated little emotion and never seemed interested in making friends. Instead, he prefers concentrating on doing his homework perfectly but never manages to finish due to constant revisions.
During the exam, the patient is emotionally detached and distant. He denies any history of significant mood episodes, psychotic symptoms, performance of rituals, intrusive thoughts, or paranoia. When asked how he feels about not having close friendships, he begins to discuss the evolutionary role of friendship in relation to survival and its impact on an individual’s safety and health. What is the most likely diagnosis?
A. Obsessive-compulsive personality disorder
B. Obsessive-compulsive disorder
C. Narcissistic personality disorder
D. Schizoaffective disorder
Correct Answer: A.
Obsessive-compulsive personality disorder
Patients with obsessive-compulsive personality disorder (OCPD) are characterized by strict adherence to rules and a lack of adaptability. OCPD is not associated with the intrusive thoughts and ritual performance that characterize obsessive-compulsive disorder (OCD). These people would colloquially be called anal or “control freaks.” This patient also demonstrates intellectualization and isolation of affect, which are 2 common defense mechanisms seen in OCPD.
Incorrect Answers:
B. OCD is associated with intrusive thoughts and ritual performance to relieve the anxiety from intrusive thoughts. These patients do not present with the same extreme rule-following.
C. Narcissistic personality disorder does not present with a focus on rule-following. These patients are much more focused on themselves.
D. Schizoaffective disorder patients would present with signs of schizophrenia with a depressive component, which is not seen in this patient.
Which pair of personality disorders is most often comorbid with generalized anxiety disorder?
A. Borderline and histrionic
B. Avoidant and dependent
C. Histrionic and dependent
D. Borderline and schizoid
Correct Answer: B.
Avoidant and dependent
Dependent and avoidant personality disorders commonly co-occur with generalized anxiety disorder (GAD), although personality disorders may result from GAD. GAD is also commonly comorbid with other anxiety and unipolar depressive disorders. In males with GAD, there is also comorbidity with substance use disorders.
Dependent personality disorder is characterized by a need to be cared for, resulting in clingy and obedient behavior. It is commonly comorbid with depressive disorders, anxiety disorders, and adjustment disorders. It also commonly co-occurs with other personality disorders, including borderline, avoidant, and histrionic.
Patients with avoidant personality disorder often feel inadequate, hypersensitive to rejection/criticism, and avoid social interactions. It is commonly comorbid with depressive, bipolar, and anxiety disorders. It also commonly co-occurs with other personality disorders, including dependent, borderline, paranoid, schizoid, and schizotypal.
Incorrect Answers:
A. Borderline personality disorder is associated with depressive and bipolar disorders, posttraumatic stress disorder, substance use disorders, attention-deficit hyperactivity disorder, and eating disorders. Histrionic personality disorder has been associated with somatic symptom disorder, functional neurological symptoms disorder, and major depressive disorder.
C. Histrionic personality disorder has been associated with somatic symptom disorder, functional neurological symptoms disorder, and major depressive disorder. Dependent PD is often comorbid with GAD.
D. Borderline personality disorder is associated with depressive and bipolar disorders, posttraumatic stress disorder, substance use disorders, attention-deficit hyperactivity disorder, and eating disorders. In some cases, schizoid personality disorder can be premorbid to schizophrenia and delusional disorder.
Vital Concept:
GAD is characterized by pervasive concern or worry and is often comorbid with dependent and avoidant personality disorders.
A certain percentage of the population meets the criteria for one or more personality disorders. What is this percentage?
A. 5%
B. 15%
C. 25%
D. 35%
Correct Answer: B.
15%
Epidemiological surveys show that 9-16% of the general population meets criteria for at least 1 personality disorder.
Incorrect Answers:
A. This percentage prevalence is too low (actual is 15%)
B., D. These percentage prevalences are too high (actual is 15%)
Impulsive, violent criminals have been found to have lower concentrations of a certain substance in their CSF. What is this substance?
A. 5-HIAA
B. Dopamine
C. Acetylcholine
D. Norepinephrine
Correct Answer: A.
5-HIAA
Impulsive offenders who have a tendency to behave aggressively, especially while intoxicated, had low CSF 5-HIAA. Another study demonstrated that impulsive violent offenders possessed lower concentrations of CSF 5-HIAA than non-impulsive violent criminals. Furthermore, a meta-analysis of 20 studies also found significantly reduced levels of 5-HIAA in the CSF of antisocial individuals compared to controls.
Incorrect Answers:
B, C, and D. These aren’t found in lower concentrations in impulsive, violent criminals’ CSF.
A 32-year-old patient who has a history of imprisonment for four counts of assault and robbery states that they just do “whatever they feel like”. Their family reports that they have been lying and cheating since they were 12 years old. They often got into fights at school and in college. They have been fired from over 12 jobs and have alienated most of their family and friends. People with this disorder are likely to demonstrate which feature?
A. Desperate attempts to prevent actual or perceived rejection or desertion by others
B. Is somewhat preoccupied with jealousy and envy- of others or suspicion of others directed at them
C. Onset of conduct disorder before age 15
D. An unfounded belief that people are trying to take advantage of them, lying to them, or otherwise hurting them
Correct Answer: C.
Onset of conduct disorder before age 15
Antisocial personality disorder is characterized by:
- consistent ignorance and abuse of surrounding people’s rights that begins by age 15 as evidenced by at least three of the following:
- dishonesty in the form of defrauding, deceiving, or misleading others for individual gain
- an agitated and sometimes hostile nature with a history of numerous altercations
- a lack of personal accountability as evidenced by a failure to fulfill financial responsibilities or deliver professionally when promised
- lack of concern regarding social norms or laws, with repeated arrests or undetected breaches of the law
- quick decision-making without consideration of consequences or preparation
- lack of concern about the well-being or security of others or oneself
- an inability to feel guilty or personally responsible following the harm, deception, or abuse of another person
The symptoms must be present outside of and notwithstanding a diagnosis of bipolar disorder, schizophrenia, or some other psychotic disorder.
Conduct disorder must be diagnosed or evident prior to age 15, but antisocial PD is not to be confirmed in an individual under 18.
Incorrect Answers:
A. Desperate attempts to prevent actual or perceived rejection or desertion by others are associated with borderline personality disorder.
B. Patients with narcissistic personality disorder are somewhat preoccupied with jealousy and envy- of others or suspicion of others directed at them.
D. Patients with paranoid personality disorder may experience an unfounded belief that people are trying to take advantage of them, lying to them, or otherwise hurting them.
Vital Concept:
A prior history of conduct disorder before the age of 15 is characteristic of antisocial personality disorder.
Which of the following personality disorders is characterized by little desire for relationships and little or no discomfort over the detachment from others?
A. Paranoid
B. Schizoid
C. Schizotypal
D. Obsessive-compulsive
Correct Answer: B.
Schizoid
Schizoid personality disorder is defined as a pattern of detachment from social relationships and restricted expression of emotions in interpersonal settings. This personality disorder typically presents by the patient’s ‘20s and is evidenced by at least four of the following:
· consistently picks actions or events that are performed alone
· does not enjoy most (or any) events, actions, or hobbies
· appears unaffected by people’s positive or negative opinions of them
· does not want or like to be emotionally close to people, including family members
· minimal (or no) desire to have a sexual relationship with someone else
· has very few (or no) people that they are emotionally close to outside of family
· appears to display minimal attachment, emotion, or responsiveness
The symptoms must be present outside of and notwithstanding a diagnosis of bipolar disorder, schizophrenia, or some other psychotic disorder, autism spectrum disorder, or other medical condition.
Incorrect Answers:
A. Paranoid personality disorder is diagnosed when someone is distrustful and suspicious of others without reason.
C. Schizotypal personality disorder is defined by social isolation related to social anxiety and difficulty interacting. These patients can have odd behaviors and appearances.
D. Obsessive-compulsive personality disorder is characterized by perfectionism, rigidity, and a consistent fascination with control, flawlessness, and organization.
Vital Concept:
Schizoid personality disorder is defined as a consistent lack of attachment to others and a limited display of emotions when interacting with others.
In the DSM-5-TR, a personality disorder diagnosis necessitates “an enduring pattern of inner experience and behavior that deviates markedly from the expectations of the individual’s culture” in two or more out of four areas. Which of the following are included in those four areas?
A. Cognition, perceptions, interpersonal functioning, impulse control
B. Cognition, affectivity, interpersonal functioning, dysphoria
C. Cognition, affectivity, interpersonal functioning, occupational functioning
D. Cognition, affectivity, interpersonal functioning, impulse control
Correct Answer: D.
Cognition, affectivity, interpersonal functioning, impulse control
Diagnosing a patient with a general personality disorder requires consistent thoughts, feelings, and actions that are significantly varied from those of the individuals around them. This must be observed in at least of the categories of thought (cognition), emotion (affectivity), interactions with others (interpersonal functioning), and the ability to manage inner desires (impulse control).
Incorrect Answers:
A. Perceptions aren’t one of the four areas
B. Dysphoria isn’t one of the four areas
C. Occupational functioning isn’t one of the four areas
Vital Concept:
Cognition, affectivity, interpersonal functioning, and impulse control comprise Criteria A for personality disorders.
Obsessive-compulsive personality disorder (OCPD) is prevalent in a certain percentage of the general population. What is its prevalence?
A. <1%
B. 1-2%
C. 2-3%
D. 5-10%
Correct Answer: D.
5-10%
Lifetime obsessive-compulsive personality disorder has a prevalence of approximately 7.8% in the general population. It is less common in younger adults and in Asians and Hispanics.
Incorrect Answers:
A, B, C. These prevalence percentages are all too low
When targeting the symptoms of impulsivity and affective dysregulation in a patient with borderline personality disorder, which of the following medications is first line treatment?
A. Lithium
B. Fluoxetine
C. Isocarboxazid
D. Lorazepam
Correct Answer: A.
Lithium
Borderline PD is thought to be secondary to many factors, including genetic, neurobiological, and psychosocial influences. However, specific genetic risk factors have not been identified. Specific brain abnormalities include reduced volume in the amygdala and hippocampus, altered amygdala activation, and hippocampal hypometabolism. Other findings include abnormalities in the prefrontal, corticostriatal, and limbic networks, which could be related to low serotonin neurotransmission and behavioral disinhibition.
The primary treatment for borderline PD is psychotherapy and medications to target symptoms. Most patients will need long-term therapy to see improved interpersonal relationships, personality changes, and overall functioning. Psychodynamic, dialectical-behavioral, transference-focused, interpersonal, and cognitive-behavioral therapies have been effective. Family therapy or couples therapy alone has not been effective. Mood stabliizers reduce impulsive aggression and depressive symptoms. Affective dysregulation is treated with an SSRI or venlafaxine, but TCAs have had inconsistent results. Benzodiazepines should be used sparingly, as research on their use with BPD is limited, and they carry more substantial risks in this population. MAOIs show strong empirical support, but their side effect profiles make them second-line. Patients with borderline PD are unlikely to respond to ECT.
Impulsive-behavioral dyscontrol symptoms that present serious risk to the patient (e.g. aggression, self-mutilation, or self-damaging behaviors like reckless spending, promiscuous sex, and substance abuse) are treated by adding an antipsychotic to an SSRI. Other alternatives include switching to an MAOI. A few studies support the use of lithium, valproate or carbamazepine to treat impulsive aggression. Cognitive-perceptual symptoms (paranoid ideation, derealization, depersonalization, or hallucinations) respond best to low-dose antipsychotics.
Incorrect Answers:
B. Fluoxetine. Fluoxetine is not first line treatment for impulsitiviy or affective dysregulation in borderline PD.
C. Isocarboxazid. MAOIs show strong empirical support, but their side effect profiles make them second-line.
D. Lorazepam. Benzodiazepines should be used sparingly, as research on their use with BPD is limited, and they carry more substantial risks in this population.
Vital Concept:
The primary treatment for borderline personality disorder is psychotherapy but medications can be utilized to target symptoms. For impulsive aggression, mood stabilizers are recommended. For affective dysregulation, use an SSRI or venlafaxine.
There are a number of criteria for avoidant personality disorder. Which of the following is one of those criteria?
A. Pervasive distrust and suspiciousness of others’ motives
B. Pervasive pattern of social inhibition
C. Pervasive and excessive need to be taken care of
D. Pervasive pattern of preoccupation with orderliness
Correct Answer: B.
Pervasive pattern of social inhibition
The diagnostic criteria for avoidant PD are:
A consistent finding of feelings of deficiency, highly touchy when presented with constructive feedback, and shyness or self-consciousness. This personality disorder typically presents by the patient’s ‘20s and is evidenced by at least four of the following:
- resistance to interacting with others unless acceptance is guaranteed prior
- consistent distraction by the prospect or threat of being disparaged or excluded
- a self-image that includes being substandard, socially awkward, and unlikable
- avoids functions or get-togethers that include interacting with others out of concern for reproach or censure
- is hesitant in romantic or close friendships due to concern for being judged or mocked
- is reserved early on in relationships due to a sense of deficiency
- is highly resistant to trying new things or taking risks due to a fear of humiliation
Incorrect Answers:
A. This is a criterion for paranoid personality disorder.
C. This is a criterion for dependent personality disorder.
D. This is a criterion for obsessive-compulsive personality disorder.
Vital Concept:
The criteria for avoidant personality disorder include behaviors that avoid social interaction, activities, or relationships out of fear that they are not good enough or will not be liked.
First-line pharmacology for aggression, due to seizure activity, involves a particular drug. What is it?
A. Antipsychotics
B. Carbamazepine
C. Lithium
D. Levetiracetam
Correct Answer: B.
Carbamazepine
First-line pharmacotherapy for aggression due to seizure activity involves carbamazepine. The second-line option would be benzodiazepines.
Incorrect Answers:
A. Antipsychotics are contraindicated in the treatment of aggressive personality changes due to epilepsy.
C. D. These drugs are neither contraindicated nor recommended for use in managing aggression due to seizures.
For treating patients with histrionic personality disorder, which therapy type is used to teach patients to focus on their unconscious motivations for being unable to commit to a stable and meaningful relationship, instead of seeking out disappointing partners?
A. Cognitive-behavioral therapy
B. Problem-solving approach therapy
C. Interpersonal therapy
D. Psychodynamic therapy
Correct Answer: D.
Psychodynamic therapy
In psychodynamic psychotherapy, the therapist works with the patient to recognize his or her unconscious or conscious motivations for making these decisions. It is especially interested in the dynamic relations between conscious motivation and unconscious motivation. No substantial treatment research exists for histrionic personality disorder (HPD), although 2 meta-analyses of psychotherapeutic treatments for personality disorders suggest that these conditions respond to both psychodynamic therapy and cognitive-behavioral therapy. Interpersonal therapy (IPT) focuses on improving relationships in the present. IPT is particularly accessible to patients who find dynamic approaches confusing or the “homework” demands of cognitive-behavioral therapy too daunting.
Incorrect Answers:
A, B, C. These aren’t indicated for treating patients with histrionic personality disorder.
In addition to an assessment of pathological personality traits, the alternative model in Section III of the DSM-5-TR requires an assessment of which category for a personality disorder diagnosis?
A. Level of impairment in personality functioning
B. Sudden onset of symptoms
C. Degree of introversion versus extraversion
D. Stability of personality traits over time
Correct Answer: A.
Level of impairment in personality functioning
The alternative model in Section III of the DSM-5-TR classifies personality disorders by personality functioning impairment and pathological personality traits. This section includes a scale for measuring the level of impairment, ranging from 0 (no impairment) to 4 (extreme impairment). A diagnosis of a personality disorder would require at least a level 2 (moderate impairment) and at least one pathological personality trait. Impairment in personality functioning predicts the presence of the personality disorder and the impairment severity predicts if the patient has multiple personality disorder or more severe personality disorders. This model is only applied to the following personality disorders: antisocial, avoidant, borderline, narcissistic, obsessive-compulsive and schizotypal.
Incorrect Answers:
B. The impairment must be stable over time with onset in adolescence or early adulthood.
C. There is no assessment in the alternative model for personality disorders, DSM-5-TR section III, for degree of introversion versus extraversion.
D. Both DSM-5-TR sections II and III require stability of personality traits over time in order to receive a diagnosis of a personality disorder.
Vital Concept:
The alternative model in Section III of the DSM-5-TR classifies personality disorders by personality functioning impairment and pathological personality traits.
A person with antisocial personality disorder will demonstrate a number of features. Which of the following is a feature of antisocial personality disorder?
A. Repeated discussions, attempts, or contemplations of suicide or intentional self-harm
B. Onset of conduct disorder before age 18
C. An inability to feel guilty or personally responsible following the harm, deception, or abuse of another person
D. Short-term thoughts of persecution/victimhood or severe derealization or depersonalization
Correct Answer: C.
An inability to feel guilty or personally responsible following the harm, deception, or abuse of another person
Antisocial personality disorder (ASPD) is characterized by:
consistent ignorance and abuse of surrounding people’s rights that begins by age 15 as evidenced by at least three of the following:
- dishonesty in the form of defrauding, deceiving, or misleading others for individual gain
- an agitated and sometimes hostile nature with a history of numerous altercations
- a lack of personal accountability as evidenced by a failure to fulfill financial responsibilities or deliver professionally when promised
- lack of concern regarding social norms or laws, with repeated arrests or undetected breaches of the law
- quick decision-making without consideration of consequences or preparation
- lack of concern about the well-being or security of others or oneself
- an inability to feel guilty or personally responsible following the harm, deception, or abuse of another person
The symptoms must be present outside of and notwithstanding a diagnosis of bipolar disorder, schizophrenia, or some other psychotic disorder.
Conduct disorder must be diagnosed or evident prior to age 15, but antisocial PD is not to be confirmed in an individual under 18.
Incorrect Answers:
A. Self-harm is associated with borderline personality disorder.
B. Conduct disorder is diagnosed in children and adolescents with a repetitive and persistent pattern of behavior violating basic rights of others or major societal norms or rules. While the majority of children with conduct disorder do not develop adult ASPD, they are at risk to do so, with an estimated 25 percent of girls and 40 percent of boys eventually developing ASPD
D. Patients with borderline personality disorder may experience short-term thoughts of persecution/victimhood or severe derealization or depersonalization.
Vital Concept:
Antisocial personality disorder should not be diagnosed in a patient under the age of 18, but a portion of the criteria stipulates that conduct disorder must have been diagnosed or obvious prior to age 15.
A certain personality disorder is diagnosed more frequently in men. Which is it?
A. Borderline
B. Antisocial
C. Avoidant
D. Histrionic
Correct Answer: B.
Antisocial
Antisocial personality disorder occurs more frequently in men. Borderline personality disorder and histrionic personality disorder occur more commonly in women, while dependent and avoidant also show a slight female predilection in some studies and gender neutrality in others. Schizoid personality disorder occurs relatively equally among the genders with a slight male predilection suggested by some sources.
Incorrect Answers:
A and D. These are diagnosed more often in women
C. This shows a slight female predilection in some studies, and gender neutrality in others
Children who develop borderline personality disorder may experience disruption of their development. According to Mahler, what stage of their development may be disrupted?
A. Symbiotic phase
B. Autistic phase
C. Rapprochement
D. Practicing
Correct Answer: C.
Rapprochement
Mahler’s theory is based on psychoanalytic work from observation of children ages 6 months to 3 years. The autistic phase occurs during the first weeks of life and is characterized by total detachment and self-absorption, as the infant spends most of his or her time sleeping. The symbiotic phase occurs until about 5 months of age. The infant recognizes his or her mother but lacks a sense of individuality.
Separation-individuation has 3 phases:
1) hatching, in which the infant shows increased interest in the outside world
2) practicing (9-16 months), in which the infant develops the physical ability to separate from the mother
3) rapprochement (15-24 months), in which the infant explores the outside world but requires that the mother is present and able to support the child appropriately and emotionally in completing the task
Mahler’s work has been expanded by other theorists to understand the basis of personality disorder. Disturbance of the rapprochement subphase is associated with a persistent longing for and dread of fusion with the object that is thought to be secondary to aggression or withdrawal in the mother. Disruption of the rapprochement subphase is thought to contribute to the development of borderline personality disorder.
Narcissistic personality disorders are likely due to inadequate soothing during the symbiotic phase and inadequate refueling during separation-individuation. Object constancy describes the child’s understanding that his or her mother is a separate individual and that he or she is also separate. Object constancy leads to the formation of internalization, which will allow the child to have an internal representation of mother, in turn allowing healthy separation, exploration, and the development of self-esteem.
Incorrect Answers:
A. This isn’t one of Mahler’s stages of infant development
B, and D. These stages of Mahler’s stages of infant development aren’t disrupted in children who develop borderline personality disorder
An 18-year-old college freshman presents with increased anxiety. He reports trouble making friends in college, which is causing him great distress. He was hoping to have “a fresh start” at college, as he did not make friends in high school because he was too worried about being rejected. The patient has always felt isolated because he thinks he is not as smart as his peers and is afraid he will be rejected for being “dumb.” He had planned to join the intramural sports league at his college but backed out because he “wasn’t good enough to play sports” and thought he would only embarrass himself. Additionally, the patient attempted to go to a meet-and-greet in his dorm but was so anxious that people would not like him that he sat in a chair and did not interact with anyone.
Which of the following diagnoses is consistent with his presentation?
A. Antisocial personality disorder
B. Schizotypal personality disorder
C. Schizoid personality disorder
D. Avoidant personality disorder
Correct Answer: D.
Avoidant personality disorder
Avoidant personality disorder is characterized by a consistent and nearly debilitating fear of rejection. These patients often desire close relationships with others, but their rumination on rejection prevents them from interacting with others.
Incorrect Answers:
A. People with antisocial personality disorder can be extroverted and superficially liked; however, they tend to be misanthropic in their actions and show little to no adherence to laws or social standards. There is often a marked legal history with these individuals.
B. Schizotypal personality disorder describes difficulty with socializing combined with a pattern of strange thinking.
C. Schizoid personality disorder is demonstrated in patients who do not form close relationships but generally have little to no desire to socialize or form relationships with others.
Vital Concept:
Avoidant personality disorder is characterized by a consistent and nearly debilitating fear of rejection. Patients with this condition often desire close relationships with others, but can’t interact with others due to their rumination on rejection.
A number of personality disorders share the common features of being anxious and fearful. Which of the following is one of the disorders in that group?
A. Antisocial personality disorder
B. Avoidant personality disorder
C. Borderline personality disorder
D. Histrionic personality disorder
Correct Answer: B.
Avoidant personality disorder
Avoidant personality disorder belongs to the Cluster C group, which shares the common features of being anxious and fearful.
Incorrect Answers:
A. Antisocial personality disorder is a cluster B disorder, characterized by drama, emotion, and impulsivity.
C. Borderline personality disorder is a cluster B disorder.
D. Histrionic personality disorder is a cluster B disorder.
Vital Concept:
Cluster C disorders all present with fear and anxiety. Avoidant PD, dependent personality disorder, and obsessive-compulsive personality disorder are all cluster C disorders
Which class of psychotropic medications is considered first-line treatment for impulsive-behavioral dyscontrol in personality-disordered patients not relieved with psychotherapy alone?
A. MAOIs
B. TCAs
C. Mood stabilizers
D. SSRIs
Correct Answer: C.
Mood stabilizers
Mood stabilizers are considered first-line agents for treating impulsive-behavioral symptoms, such as impulsivity, self-injury, theft, and aggression/interpersonal conflict, in personality-disordered patients. By contrast cognitive-perceptual symptoms, such as hallucinations and delusions/paranoia are treated with a low-dose of antipsychotic medication
Affective dysregulation, which may present with depressed or anxious mood, lability, or anger, is also treated with either a low-dose antipsychotic medication or a mood stabilizer.
Incorrect Answers:
A and B- MAOIs and TCAs have many adverse side effects and are not considered first-line treatment. TCAs can also be fatal in overdose.
D- SSRIs are not first-line therapy for impulsive-behavioral symptoms in patients with personality disorder.
Vital Concept:
Personality disorders are typically treated nonpharmacologically with psychotherapy. If symptoms significantly impair function, some symptoms may be managed with medication. Mood stabilizers are often used for impulsive-behavioral symptoms or affective dysregulation.
A 45-year-old man presents for his first appointment. He introduces himself as “John the spaceman” and is wearing a neon orange tie, a puffy silver coat, and multicolored pants. His mother is deceased, and the only people he feels comfortable speaking with are his father and brother. The patient does not trust other people because he never knows “what they may be plotting.” He works as a mail-sorter in the back of the local post office. Which of the following personality disorders is the most likely diagnosis?
A. Schizotypal
B. Antisocial
C. Histrionic
D. Paranoid
Correct Answer: A.
Schizotypal
This patient exhibits odd beliefs, paranoid ideation, an “off” appearance, a lack of close friends, and social anxiety secondary to paranoid fears. These features are consistent with schizotypal personality disorder.
Incorrect Answers:
B. This patient’s behavior doesn’t exhibit the aggression that’d be seen in antisocial disorder
C. This patient isn’t overly dramatic or emotional, as you’d see in a patient with histrionic disorder
D. There’s no evidence of unreasonable suspicion, as you’d see in paranoid disorder
Which of the following personality disorder(s) have FDA-approved medication(s)?
A. Schizotypal
B. Paranoid
C. Borderline
D. No FDA-approved medications for any personality disorder
Correct Answer: D.
No FDA-approved medications for any personality disorder
Currently, there are no FDA-approved medications for any personality disorder. Psychotherapy is the mainstay of personality disorder treatment at this time. Medications can be used in combination with psychotherapy in severe cases and based on specific symptoms present that could justify adding an antipsychotic, antidepressant or anti-anxiety medication.
Incorrect Answers:
A. There are no FDA-approved medications for schizotypal personality disorder.
B. There are no FDA-approved medications for paranoid personality disorder.
C. There are no FDA-approved medications for borderline personality disorder.
Vital Concept:
Psychotherapy is the mainstay of personality disorder treatment. There are no FDA-approved medications for any personality disorder.
A patient is treated in the emergency department for a laceration after an altercation with their roommate. The patient tells the health care provider in confidence that they plan to kill their roommate. They report having a hunting rifle in their car with which they plan to shoot their roommate. The patient is not intoxicated and appears competent.
Which of the following is the most appropriate course of action?
A. Frankly discuss the potential consequences of this plan with the patient.
B. Keep the patient in the emergency department and inform the police and the intended target.
C. Call the hospital risk management department to determine the best course of action.
D. To keep the patient’s confidentiality, advise him to wait until his temper cools and set a follow-up appointment for tomorrow.
Which of the following statements about depersonalization/derealization disorder is correct?
A. Symptoms are typically persistent.
B. Mean age of onset is 35 years.
C. Depersonalization/derealization disorder is diagnosed in men more often than women.
D. Subjectively altered sense of time is common.
Correct Answer: D.
Subjectively altered sense of time is common.
Common symptoms of depersonalization/derealization disorder include a subjectively altered sense of time, subjective difficulty in vividly recalling past memories, vague somatic symptoms, and fear of irreversible brain damage. There is an association with childhood interpersonal trauma, and it can be precipitated by severe stress or illicit drug use (most notably hallucinogens, ketamine, MDMA, and salvia). Marijuana can precipitate new-onset panic attacks and depersonalization/derealization disorder at the same time.
Incorrect Answers:
A. Symptoms can wax and wane, but in some cases, there is an unwavering, persistent course.
B. The average age of onset is 16 years, and <20% of patients experience onset after age 20 (and 5% after age 25).
C. DDD is equally prevalent in men and women.
Vital Concept:
DDD is often associated with a subjectively altered sense of time.
Fetishistic disorder is considered a paraphilic disorder. How is it defined?
A. Fantasies about watching unsuspecting individuals undressed or in sexual activity
B. Exposing one’s genitals to strangers
C. Rubbing against non-consenting people
D. Use of inanimate objects for sexual stimulation
Correct Answer: D.
Use of inanimate objects for sexual stimulation
Fetishism is characterized by feelings of sexual excitement related to inanimate objects or nonsexual body parts. This occurs over a period of 6 or more months, with associated daydreams and desires.
The inanimate objects cannot be limited to articles of clothing involved in dressing as the opposite gender or those designed for sexual gratification (e.g., vibrator).
The patient has either followed through with their desires or describes them as leading to substantial anguish or dysfunction.
Incorrect Answers:
A. This describes voyeuristic disorder.
B. This describes exhibitionistic disorder.
C. This describes frotteuristic disorder.
Vital Concept:
Fetishism is characterized by feelings of sexual excitement related to inanimate objects or nonsexual body parts.
A 25-year-old woman with a past medical history of cocaine use disorder and bipolar disorder-type I gives birth at 37 weeks of gestation to a baby boy whose APGAR scores were 7 and 8 at 1 and 5 minutes, respectively, losing points for cyanosis. A few hours later, the baby develops tachycardia with heart rate at 180, as well as tachypnea with a respiratory rate at 100. Nasal flaring is visible on physical exam, along with central cyanosis. A chest X-ray reveals a normal cardiac silhouette. However, a transthoracic echocardiogram shows a significantly large right atrium with a very small right ventricle due to apical displacement of the tricuspid valve. Which of the following medications were most likely consumed during the mother’s pregnancy?
A. Valproate
B. Lithium
C. Fluoxetine
D. Cocaine
Correct Answer: B.
Lithium
Not all medications are safe during pregnancy. They are classified according to the level of evidence of their teratogenicity (i.e. birth anomaly induction). The older classification was Class A means no known adverse effects, Class B: no risks have been identified in human beings, Class C: drug safety is unknown due to lack of sufficient research, Class D: research has shown adverse effects in humans, Class X: should be completely avoided in pregnancy. Due to the confusing nature of this system, the FDA required that it be replaced with a narrative section and subsection. Those sections are pregnancy, with risk summaries, registries of cases, clinical data, and clinical considerations. Another section involves lactation, also containing data relevant to the drug’s effect on lactation, clinical considerations, and a synopsis of the adverse effects. The last section deals with the female and male reproductive systems, with subsections dealing with contraception, infertility, and pregnancy testing.
The mother has a history of bipolar disorder-type I. Therefore, suspicion for pharmacotherapy should be present. Commonly, bipolar disorder is treated with Lithium. Lithium causes Ebstein anomaly, which is atrialization of the right ventricle by which the tricuspid valve moves towards the apex of the heart, thus enlarging the right atrium and shrinking the right ventricle. Ebstein anomaly is highly specific for Lithium use during pregnancy. This anomaly leads to poor pulmonary circulation and, consequently, poor blood oxygenation; poor oxygenation leads to tachypnea and tachycardia.
Incorrect Answers:
A. Valproate: it is an anticonvulsant that is used mainly to treat epilepsy, mania, and migraines. It is recommended not to use valproate in women of reproductive age unless it’s absolutely necessary. Its use during pregnancy has been correlated with neural tube defects (such as spina bifida) and low IQ scores. Therefore, it is absolutely contraindicated for migraine prevention in pregnant women and it is highly recommended against for other conditions in the same population. In women of childbearing age using valproate, contraception is usually instituted concurrently.
C. Fluoxetine: it is a Selective Serotonin Reuptake Inhibitor (SSRI) that is used as an antidepressant. Its main use is for depressive disorders, obsessive compulsive disorder, and bipolar type I in combination with olanzapine for treatment-resistant cases. It is a class C in the old classification system, which means that adverse events may have been noticed in animals during gestation. Human results have not been conclusive. SSRIs may be used during pregnancy on individual basis. However, no SSRI has been associated with the Ebstein anomaly. Therefore, this answer choice is incorrect.
D. Cocaine: it is an indirect sympathomimetic. It works by blocking the re-uptake of the catecholamines at the synaptic cleft, thus strengthening their effects. Epinephrine binding to alpha-1 receptors on blood vessels causes vasoconstriction. Cocaine use during pregnancy has been associated with Intra-Uterine Growth Restriction (IUGR), and Placental Abruption. Babies born to mothers who used cocaine during pregnancy are susceptible to being small for gestational age (SGA). However, the newborn in the clinical vignette has a cardiac anomaly that has not been shown to be associated with cocaine use.
A 24-year-old postpartum woman with multiple psychiatric disorders and medications presents, stating that she is planning on breastfeeding her baby. Which of her medications is contraindicated in breastfeeding and requires rigorous monitoring if a mother is planning to breastfeed?
A. Olanzapine
B. Trazodone
C. Lithium
D. Paroxetine
Correct Answer: C.
Lithium
Data suggests that all psychotropic medications (including antipsychotic agents, antidepressant agents, lithium carbonate, and benzodiazepines) are excreted into breast milk. Concentrations of these agents in breast milk can vary considerably. Lithium is relatively contraindicated in breastfeeding; if a mother is taking lithium, breastfeeding is only feasible with rigorous monitoring. Weight should be monitored closely, as well as signs of dehydration, lethargy, and feeding concerns. Laboratory monitoring should include a lithium level, CBC, BUN/Cr, and TSH.
Incorrect Answers:
A. Clinical exposure via breast milk is generally considered to be clinically insignificant. Infants whose mothers took an antipsychotic during pregnancy are generally encouraged to continue this during pregnancy. the infant should be monitored for extrapyramidal symptoms (EPS).
B. Trazodone levels in breastfed infants is relatively low and not expected to cause adverse effects.
D. Paroxetine use during breastfeeding is a low-risk option, with no detectable serum levels of the drug found in breastfed infants and minimal side effects.
Vital Concept:
Of the medications listed, lithium is the most concerning to be administered to a breastfeeding mother.
When bulimia nervosa presents on its own without bipolar disorder, the treatment course is different than for bulimia that’s comorbid with bipolar and substance use disorders. Which of the following treatment options is recommended for bulimia nervosa in the absence of bipolar disorder?
A. SSRI alone
B. Antipsychotic alone
C. Mood stabilizer alone
D. Cognitive behavioral therapy and SSRI
Correct Answer: D.
Cognitive behavioral therapy and SSRI
CBT alone or CBT plus an SSRI is considered adequate treatment for bulimia nervosa. The only SSRI currently FDA-approved for bulimia is fluoxetine. Studies are also suggesting topiramate for use in bulimia.
Incorrect Answers:
A. An SSRI alone isn’t considered adequate for bulimia treatment.
B. An antipsychotic won’t help treat bulimia, as it doesn’t treat the issue at hand.
C. A mood stabilizer alone won’t help treat bulimia, as there’s also a behavioral component that needs to be addressed.
Vital Concept:
Bulimia is often comorbid with bipolar and substance use disorders. The most effective treatment for bulimia overall is CBT. An SSRI alone is not considered adequate treatment for this condition.
Which of the following risks is associated with lithium exposure during gestation?
A. Increased risk of cardiac malformation
B. Increased risk of perinatal mortality, especially when used during the third trimester
C. Low birth weight
D. Increased risk of diabetes mellitus
Correct Answer: A.
Increased risk of cardiac malformation
Antenatal lithium exposure is thought to cause teratogenic effects, especially in the heart and when used during the first trimester. This may include Ebstein anomaly (abnormalities of the tricuspid valve and right ventricle), right ventricular outflow tract obstruction defects, coarctation of the aorta, and mitral atresia.
Incorrect Answers:
B. Antepartum exposure to lithium has not been proven to dramatically affect perinatal mortality, as the research seems contradictory and unclear.
C. Lithium use may be associated with a slightly increased birth weight, but this is often not clinically significant.
D. Lithium exposure during pregnancy has been associated with nephrogenic diabetes insipidus, not diabetes mellitus.
Vital Concept:
The potential risks associated with lithium use during pregnancy are significant and should be explored and explained to all patients of childbearing age.
A 2-year-old girl with 21-hydroxylase deficiency undergoes genitoplasty. Which statement is true concerning her condition?
A. Her penis has been feminized in utero.
B. She has a lower-than-average chance of homosexual orientation.
C. There is a correlation between virilization degree and gender dysphoria.
D. Adequate hormone replacement therapy prevents gender identity disorder.
Correct Answer: C.
There is a correlation between virilization degree and gender dysphoria.
The degree of virilization (graded on the Prader scale) is correlated with the development of gender dysphoria and homosexuality.
Incorrect Answers:
A. Females with congenital adrenal hyperplasia (CAH) have a virilized clitoris. Boys with classic CAH have normal-appearing genitalia.
B. A higher rate of homosexuality has been reported in individuals with CAH, which correlates with the degree of virilization.
D. Successful HRT, while helpful, does not eliminate psychological issues with gender in intersex patients such as females with CAH.
A mother presents to a pediatrician with her 24-month-old son. She explains that the child cries every night non-stop until he is allowed to sleep in the parents’ bed with them. He is otherwise healthy. The mother is somewhat distraught about this situation and relates that she and her husband have had frequently argued about what to do; they both feel it is a problem. Once the child is asleep, he does not snore and does not awaken with nightmares. What is the most appropriate way to advise these parents?
A. Arranging sleep study
B. Putting child to sleep when awake but drowsy
C. Letting child cry through night, as he will fall asleep eventually
D. Planning appropriate limit-setting
Correct Answer: D.
Planning appropriate limit-setting
It is not ideal for these parents to have their 2-year-old sleeping in their bed, especially since they have disagreements over it. The parents should agree on appropriate bed time and bedtime rituals, which must be enforced by both parents with equal rigor.
Incorrect Answers:
A. The child does not have trouble sleeping when in the right environment, which is also the wrong environment for the parents.
B. Putting the child to sleep when awake and drowsy, but not yet sleeping, is helpful but unlikely to solve the problem in this case.
C. Extinction of the behavior may work, and it could be tried in addition to appropriate limit-setting.
Family therapy can be helpful for some anorexia nervosa patients. In particular, which patient population can family therapy be useful for?
A. Family therapy is not particularly helpful in patients with anorexia nervosa.
B. Older patients with longer illness
C. Older patients with shorter illness
D. Younger patients with shorter illness
Correct Answer: D.
Younger patients with shorter illness
Family therapy is most helpful in younger patients with a short duration of illness.
Incorrect Answers:
A. Family therapy can be useful for patients who are younger with shorter illness.
B. Older patients with longer illness likely won’t see much benefit from family therapy.
C. Older patients with shorter illness likely won’t see much benefit from family therapy.
Which of the following is included in the diagnostic criteria for bulimia nervosa in the DSM-5-TR?
A. The ingestion of a much bigger quantity of food than a typical person would ingest over two weeks
B. The perception of being able to regulate the quantity or selection of food consumed during the occurrences
C. In response, the patient will attempt to avoid an increase in weight through purging, which may include exercise, vomiting, fasting, or the use of medications to empty the gut, such as laxatives or diuretics
D. The symptoms coincide with occurrences of anorexia nervosa
Correct Answer: C.
In response, the patient will attempt to avoid an increase in weight through purging, which may include exercise, vomiting, fasting, or the use of medications to empty the gut, such as laxatives or diuretics
Recurrent inappropriate compensatory behaviors to prevent weight gain, such as self-induced vomiting; misuse of laxatives, diuretics, or other medications; fasting; or excessive exercise, are features of bulimia nervosa. The DSM-5-TR diagnostic criteria include:
Binge eating is the ingestion of a much bigger quantity of food within a window of time (i.e., typically no more than 2 hours) than a typical person would ingest at the same time, with the perception of not being able to regulate the quantity or selection of food consumed during the occurrences.
In response, the patient will attempt to counteract the eating occurrences described above and avoid an increase in weight through purging, which may include exercise, vomiting, fasting, or the use of medications to empty the gut, such as laxatives or diuretics.
The eating occurrences and purging are experienced once or more per week for 12 weeks or more.
The patient’s body weight and size directly inform their self-regard and self-esteem.
The symptoms do not coincide with occurrences of anorexia nervosa.
Incorrect Answers:
A. Binge eating is the ingestion of a much bigger quantity of food within a window of time (i.e., typically no more than 2 hours) than a typical person would ingest, not over 2 weeks.
B. Binge eating occurs with the perception of not being able to regulate the quantity or selection of food consumed during the occurrences
D. The symptoms should not coincide with occurrences of anorexia nervosa.
Vital Concept:
Episodes of binge eating with repeated compensatory behaviors to prevent weight gain, such as self-induced vomiting; misuse of laxatives, diuretics, or other medications; fasting; or excessive exercise, are features of bulimia nervosa.
Behavioral therapy may help patients with anorexia or bulimia. Which of the following is a behavioral therapy that may help patients with anorexia or bulimia who binge and purge?
A. Have another person portion out their meals
B. Have a supervisor ensure that they do not binge
C. Have a supervisor watch them for 2-3 hours after every meal
D. Keep a log of when they feel like binging or purging
Correct Answer: C.
Have a supervisor watch them for 2-3 hours after every meal
Patients with these disorders rarely feel comfortable self-inducing vomiting in front of another person. Having them supervised for 2-3 hours after the meal ensures that the food is digested before they would purge it. The goal is to stop the cycle of binging and purging altogether. DSM-5-TR criteria for anorexia include
limitation of caloric intake resulting in a markedly decreased body mass index (BMI) or weight as compared to similar individuals of the same sex, age, or height.
extreme dread of an increase in weight or of being overweight, or actions that prevent weight gain, despite being severely underweight
distorted perception of their own body and their weight, an exaggerated impact of their weight on their self-regard, or a consistent disregard or lack of awareness of the consequences of their health condition or their behavior
Incorrect Answers:
A. Having another person portion out their meals would not help these patients to avoid purging.
B. Stopping the entire cycle is needed; it is insufficient to avoid binge eating. The patient may also need assistance to avoid purging after typical-sized meals.
D. While it is important to facilitate the recognition of triggers and urges, this alone won’t necessarily stop the targeted behavior
Vital Concept:
A patient that is struggling to avoid purging may find it beneficial to have accountability through supervision for 2-3 hours after eating a meal.
If a patient with an eating disorder presents with certain criteria, they should be admitted. Which of the following is an indication for admission?
A. Heart rate 65bpm
B. Glucose 80 mg/dL
C. Potassium <4.0mEg/L
D. Temperature <97.0°F
Correct Answer: D.
Temperature <97.0°F
Incorrect Answers:
A. This is a normal resting heart rate (60-100bpm is normal range), so it’s not an indication for admission.
B. This is a normal glucose reading (70-130 mg/dL is normal range), so it’s not an indication for admission
C. Potassium has to be below <3mEq/L to be an indication for admission — this is above that threshold.
The predominances of different P450 enzymes varies throughout people’s lives. Which cytochrome P450 enzyme is predominantly present at birth?
A. 1A2
B. 2D6
C. 2C19
D. 3A7
Correct Answer: D.
3A7
Clearance is the volume of plasma from which a drug is removed per unit of time. Most psychotropic drugs are eliminated by hepatic metabolism followed by renal excretion. Hepatic metabolism is broken into Phase I and Phase II reactions. Phase I reactions involve the formation of a more polar metabolite that can be eliminated renally or further metabolized by Phase II conjugation. Phase I reactions include oxidation, reduction, and hydrolysis. Most Phase I oxidation reactions are carried out by the hepatic cytochrome P450 system. The CYP enzymes most involved in the metabolism of drugs are CYP 1A2, 2C9, 2C19, 2D6, and 3A4. Adult values for the cytochrome P450 enzymes are generally achieved by 1 year of age. CYP3A7 is the predominant P450 enzyme at birth, but its levels fall quickly and are undetectable in most adults. Because psychotropic medications are not generally prescribed to children under the age of 5, CYP3A7 is the enzyme that is least important in the metabolism of psychotropic drugs of the enzymes listed.
Incorrect Answers:
A. CYP1A2 is the last cytochrome to develop.
B. C. CYP3A4, 2D6, and 2C enzymes are present in newborns at very low concentrations but surge quickly to about 20-30% of adult values within the first week of life.
Paraphilias are abnormal sexual desires. Which of the following statements about paraphilias is true?
A. They include only fetishism, pornography addiction, sexual masochism, and exhibitionism.
B. People with paraphilias seldom act out their fantasies.
C. They are commonly diagnosed in clinical settings.
D. They are almost never diagnosed in females.
Correct Answer: D.
They are almost never diagnosed in females.
Paraphilias are almost never diagnosed in females, though there are case reports of some.
Incorrect Answers:
A. The chapter on paraphilic disorders in DSM-5-TR includes 8 conditions: exhibitionistic disorder, fetishistic disorder, frotteuristic disorder, pedophilic disorder, sexual masochism disorder, sexual sadism disorder, transvestic disorder, and voyeuristic disorder.
B. People often do act out their fantasies on willing or unwilling/unknowing victims.
C. They are rarely diagnosed in the clinical setting but are likely quite prevalent.
Vital Concept:
Paraphilic disorders are rarely diagnosed in females and are predominantly diagnosed in males.
A patient who had been reported missing by their spouse 10 days prior was identified in a town over 400 miles from home. Coworkers state that on the day they disappeared, they were acting normally but never returned from lunch. When found, the patient was using a different name, was staying at a motel, and did not recall their previous name or their spouse. They seem truly puzzled and upset by the police stating that they are someone else and bringing them home. Physical exam, laboratory workup, and CT imaging reveal no abnormalities. What is the most likely diagnosis?
A. Depersonalization/derealization disorder
B. Dissociative amnesia with dissociative fugue
C. Transient global amnesia
D. Transient ischemia (TIA)
Correct Answer: B.
Dissociative amnesia with dissociative fugue
This is dissociative amnesia with a dissociative fugue (DSM-5-TR). Dissociative amnesia with dissociative fugue criteria includes an inability to recall autobiographical information and apparently purposeful travel associated with amnesia. The 12-month prevalence for dissociative amnesia is approximately 1.8% (1% for men, 2.6% for women). Onset is usually sudden, and memory loss may be refractory. There is a significantly elevated suicide risk in patients with dissociative amnesia, particularly when amnesia remits. Dissociative fugue is present If the patient presents with intentional travel or dazed roaming to uncover lost memories and information.
Incorrect Answers:
A. Depersonalization/derealization disorder is often characterized by the patient as feeling detached from their body. They often describe the experience as if they are watching from outside the body. There is no associated memory loss or change in personality or behavior.
C. Transient global amnesia is a temporary episode of memory loss that is not associated with a neurological cause. The onset is sudden, and the person is unable to recall who there are, how they got there, or what they were doing.
D. A transient ischemia attack (TIA), often referred to as a ministroke, produces effects similar to a stroke, but they resolve rapidly. This may include motor or sensory deficits. There is no associated change in memory or personality.
Vital Concept:
Dissociative amnesia is associated with generalized, localized, or selective memory loss.
Children develop gender identity at a fairly young age. When do they begin to develop gender identity?
A. 2
B. 3
C. 4
D. 5
Correct Answer: A.
2
Gender identity usually begins to form by 18-24 months and is extremely unlikely to change after that point. Gender identity becomes rigid/inflexible around the ages of 5–7 years, and socially defined gender roles relax somewhat after that age.
Incorrect Answers:
B. C. These aren’t meaningful ages in terms of gender identity development.
D. This is the age at which gender identity starts becoming rigid/inflexible. Gender identity becomes rigid/inflexible around ages 5-7.
Interpersonal psychotherapy may be used to treat individuals with bulimia nervosa. When used in this way, what’s the goal of this treatment?
A. Encourage patients to enjoy food without feeling the need to binge and purge
B. Correct inappropriate beliefs they have about themselves and their disorder
C. Alter their response to periods of extreme stress or anxiety
D. Address relationship-related stressors contributing to eating disturbance
Correct Answer: D.
Address relationship-related stressors contributing to eating disturbance
Interpersonal psychotherapy normalizes the disturbed eating behavior, decreasing the number of binge-purge episodes by addressing interpersonal sources of stress believed to precipitate or promote the behavior. This therapy is also effective in patients with binge-eating disorder.
Incorrect Answers:
A. Interpersonal psychotherapy doesn’t encourage patients to enjoy food.
B. Interpersonal psychotherapy doesn’t correct patients’ beliefs about themselves or their bulimia disorder.
C. Interpersonal psychotherapy doesn’t alter patients’ response to extreme stress or anxiety
Certain indicators require hospitalization of patients with eating disorders. Of the following characteristics, which requires hospitalization?
A. Nausea
B. Vomiting
C. Hypotension
D. Depression
Correct Answer: C.
Hypotension
Hypotension, hypothermia, severe starvation, weight loss, and electrolyte imbalance are the main indications for hospitalization.
Incorrect Answers:
A. Nausea doesn’t indicate inpatient treatment — but it might be a sign of starvation or electrolyte imbalances, and may worsen those imbalances
B. Vomiting doesn’t indicate inpatient treatment — but it might be a sign of starvation or electrolyte imbalances, and may worsen those imbalances
D. Depression on its own doesn’t indicate inpatient treatment — only depressed patients with eating disorders with suicidal ideations or psychosis are indicated for inpatient treatment
Generally, any SSRI can be considered to treat bulimia. However, only one medication is FDA-approved. Which of the following is it?
A. Mirtazapine
B. Fluoxetine
C. Amitriptyline
D. Bupropion
Correct Answer: B.
Fluoxetine
In general, any SSRI can be considered for treatment of bulimia, but only fluoxetine is FDA-approved. TCAs should be avoided due to their potential to increase cardiac toxicity. MAOIs should be avoided in these patients. Bupropion should be avoided in all eating disorder patients due to seizure risk.
Incorrect Answers:
A Mirtazapine is an atypical antidepressant. It is not FDA approved for the treatment of bulimia.
C. Amitriptyline is a tricyclic antidepressant; these should be avoided in bulimia patients because of their potential to increase cardiac toxicity.
D. Bupropion should be avoided in all eating disorder patients due to seizure risk.
Vital Concept:
Fluoxetine is the only medication approved by the FDA for the treatment of bulimia nervosa.
Christopher, who is 10 months old is playing with his favorite toy elephant. His older brother comes and hides it behind their play kitchen. Christopher begins fussing until his mother helps him retrieve the toy elephant. What does Christopher’s understanding of the toy still being there even though it’s out of sight demonstrate?
A. Object permanence
B. Reversibility
C. Conservation
D. Sensorimotor
Correct Answer: A.
Object permanence
Object permanence is the ability to understand that objects have an existence independent of the child’s involvement with them. Reversibility is the realization that one thing can turn into another and back again. Conservation is the ability to recognize that although the shape of an object may change, it will still maintain characteristics that enable it to be recognized as that object.
Incorrect Answers:
B. This is the realization that things can turn into each other and back again; it doesn’t apply here.
C. This is the ability to recognize that although an object’s shape may change, it will maintain characteristics that allow it to be recognized as that object; it doesn’t apply here.
D. This is the first of Piaget’s four stages of cognitive development. It doesn’t explain Christopher’s understanding of the toy still being there.
A clinician is looking at the MRI of a patient’s brain. If the patient has a history of chronic and persistent schizophrenia, which of the following features will they most likely see on the scan?
A. Diminished size of body and temporal horn of lateral ventricles
B. Hippocampal hypertrophy
C. Decreased volume of lateral temporal lobe
D. Smaller hippocampal volume
Correct Answer: D.
Smaller hippocampal volume
Structural differences in the hippocampus have been well documented, showing smaller volumes of the hippocampus with similar volumetric changes within the medial temporal and prefrontal lobes. The lateral ventricles may also be enlarged. Schizophrenia affects 1% of the population worldwide. It is characterized by positive symptoms (hallucinations, delusions, and disorganized speech and behavior) and negative symptoms (flattened affect, alogia), as well as other symptoms, including inattentiveness. Some patients with schizophrenia also lack insight regarding their condition. There is usually significant impairment in their social, occupational, and interpersonal function. To diagnose schizophrenia, symptoms must be present for at least 6 months, including at least 1 month of positive or negative symptoms.
The diagnosis of schizophrenia has expanded in the DSM-5. Criterion A now includes five items: delusions; hallucinations; disorganized speech (e.g., frequent derailment or incoherence); grossly disorganized or catatonic behavior; and negative symptoms (diminished emotional expression or avolition). At least 2 of the 5 symptoms must be present for at least 1 month, and 1 of the 2 symptoms must be delusions, hallucinations, or disorganized speech. Negative symptoms, which impair function the most, are now officially included. Continuous signs of disturbance must be present for at least 6 months with at least 1 month of the symptoms listed above.
Incorrect Answers:
A. The lateral ventricles are generally enlarged, not smaller, as the hippocampus atrophies.
B. The hippocampal volume is typically decreased in patients with schizophrenia.
C. Smaller volume is evident in the medial temporal lobe, not the lateral temporal lobe.
Vital Concept:
Consistent changes have been observed in patients with chronic schizophrenia, such as enlarged lateral ventricles and decreased volume in the hippocampus, medial temporal, and prefrontal lobes.
A 14-month-old child cries and becomes irritable when her mother leaves her with the neighbor for a few minutes. The child is familiar with the neighbor, who often visits. When the mother returns, the child is comforted, stops crying, and shows an affectively positive response to her mother. What does the child’s response to the mother’s absence indicate?
A. Normal developmental process of separation anxiety
B. Insecure attachment to mother
C. Recent abuse or maltreatment by neighbor
D. Stranger anxiety (child is too young to recognize neighbor)
Correct Answer: A.
Normal developmental process of separation anxiety
Separation anxiety is a normal developmental process. It is most commonly seen at 10-18 months of age.
Incorrect Answers:
B. C. Nothing in this scenario indicates insecure attachment to the mother (since the child is comforted and shows a positive response when the mother returns) or abuse or maltreatment by the neighbor.
D. Stranger anxiety is typically seen around 8 months of age and would be unlikely, as the baby is familiar with the neighbor.
A 30-year-old male with a recent 6-month first episode of depression is currently being treated with paroxetine. He presents with concerns about his sexual function. He is having difficulty maintaining arousal and achieving orgasm during vaginal intercourse. The patient does become aroused and is able to achieve orgasm during oral sex but maintains minimal interest in penetrative sex. In his previous sexual encounters, since his first at age 16, he has solely participated in oral sex and had avoided vaginal or anal intercourse due to his lack of interest in or arousal by these acts. He is only attempting vaginal sex at this time because he and his wife are trying to conceive. Which of the following diagnoses explains the patient’s concerns?
A. Medication side effect
B. Non-pathological result of stress from trying to conceive
C. Frotteurism
D. Partialism
Correct Answer: D.
Partialism
Partialism or oralism is when an individual is only aroused by and can only achieve orgasm through a single type of sexual contact, usually relegated to a specific anatomical location. These individuals may even attempt to avoid penetrative sex due to a lack of interest. This is a type of paraphilia.
Incorrect Answers:
A. While SSRIs like paroxetine can have sexual side effects, these are mostly manifested as delayed ejaculation or difficulty maintaining an erection. Although this patient reports difficulty maintaining arousal, his difficulty is isolated to penetrative sex and is not present when engaging in oral sex. Additionally, the patient’s first depressive episode occurred 6 months ago, and the medication was likely started at that time. The patient reports a lack of interest in vaginal intercourse consistently since his first sexual encounter.
B. The patient reports a lack of interest in penetrative sex persistently since age 16. Because his lack of interest is not temporally correlated with his and his wife’s attempts to conceive, this is unlikely the explanation.
C. Frotteurism is a paraphilic disorder characterized by arousal by or orgasm from touching or rubbing against a fully clothed sexual partner or a non-consenting individual in a public place.
An adult male is referred following his release from jail. He was charged with sexual misconduct for rubbing against and grabbing a 19-year-old woman’s buttocks and breasts while riding the bus. He has done this countless times but was only caught once. What is this disorder called?
A. Fetishistic disorder
B. Exhibitionistic disorder
C. Pedophilic disorder
D. Frotteuristic disorder
Correct Answer: D.
Frotteuristic disorder
Frotteuristic disorder is characterized by feelings of sexual excitement related to coming into contact with or brushing up against an individual without their knowledge. This occurs over a period of 6 or more months, with associated daydreams and desires.
The patient has either followed through with their desires or describes them as leading to substantial anguish or dysfunction.
Incorrect Answers:
A. Fetishistic disorder is the sexual use of non-living objects.
B. Exhibitionistic disorder involves exposing one’s genitals to a stranger.
C. Pedophilic disorder is sexual activity with a prepubescent child.
Vital Concept:
Frotteuristic disorder is characterized by feelings of sexual excitement related to coming into contact with or brushing up against an individual without their knowledge.
Patients who display factitious disorder by proxy are also likely to have a certain condition. What is this condition?
A. Family history of schizophrenia
B. Medical history of psychosis
C. Major medical condition
D. Personality disorder
Correct Answer: D.
Personality disorder
Patients who have factitious disorder by proxy (Munchausen syndrome) are more likely to have personality disorders and other psychiatric disorders.
Incorrect Answers:
A. B. They are not likely to have a disorder involving psychosis.
C. A major medical condition may or may not be present in a person with factitious disorder by proxy.
Which of the following diagnoses has a higher female prevalence?
A. Bipolar I disorder
B. ADHD
C. Schizophrenia
D. Major depressive disorder
Correct Answer: D.
Major depressive disorder
Correct Answer: D. Major depressive disorder
Major depressive disorder has a female-to-male predominance of 1.5:1 to 3:1 in some studies, beginning in early adolescence.
Incorrect Answers:
A. Bipolar I disorder has a male-to-female ratio of 1.1:1.
B. ADHD is more frequent in males.
C. Schizophrenia is generally considered to have a male-to-female ratio of 1.4:1, but this differs across samples depending on the emphasis on negative symptoms and poorer outcomes (male predominance) versus inclusion of more mood symptoms and brief presentations (equal predominance 1:1 for both sexes).
Vital Concept:
Depression affects roughly twice as many women as it does men according to most prevalence studies.
In the DSM-5-TR, there is a category called “somatic symptom and related disorders” which all share the prominence of somatic symptoms associated with significant distress and impairment. Which of the following disorders falls into this category?
A. Somatization disorder
B. Hypochondriasis
C. Pain disorder
D. Illness anxiety disorder
Correct Answer: D.
Illness anxiety disorder
In the DSM-5-TR, the category “somatic symptom and related disorders” includes the diagnoses of somatic symptom disorder (SSD), illness anxiety disorder, conversion disorder (functional neurological symptom disorder), psychological factors affecting other medical conditions, factitious disorder, other specified somatic symptom and related disorder, and unspecified somatic symptom and related disorder. All of the disorders in this chapter share the prominence of somatic symptoms associated with significant distress and impairment.
Incorrect Answers:
A. B. and C. These have all been removed from the DSM-5, and many of the individuals who’d previously been diagnosed with one of these disorders can now be diagnosed with somatic symptom disorder (SSD), which is part of the category of somatic symptom and related disorders.
Vital Concept:
Somatic symptoms and related disorders includes somatic symptom disorder (SSD), illness anxiety disorder, conversion disorder (functional neurological symptom disorder), psychological factors affecting other medical conditions, factitious disorder, other specified somatic symptom and related disorder, and unspecified somatic symptom and related disorder.
A 19-year-old female patient is referred for evaluation. She weighs approximately 78% of her ideal body weight for her height. She has been amenorrheic for several months. The patient states that she is fat and refuses to gain weight, despite appearing cachectic on physical exam. She also admits to using laxatives and diuretics daily. Which of the following findings are expected in this patient?
A. Hypokalemic alkalosis
B. Hypokalemic acidosis
C. Hypochloremic acidosis
D. Hyperkalemic alkalosis
Correct Answer: A.
Hypokalemic alkalosis
Hypokalemic alkalosis is most likely due to the daily use of laxatives and diuretics. Diarrhea associated with chronic laxative abuse is rich in potassium and chloride (70–90mEq/L), in combination with increased renal-bicarbonate reabsorption and volume contraction due to a profound loss of sodium and water, resulting in hypokalemia and a saline (chloride)-responsive metabolic alkalosis. Diuretics also cause metabolic alkalosis, likely derived from multiple contributing mechanisms. Secondary hyperaldosteronism often develops due to volume depletion, renal chloride loss, or a contraction alkalosis. Chloride-unresponsiveness may also develop due to a profound K depletion from either chronic thiazide or loop-diuretic exposure. Patients might also have leukopenia with relative lymphocytosis, elevated bicarbonate, and hypochloremia. These electrolyte abnormalities can cause cardiac arrhythmias and even death from cardiac arrest.
Incorrect Answers:
B. Acute diarrhea usually produces a hyperchloremic metabolic acidosis from bicarbonate losses in stool, but diarrhea associated with chronic laxative abuse is rich in potassium and chloride (70–90mEq/L), resulting in hypokalemic alkalosis.
C. Acute diarrhea usually produces a hyperchloremic metabolic acidosis from bicarbonate losses in stool, but diarrhea associated with chronic laxative abuse is rich in potassium and chloride (70–90mEq/L), resulting in hypokalemic alkalosis.
D. Hyperkalemic alkalosis may occur with certain endocrine disorders (e.g., Cushing’s syndrome) and conditions affecting the renal system (e.g., Barrter’s and Gitelman’s).
Vital Concept:
A patient utilizing laxatives and diuretics daily as a component of a pathological eating disorder or body dysmorphia will most likely present with hypokalemic alkalosis due to the loss of potassium, chloride, and water.
Which of the following descriptions of dissociative identity disorder (DID) is correct?
A. DID has a prevalence among psychiatric inpatients of <0.5%
B. DID is a temporary disorder
C. Patients with DID typically recall events once the dissacocation is over
D. DID usually occurs with comorbid psychiatric conditions
Correct Answer: D.
DID usually occurs with comorbid psychiatric conditions
Dissociative identity disorder criteria was updated in the DSM-5. Criterion A was expanded to include some possession-form phenomena and functional neurological symptoms to account for more diverse presentations of the disorder. Also, Criterion A specified that transitions in identity may be observable by others or self-reported. Lastly, according to Criterion B, individuals with dissociative identity disorder may report recurrent gaps in recall of everyday events, not just of traumatic experiences. The diagnostic criteria in the DSM-5-TR now include:
The presence of more than two distinct personalities that disrupt an individual’s identity or sense of self (some cultures refer to this as being possessed). Symptoms that are reported by the individual or observed by others include changes in:
- cognition
- emotional response
- conduct
- sensory-motor functioning
- alertness or awareness
- memory or recollection
- awareness or perception
Symptoms affect the individual’s social and professional functioning.
There are recurrent gaps in memories that cannot be attributed to forgetfulness.
The symptoms cannot be attributed to a cultural or religious practice; in children, the symptoms are not explained by expected imaginative play for their age.
Symptoms cannot be attributed to a medical condition or the use of mind-altering substances.
Incorrect Answers:
A. DID has a prevalence among psychiatric inpatients of approximately 3%. DID is present in 1% of the general population and comprises around 10% of dissociative disorders.
B. DID is a chronic and recurrent disorder.
C. Patients with DID typically have issues recalling events after they have occurred.
Vital Concept:
Patients with DID often present with comorbid depression, anxiety, substance abuse, self-injury, and non-epileptic events.
The process of drug absorption and transportation differs in children and adults until children reach a certain age. At what age does the process of absorption and transportation of drugs become comparable between children and adults?
A. 4 months
B. 12 months
C. 2 years
D. 4 years
Correct Answer: C.
2 years
Pharmacokinetics refers to the rates of absorption, distribution, metabolism, and elimination of drugs. Bioavailability, volume of distribution, half-life, and clearance dictate how much exposure a patient will have to a drug at a particular dose. At birth, the absorption and transport of drugs are affected by a high gastric pH, slowed gastric emptying, and irregular peristalsis. By 24 months, these processes have matured enough to be comparable to adult metabolism, albeit somewhat slower. The immaturity of drug metabolizing enzymes in the newborn can lead to a higher bioavailability of drugs that are extensively metabolized by first-pass metabolism. Topical administration of drugs to infants can also result in higher plasma concentrations because infants have a thinner skin (specifically the outermost layer of the epidermis), greater hydration of the epidermis, and a much higher ratio of surface area to body mass.
Incorrect Answers:
A. Gastric pH reaches adult values around 2 years of age.
B. Glomerular filtration rate increases rapidly after birth and reaches adult values around 1 year of age. Adult values for the cytochrome P450 enzymes are also generally achieved by this age.
D. There are few clinically significant differences in the volume of distribution between older children and adults. Psychotropic medications are not generally prescribed to children under the age of 5; by this time, the process of drug metabolism has matured sufficiently. Dosing medication in older children and adolescents mainly relies on differences in body size between child and adult dosing.
Which of the following is a postulated physiological disturbance that may explain some symptoms of panic?
A. Decreased catecholamines in the central nervous system
B. Abnormality of locus coeruleus
C. Respiratory acidosis
D. Activation of the parasympathetic nervous system
Correct Answer: B.
Abnormality of locus coeruleus
Fear triggers the amygdala that activates the sympathetic nervous system and the hypothalamic pituitary adrenal (HPA) axis. The sympathetic nervous system activates the locus coeruleus and adrenal medulla (increases epinephrine and norepinephrine). The locus coeruleus is a nucleus in the pons of the brainstem and produces norepinephrine . The HPA axis releases corticotropin releasing factor (CRF) activating secretion adrenocorticotropic hormone (ACTH) causing secretion of cortisol.
Incorrect Answers:
(A) Decreased catecholamines in the central nervous system. With panic attacks get increased catecholamines in the CNS including epinephrine and norepinephrine released from the adrenal medulla.
(C) Respiratory acidosis. Hyperventilation during panic (causing decreased CO2) can cause respiratory alkalosis, not respiratory acidosis, and hypocapnia
(D) Activation of the parasympathetic nervous system. Panic attacks activate the sympathetic nervous system, not parasympathetic, stimulating the flight or fight response.
A 19-year-old actress presents who is amenorrheic. The patient is 85% of her expected weight for her age and height and refuses to eat out of fear of being “too fat to get a part in movies.” Her vital signs and labs (CBC and chemistry panel) are within normal limits. Psychiatric hospital admission is recommended, but she refuses to comply. What is the appropriate response?
A. Admit patient involuntarily
B. Ask ethics committee to review case
C. Refuse to treat patient if she will not be admitted
D. Allow patient to return home, as risk of imminent danger is low
Correct Answer: D.
Allow patient to return home, as risk of imminent danger is low
A patient with anorexia nervosa presenting to a physician should only be admitted against his or her will when there is a risk of probable death from complications of malnutrition. This patient’s labs and vital signs are normal. The advice of an ethics committee is not required. Do not threaten the patient with abandonment or refer her, as these actions do not address her problem.
Incorrect Answers:
A. Patients with anorexia should only be admitted against their will when there’s a risk of probable death from malnutrition complications — this case doesn’t reach that standard.
B. The ethics committee isn’t needed to weigh in on this case.
C. Refusing to treat the patient doesn’t address her problem.
Which neurotransmitter is thought to play the most significant role in social phobia?
A. Cortisol
B. Gamma-aminobutyric acid (GABA)
C. Glutamate
D. Dopamine
Correct Answer: D.
Dopamine
Dopamine, serotonin, and noradrenaline have all been suggested to play some role in social phobia. Dopamine is thought to mediate social interest, gregariousness, confidence, and sensitivity to rejection. People with low levels of dopamine in the CSF and Parkinson’s disease are more likely to be introverted and to have social phobia as well.
Incorrect Answers:
(A) Cortisol. Cortisol does not play the most significant role in social phobia but does in stress response and panic disorder.
(B) Gamma-aminobutyric acid (GABA). Reduced GABA has been associated with panic disorder and depression.
(C) Glutamate. Glutamate impairment has been associated with anxiety disorder but dopamine is the best answer choice for playing the most significant role in social phobia.
The PMHNP is conducting an interview at 1pm via Telehealth on a Veteran who was discharged from the navy one year ago. The patient is being followed by the Veteran’s Administration (VA) for bipolar disorder and olanzapine (Zyprexa) therapy. The patient states “I am feeling kind of sluggish, but what is the worst is the weight gain”. When questioned about weight gain, the patient stands up to be fully visible on camera and states “I’ve gained 100 pounds in the last year”. Which of the following labs would be accurate and readily available that day to help in evaluating this patient?
A. Random glucose
B. Random cholesterol
C. Hemoglobin A1C
D. Fasting glucose
Correct Answer: C.
Hemoglobin A1C
It does not require fasting and is very accurate in evaluating overall glucose metabolism. In most labs it is readily available the same day.
Incorrect Answers:
A. Random glucose. A random glucose is a glucose level that can be drawn at any time. It can be helpful depending on the level, but is not as accurate. A fasting glucose would be needed to support the diagnosis of metabolic syndrome.
B. Random cholesterol. A random cholesterol is a cholesterol that can be drawn at any time. The patient’s appointment is at 1pm; therefore the patient should be instructed to have the labs drawn the next day after fasting 8-10 hours. In addition to a fasting cholesterol, a High Density Lipid (HDL) is another indicator for the diagnosis of metabolic syndrome. If the HDL is <40 in a male patient (<50mg/dL in females), the HDL can be a positive indicator of metabolic syndrome and further follow up will be required.
D. Fasting glucose. A fasting glucose would be helpful in evaluating this patient’s glucose, but it requires the patient be NPO for 8-10 prior to testing. The patient’s appointment is at 1pm; therefore the patient should be instructed to have the labs drawn the next day. If the fasting glucose is >110 mg/dL, that is a positive indicator of metabolic syndrome and further follow up will be required.
Vital Concept: A Hemoglobin A1C measures the concentration of glucose over 120 days in red blood cells.
A 5-year-old child is brought in for evaluation because the parents are concerned about the child’s behavior. The child was assigned male sex at birth. At home, the child likes to go through the parents’ closet and wear the mother’s dresses. The child uses action figures and cars in elaborate tea-time rituals. When asked by the physician why they like to wear women’s clothing, the child matter-of-factly replies, “because I am a girl.” According to the parents, the child has stated that they are a girl since age 3. On physical exam, the patient has normal-appearing male genitalia. Which of the following describes the child’s sexual characteristics?
A. Male sex assignment and male gender identity
B. Male sex assignment and female gender identity
C. Female sex assignment and female gender identity
D. Female sex assignment and male gender identity
Correct Answer: B.
Male sex assignment and female gender identity
The four interrelated psychosexual factors are sexual identity, gender identity, sexual orientation, and sexual behavior. Sexual identity reflects inborn biological sexual characteristics: chromosomes, genitalia, hormones, etc. Gender identity reflects a person’s individual sense of being male or female. It is generally set by age 3 and very difficult to change. This child’s sex assignment is male, but they associate with the female gender. Thus, the child’s sexual identity is male, but their gender identity is female.
In DSM-5-TR, people whose gender at birth is contrary to their identified gender leading to anguish and dysfunction are diagnosed with gender dysphoria. While sexual orientation is not necessarily fully revealed in early childhood, both sexual and gender identities are present early in development.
Incorrect Answers:
A. The boy saying “because I am a girl” indicates female, not male, gender identity.
C. D. As the child has male genitalia, they have a male, not female, sex assignment. This means that the child was assigned dmale sex at birth.
Vital Concept:
Gender dysphoria describes a misalignment or disagreement between the patient’s identified gender and anatomical sex.
Certain attitudes, motivations, and behaviors are attributable to factitious disorder and malingering. Which of the following statements accurately describes a behavior or motivation involved in factitious disorder and malingering?
A. Factitious disorder involves secondary gain such as getting out of criminal charges.
B. Malingering involves assuming a sick role to get attention from others.
C. When considering underlying disorders contributing to malingering behaviors, antisocial personality disorder is a common feature.
D. Factitious disorder can include feigning sickness to get a prescription of narcotics.
Correct Answer: C.
When considering underlying disorders contributing to malingering behaviors, antisocial personality disorder is a common feature.
Malingering, especially in criminal cases where persons are trying to get out of criminal charges by feigning mental illness or other illness, can be associated with antisocial personality disorder. Patients are diagnosed with a factitious disorder or malingering because they are “faking” or even creating symptoms that have no real pathological basis.
Differentiation between factitious disorder and malingering involves determining the patient’s motivation.
In malingering, there is some secondary gain (e.g. getting out of a crime by “pleading insanity,” making money).
In factitious disorder or factitious disorder by proxy, the patient wants to assume the “sick role” (such as in Munchausen syndrome). The 2 disorders require different treatments. Treatment of these disorders is tricky, and the physician must be sure that no real pathology is causing the symptoms first in order to avoid medico-legal consequences. Factitious disorder by proxy is considered child maltreatment and needs to be reported to the authorities.
Incorrect Answers:
A. As stated above, malingering is the condition involving secondary gain as motivators.
B. This is associated with factitious disorder.
D. Factitious disorder does not involve motives of secondary gain.
Which of the following terms is matched with its correct definition?
A. Localized amnesia: loss of memory subsequent to an event until and including the present
B. Selective amnesia: lack of memories from a circumscribed period of time, often immediately following a traumatic event
C. Generalized amnesia: patient can recall some but not all events from a particular period
D. Systematized amnesia: loss of memory relating to a particular category of information
Correct Answer: D.
Systematized amnesia: loss of memory relating to a particular category of information
Systematized amnesia is the loss of memory relating to a particular category of information. Previously referred to as psychogenic amnesia, the DSM-5-TR describes it as a type of dissociative amnesia where the patient has no memory of a specific event or grouping of information or the memories are fragmented (e.g., the individual remembers the home they grew up in but does not remember the presence of a specific room of the house). Individuals rarely discuss these symptoms with others and attempt to justify or downplay the loss of particular memories.
Incorrect Answers:
A. Localized amnesia is the lack of memories from a circumscribed period of time, usually immediately following a traumatic event.
B. In selective amnesia, the person can recall some but not all events from a particular time period.
C. Generalized amnesia is the lack of memory of the patient’s entire life.
Vital Concept:
Systematized amnesia is the loss of memory relating to a particular category of information.
While child maltreatment can occur to any child, there are certain risk factors make particular children more prone to maltreatment. Which of the following puts a child at elevated risk of maltreatment?
A. Easy to console children
B. Female gender
C. Physical disability
D. Being an only child
Correct Answer: C.
Physical disability
Prematurity, developmental disability, and physical disability are risk factors for abuse. Children with any of these factors are more likely to be abused. Other factors that make a child more likely to be abused include age younger than 3 with less than 1 being the highest group that is victimized and being male.
Incorrect Answers:
A. Children with difficult temperaments tend to be abused more frequently than their peers.
B. Male children have a higher risk of child maltreatment than female children. Female children are more likely to experience neglect.
D. Only children do not have an increased risk of child abuse.
Vital Concepts:
Age under 3, prematurity, and developmental disability or physical disability are risk factors for abuse.
A paraphilic disorder involves the observation of unsuspecting people either undressing or engaging in sexual activity. What is this disorder?
A. Fetishistic disorder
B. Voyeuristic disorder
C. Frotteuristic disorder
D. Pedophilic disorder
Correct Answer: B.
Voyeuristic disorder
Voyeuristic disorder involves feelings of sexual excitement related to viewing an individual while undressing, undressed, or having sex without their knowledge. This occurs over a period of 6 or more months, with associated daydreams and desires.
The patient has either followed through with their desires or describes them as leading to substantial anguish or dysfunction.
The patient must be at least 18 years old
Incorrect Answers:
A. Fetishistic disorder is sexual excitement related to inanimate objects.
C. Frotteuristic disorder involves sexual excitement related to rubbing against non-consenting people.
D. Pedophilic disorder involves sexual excitement related to prepubescent children.
Vital Concept:
Voyeuristic disorder involves feelings of sexual excitement related to viewing an individual while undressing, undressed, or having sex without their knowledge.
Lucy is a 26-year-old woman who is referred by her primary care physician. She admits to frequent binge eating followed by purging through self-induced vomiting several times a day for the past several months. In terms of her disorder, what does “Russell’s sign” indicate?
A. Halitosis and tooth decay due to stomach acids
B. Scars and abrasions on hands from using them to induce vomiting
C. Enlarged parotid glands and elevated amylase
D. Hypokalemia due to vomiting
Correct Answer: B.
Scars and abrasions on hands from using them to induce vomiting
Russell’s sign refers to the scars and abrasions found on the hands of anorexic and bulimic patients who induce vomiting.
Russel’s sign
Incorrect Answers:
A. Tooth decay and tooth sensitivity are common in patients who vomit frequently.
C. Parotid gland enlargement and elevated amylase levels are also common in patients who induce emesis.
D. Other sequelae of self-induced purging include hypokalemic alkalosis, elevated bicarb, hypochloremia, and dehydration. Electrolyte imbalances can cause cardiac arrhythmias and sudden cardiac death. Ipecac intoxication can cause cardiomyopathy and subsequent cardiac failure and death.
Vital Concept:
This type of scarring is considered one of the physical indicators of a mental illness, and Russell’s sign is primarily found in patients with an eating disorder such as bulimia nervosa or anorexia nervosa. However, it is not always a reliable indicator of an eating disorder; there are many more factors associated with it. Bulimics who are capable of “handsfree purging”, or the induction of vomiting by the willful opening of the esophageal sphincter in a manner similar to belching, while contracting the stomach muscles, do not have Russell’s sign.
A 19-year-old woman admits to eating large quantities of food in spurts. She then takes diuretics and exercises strenuously to try to “make up for the calories.” The patient does this a few times a week. Otherwise, she claims to eat a “normal, healthy diet” and is a normal weight. What is the first-line of treatment?
A. Nothing (patient is displaying a variant of normal eating and dieting behavior)
B. Fluoxetine
C. Bupropion
D. Paroxetine
Correct Answer: B.
Fluoxetine
Fluoxetine is the only medication approved by the FDA for the treatment of bulimia nervosa. In general, any SSRI should be considered first-line in treating bulimia. TCAs should be avoided due to their potential to increase cardiac toxicity. MAOIs should be avoided in these patients. Bupropion should be avoided in all eating disorder patients due to seizure risk.
Incorrect Answers:
A. The patient displays bulimia nervosa, and needs to be treated for it.
C. Bupropion shouldn’t be used in eating disorder patients, as there’s seizure risk.
D. Paroxetine, as an SSRI, is considered first-line treatment for treating bulimia, but isn’t FDA-approved as fluoxetine is.
How long, at minimum, should patients with eating disorders be observed after meals?
A. 15 minutes
B. 30 minutes
C. 1 hour
D. 2 hours
Correct Answer: D.
2 hours
Patients should be observed for a minimum of 2 hours after eating, even if this requires attendants to follow them to the bathroom. The primary purpose of the observation is to prevent purging of the meal.
Incorrect Answers:
A. B. and C. These durations are all too short.
A 19-year-old actress presents with amenorrhea. The patient is 70% of her expected weight for her age and height and refuses to eat out of fear of being “too fat to get a part in movies.” She is admitted to the inpatient psychiatric unit. On day 2 of admission, she asks for treatment for her severe constipation. What should be administered?
A. Lactulose
B. Senna
C. Magnesium hydroxide
D. Docusate
Correct Answer: D.
Docusate
Patients with anorexia nervosa who are hospitalized for a treatment program may complain of constipation. This symptom is usually relieved when they begin to eat normally. Never give these patients laxatives; stool softeners are permissible. Lactulose, senna, magnesium hydroxide, and bisacodyl all have laxative activity. Docusate is a stool softener and not a laxative. It may be given in this setting.
Incorrect Answers:
A. B. and C. Lactulose, senna, and magnesium hydroxide are all laxatives, which shouldn’t be given to patients with anorexia nervosa.
Up to 25% of bulimia patients have a particular personality disorder. What is this common personality disorder among bulimia patients?
A. Histrionic
B. Antisocial
C. Narcissistic
D. Borderline
Correct Answer: D.
Borderline
Approximately 25% of bulimia patients have borderline personality disorder. Just over 40% of bulimic patients also have suffered from an affective disorder at some point, and approximately 20% have a substance abuse history.
Incorrect Answers:
A, B, and C. These other personality disorders aren’t as common as borderline personality disorder in bulimia patients.
A 28-year-old female with a past medical history of epilepsy gives birth to a 2.5-kg baby girl. On physical exam, the baby is not in acute distress, acyanotic, and normocephalic. However, the baby has phalangeal hypoplasia bilaterally. Even though the mother did not seek prenatal care, she was taking folate supplements. Which of the following medications is most likely to have caused this teratogenicity?
A. Citalopram
B. Lithium
C. Lorazepam
D. Phenytoin
Correct Answer: D.
Phenytoin
Phenytoin is an anti-epileptic that is known for causing fetal hydantoin syndrome, which is characterized by short limbs and phalangeal hypoplasia. Phenytoin has also been known to cause neural tube defects. Folate is necessary to prevent such defects, and it is recommended for all pregnant women. Higher doses of folate are recommended for women taking anti-epileptics, such as phenytoin or valproate.
Therefore, in women whose risk of developing serious seizures outweighs the benefits of stopping the antiepileptic, it is necessary to ensure that higher doses of folate are administered.
Incorrect Answers:
A. Citalopram: it is another Selective Serotonin Reuptake Inhibitor (SSRI) used for the treatment of depressive disorders. Evidence using Citalopram during pregnancy has not been very clear, and a clinician should always weighs the risks and benefits of using an SSRI during pregnancy. However, phalangeal hypoplasia has not been associated with SSRIs.
B. Lithium: bipolar disorder is treated with Lithium. Lithium causes Ebstein anomaly, which is atrialization of the right ventricle by which the tricuspid valve moves towards the apex of the heart, thus enlarging the right atrium and shrinking the right ventricle. Ebstein anomaly is highly specific for Lithium use during pregnancy. This anomaly leads to poor pulmonary circulation and, consequently, poor blood oxygenation; poor oxygenation leads to tachypnea and tachycardia.
C. Lorazepam: Lorazepam is a benzodiazepine (BDZ). BDZs are a group of drugs that bind to the GABA receptors in the Central Nervous System, and induce relaxation and sedation. Lorazepam is routinely given to abort seizures and to avoid severe alcohol withdrawal symptoms in patients with chronic alcohol use disorder. BDZs are not associated with fetal hydantoin syndrome when taken during pregnancy. BDZs have not been shown to cause teratogenicity in most studies.
Infantile anorexia interferes with an important developmental event. Which developmental event does it interfere with?
A. Somatopsychological differentiation
B. Hunger vs. satiety differentiation
C. Self-feeding
D. Fine motor development of oropharynx and fingers
Correct Answer: A.
Somatopsychological differentiation
Infantile anorexia is characterized by food refusal by the infant, which can lead to malnutrition. The disorder usually occurs between 6 months and 3 years of age and can vary between meals and feeders. The parent may resort to bribery, distraction, or force-feeding. The infant’s feeding is directed by his or her emotional needs instead of physiological sensations of hunger and satiety, and he or she fails to develop somatopsychological differentiation, which is the ability to differentiate sensations in the body. The best way to address infantile anorexia and to restore normal growth is by helping the parents reduce stress and control issues around mealtimes. Infantile anorexia must be differentiated from posttraumatic feeding disorder (PTFD), which usually has a sudden onset after a traumatic event like choking or the insertion of a gastric tube.
Incorrect Answers:
B. C. and D. These developmental events aren’t affected by infantile anorexia.
Anorexia nervosa is associated with a number of medical complications. Of the following, which is one of the medical complications associated with anorexia nervosa?
A. Ankylosing spondylitis
B. Lanugo
C. Sjogren’s syndrome
D. Selective IgA deficiency
Correct Answer: B.
Lanugo
Medical complications associated with anorexia include (but are not limited to) bradycardia, pancytopenia, lanugo, osteopenia, metabolic encephalopathy, arrhythmias, elevated LFTs, elevated BUN, decreased T3 and T4, parotid gland enlargement, seizures, and peripheral neuropathy. Ankylosing spondylitis, selective IgA deficiency, Sjogren’s syndrome, and ulcerative colitis are autoimmune diseases.
Incorrect Answers:
A. C. and D. These are all autoimmune diseases, and as such aren’t associated with anorexia nervosa (which is an eating disorder).
Certain disorders are comorbid with anorexia nervosa. Of the following disorders, which is most commonly comorbid with anorexia nervosa?
A. Somatic disorders
B. Personality disorders
C. Major depressive disorder
D. Bipolar disorders
Correct Answer: C.
Major depressive disorder
Major depressive disorder is the most commonly comorbid condition in patients with anorexia nervosa. A nationally representative survey found 56% of anorexic patients had comorbid psychiatric diagnoses. Unipolar major depression was the most prevalent comorbid disorder (39%), followed by alcohol use disorders, specific phobias, and social anxiety disorder (each in approximately 25% of anorexic patients). In the clinical setting, unipolar depression is seen in up to 71% of anorexics, and obsessive compulsive disorder is also common (up to 35% of anorexic patients).
Incorrect Answers:
A. B. and D. These are less commonly comorbid with anorexia nervosa as compared to major depressive disorder.
A 17-year-old thin-appearing female with a history of major depressive disorder presents with increased fatigue. The patient states that while she normally runs and exercises after each meal until she has expended the number of calories she’s eaten, she has been feeling more tired during these workouts and is recently having difficulty completing them. She states that she felt similarly during prior depressive episodes and thinks sertraline is no longer controlling her symptoms. The patient also notes that she has begun to feel “ugly” due to the appearance of thin soft hairs on her back and arms. She is concerned because her stress has become so severe she has stopped menstruating recently. What part of the patient’s presentation is a relative contraindication to augmenting her depression treatment with bupropion?
A. Bupropion should never be used with sertraline; this combination can induce tyramine crisis.
B. Patient is at increased risk for seizures.
C. Patient is too young to be treated safely with bupropion.
D. If sertraline was ineffective, bupropion will also be ineffective.
Correct Answer: B.
Patient is at increased risk for seizures.
The patient is demonstrating signs concerning for anorexia, as indicated by excessive exercise and distorted self-perception. The DSM-5-TR includes extreme actions that prevent weight gain and distorted perception of their own body and their weight within the diagnostic criteria for anorexia. Patients with eating disorders are at increased risk for seizures, and bupropion further lowers a patient’s seizure threshold. Using bupropion in a patient with an eating disorder is relatively contraindicated.
Incorrect Answers:
A. MAOIs are associated with the risk of a tyramine crisis. Neither of these medications is an MAOI, and bupropion is a commonly used and safe augment to SSRIs.
C. Bupropion is not contraindicated for use in children, but like most other psychiatric medications, it may require closer monitoring and slower titration.
D. Patients often respond differently to different medications, even if they are in the same class (i.e., patients who do not improve sufficiently with one SSRI may find success with another SSRI). This is often true if switching classes. If a patient does not see improvement with sertraline, an SSRI, then trialing bupropion, an NDRI, is a reasonable next step.
Vital Concept:
This patient is displaying indicators of a possible eating disorder, which increases their risk of seizures. Bupropion is relatively contraindicated in this patient as it also lowers the seizure threshold.
Exhibitionism is the act of exposing parts of the body normally not exposed (e.g., genitalia areas) in public or semi-public settings. Which of the following statements about this behavior is correct?
A. It can be diagnosed with a single incident.
B. Onset is usually in the mid- to late-twenties.
C. Sexual urges or fantasies cause substantial or interpersonal problems.
D. When patients act on their urges, they generally seek further sexual activity with the stranger.
Correct Answer: C.
Sexual urges or fantasies cause substantial or interpersonal problems.
Exhibitionistic disorder is characterized by feelings of sexual excitement related to showing private parts of their body to an individual without their prior knowledge or consent. This occurs over a period of 6 or more months, with associated daydreams and desires.
The patient has either followed through with their desires or describes them as leading to substantial anguish or dysfunction.
Incorrect Answers:
A. Exhibitionism needs to occur over at least 6 months to be diagnosed.
B. Onset is usually before age 18.
D. When exhibitionists act on their urges, they generally don’t make attempts to have further sexual activity with the stranger(s) to whom they expose themselves.
Vital Concept:
In exhibitionistic disorder, the patient has either followed through with their desires or describes them as leading to substantial anguish or dysfunction.
A transgender male is sexually attracted to men. What is his sexual orientation?
A. Gynephilic
B. Androphilic
C. Analophilic
D. Asexual
Correct Answer: B.
Androphilic
The use of homosexual and heterosexual designators in transgender individuals is problematic. Historically, most medical professions classified orientation based on the individual’s genetic/assigned sex; the individual, however, defined it with regard to his or her perceived gender. Androphilia describes sexual attraction to men or masculinity; gynephilia describes the sexual attraction to women or femininity. Ambiphilia describes the combination of both androphilia and gynephilia.
Incorrect Answers:
A. Gynephilic means attracted to women.
C. Analophilic means asexual (attracted to neither men nor women).
D. Asexual is synonymous with analophilic.
A 35-year-old female presents to the outpatient clinic with depressed mood, irritability, and sleep disturbance. She recently gave birth to a child with microphthalmia, growth retardation, microcephaly, a thin upper lip, and broad-bridged nose. What is the mother’s most likely diagnosis?
A. Bipolar disorder
B. Major depression
C. Alcohol dependence (alcohol use disorder)
D. Illness anxiety disorder
Correct Answer: C.
Alcohol dependence (alcohol use disorder)
The patient is suffering from alcohol use disorder (formerly alcohol dependence), and her baby has congenital abnormalities due to fetal alcohol syndrome (FAS), which is a complication of prolonged alcohol use by the mother. No amount of alcohol can be considered safe during pregnancy. About 5% of mothers with chronic alcohol use disorder give birth to children with FAS. Intellectual disability (formerly mental retardation) is common, and 44% of children with fetal alcohol syndrome have an IQ of 79 or below. Other congenital defects include wide-set eyes, short palpebral fissure, a short and broad-bridged nose, hypoplastic philtrum, thinned upper lip, and flattened mid-face. Maternal alcohol use with breastfeeding has been shown to impair a child’s motor but not mental development.
Incorrect Answers:
A. Bipolar disorder is associated with mood swings involving an irritable or elated mood and dysphoria.
B. Major depression is an unlikely diagnosis in a female who has no previous episode of low mood or other biological and depressive symptoms associated with depression.
D. This female has alcohol dependence with complication development, making illness anxiety disorder unlikely.
An adolescent patient with a BMI of 16 is intensely fearful of gaining weight, limits themself to <1,000 calories per day, and views themself as fat. They sometimes force themselves to vomit after eating what others would consider a normal meal. The patient has normal menses. What is the most likely DSM-5-TR diagnosis?
A. Bulimia nervosa
B. Eating disorder NOS
C. Anorexia nervosa, binge-eating/purging type
D. Unspecified eating disorder
Correct Answer: C.
Anorexia nervosa, binge-eating/purging type
In the DSM-5-TR, the diagnosis of anorexia nervosa no longer requires irregular menses in postmenarcheal females. This patient should be diagnosed with anorexia nervosa, binge-eating/purging type in the DSM-5-TR. The criteria include
limitation of caloric intake resulting in a markedly decreased body mass index (BMI) or weight as compared to similar individuals of the same sex, age, or height.
extreme dread of an increase in weight or of being overweight, or actions that prevent weight gain, despite being severely underweight
distorted perception of their own body and their weight, an exaggerated impact of their weight on their self-regard, or a consistent disregard or lack of awareness of the consequences of their health condition or their behavior
Incorrect Answers:
A. Although the patient engages in purging behavior, the presence of anorexic behaviors eliminates a sole diagnosis of bulimia nervosa.
B. D. Unspecified eating disorder is not appropriate in this case, given that they meet the diagnostic criteria for anorexia
Vital Concept:
This patient appears to satisfy the criteria for anorexia nervosa.
A 17-year-old ballet student presents for treatment of amenorrhea for 1 year. On physical exam, her weight is 70% of that expected for her age and height. When this is mentioned, the patient replies that she looks fat in her ballet outfit and refuses to eat because she cannot afford to gain any more weight. Her CMP shows mild hypokalemia, and CBC is within normal limits. Blood pressure is 79/60, and heart rate is 54. What is the next step in management?
A. Discharge and draw blood for CMP and CBC in 1 week
B. Refer to nutritionist as outpatient
C. Suggest seeing psychiatrist for weekly psychotherapy
D. Admit to hospital
Correct Answer: D.
Admit to hospital
This patient suffers from anorexia nervosa and needs admission because her blood pressure is <80 systolic and her weight is 30% below expected. While a blood draw in a week, a referral to a nutritionist, and psychotherapy may at some point play a role in her care, this ill patient requires hospitalization. She is at considerable risk for developing cardiac arrhythmias, an EKG is warranted. Other criteria often used for admission are: <75% ideal body weight or ongoing weight loss despite intensive management, refusal to eat, body fat <10%, heart rate <50bpm daytime or <45bpm nighttime, systolic pressure <80, orthostatic changes in pulse (increased >35bpm) or blood pressure (decreased >10 mmHg), hypothermia and arrhythmia.
Incorrect Answers:
A. The patient can’t be discharged, as her blood pressure and weight are too low. While a blood draw in a week might be required later, she needs hospitalization now.
B. A referral to a nutritionist may be needed later, but hospitalization is needed now.
C. Psychotherapy may be needed later, but hospitalization is needed now.
A 32-year-old woman presents to the emergency department. She says she cannot remember how she got there or what her name is. On examination, she has intact new learning ability with a loss of remote memory, including autobiographical memory. What is the most likely diagnosis?
A. Wernicke’s encephalopathy
B. Korsakoff’s disease
C. Transient global amnesia
D. Dissociative amnesia
Correct Answer: D.
Dissociative amnesia
Dissociative (psychogenic) amnesia includes loss of autobiographical memory, sometimes with preserved ability for new learning.
Incorrect Answers:
A. Wernicke’s encephalopathy, which results from thiamine (vitamin B1) deficiency, occurs in malnourishment such as in chronic alcohol use disorder. It is defined by the triad of confusion, ataxia, and ophthalmoplegia.
B. Korsakoff’s disease, the chronic phase of thiamine deficiency, presents with anterograde and retrograde amnesia. It is classically associated with confabulation as a result of the poor memory.
C. Transient global amnesia leads to anterograde amnesia, the inability to encode new memories. The patient retains immediate recall as well as remote memory. The remainder of the neurological exam is normal.
A 27-year-old patient presents to clinic for an annual visit. They report that for the past 8 months they have had twice-weekly episodes where they binge eat then either take laxatives or induce vomiting because they feel guilty about the binge and are afraid they will gain weight. They do not otherwise restrict their food intake. The patient lives on their own and prepares most of their meals at home. Their vital signs and physical exam are unremarkable. Which of the following is the most appropriate treatment for this patient?
A. Family-based therapy
B. Cognitive behavioral therapy
C. Biofeedback therapy
D. Food journal to raise awareness of eating patterns
Correct Answer: B.
Cognitive behavioral therapy
This patient has bulimia nervosa, an eating disorder characterized by consuming an abnormally large amount of food followed by compensatory behaviors, which may include exercise, vomiting, diuretics, emetics, and laxatives. The Diagnostic and Statistical Manual, 5th Edition (DSM-V) set diagnostic criteria for bulimia nervosa as follows:
Recurrent episodes of binge eating
Recurrent compensatory behaviors
Binge eating and compensatory behavior each happen at least once per week for three months
Self-evaluation unduly influenced by body shape and weight
Does not happen exclusively during episodes of anorexia nervosa
Cognitive-behavioral therapy (CBT) is the first-line therapy for bulimia nervosa. CBT seeks to correct the inappropriate thoughts bulimic patients have regarding themselves and their disorder. CBT can reduce the number of binge-purge episodes and is also effective for patients with binge-eating disorders.
Incorrect Answers:
A. Family-based therapy is one of the treatment options of choice for children and adolescents with anorexia nervosa. It requires that the patient lives in a household where parents or family members can consistently enforce specific eating behaviors before eventually transitioning back to more autonomous eating by the patient. This patient does not have anorexia nervosa, and living alone would make this treatment option logistically very challenging.
C. Biofeedback therapy can effectively reduce the symptom burden of panic disorder but does not have a role in treating bulimia nervosa.
D. Food journals are not a component of treatment for any eating disorder. Instead, they may inappropriately cause fixation on food intake and encourage disordered eating behaviors.
Vital Concept:
Cognitive-behavioral therapy (CBT) is the first-line therapy for the treatment of bulimia nervosa and aims to change the way the patient thinks about themselves and their disorder.
A 50-year-old female with no significant past medical history presents to the clinic for a regular check-up in June. A review of her medical record reveals that she never received the HPV vaccine. Her last Tdap was at age 25. She is up-to-date on all her other vaccines. Her vitals are a temperature of 98.9 F, respiratory rate 18, pulse 73, and her blood pressure 118/78 mmHg. Her physical exam is unremarkable.
Which of the following vaccines is indicated at this visit?
A. Pneumococcal
B. Td booster
C. HPV vaccine
D. Intranasal Influenza vaccine
Correct Answer: B.
Td booster
A tetanus booster shot is recommended every ten years for everyone. The current recommendation is to give Tetanus, diphtheria, and acellular pertussis (Tdap) once for patients 11 years or older who have not received Tdap or whose immunization status is unknown, and then to use the Tetanus diphtheria (Td) or Tdap vaccine every 10 years as a booster. This patient is due for her booster tetanus shot.
Incorrect Answers:
A. Pneumococcal vaccine is indicated for individuals 65 years or older or those ages 19-64 years with chronic medical conditions.
C. Human Papilloma Virus (HPV) vaccine is indicated for patients ages 9-26, and has been approved up to age 45 years. There are two types of the vaccine (HPV 4 or HPV 2, containing 4 and 2 strains of the virus, respectively). A complete series consists of 3 doses of either HPV 2 or HPV 4 vaccines, with the 2nd dose administered at 4-8 weeks after the 1st one, and the third dose at 24 weeks after the 1st one (16 weeks after the 2nd ). This patient is older than the recommended age. Therefore, the HPV vaccine is not indicated at this time.
D. The Center for Disease Control recommends annual influenza vaccine for all patients aged 6 months or older. Most patients receive the inactivated, intramuscular influenza vaccine. The intranasal, live attenuated influenza vaccine is not as commonly administered as the inactivated one. Most patients receive the flu vaccine in the fall season. The patient in this vignette is visiting in June and she is up-to-date on all her vaccines (except HPV and Td). The flu vaccine is not indicated at this visit.
Vital Concepts:
The current recommendation is to give Tetanus, diphtheria, and acellular pertussis (Tdap) once for patients 11 years or older who have not received Tdap or whose immunization status is unknown, and then to use the Tetanus diphtheria (Td) or Tdap vaccine every 10 years as a booster.
Per the DSM-5-TR, what is the minimum age in years a person needs to be to receive a diagnosis of pedophilic disorder?
A. 5
B. 13
C. 16
D. 18
Correct Answer: C.
16
Pedophilic disorder criteria:
Feelings of sexual excitement involving prepubescent children (typically under the age of 14). This occurs over a period of 6 or more months, with associated daydreams and desires.
The patient has either followed through with their desires or describes them as leading to substantial anguish or dysfunction.
The patient must be at least 16 years old, with at least a 5-year age gap
May specify if the intended individual is a male, female, or both, and whether the patient is exclusively attracted to children or also attracted to adults
Incorrect Answers:
A. The person must be 16 years or older and at least 5 years older than the child sexually interested in. This question asked how old the person has to be for a diagnosis of pedophilic disorder, not how much older than the child the person has to be.
B. The child/children of interest are typically prepubescent child/children (13 years or younger).
D. An individual must be 16 years or older to be diagnosed with pedophilic disorder, not 18 years old.
Vital Concept:
Must be 16 years or older and at least 5 years older than the child sexually interested in to be diagnosed with pedophilic disorder.
A 47-year-old woman is brought to the ED by her husband after a change in mental status. The husband explains that his wife did not know their names or home addresses but was still able to learn new information and remember select past memories. She recently discovered he was having an affair with their babysitter. Physical exam, laboratory results, and a CT scan of her head without contrast are normal. What is the most likely diagnosis?
A. Dissociative episode
B. Transient ischemic attack
C. Transient global amnesia
D. Cardiovascular accident
Correct Answer: A.
Dissociative episode
Amnestic disorders are secondary syndromes caused by systemic medical or primary cerebral diseases, substance use disorders, or medication adverse effects, as evidenced by findings from clinical history, physical exam, or laboratory data. In contrast to transient global amnesia, dissociative amnesia typically does not involve deficits in learning and recalling new information. Instead, patients present with a circumscribed inability to recall previously learned information but continue to function normally in the present. The dissociative disorders can sometimes be difficult to differentiate from the amnestic disorders. Patients with dissociative disorders are more likely to have lost their orientation to self and may have more selective memory deficits than patients with amnestic disorders. For example, patients with dissociative disorders may not know their names or home addresses but are able to learn new information and remember selected past memories. Dissociative disorders are also often associated with emotionally stressful life events involving money, the legal system, or troubled relationships. Episodes of psychogenic amnesia end abruptly and are typically associated with an awareness of having no memories for the time period of the amnestic or fugue state.
Incorrect Answers:
B. Transient ischemic attack (TIA). A TIA is like a stroke but only last for a few minutes and does not cause lasting damage. These few minutes consist of physical symptoms, such as weakness, numbness, slurred speech, or change in vision.
C. Transient global amnesia. Transient global amnesia typically involves deficits in learning and recalling new information.
D. Cardiovascular accident. A cardiovascular accident (CVA), a stroke, presents with physical symptoms in addition to confusion, such as weakness, numbness, slurred speech, or change in vision.
Numerous medications have sexual side effects. Which of the following medications’ sexual side effects is correctly described?
A. Anti-hypertensives - impaired vaginal lubrication due to antiadrenergic effects
B. Antipsychotics - impaired arousal and orgasm due to adrenergic effects
C. Antidepressants - inhibited orgasm through antiadrenergic effects
D. Spironolactone - decreased sexual desire through serotonergic effects
Correct Answer: A.
Anti-hypertensives - impaired vaginal lubrication due to antiadrenergic effects
Anti-hypertensives cause erectile dysfunction and impaired vaginal lubrication secondary to antiadrenergic effects.
Incorrect Answers:
B. Antipsychotics cause erectile dysfunction and impaired vaginal lubrication secondary to anticholinergic as well an alpha 2 receptor effects. In addition, there is impaired arousal and orgasm through dopamine-blocking.
C. TCA and MAOI antidepressants can cause erectile dysfunction and impaired vaginal lubrication secondary to anticholinergic effects. SSRIs can cause inhibition of arousal and orgasm by increased serotonergic activity.
D. Spironolactone, estrogen, and steroids cause decreased sexual desire secondary to decreased testosterone.
The NP is seeing a patient for depression who recently had a thyroidectomy for a large, benign goiter. The patient states she has been experiencing “numbness and tingling” of fingers, toes and circumoral region. Which of the laboratory values would support the NP’s suspicions that the patient’s parathyroid was removed during the thyroidectomy?
A. Creatinine 0.7mg/dl
B. Calcium 7.9 mg/dl
C. Chloride 100 mEq/L
D. Magnesium 2.0 mEq/L
Correct Answer: B.
Calcium 7.9 mg/dl
Calcium normal range is 8.6-10.2 mg/dl. Several factors can cause hypocalcemia, including primary hypoparathyroidism and surgical hypoparathyroidism. Hypocalcemia is associated with thyroid and parathyroid surgery. Numbness and tingling of finger, toes and circumoral region are signs of hypocalcemia. Additional signs of hypocalcemia include: confusion, muscle spasms/cramps, weak nails, hallucinations, and depression. Hypocalcemia can be verified by a positive Chvostek’s sign or Trousseau’s sign in addition to obtaining a serum calcium level. In the case of this patient, the parathyroid was removed along with the thyroid.
Incorrect Answers:
A. Creatinine 0.7mg/dl. Creatinine normal values are 0.5-1.5 mg/dl and monitors kidney function and is not associated with the parathyroid.
C. Chloride 100 mEq/L. Chloride normal values are 96-106 mEq/L. This is a normal value, however, hypochloremia is most often associated with severe vomiting and diarrhea, gastric surgery and GI tube drainage/gastric suctioning.
D. Magnesium 2.0 mEq/L. Magnesium normal values are 1.5-2.5 mEq/L. While this is a normal magnesium level, hypomagnesemia is most often associated with nasogastric suction, diarrhea or fistulas. Chronic alcohol abuse is also a major cause of hypomagnesemia. Clinical symptoms present as ataxia, dizziness, depressed mood, insomnia, increased deep tendon reflexes and confusion. Hypermagnesemia is most associated with renal failure and clinical manifestation includes flushing, hypotension, muscle weakness, hypoactive reflexes, depressed respirations and diaphoresis.
Vital Concepts:
The calcium normal range is 8.6-10.2 mg/dl. Several factors can cause hypocalcemia, including primary hypoparathyroidism and surgical hypoparathyroidism Signs of hypocalcemia include: numbness and tingling of fingers or toes, confusion, muscle spasms/cramps, weak nails, hallucinations, and depression
Which of the following testosterone levels would be indicative of Tanner Stage V in a male patient?
A. 2-23 ng/dl
B. 5-70 ng/dl
C. 15-280 ng/dl
D. 265-800 ng/dl
Correct Answer: D.
265-800 ng/dl
265-800 ng/dl is the level of testosterone for males in Tanner Stage V (i.e., adult genitalia). Conceptually, pubertal maturation can be described in terms of sequence, timing, and tempo. Puberty consists of a series of predictable events, and the sequence of changes in secondary sexual characteristics has been categorized by several groups. The staging system utilized most frequently is that published by Marshall and Tanner and the sequence of changes, commonly referred to as “Tanner stages”. A testosterone level above 300ng/dL is considered normal for an adult male
Incorrect Answers:
A. 2-23 ng/dl is the testosterone level for Tanner Stage I (i.e., prepubertal).
B. 5-70 ng/dl is the testosterone level for Tanner Stage II (i.e., enlargement of scrotum and testes; scrotum skin reddens and changes in texture). Early puberty levels may range from 30-100 ng/dL.
C. 15-280 ng/dl is the testosterone level for Tanner Stage III (i.e., enlargement of penis [length at first] and further growth of testes). Testosterone levels in mid to late puberty may range from 100-300 ng/dL.
Vital Concept:
The testosterone levels can be used to assess a patient’s expected (and corresponding) stage of physical pubertal development.
Some treatments can harm a developing fetus’ cardiac formation. Which of the following treatments can cause cardiac malformations in the developing fetus if used in a pregnant patient?
A. Hospitalization
B. Antipsychotic agents
C. Electroconvulsive therapy
D. Lithium
Correct Answer: D.
Lithium
The approximate incidence of major congenital anomalies in children of pregnant women treated with lithium in the first trimester is 4-12%. Ebstein’s anomaly is considered the most common fetal cardiac defect caused by lithium, occurring in approximately 1/1000, compared to the general population incidence of 1/20,000. Ebstein anomaly is when the tricuspid valve and right ventricle develop abnormally. Lithium during the second and third trimesters can result in neonatal lithium toxicity. Lithium remains a category D drug during pregnancy.
Incorrect Answers:
A. Hospitalization. Severely depressed, actively psychotic, or suicidal patients need to be hospitalized. However, this alone doesn’t cause fetal cardiac malformations.
B. Antipsychotic agents. Most studies have shown that second-generation antipsychotics do not increase the risk of major physical malformations above general population rates. Quetiapine, olanzapine, and risperidone are among the antipsychotics that have the most safety data in pregnancy, although they all remain category C.
C. Electroconvulsive therapy. Electroconvulsive therapy can be utilized for severe depression and mania in pregnant women. Two reviews of ECT during pregnancy note the efficacy and safety of this procedure. ECT may also be considered as an alternative to pharmacotherapy for women who seek to avoid exposure to psychotropic medications during pregnancy or for women who do not respond to standard treatments.
According to a recent study, the volumes of which brain structures are significantly smaller in patients with dissociative identity disorder versus healthy subjects?
A. Pons and cortex
B. Amygdala and corpus callosum
C. Frontal cortex
D. Hippocampus
Correct Answer: D.
Hippocampus
On MRI, the hippocampus of dissociative identity disorder patients was smaller than healthy volunteers.
Incorrect Answers:
A. Pons and cortex is incorrect as the recent research shows the hippocampus of dissociative identity disorder patients was smaller than healthy volunteers.
B. Amygdala and corpus callosum is incorrect as the recent research shows the hippocampus of dissociative identity disorder patients was smaller than healthy volunteers.
C. Frontal cortex is incorrect as the recent research shows the hippocampus of dissociative identity disorder patients was smaller than healthy volunteers.
A certain atypical antipsychotic is absorbed much more quickly if administered within an hour of a meal, as compared to while fasting. Which of the following medications is this?
A. Risperidone
B. Quetiapine
C. Ziprasidone
D. Aripiprazole
Correct Answer: C.
Ziprasidone
Absorption of Ziprasidone after oral administration is significantly more rapid when administered within 1 hour of a meal compared to fasting. A fatty meal increased total absorption compared to fasting by 68% and decreased the serum half-life from 6.6 hours to 4.7 hours.
Incorrect Answers:
A, B, D. None of these is affected by food in the stomach.
Which of the following is a three question screening tool used for diagnosis of alcohol abuse and dependence with a patient who is acutely ill or otherwise poorly disposed to speak with the practitioner?
A. AUDIT
B. AUDIT-C
C. CAGE
D. MAST
Correct Answer: B.
AUDIT-C
AUDIT-C is a three-item version of AUDIT that can be used in an effort to allow cooperation with a client who is acutely ill or otherwise poorly disposed to speaking with a provider. AUDIT-C has nearly identical sensitivity to the full AUDIT.
Incorrect Answers:
A. AUDIT, Alcohol Use Disorders Identification Test, consists of 10 multiple-choice questions regarding the quantity and frequency of a patient’s alcohol consumption, drinking behavior, and alcohol-related problems. AUDIT is the gold standard for screening.
C. CAGE is four-item screening tool with a 50-75% sensitivity and specificity of about 80%. The questions begin with “Have you ever….” C-thought you should cut back, A –felt annoyed by others criticizing your drinking, G-felt guilty or bad about your drinking, E-had a morning eye-opener.
D. MAST Michigan Alcohol Screening Test is a 25 question screening tool that has simple yes and no answers.
To minimize the risk of agranulocytosis, a patient’s baseline ANC needs to exceed a certain level to initiate treatment with clozapine. What is this minimum baseline ANC level?
A. 1,000
B. 1,500
C. 2,000
D. 2,500
Correct Answer: B.
1,500
ANC must be >1,500 to initiate treatment with clozapine. Patients usually recover from clozapine-induced agranulocytosis within 14-24 days of stopping the drug, but rechallenge with clozapine is not recommended if the ANC drops below 1,000 because of the very high rate of recurrence, unless the physician determines that the benefit outweighs the risk. An exception is made for patients with benign ethnic neutropenia.
Incorrect Answers:
A. This level is too low; the baseline ANC is 1,500 to initiate treatment with clozapine.
C, D. These levels are too high; the baseline ANC is 1,500 to initiate treatment with clozapine.
References:
Medicare and Medicaid have instituted a program or concept to improve quality, safety, and efficiency in healthcare through the use of certified electronic records. What is the name of this concept or program?
A. HITECH (Health Information Technology for Economic and Clinical Health)
B. HIPAA (Health Insurance Portability and Accountability Act)
C. Meaningful Use
D. ACA (Affordable Care Act)
Correct Answer: C.
Meaningful Use
The Medicare and Medicaid EHR Incentive Programs provide financial incentives for the “meaningful use” of certified EHR technology. This program includes incentives for participation, demonstrated by meeting certain requirements over three stages.
Incorrect Answers:
A. HITECH refers to the Health Information Technology for Economic and Clinical Health Act. This act was enacted under the American Recovery and Reinvestment Act of 2009 to promote and expand the adoption of health information technology.
B. HIPAA refers to the Health Insurance Portability and Accountability Act, enacted in 1996. This act ensures equal access to certain health and human services and protects the privacy and security of health information.
D. ACA refers to the Affordable Care Act, designed to provide increased access to health care for all Americans.
Vital Concepts:
The Medicare and Medicaid EHR Incentive Programs provide financial incentives for the “meaningful use” of certified EHR technology. This program includes incentives for participation, demonstrated by meeting certain requirements over three stages.
References:
What is the most widely used brief psychotherapy?
A. Eclectic
B. Cognitive-behavioral
C. Psychodynamic
D. Interpersonal
Correct Answer: B.
Cognitive-behavioral
Cognitive-behavioral therapy is the most widely used brief psychotherapy.
Incorrect Answers:
A. Eclectic brief psychotherapy is the second most popular psychotherapy. It’s made up of an integration of multiple techniques. The most popular version, proposed by Budman and Gurman, emphasizes the three dimensions of mental life: interpersonal, developmental, and existential.
C. Psychodynamic therapy is the third most popular psychotherapy. It focuses on unconscious conflicts, repressed feelings, and early relationships.
D. Interpersonal is the fourth most popular psychotherapy. It’s primarily used to treat depression and focus on current relationships. Its four core issues are grief, role transition, role dispute, and interpersonal deficits.
References:
A patient has become tolerant of a medication. In this case, what change can be expected in the dose-response curve?
A. Depends on whether tolerance is due to pharmacokinetic or pharmacodynamic effect
B. Linear curve shift
C. Shift to left
D. Shift to right
Correct Answer: D.
Shift to right
When tolerance develops, higher doses are required to achieve the same effect. In a graph with the dose on the X axis and the response on the Y axis, tolerance corresponds with a shift to the right. While tolerance can develop due to pharmacokinetic effects, pharmacodynamics effects, or both, the shift in the curve is to the right regardless.
Incorrect Answers:
A. The rightward shift occurs regardless of the cause of the tolerance
B. The shape of the curve doesn’t change — only its position on the X axis
C. A shift to the left would indicate decreased tolerance, which isn’t correct
References:
A patient with a history of comorbid opioid use disorder and alcohol use disorder is committed to maintaining abstinence from both substances. Which of the following is an FDA-approved medication for alcohol use disorder and opioid use disorder that would be appropriate for this patient?
A. Naltrexone
B. Acamprosate
C. Gabapentin
D. Naloxone
Correct Answer: A.
Naltrexone
FDA-approved medications for alcohol use disorder include naltrexone, acamprosate, and disulfiram. Naltrexone is approved for treatment of opioid use disorder and can be considered in patients with comorbid alcohol use disorder. However, patients must be detoxified from opioids and committed to abstaining, as use of opioids while taking naltrexone may precipitate opioid withdrawal.
Incorrect Answers:
B. Acamprosate can be given for alcohol use disorder but does not have additional benefits for opioid use.
C. Gabapentin can be used for pain and treatment of withdrawal symptoms in opiate withdrawal. It is also an off-label treatment for ETOH use disorder.
D. Naloxone does not have an FDA-approved indication for the treatment of alcohol use disorder.
References:
The NP is caring for a patient dying of lymphoma. The patient’s wife is struggling with this diagnosis and asks the NP, what if we had seen you sooner, would my husband have survived? The NP suspects that this individual is in which stage of grief according to Elizabeth Kubler-Ross?
A. Acceptance
B. Bargaining
C. Denial
D. Depression
Correct Answer: B.
Bargaining
According to Kubler-Ross, bargaining involves thinking in “what-if” statements, such as “what if we had seen the doctor sooner.”
Incorrect Answers:
A. In the acceptance stage of grief, the individual is resolved to their condition and accepts that nothing they can do will change the outcome.
C. Denial involves resisting or refusing to remember information that has been supplied.
D. Depression involves being tearful or withdrawn from the situation.
Vital Concepts:
According to Kubler-Ross, bargaining involves thinking in “if-then” terms. If we change practitioners, then my loved one will not die.
References:
A 45-year-old man presents to the emergency room for a “panic attack.” He appears agitated, confused, and dizzy. His speech is slurred. The patient becomes more agitated and combative with staff upon further questioning. He states that he was prescribed Xanax at 0.5mg 3x daily by an outpatient psychiatrist. Because it is after hours, a message was left with his psychiatrist’s answering service. The patient’s vital signs are within normal limits. What is the most likely diagnosis?
A. Panic attack
B. Benzodiazepine withdrawal
C. Benzodiazepine intoxication
D. Delirium
Correct Answer: C.
Benzodiazepine intoxication
When benzodiazepine dependence arises during therapeutic treatment, a predictable clinical course often occurs, especially during long-term treatment of a generalized anxiety disorder, panic disorder, or severe insomnia. When treatment is started, patients often have initial side effects such as drowsiness, psychomotor impairment, or memory impairment. Benzodiazepine intoxication ranges from mild to severe. Mild symptoms include agitation, confusion, drowsiness, blurred vision, and dizziness. Severe symptoms include respiratory depression, unresponsiveness, and coma. New symptoms, particularly alterations in sensory perception, suggest the beginning of a withdrawal syndrome.
Incorrect Answers:
A. Panic attack. Physical symptoms of a panic attack can include chest pain, palpitations, difficulty breathing, dizziness, sweating, and paresthesias. The key symptom of panic is hyperventilation. Patient feels like they are “going crazy” or having a “heart attack,” but the only symptom present is sinus tachycardia.
B. Benzodiazepine withdrawal. Signs of withdrawal include anxiety, tremors, nightmares, insomnia, anorexia, nausea, vomiting, postural hypotension, seizures, delirium, and hyperpyrexia. Abrupt discontinuation of sedative-hypnotics in patients who are severely physically dependent on them can result in serious medical complications and even death.
D. Delirium. Withdrawal delirium may include confusion and visual and auditory hallucinations. Delirium generally follows a period of insomnia. Some patients may have only delirium, others may have only seizures, and others may have both.
References:
Health care reimbursement differs between groups. Which of the following describes a group with fixed reimbursement amounts that are adjusted based on case severity, regional costs, and teaching costs?
A. ICD-10
B. Diagnosis-related groups
C. Modifier
D. Coding
Correct Answer: B.
Diagnosis-related groups
Diagnosis-related groups (DRGs) are groups related to a diagnosis with a fixed reimbursement amount. Adjustments are based upon the severity of the case, teaching costs, and area of service (rural/urban/regional). Hospitals receive a set dollar amount for each client based upon the DRG, regardless of the client’s length of stay or use of services.
Incorrect Answers:
A. This is used for coding and classifying mortality data from death certificates.
C. This is a code that provides the means by which the reporting physician can indicate that a service or procedure that has been performed has been altered by some specific circumstance but has not changed in its definition or code.
D. This is the process of using universal medical alphanumeric codes to document healthcare diagnosis, procedures, medical services, and equipment.
Vital Concepts:
Diagnosis-related groups (DRGs) are groups related to a diagnosis with a fixed reimbursement amount.
References:
The NP is arranging proper placement for a patient. Which of the following is accurate about a Skilled Nursing Facility (SNF)?
A. Medicare typically covers the cost of SNF care.
B. One can obtain surgical procedures at a SNF.
C. Custodial care alone is enough to qualify for placement in a SNF.
D. Psychiatric therapy is a hallmark of SNF care.
Correct Answer: A.
Medicare typically covers the cost of SNF care.
A skilled nursing facility can submit bills to Medicare for skilled nursing and physician/medical care. However, CMS does limit the number of days it will cover.
Incorrect Answers:
B. Skilled nursing care is the primary reason that a patient is admitted to a SNF rather than a nursing home or long term care facility. Surgery is generally not performed at a SNF.
C. Custodial care is typically done in a nursing home. Without the need for more advanced care, a patient will usually not be sent to a SNF.
D. A SNF will provide appropriate physical and occupational therapy but most do not offer psychiatric therapy services.
Vital Concepts:
A skilled nursing facility can submit bills to Medicare for skilled nursing and physician/medical care. However, CMS does limit the number of days it will cover.
References:
Drugs tend to be absorbed more slowly when a patient has a full stomach. Which medication should be taken on an empty stomach?
A. Sedatives/hypnotics
B. SSRIs
C. SNRIs
D. Carbamazepine
Correct Answer: A.
Sedatives/hypnotics
Sedative-hypnotics are used for rapid/acute treatment. The faster they are absorbed, the more effectively and quickly the drugs can begin to work. SSRIs and SNRIs can be affected by food in the stomach. However, these drugs are generally beneficial over a long treatment period, and their efficacy is more dependent on the built-up concentration of the compounds in the system, which is unlikely to be affected by occasionally taking the drug with food.
Incorrect Answers:
B, C. While SSRIs and SNRIs can be affected by food in the stomach, their efficacy is more dependent on the built-up concentration of the compounds in the system; so occasionally taking the drug with food is unlikely to affect the effects
D. This should only be taken with meals
References:
A number of medications have been studied for the treatment of drug-induced psychosis in patients with Parkinson’s disease. Which of the following medications has the most evidence of its effectiveness for treating drug-induced psychosis in patients with Parkinson’s disease?
A. Risperidone
B. Clozapine
C. Olanzapine
D. Quetiapine
Correct Answer: B.
Clozapine
Clozapine is the most studied treatment for drug-induced psychosis in patients with Parkinson’s disease; olanzapine is a close second.
Incorrect Answers:
A. C. Risperidone has the greatest number of parkinsonian side effects, followed by olanzapine and finally clozapine. Use of clozapine is limited due to significant hematological, cognitive, metabolic, cardiac, and neurological side effects. There is little clinical data supporting the use of ziprasidone or aripiprazole in dementia.
D. Quetiapine is often used anecdotally, but there is a lack of evidence for its use at this time.
References:
A 10-year-old Caucasian boy is being treated for ADHD with atomoxetine. However, even at very low doses, he complains of side effects including dry mouth, loss of appetite, and insomnia. The child’s mother gives his medication every morning and confirms proper compliance with the prescribing instructions. He does not take any other prescribed or OTC medications and has no significant past medical history. The child has a history of sensitivity to the side effects of some medications but not all. Which of the following measures might help the practitioner understand why the child is experiencing side effects from this medication?
A. Genotyping
B. Urine toxicology
C. Creatinine clearance
D. Liver function tests
Correct Answer: A.
Genotyping
The child is likely a poor metabolizer. Many psychotropic medications are metabolized by CYP2D6; over 70 different mutated alleles described in research. Nonfunctional or reduced function alleles occur in approximately 25-30% of Caucasians, resulting in about 5-10% of the population having the poor metabolizer genotype. Under 2% of Asians have the poor metabolizer genotype, and the frequency among African Americans is somewhere between. Poor metabolizers at CYP2D6 taking a substrate of CYP2D6 can be at significantly increased risk of adverse events and are much more likely to discontinue treatment because of the side-effect burden. Atomoxetine is a substrate of CYP2D6; poor metabolizers can develop plasma concentrations of atomoxetine 10x higher than unaffected individuals. Genotyping might help establish the presence of a nonfunctioning allele and confirm the poor metabolizer phenotype.
Incorrect Answers:
B. Urine toxicology may establish illicit drug use but would not explain current or previous instances of sensitivity to particular medications.
C. D. Renal and hepatic impairment can affect drug metabolism but are less likely to be the culprit in a healthy 10-year-old.
References:
Individuals with panic disorder are most commonly affected by which other psychiatric disorder?
A. Posttraumatic stress disorder
B. Impulse control disorders
C. Agoraphobia
D. Psychotic disorders
Correct Answer: C.
Agoraphobia
Panic disorder is most commonly associated with anxiety disorders such as agoraphobia. In addition to other anxiety disorders, panic disorder has a higher prevalence in individuals with major depression, bipolar disorder, and possibly mild alcohol use disorder.
Incorrect Answers:
A. Although patients with posttraumatic stress disorder can also have panic disorder, it is more commonly found with agoraphobia.
B. Impulse control disorders include oppositional defiant disorder, conduct disorder, intermittent explosive disorder, kleptomania, and pyromania. They can occur in patients with other psychiatric disorders and/or substance use disorders, but are not commonly found with panic disorder.
D. Psychotic disorders are not comorbid with panic disorder.
Vital Concept:
Panic disorder is most commonly associated with mood and anxiety disorders.
References:
Nurse practitioners need to meet certain requirements. Which of the following is a legal requirement to practice as a nurse practitioner?
A. Certification
B. Licensure
C. Collaborative agreement
D. National Provider Identification number
Correct Answer: B.
Licensure
Licensure is a legal requirement to practice as a nurse practitioner.
Incorrect Answers:
A. Certification is a voluntary process granted by a nongovernmental association or organization, and certification is now mandated in most states in the United States as a condition of licensure.
C. A collaborative agreement refers to a written agreement between a supervising physician and nurse practitioner that outlines the nurse practitioner’s role and responsibility in clinical practice. Many states have granted NPs full practice authority and no longer require a collaborative practice agreement.
D. A National Provider Identification Number (NPI) is required for billing.
Vital Concepts:
Licensure is granted by a governmental agency, the state board of nursing. To become licensed, a nurse must meet minimal educational and clinical requirements.
References:
A former patient who you treated for a year for depression has moved away. While back in town visiting family, they call to ask you on a date. They are now divorced and would love to date someone who “understands them so well.” They are attractive and engaging and have not been your patient for 2 years. What is the appropriate course of action?
A. Since they are no longer your patient, it is acceptable to go on a date.
B. If the patient signs a waiver releasing you from liability, you can consider their request.
C. Inform the patient that it is inappropriate to suggest a relationship with their healthcare provider.
D. Politely decline their offer because it is unethical.
Correct Answer: D.
Politely decline their offer because it is unethical.
It is unethical for a healthcare provider to enter into a sexual relationship with a former patient, even when the patient initiates the contact, particularly if the healthcare provider uses or exploits emotions, trust, influence, or knowledge derived from the past provider-patient relationship. The extent of the previous provider-patient relationship, degree of emotional dependence on the healthcare provider, the extent to which the patient has confided private or personal information to the healthcare provider, and extent of the provider’s general knowledge about the patient’s life may contribute to the intimacy of the provider-patient relationship and render a romantic or sexual relationship with a former patient unethical. However, the healthcare provider should not reprimand the patient but should simply explain the situation.
Incorrect Answers:
A. This is not true, it’s still unethical.
B. There’s nothing a patient can sign to release you from your ethical obligations.
C. You shouldn’t reprimand the patient.
Vital Concept:
It is unethical for a healthcare provider to enter into a sexual relationship with a former patient, even when the patient initiates the contact, particularly if the healthcare provider uses or exploits emotions, trust, influence, or knowledge derived from the past provider-patient relationship.
References:
A patient who has recurrent, multiple physical symptoms and pain over 6 months that are not fully explained by physical factors and that result in medical attention would most likely be diagnosed with which disorder?
A. Functional neurological symptom disorder
B. Factitious disorder
C. Illness anxiety disorder
D. Somatic symptom disorder
Correct Answer: D.
Somatic symptom disorder
SSD diagnosis does not require that somatic symptoms are medically unexplained. In other words, symptoms may or may not be associated with another medical condition or diagnosis. To meet the criteria for SSD, patients must have:
At least one somatic symptom that substantially impacts the patient’s daily functioning or leads to anguish
The patient then develops extraordinary actions, emotions, or ruminations regarding the symptom(s) or related medical condition(s), as evidenced by one or more of the following:
· consistent worry or concern about their physical wellbeing
· extraordinary devotion of resources (e.g., energy, time) to the symptom(s) or related medical condition(s)
· ruminations that are consistent and unbalanced regarding the gravity and meaning underlying these symptoms
These conditions persist for at least 6 months, although the specific symptom of focus may vary or shift during that time
Specifiers include:
· with prominent pain if the symptoms involve physical discomfort
· persistent if the symptoms are severe, enduring (greater than 6 months), and cause substantial dysfunction
· mild/moderate/severe based on the number of symptoms listed above that are present (1/2/2+ multiple symptoms)
Incorrect Answers:
A. Functional neurological symptom disorder (FNSD) requires the presence of one or more voluntary motor or sensory symptoms that are incompatible with known neurological or medical conditions.
B. There’s no evidence that the patient is faking their condition to assume the role of the patient, which is the underlying motivation in factitious disorder.
C. Illness anxiety disorder is characterized by a preoccupation with having or acquiring an illness and the presence of a high level of anxiety regarding one’s health despite no somatic symptoms present (or only mild in intensity).
Vital Concept:
Somatic symptom disorder (SSD) is a disease with somatic symptoms that must be significantly distressing or disruptive to daily life and must be accompanied by excessive thoughts, feelings, or behaviors.
References:
A 70-year-old female with depression for the past 3 months presents to urgent care with complaints of dizziness and lightheadedness. She also has diabetes, atrial fibrillation, Parkinson’s disease, congenital prolonged QTc, and bilateral knee replacements. The patient has had the same medication regimen “for years,” including levodopa, metformin, and clopidogrel. She was also started on an antidepressant recently but cannot remember the name. Physical exam reveals no concerning signs. However, her lab work reveals a serum sodium of 127. Which medication is the patient most likely using to treat her depression?
A. Sertraline
B. Selegiline
C. Buprenorphine
D. Zolpidem
Correct Answer: A.
Sertraline
SSRIs like sertraline have been associated with hyponatremia and SIADH, particularly in elderly females.
Incorrect Answers:
B. MAOIs like selegiline are usually not associated with hyponatremia, but may cause dizziness in some patients. MAOIs are not used as first-line treatment for depression, especially with this patient’s multiple medical comorbidities. Although MAOIs can be used in treatment refractory depression, this patient has only been experiencing depression for the past 3 months, which is not enough time for adequate trials of first-line treatment options.
C. Buprenorphine is used in the treatment of opioid use disorder and is not used to treat depression.
D. Zolpidem is a sleep aid; it is not used in the treatment of depression.
References:
In which of the following ways does necrosis differ from apoptosis?
A. Phagocytosis by macrophages
B. Inflammatory response
C. Membrane blebbing
D. Degradation of DNA
Correct Answer: B.
Inflammatory response
Apoptosis is genetically programmed cell death. It is an active process that requires RNA and protein synthesis and is characterized by cytoplasmic shrinkage, chromatin condensation, and degradation of DNA into oligonucleosomal fragments. Reactive oxygen species at intermediate concentrations can trigger apoptosis, as can a deprivation of neurotrophic factors like nerve growth factor (NGF). Reactive oxygen species at higher concentrations can induce necrotic cell death. Evidence accumulated since the 1990s suggests that apoptosis may actually be the default program for most cells and that the continual presence of survival signals is necessary to prevent widespread cell suicide. Unlike necrosis, the process of apoptosis does not stimulate an inflammatory response. In necrosis usually get a disruption of the cell membrane resulting in cell death products being released into the extracellular space triggering the inflammatory response.
Incorrect Answers:
(A) Phagocytosis by macrophages. Phagocytosis by macrophages occurs in both necrosis and apoptosis. In apoptosis phagocytosis can also occur by adjacent cells.
(C) Membrane blebbing. Membrane blebbing occurs in both necrosis and apoptosis. However, loss of membrane integrity is only in necrosis, not apoptosis.
(D) Degradation of DNA. Degradation of DNA occurs in both apoptosis and necrosis. In necrosis the digestion of DNA is random and has postlytic DNA fragmentation. In apoptosis there is prelytic DNA fragmentation.
References:
Which of the following medications has been shown to significantly reduce relapse rates and diminish the severity and duration of bipolar disorder in postpartum women when used as prophylaxis?
A. Valproic acid
B. Lithium
C. Quetiapine
D. Aripiprazole
Correct Answer: B.
Lithium
Several studies demonstrate that women with histories of bipolar disorder or peripartum psychosis benefit from prophylactic treatment with lithium instituted either before delivery (at 36 weeks of gestation) or no later than the first 48 hours following delivery. Prophylactic lithium appears to reduce relapse rates significantly and diminish the severity and duration of postpartum illness.
Incorrect Answers:
A. Valproic acid use during the first trimester increases the risk of neural tube defects. Lithium, not valproic acid, has been studied as the preferred prophylactic treatment either before delivery or no later than the first 48 hours following delivery to reduce relapse rates in bipolar disorder.
C. There have been no studies demonstrating prophylactic treatment with quetiapine instituted either before or after delivery that reduces relapse rates in bipolar disorder.
D. There have been no studies demonstrating prophylactic treatment with aripiprazole instituted either before or after delivery that reduces relapse rates in bipolar disorder.
Vital Concept:
Lithium has been shown to reduce relapse rates and decrease the severity and duration of illness when given to women with a history of bipolar disorder before or within 48 hours of delivery.
References:
Atypical antipsychotics can be used to treat children with aggression. Which of the following is true about this treatment choice?
A. Atypical antipsychotics are first-line treatment in treating children with both ADHD and aggression.
B. Tardive dyskinesia occurs primarily on initiation of therapy.
C. Atypical antipsychotics should be tapered slowly.
D. Sedation usually occurs after months of therapy.
Correct Answer: C.
Atypical antipsychotics should be tapered slowly.
Atypical antipsychotics should be tapered slowly to allow adjustment to the lowered dose and to prevent withdrawal akathisias. Aripiprazole (in 6-17 years old) and risperidone (in 5-16 years old) are FDA approved for irritability in autism (6-17 years old). Most atypical antipsychotics are used off-label for behavioral disturbances in children and adolescents. Clozapine is not due to side effect profile. Pimavanserin is not because it is only utilized for patients with Parkinson’s disease hallucinations.
Incorrect Answers:
A. Psychosocial/educational intervention and appropriate pharmacotherapy of primary psychiatric disorder should be optimized prior to atypical antipsychotics use.
B. Tardive dyskinesia, involuntary movements, is seen after months to years on antipsychotic therapy, not upon initiation.
D. Sedation is seen primarily with the initiation of therapy, not after months.
Vital Concept:
Atypical antipsychotics should be tapered slowly to allow adjustment to the lowered dose and to prevent withdrawal akathisias.
References:
Galantamine is used to reverse the muscular effects of gallamine triethiodide and tubocurarine. It has also been studied as a treatment for Alzheimer’s disease and other CNS disorders. What is its mechanism of action?
A. Monoamine oxidase inhibitor
B. Antagonist at glutamatergic NMDA receptors
C. Competitive inhibitor of acetylcholinesterase
D. Selective serotonin reuptake inhibitor
Correct Answer: C.
Competitive inhibitor of acetylcholinesterase
Galantamine has a dual mode of action. First, it acts as a reversible competitive inhibitor of acetylcholinesterase (AChE). Second, it is an allosteric modulator of nicotinic acetylcholine receptors (nAChRs). Galantamine can improve and stabilize cognitive performance, activities of daily living, and behavioral symptoms of dementia. The efficacy and tolerability of galantamine are comparable to other cholinesterase inhibitors. Memantine is an antagonist at glutamatergic NMDA receptors.
Incorrect Answers:
A. Monoamine oxidase inhibitors (MAOIs) are a class of drugs that inhibit the activity of one or both monoamine oxidase enzymes: monoamine oxidase A (MAO-A) and monoamine oxidase B (MAO-B).
B. This is the MOA of memantine.
D. Selective Serotonin Reuptake Inhibitors (SSRIs) increase serotonin levels in the brain.
References:
Children with anxiety disorders may have certain developmental considerations that are unique to their disorder. Which of the following is a concern for children with anxiety disorder?
A. Excessive separation anxiety develops in 30% of children.
B. Children with obsessive-compulsive disorder (OCD) recognize their compulsions or obsessions as abnormal or unreasonable.
C. Children with generalized anxiety disorder (GAD) describe more worries or fears than somatic complaints.
D. Children with posttraumatic stress disorder (PTSD) may have generalized nightmares of monsters, rescues, and threats rather than recurring dreams of traumatic events.
Correct Answer: D.
Children with posttraumatic stress disorder (PTSD) may have generalized nightmares of monsters, rescues, and threats rather than recurring dreams of traumatic events.
Children with PTSD may have generalized nightmares of monsters, rescues, and threats rather than recurring dreams of the traumatic event. Additionally, children with PTSD may also exhibit fear of betraying adults and may resist descriptions of traumatic events.
Incorrect Answers:
A. Separation anxiety is a normal part of child development, particularly at ages 8-10 months and 18-24 months. About 2-5% of children develop separation anxiety to the extent that they may display excessive anxiety when going to school, going to sleep, or if separated from a parent when he or she is going on an errand.
B. Children with OCD may not acknowledge their symptoms or may fail to recognize them as abnormal or unreasonable.
C. Somatic complaints are more common than worries and fears in children with GAD.
Vital Concept:
The nightmares and flashbacks in children with PTSD may be generalized and not specific to their history of trauma.
References:
A 55-year-old male is treated for depression with amitriptyline and bupropion. He complains of increased blurred vision and urinary retention. His amitriptyline dose has not changed, what is a possible explanation for this?
A. Patient has developed benign prostatic hyperplasia (BPH).
B. Bupropion dose was increased by his primary care doctor.
C. Bupropion dose was decreased by his primary care doctor.
D. Patient is experiencing adverse effects of bupropion.
Correct Answer: B.
Bupropion dose was increased by his primary care doctor.
Common anticholinergic side effects of the tricyclic antidepressants (TCAs) (e.g. amitriptyline) are blurred vision, urinary retention, constipation, and dry mouth. Primarily CYP2D6 and 1A2 enzymes metabolize amitriptyline. Bupropion is a strong inhibitor of the CYP2D6 enzyme and can increase levels of amitriptyline. Increasing the level of bupropion would slow the metabolism of amitriptyline and increase anticholinergic side effects.
Incorrect Answers:
A. Patient has developed benign prostatic hyperplasia (BPH). This is a distracter. BPH does cause urinary retention but does not account for blurred vision. Increasing bupropion leading to a higher concentration of the TCA would account for both symptoms.
C. Bupropion dose was decreased by his primary care doctor. Bupropion, is an inhibitor of the CYP2D6 system which decreases the metabolism of TCAs like amitriptyline increasing TCA blood levels. Decreasing bupropion would lead to increased metabolism resulting in a lower TCA level. This would not cause the anticholinergic side effects this patient is experiencing.
D. Patient is experiencing adverse effects of bupropion. TCAs have significant anticholinergic activity versus bupropion does not have anticholinergic activity.
References:
CYP3A4 is a member of the cytochrome P450 family of enzymes. Which of the following is a CYP3A4 inducer?
A. Clarithromycin
B. Modafinil
C. fluconazole
D. ketoconazole
Correct Answer: B.
Modafinil
Inhibitors include Clarithromycin, fluconazole, and ketoconazole. Inducers include phenobarbitol, St. John’s Wort, and dexamethasone.
Incorrect Answers:
A. Clarithromycin is a CYP3A4 inhibitor, not inducer
C. fluconazole is a CYP3A4 substrate, not inducer.
D. ketoconazole is a CYP3A4 inhibitor.
References:
A 7-year-old child presents to the pediatrician’s office with her foster parents. She is profoundly underweight and demonstrates delayed language development. The child appears shy but can answer questions on a review of systems, all of which are negative. The child’s foster parents report her biological parents suffer from substance use disorders, and she was taken from her home when neighbors called the police, suspecting she and her 5-year-old brother had been abandoned. What is the most likely etiology of this patient’s symptoms?
A. Anemia
B. Eating disorder
C. Neoplastic disease
D. Emotional abuse or neglect
E. Viral infection
Correct Answer: D.
Emotional abuse or neglect
Although further testing should be completed to rule out endocrine or metabolic causes of weight loss (e.g., diabetes, celiac disease, or adrenal insufficiency), toxic exposures such as lead, and other organic causes of weight loss, the most likely causes of weight loss/failure to gain weight, in this case, are emotional abuse and neglect. Emotional neglect of children can result from parents with substance use disorders and severe mental health disorders. Domestic violence and intimate partner violence increase the risk of emotional neglect for children.
Incorrect Answers:
A, C, and E. While neoplastic disease, infections (including parasitic infections or viral infections such as HIV), and anemia can result in weight loss or failure to gain weight, these are not the most likely causes of this patient’s low weight.
B. Eating disorders usually affect older children and adolescents.
Vital Concept:
Emotional abuse or neglect in children can have physical, developmental, and psychological consequences. Weight loss in young children may indicate neglect.
References:
Race may have an effect on how individuals experience traumatic events and/or develop PTSD. Which of the following statements regarding traumatic event exposure, development of PTSD, and racial and ethnic groups is true?
A. Native Americans have a rate of violent victimization at 4x national average.
B. PTSD rates among Native Americans is 2x national average.
C. European American combat veterans experience higher rates of PTSD than African Americans; this may be related to more traumatic early experiences.
D. Latino combat veterans experience a higher rate of PTSD than their African American and Euro-American counterparts.
Correct Answer: D.
Latino combat veterans experience a higher rate of PTSD than their African American and Euro-American counterparts.
This is especially true for Puerto Rican combat veterans.
Incorrect Answers:
A. Native Americans have a rate of violent victimization that is 2x the national average.
B. PTSD rates among Native Americans is 3x higher than the national average.
C. African American combat veterans experience higher rates than European Americans. This may be related to a higher rate of traumatic early experiences, as more African Americans live in urban environments that have more violence.
References:
A 48-year-old African female is admitted with profuse sweating, diarrhea, abdominal cramps, vomiting, yawning, muscle aches, restlessness, and insomnia for one day. A physical exam reveals a BP of 155/105 mmHg and a temperature of 100°F. The patient is cold with muscle twitches and dilated pupils. She is highly motivated to detoxify. Which medication is the best choice for her?
A. Lithium
B. Chlordiazepoxide
C. Methadone
D. Haloperidol
Correct Answer: C.
Methadone
Management of acute opioid withdrawal involves a combination of general supportive measures in conjunction with pharmacotherapy. Treatment consists of short-term detoxification and long-term maintenance. Methadone, clonidine, buprenorphine, and clonidine-naltrexone are used for detoxification, while methadone, buprenorphine, and naltrexone are used for maintenance therapy. Another less-recommended option is the use of ultra-rapid opioid detoxification. This method uses general anesthesia with propofol or conscious sedation with midazolam as well as naltrexone (or naloxone), ondansetron (an antiemetic), octreotide (an antidiarrheal), clonidine, and other benzodiazepines.
Incorrect Answers:
A. Lithium is the drug of choice for bipolar disorders.
B. Chlordiazepoxide is a benzodiazepine used in alcohol withdrawal.
D. Haloperidol is an antipsychotic drug that is useful for psychotic episodes and uncontrolled agitated behavior.
References:
An adult patient is brought to a psychiatrist’s office by their spouse. The spouse states the patient is convinced they are being monitored by the NSA. The patient is a computer programmer, and a close friend at work was arrested approximately 2 months ago, but the details of the case are not clear. The patient’s spouse states the patient has endorsed these concerns for roughly 6 weeks. Last week, the spouse noticed all the smoke detectors in their home had been dismantled, and the patient had broken apart both of their cell phones. The patient said they were looking for listening devices. The patient did not want to attend the appointment, but the spouse insisted. Despite these symptoms, the patient has been performing well at work, and the spouse reports they recently went on a camping trip where the patient seemed “completely back” to their normal self.
On evaluation, the patient is alert and oriented. They deny other psychiatric and neurologic symptoms, including hallucinations, increased energy, or sleep disruptions. They report a history of mild depressive symptoms during college, but this never required treatment, and the patient denies current low mood. What is the most likely diagnosis for this patient?
A. Paranoid personality disorder
B. Schizoaffective disorder
C. Schizophrenia
D. Delusional disorder
E. Major depression with psychotic symptoms
Correct Answer: D.
Delusional disorder
To meet the criteria for delusional disorder, a patient must have one or more delusions for at least a month in the absence of other prominent mood or psychotic symptoms. Delusions are defined as a fixed false belief; in this case, the patient believes they’re being monitored by government agencies. To make a diagnosis of delusional disorder, other causes must be excluded, including medications or illicit drug use, certain medical conditions, and psychiatric disorders such as schizophrenia.
Delusional disorder is relatively rare, with a lifetime estimated prevalence of 0.2 percent. There are multiple subtypes of delusional disorder, including persecutory (which is the subtype applicable in this case), grandiose, jealous, somatic, erotomanic, and mixed. Mood disorders are common comorbid conditions, particularly depression; a family history of paranoid personality disorder is a risk factor.
Patients with delusional disorder often present in their 30s and 40s and do not have functional impairments outside of their delusions. They often insist their delusions are true and do not accept treatment. Antipsychotics are recommended. Although no randomized controlled trials have been completed for this disorder, case series and open-label studies have demonstrated the efficacy of this treatment with varied response rates. A strong therapeutic alliance between the physician and the patient is an essential part of treating delusional disorder because patients lack insight and may resist treatment.
Incorrect Answers:
A. Paranoid personality disorder is characterized by suspicion of other people and distrust of their motives. Individuals with paranoid personality disorder may develop delusional disorder. This patient is unlikely to have paranoid personality disorder, as the symptoms only appeared a few weeks ago, and they only endorse a single specific delusion rather than more generalized mistrust and suspicion.
B. To meet the criteria for schizoaffective disorder, a patient must have features of both schizophrenia and a major mood disorder.
C. Although this patient has a specific delusion, a diagnosis of schizophrenia requires at least two characteristic symptoms (including hallucinations, disorganized speech, and negative symptoms) that persist for a month, evidence of impaired functioning since the onset of symptoms, and at least 6 months of continuous signs of the disturbance.
E. This patient does not currently endorse any symptoms of depression, making major depression with psychotic symptoms unlikely.
Vital Concept:
Delusional disorder is characterized by at least one delusion lasting for at least one month in patients who otherwise lack functional impairments and have no other medical, substance-induced, or psychiatric reason for their symptoms. Patients with this rare disorder often lack insight into their disease. Treatment with antipsychotics is recommended.
References:
The National Quality Measure Clearinghouse (NQMC) is evaluating a new quality measure to assess the quality of diabetic care in the elderly. The evidence used to create the measure consists of a clinical practice guideline and several research studies published in peer-reviewed journals and indexed in the National Library of Medicine. Scientific evidence provides support for which of the following measures?
A. Reliability
B. Validity
C. Sensitivity
D. Specificity
Correct Answer: B.
Validity
The validity of a quality measure refers to how well it addresses the purpose chosen by the user and is a function of how the measure is built. The user should consider how well the intended use matches the intent of the developer of the measure, which is identified in the NQMC summary section called State of Use of the Measure. The NQMC summary of a measure includes six fields intended to answer these questions. They include these questions:
- How strong is the scientific evidence supporting the validity of this measure as a quality measure?
- Are all individuals represented in the denominator equally eligible for inclusion in the numerator?
- Is the measure result under control of those whom the measure evaluates?
- How well do the measure specifications capture the subject of the measure?
- Does the measure provide for fair comparisons of performance of providers, facilities, health plans, or geographic areas?
- Does the measure allow for adjustment to exclude patients with rare performance-related characteristics when appropriate?
Incorrect Answers:
A. Reliability is the overall consistency of a measure.
C. Sensitivity is the ability of a test to correctly identify those with the disease (true positive rate), whereas test specificity is the ability of the test to correctly identify those without the disease (true negative rate).
Vital Concepts:
The validity of a quality measure refers to how well it addresses the purpose chosen by the user and is a function of how the measure is built.
References:
A patient is able to obtain, process, and understand basic health information and services in order to make educated health decisions. What is this patient viewed as having?
A. True insight
B. Secondary process thinking
C. Primary process thinking
D. Health literacy
Correct Answer: D.
Health literacy
This is the definition of health literacy and it is important to know the meaning of all the answers. It is a common theme in healthy people 2020.
Incorrect Answers:
A. Understanding of the objective reality of the situation coupled with the motivational and emotional impetus to master the situation or change behavior is true insight.
B. The form of thinking that is logical, organized, reality oriented, and influenced by the demands of the environment is second process thinking.
C. Unconscious defense mechanism in which a person attributes to another these unconscious ideas, thoughts, and feelings, impulses that in themselves is a form of protection from anxiety.
References:
Concurrently administering valproic acid with certain medications is likely to decrease valproic acid levels. Which of the following medications may lead to decreased valproic acid levels?
A. Felbamate
B. Cimetidine
C. Carbamazepine
D. Erythromycin
Correct Answer: C.
Carbamazepine
Valproate metabolism may be induced by other anticonvulsants, including carbamazepine, phenytoin, primidone, and phenobarbital, resulting in an increased total clearance of valproate.
Incorrect Answers:
A. B. D. Drugs that may increase valproate levels include cimetidine, macrolide antibiotics (e.g. erythromycin), and felbamate.
References:
What is the major risk regarding fetal exposure to lithium?
A. Gastrointestinal abnormalities
B. Cardiovascular abnormalities
C. Pulmonary abnormalities
D. Ocular abnormalities
Correct Answer: B.
Cardiovascular abnormalities
The approximate incidence of major congenital anomalies in children of pregnant women treated with lithium in the first trimester is 4-12%. Ebstein’s anomaly is considered the most common fetal cardiac defect caused by lithium, occurring in approximately 1/1,000, compared to the general population incidence of 1/20,000. Ebstein’s anomaly is when the tricuspid valve and right ventricle develop abnormally. Lithium remains a category D drug during pregnancy. Have to weigh the risks and benefits. If have bipolar disorder and discontinue lithium during pregnancy there is a 70% risk of mania relapse.
It is recommended to monitor lithium levels monthly in pregnancy until 34 weeks then monitor weekly. During first and second trimester there is a decrease in lithium blood levels due to increased total body water, plasma volume and GFR. Lithium during the third trimester returns back to preconception levels and therefore can result in lithium toxicity.
Incorrect Answers:
A. There has been no association with fetal exposure to lithium and gastrointestinal abnormalities.
C. There has been no association with fetal exposure to lithium and pulmonary abnormalities.
D. There has been no association with fetal exposure to lithium and ocular abnormalities.
Vital Concept:
The most congenital malformations from neonatal lithium exposure is cardiovascular abnormalities, in particular Ebstein’s anomaly.
References:
The NP is caring for a patient and receives a call from a physician requesting the radiology results for the patient. The NP should:
A. Since the caller is a physician, the NP should give the results.
B. Not give the caller the results.
C. Verify that the caller has a need to know the information.
D. Ask the physician to come to the unit to access the information.
Correct Answer: C.
Verify that the caller has a need to know the information.
The NP must determine if the caller on the phone is the physician of record or has a need to know the information.
Incorrect Answers:
A. Until the NP determines that the caller has a true need to know the information, results should not give out the information.
B. The caller may be the physician of record and may have a need to know the information, so the NP should not refuse to give out the information until caller identity is identified.
D. Unless the NP cannot confirm the identity of the caller, there is no need to require the physician to come to the unit to access the information.
Vital Concept:
To ensure protected health information (PHI), verification of the identity of the caller is important to remain in compliance with HIPPA.
References:
ECT was first used in the U.S. in the 20th century. What decade specifically was it first used?
A. 1920s
B. 1940s
C. 1970s
D. 1990s
Correct Answer: B.
1940s
The first use in the U.S. of electroconvulsive therapy (ECT) was in the 1940s. ECT had been used for several years prior in Europe.
ECT has FDA indications to treat catatonia and treatment-resistant or severe depression (unipolar or bipolar). ECT is used off label for psychotic depression, suicidality, severe schizophrenia, and neuroleptic malignant syndrome. ECT has also been used as last resort treatments in OCD, dyskinesias, Gilles de la Tourette syndrome, epilepsy and Parkinson’s disease.
ECT produces a generalized seizure, usually 30-60 seconds in duration, with an unknown mechanism of action for these indications. Treatments are 3 days per week and vary from 7-12 treatments in an acute series. Most common side effects include acute confusion, memory loss, headaches, jaw pain and nausea.
Incorrect Answers:
A. ECT was first used in the U.S. in the 1940s, not 1920s.
C. ECT was first used in the U.S. in the 1940s, not 1970s.
D. ECT was first used in the U.S. in the 1940s, not 1990s.
Vital Concept:
The first use in the U.S. of electroconvulsive therapy (ECT) was in the 1940s.
References:
Medicare direct reimbursement to NPs is allowed under federal law. Which of the following allows this practice?
A. Affordable Care Act
B. Balanced Budget Act of 1997
C. COBRA
D. CMS
Correct Answer: B.
Balanced Budget Act of 1997
The Balanced Budget Act of 1997 was signed into law by President Bill Clinton and allowed direct reimbursement of nurse practitioners by Medicare. Nurse practitioners can be directly reimbursed by Medicare Part B, Medicaid, Tricare, and some health insurance plans. Medicare permits NP reimbursement at 85% of the usual and customary fee, referring to the fee paid to a physician. The NP must file charges under his or her name or provider number for reimbursement. Prior to this act, only NPs who practiced in certain designated or rural areas in the US were permitted to bill Medicare directly.
Incorrect Answers:
A. The Affordable Care Act went into effect in 2014 and is intended to broaden healthcare access.
C. COBRA stands for Consolidated Omnibus Budget Reconciliation Act, which provides for continuation of an employer’s preexisting group health insurance for a fixed period for an individual who loses coverage.
D. CMS refers to the Centers for Medicaid and Medicare Service.
Vital Concepts:
The Balanced Budget Act of 1997 was signed into law by President Bill Clinton and allowed direct reimbursement of nurse practitioners by Medicare. Nurse practitioners can be directly reimbursed by Medicare Part B, Medicaid, Tricare, and some health insurance plans.
References:
The Substance Abuse and Mental Health Services Administration (SAMHSA) says, “peer support services are a way to potentially extend the reach of treatment beyond the clinical setting into the everyday environment of those seeking a successful sustained recovery process.” Which of the following statements regarding peer support services is true?
A. Peer-based services are limited to face-to-face self-help groups.
B. Peer-based support does not have any demonstrable therapeutic effects.
C. There is often a disconnect between peer-based support services and traditional diagnostic-driven programs.
D. Peer-based programs often emphasize formal psychiatric diagnoses.
Correct Answer: C.
There is often a disconnect between peer-based support services and traditional diagnostic-driven programs.
Peer-based programs tend to focus on supporting individuals’ return to independent living.
Incorrect Answers:
A. There are a variety of peer-based service models (including self-help groups, face to face, or Internet), peer-delivered services, peer partnerships, and peer employees.
B. Peer-based programs do have benefit, but further studies are needed to characterize their benefit.
D. Peer-based programs tend to focus on supporting individuals’ return to independent living rather than diagnostic labels.
References:
A woman and her husband are patients at the same practice, and it was understood that the spouses’ medical conditions could be discussed by the physician with the other spouse. The woman has recently been diagnosed with stage IV colon cancer and asks the provider not to tell her husband. She plans to seek aggressive treatment, but her husband just accepted a job in another city, and she does not want to disrupt his career.
Which of the following is the most appropriate next step?
A. Hold a family conference to encourage the patient to discuss her diagnosis with her husband.
B. Tell the patient that her husband will not be informed if she agrees to seek marital counseling.
C. Explain to the patient that her condition will not be discussed with anyone without her permission.
D. Tell the patient’s husband about her diagnosis confidentially but ask him not to indicate awareness of her condition.
Correct Answer: C.
Explain to the patient that her condition will not be discussed with anyone without her permission.
A clinician may not disclose a patient’s medical condition to anyone, even a spouse, without the patient’s permission. The provider can encourage the patient to reveal her medical condition to her husband and offer to meet with them jointly to address any questions or concerns. Some individuals may prefer limited or no disclosure of information to family members. The patient’s confidentiality should be respected unless a third party is at risk of harm.
Incorrect Answers:
A and D. A clinician may not disclose a patient’s medical condition to anyone without the patient’s permission.
B. Regardless of whether the patient agrees to marital counseling, the clinician may not disclose her medical condition to her spouse without her permission.
Vital Concept:
A clinician may not disclose a patient’s medical condition to anyone without the patient’s permission.
References:
Immediately after a needle stick, you are tested and are HIV positive. You were negative when last tested 9 months ago. Over the past several months, you have performed invasive procedures on several patients and plan to continue. Are you required to share this information with your patients (past and present)?
A. Yes, all surgical patients should be aware if their surgeon has a potentially communicable disease.
B. No, continue to use universal precautions; you are not required to disclose your status to patients.
C. Yes, inform your hospital and your patients of your HIV status.
D. You are only required to inform your patients after you have been diagnosed with an AIDS-defining illness.
Correct Answer: B.
No, continue to use universal precautions; you are not required to disclose your status to patients.
Healthcare workers should use universal precautions at all times. Whether an infected physician should disclose the condition depends on the likelihood of risk to the patient and the relevant regulations, but if it becomes clear that the risk to a patient associated with contact or a procedure is high despite appropriate preventive measures, physicians should remove himself/herself from care. If a clinically significant exposure occurs, the physician is obligated to disclose the condition after the fact. A healthcare worker with a blood-borne illness has an ethical obligation to notify the hospital or employer, but this information cannot be used to terminate his/her employment.
Incorrect Answers:
A. There is no requirement of an infected physician to disclose if they have a communicable disease unless there is a high likelihood of risk to the patient.
C. The physician is obligated to disclose HIV status to the hospital but not to the patients.
D. There is no requirement of an infected physician to disclose if they have a communicable disease unless there is a high likelihood of risk to the patient regardless if it is HIV+ versus AIDS-defining illness.
Vital Concept:
A healthcare worker with a blood-borne illness has an ethical obligation to notify the hospital, but not their patients. The healthcare worker should use preventive measures and avoid contact/procedures that puts the patients at high risk despite the preventative measures.
References:
A 34 year old man presents to the office due to marital issues which are causing extreme difficulties at work. He has received a written warning of poor job performance and admits his work production is down considerably. He has complaints of being tired all the time and is unable to fall asleep at night. After inquiring about his feelings regarding the situation, he focuses on his boss and finds him completely unreasonable and he is driving him crazy. This patient is demonstrating which two defense mechanisms?
A. Displacement and repression
B. Rationalization and suppression
C. Suppression and repression
D. Displacement and rationalization
Correct Answer: A.
Displacement and repression
The man is demonstrating displacement by focusing his anger toward his boss even though he admits his productivity is down at work and his boss is having a reasonable reaction. He’s also demonstrating repression by not connecting his exhaustion and sleeping issues with his marital problems.
Incorrect Answers:
B. The man isn’t giving false reasons to show why he can’t be bothered, which would be rationalization. He also isn’t demonstrating suppression, which is when emotions are recognized but they’re not thought about it, as he’s focusing on the situation.
C. The man isn’t demonstrating suppression, which is when emotions are recognized but they’re not thought about it, as he’s focusing on the situation. He is demonstrating repression by not connecting his exhaustion and sleeping issues with his marital problems.
D. The man is demonstrating displacement by focusing his anger toward his boss even though he admits his productivity is down at work and his boss is having a reasonable reaction. He isn’t giving false reasons to show why he can’t be bothered, which would be rationalization.
References:
Some antidepressants increase dopamine significantly. Which of the following antidepressants significantly increases dopamine?
A. Duloxetine
B. Trazodone
C. Selegiline
D. Reboxetine
Correct Answer: C.
Selegiline
MAOIs like selegiline increase norepinephrine, serotonin, and dopamine (see table below). Bupropion has a slight dopamine increase as well. Most SSRIs increase serotonin predominantly, and TCAs generally increase serotonin and norepinephrine.
Incorrect Answers:
A, B, C. Do not increase dopamine
References:
A 34-year-old medical resident who was recently diagnosed with social phobia asks if he is at risk for another mental illness. Which of the following disorders is commonly comorbid with social phobia?
A. Schizophrenia
B. Borderline personality disorder
C. Gender dysphoria
D. Alcohol use disorder
Correct Answer: D.
Alcohol use disorder
Comorbid disorders with social phobics include other anxiety disorders, posttraumatic stress disorder, major depressive disorder, bipolar disorder, body dysmorphic disorder and substance use disorder. The lifetime prevalence of social phobia is around 13%, and it is slightly more common in women. Other factors that increase risk are Native American heritage, younger age, and low socioeconomic status. Factors that decrease risk are Asian, black, or Latino heritage and living in an urban setting. Onset is usually in adolescence or early adulthood.
Psychological theories of the development of social phobia indicate that being exposed to a traumatic social situation in the past or seeing others experience such an event can impart social phobia. Children who are not exposed to social situations and whose parents are controlling and overprotective have a higher likelihood of developing the disorder as well.
Incorrect Answers:
(A) Schizophrenia. Psychotic disorders have not been associated as comorbid with social phobia.
(B) Borderline personality disorder. Borderline personality disorder has not been associated as comorbid with social phobia.
(C) Gender dysphoria. Gender dysphoria has not been associated as comorbid with social phobia.
References:
A 45-year-old male with a history of chronic alcohol use disorder presents with ataxia, confusion, memory loss, and seizures. The patient receives 500mL glucose and saline solution on arrival. After a few hours, he develops diplopia, horizontal nystagmus, lateral rectus palsy, diminished deep tendon reflexes, and a positive Babinski’s sign on the left side. What is the cause of these symptoms?
A. Central pontine myelinolysis due to hyponatremia
B. Brain tumor
C. Wernicke’s encephalopathy
D. Encephalitis
Correct Answer: C.
Wernicke’s encephalopathy
This patient with chronic alcohol use disorder has developed ataxia, ophthalmoplegia, fever, and confusion. His symptoms worsened after glucose administration. Wernicke’s encephalopathy is the most likely diagnosis. Bilateral hyperintensity of the mammillary bodies are involved in most cases.
Incorrect Answers:
A. Central pontine myelinolysis due to hyponatremia -Patients with chronic alcohol use disorder may present with altered mental status and falls secondary to low sodium. This is described in the literature as beer potomania. Clinically, Wernicke’s encephalopathy and osmotic demyelination syndrome can be distinguished by ocular involvement and progression when glucose is given without thiamine. The MRI of a patient with osmotic demyelination syndrome classically shows pontine myelinolysis.
B. Brain tumor is incorrect. The triad of confusion, ophthalmoplegia, and ataxia worsening after administration of glucose suggest Wernicke’s encephalopathy. Symmetrical, bilateral lesions on MRI are more specific to an encephalopathy, and the onset of symptoms are rapid onset for a tumor.
D. Encephalitis (viral) often is displayed as changed in mental status changes in deep tendon reflexes and are often confused and agitation. The above patient meets criteria for Wernicke’s encephalopathy
References:
According to the DSM-5-TR, which of the following statements is true regarding social anxiety disorder?
A. It must be diagnosed before age 18.
B. It is classified as an obsessive-compulsive-related disorder.
C. It is diagnosed with onset at any age.
D. It has been removed from the DSM.
Correct Answer: C.
It is diagnosed with onset at any age.
In the DSM-5-TR, the diagnostic criteria do not specify the age of onset. Also, a duration criterion of “typically lasting for 6 months or more” has been added for adults to minimize the overdiagnosis of transient fears.
Social anxiety disorder (social phobia) DSM 5-TR criteria include:
· Terror or significant concern about a certain environment that exposes the patient to potential judgment by the public or individuals. This may be when being watched (e.g., during observations, dining in public spaces), presenting (e.g., public speaking), or socializing (e.g., talking or interacting with new people).
· The patient is concerned that they will behave poorly and be judged by others (i.e., they will be rejected, mortified)
· The patient reports sudden terror or significant concern every time they are exposed to the certain environment
· The patient evades the certain environment
· The actual risk or threat posed by the environment is insignificant in comparison to the patient’s emotional response and concern
· The terror or concern is consistent for at least 6 months
· The terror/concern or active evasion of the environment leads to dysfunction (academic, professional, social, or otherwise) or substantial anguish
· The patient’s symptoms are not directly related to the use of a substance, medication, or pre-existing medical diagnosis or health concern. The concern is not due to a more appropriate psychiatric condition such as autism spectrum disorder, panic attacks (i.e., panic disorder), past trauma (i.e., post-traumatic stress disorder), separation from a loved one (i.e., separation anxiety disorder), a specific trigger (i.e., specific phobia), physical judgment (i.e., body dysmorphic disorder), or a recurrent thought (i.e., obsessive-compulsive disorder).
· In pediatric patients, the terror/concern must be present when interacting with peers (not only grown-ups) and may present as dependence, immobility, mutism, outbursts, or fits
· The terror/concern is disproportionate or unconnected in those with a physical attribute that causes them discomfort (e.g., significant scars, facial or other obvious physical difference
· May specify performance only if applicable
Incorrect Answers:
(A)There is no age of onset requirement for a diagnosis of social anxiety disorder.
(B) Social anxiety disorder is classified as an anxiety disorder.
(D) Social anxiety disorder (social phobia) is a DSM diagnosis.
Vital Concept:
Social anxiety disorder is characterized by excessive fear or embarrassment and humiliation in social situations, causing significant distress and impaired function.
References:
Electroconvulsive therapy (ECT) deliberately induces a specific phenomenon. What is this phenomenon?
A. Partial complex seizures
B. Generalized tonic-clonic seizures
C. Atonic seizures
D. Myoclonic seizures
Correct Answer: B.
Generalized tonic-clonic seizures
ECT causes generalized tonic-clonic seizures by using alternating-current electrical pulses via scalp electrodes under general anesthesia. Without causing a seizure, ECT has not been found to be effective, so the electrical input must be sufficient to cause a seizure.
Incorrect Answers:
A. ECT indicates generalized tonic-clonic, not partial complex, seizures
C. ECT indicates generalized tonic-clonic, not atonic, seizures
D. ECT induces generalized tonic-clonic, not myoclonic, seizures
References:
Certain medications may predispose patients to serotonin syndrome. Which of the following medications is most likely to predispose patients to serotonin syndrome?
A. Marijuana
B. Meperidine
C. Phenytoin
D. Haloperidol
Correct Answer: B.
Meperidine
Serotonin syndrome is an adverse event, resulting from serotonin excess from the use of serotonergic medication. Clinical signs of serotonin syndrome include confusion, hyperthermia, myoclonus, tachycardia, and hyperreflexia. Severe sequelae can include rhabdomyolysis, metabolic acidosis, and renal failure.
Meperidine can cause serotonin syndrome in patients taking serotonergic medications. Dextromethorphan also exhibits this effect. Pain medications like fentanyl, tramadol, and cyclobenzaprine also are associated with serotonin syndrome, and propoxyphene has some weak serotonergic activity. Other medications associated with serotonin syndrome include amphetamine, analgesics, buspirone, lithium, monoamine oxidase inhibitors, SSRIs, SNRIs, antiemetics, ergot alkaloids, triptans, valproic acid, linezolid, and tryptophans.
Incorrect Answers:
A. Marijuana does not predispose patients to serotonin syndrome.
C. Phenytoin does not predispose patients to serotonin syndrome.
D. Haloperidol does not predispose patients to serotonin syndrome. Haloperidol has the potential to cause neuroleptic malignant syndrome (NMS). NMS can be confused with serotonin syndrome because they both cause mental status change, fever, and autonomic changes. Important in distinguishing NMS from serotonin syndrome are neuromuscular findings and reflexes. Serotonin syndrome is associated with myoclonus and hyperreflexia versus NMS is associated with diffuse rigidity and hyporeflexia.
Vital Concept:
Meperidine can cause serotonin syndrome in patients taking serotonergic medications. Clinical signs include confusion, hyperthermia, myoclonus, tachycardia, and hyperreflexia.
References:
A patient is beginning pharmacological treatment for alcohol use disorder. Which of the following statements about pharmacological treatment of AUD is true?
A. Naltrexone maintenance dose is usually 50mg once daily.
B. Naltrexone is a GABA agonist.
C. Acamprosate could be an NMDA agonist.
D. Disulfiram induces aldehyde dehydrogenase.
Correct Answer: A.
Naltrexone maintenance dose is usually 50mg once daily.
When treating alcohol use disorder, the general consensus is that in a patient without a psychiatric comorbidity, the evidence for pharmacological treatment is less robust than for psychotherapist interventions. Adequately treating any psychiatric condition, if medication is indicated, will help prevent relapse. Naltrexone is given at 25mg initially (with 1-hour observation for reactions), then 50mg once daily starting on day 2. It can also be given at 380mg IM every 4 weeks for maintenance of abstinence.
Incorrect Answers:
B. Naltrexone is an opioid blocker. Studies providing much psychosocial support and additional treatment resources have shown modest results, while other trials have shown no significant difference. Side effects include GI upset, elevated LFTs, and mild dysthymia.
C. Acamprosate structurally resembles GABA and is possibly an NMDA antagonist. Doses are around 2000mg/day. Most trials show modest efficacy in comparison to placebo. Side effects are mild and include stomach upset. Some trials indicate that combining acamprosate and naltrexone may be more beneficial than taking either drug alone.
D. Disulfiram is an aldehyde dehydrogenase inhibitor prescribed at about 250mg daily. The patient should wait at least 24 hours (but up to 5 days in some cases) to resume drinking, as doing so earlier may induce nausea, diarrhea, and raise blood pressure. Double-blinded studies have not convincingly shown its efficacy. However, when used appropriately, the medication does have clinical efficacy.
References:
A 24 year old female comes to your practice and tells you she is pregnant, upon review of her medications, which would you tell her to stop immediately? Her medications include: Valproate, Isotretinoin, Acetaminophen, Multivitamins
A. Valproate
B. Acetaminophen
C. Isotretinoin
D. Multivitamin
Correct Answer: C.
Isotretinoin
This includes taking isotretinoin in any amount even for a short period of time. It must abruptly be stopped. Anticonvulsants like Depakote (valproate) should not be discontinued abruptly because of the possibility of increasing seizures or even status epilepticus. It should be tapered and transitioned to a new medication.
Incorrect Answers:
A. While there’s still debate over whether valproate is safe for women of childbearing age and the UK has recently banned valproate use in women not in a pregnancy prevention program, there’s still no clinical consensus on not giving women valproate while pregnant
B. This is harmless, and can be continued during pregnancy
D. Pregnant women should take multivitamins (ideally prenatal vitamins)
References:
In the PDSA cycle used to direct quality improvement and facilitate change, which of the following refers to the phase of modification of change after examining a test result?
A. Plan
B. Do
C. Study
D. Act
Correct Answer: D.
Act
During the “act” phase of the PDSA (Plan Do Study Act) cycle, a change that has been made is modified based on results from a test of the change. Further modifications are made as needed and preparations are made for the next test.
Incorrect Answers:
A. During the “plan” phase, the change is identified, with ideas for its implementation. Specific measurable objectives are developed with a hypothesis.
B. During the “do” phase of a PDSA cycle, a test of a change is carried out in a pilot study and observations, including notation of obstacles, are documented. Preliminary analysis of data begins during this stage.
C. During the “study” phase, data are analyzed and compared to the hypothesis. The team gathers together to summarize the results and consider the implications of the findings.
Vital Concepts:
During the “act” phase of the PDSA (Plan Do Study Act) cycle, a change that has been made is modified based on results from a test of the change. Further modifications are made as needed and preparations are made for the next tes.
References:
The Institute of Medicine has a specific definition of healthcare performance and quality. Which of the following defines “healthcare performance” according to the Institute of Medicine?
A. How things are done now
B. The efficiency and outcome of care by an organization, including patient satisfaction
C. Healthcare delivery system inputs
D. Activities within a healthcare organization
Correct Answer: B.
The efficiency and outcome of care by an organization, including patient satisfaction
From the perspective of the Institute of Medicine, a recognized leader and advisor on improving the nation’s healthcare, quality in healthcare is defined as a direct correlation between the levels of improved health services and desired health outcomes. It is defined by an organization’s efficiency and outcome of care, and by patient satisfaction.
Incorrect Answers:
A. An organization’s current system incorporates how things are done now.
C. Resources, activities, and results that make up a healthcare delivery system are also called inputs, processes, and outcomes.
D. Activities within a healthcare organization can be addressed to improve quality by addressing what care is provided and how, when, and by whom the care is delivered.
Vital Concepts:
Quality in healthcare is defined as a direct correlation between the levels of improved health services and desired health outcomes. It is defined by an organization’s efficiency and outcome of care, and by patient satisfaction.
References:
A health care provider suspects a patient has schizophrenia and orders an MRI or CT. What is the value of ordering these tests?
A. No value, as these tests rarely lead to diagnosis of a clinically relevant disorder without other localizing signs
B. High value, as psychosis is caused by diverse symptoms, and patients often want to rule out everything before receiving a diagnosis of schizophrenia
C. While controversial, a normal MRI or CT can exclude some treatable causes of psychosis and support diagnosis of schizophrenia
D. Little value, as you expect a normal test; if it’s not, then you are unable to explain your rationale for ordering test
Correct Answer: C.
While controversial, a normal MRI or CT can exclude some treatable causes of psychosis and support diagnosis of schizophrenia
The value of CT or MRI in suspected schizophrenia is controversial. Accessing head imaging is common in new onset psychosis to rule out organic causes. A normal MRI or CT would support a diagnosis of schizophrenia.
Incorrect Answers:
A. Ordering head imaging can have value. A normal MRI or CT supports the diagnosis of schizophrenia.
B. Head imaging is controversial; it does have value but is not considered highly valuable. Also the testing is not preformed upon patient request, it is preformed if a doctor suspects other possible causes of psychosis.
D. Ordering head imaging can have value. A normal MRI or CT supports the diagnosis of schizophrenia.
Vital Concept:
While controversial, a normal MRI or CT can exclude some treatable causes of psychosis and support diagnosis of schizophrenia.
References:
Many health systems and providers use electronic health records. Which of the following statements about these systems and databases is true?
A. Administrators cannot identify everyone who has accessed the patient’s medical record
B. All data, including deidentified data that is stored on an electronic health record, may only be released with patient authorization
C. The use of a separate psychiatric system and a medical system within a hospital does not protect the privacy of psychiatric patients and is not helpful to patients
D. HIPAA prevents unauthorized release of protected health information by preventing release to non-employees of a health facility
Correct Answer: C.
The use of a separate psychiatric system and a medical system within a hospital does not protect the privacy of psychiatric patients and is not helpful to patients
C. Some hospitals have implemented systems in which the protected health information of psychiatric patients is limited to psychiatric staff, who can access both psych and medical records. This has been shown to harm patients, compared to systems where medical doctors also have full access to both psychiatric and medical protected health information.
Incorrect Answers:
A. Administrators should be able to view who has logged in and viewed a patient’s medical record.
B. De-identified data can be distributed without significant limitations.
D. HIPAA tries to limit unauthorized release of protected health information to facilities that are not “Covered Entities,” “Business Associates,” or “Subcontractors.”
Vital Concepts:
A streamlined system that contains psychiatric and medical information has been shown to be safer for patients.
References:
An adult patient presents with their spouse for evaluation. The patient states that over the past 10 years, they have had periods lasting weeks to months at a time of being happy and energetic, followed by months of feeling irritable and “wanting to stay in bed all day alone.” The symptoms have, at times, impaired their relationships or work performance.
Which of the following is the most likely diagnosis?
A. Malingering
B. Major depressive disorder
C. Bipolar I disorder
D. Borderline personality disorder
E. Cyclothymic disorder
Correct Answer: E.
Cyclothymic disorder
Cyclothymic disorder is diagnosed when there are multiple periods of hypomanic symptoms (yet insufficient to meet criteria for a hypomanic episode) and then periods of depressive symptoms (yet insufficient to meet criteria for a major depressive episode). The diagnostic criteria state:
Symptomatic periods fluctuate between those included in the diagnostic criteria for a hypomania episode and those for major depressive episode (MDE), but insufficient to meet the full criteria for either
In adults, symptoms must persist for 24 months or more (12 months in children and adolescents) and must remain present during at least half of that time; there are no asymptomatic periods that last longer than 8 consecutive weeks
The patient has never reported symptoms sufficient to meet the criteria for: major depressive episode, manic episode, hypomanic episode
The symptoms result in substantial dysfunction (professional, interpersonal, academic) or anguish for the patient
These symptoms are not more appropriately attributed to a substance of abuse or another mental health condition, such as another mood disorder or psychotic disorder
Incorrect Answers:
A. Malingering refers to exaggerating or feigning illness for secondary gains, such as in order to escape duty or work.
B. There is no mention of associated symptoms necessary for the diagnosis of a major depressive episode, such as anhedonia, suicidal ideation, guilt, decreased psychomotor activity, energy, concentration, appetite, or sleep.
C. For a diagnosis of bipolar I disorder, the criteria for a manic episode would need to be met. This simply describes periods of happiness and increased energy without the need for hospitalization or symptoms of psychosis that would indicate mania.
D. Borderline personality disorder diagnosis does not include hypomanic symptoms or depressive symptoms. Symptoms of borderline personality disorder include inappropriate anger, impulsivity, unstable relationships, and unstable self-image or sense of self.
Vital Concept:
Cyclothymic disorder is diagnosed when there are multiple periods of hypomanic symptoms and periods of depressive symptoms without meeting DSM criteria for either. Symptoms must be present for over 2 consecutive years and not symptom-free for more than 2 consecutive months.
References:
Language development is an important part of children’s development at ages 2-5. Which of the following statements is true about their language development at this age?
A. 8% of children this age have childhood-onset fluency disorder (formerly stuttering).
B. A preschooler’s vocabulary typically lags behind word articulation.
C. There is wide variation in normal language development at this age.
D. Most word articulation errors end by age 3.
Correct Answer: C.
There is wide variation in normal language development at this age.
Wide variation in normal language development is influenced by environmental factors, including the amount of language a child is exposed to, how much caregivers engage a child in language, and how much they encourage the child to speak.
Incorrect Answers:
A. Childhood-onset fluency disorder (formerly stuttering) occurs in 3% of children at this age. Often, it will resolve over time. Chronic or severe stuttering may require special speech and language services.
B. Word articulation typically lags behind vocabulary. Many preschoolers will make articulation errors, including dropping, altering, or shortening speech sounds.
D. Most articulation errors end by age 4, and people outside of their immediate environment can understand their speech.
References:
A researcher is studying effects of a new blood pressure medication on two groups of men who have hypertension. She assigns one group to receive a placebo, or inert pill. The second group receives an active medication she would like to test. Which term refers to the active medication?
A. Control
B. Dependent variable
C. Independent variable
D. Correlated variable
Correct Answer: C.
Independent variable
In an experimental study design, the variable is a characteristic, object, or event that is being measured. The independent variable is the variable that is not affected by the other variables. It is the variable that is manipulated, in this case, the medication.
The dependent variable is the variable that changes in responses to the manipulation of the independent variable. The control refers to the non-treatment group, or the group that receives a current standard treatment when testing another group with a new treatment.
Incorrect Answers:
A. The control is the non-treatment group
B. This is the variable being measured in response to manipulation of the independent variable (active medication)
D. This is a variable that moves consistently in relation to another variable
Vital Concepts:
The independent variable is the variable that is not affected by the other variables. It is the variable that is manipulated, in this case, the medication.
References:
Amiloride may be used to treat a side effect of lithium. What is that side effect?
A. Renal side effects
B. Cardiovascular side effects
C. Dermatological side effects
D. Hyperglycemic side effects
Correct Answer: A.
Renal side effects
Lithium causes polyuria due to decreased resorption of fluid from the distal tubules of the kidney. Amiloride blocks lithium entry through the sodium channel, reducing the resultant nephrogenic diabetes insipidus.
Incorrect Answers:
B, C, and D. These aren’t side effects of lithium, so amiloride wouldn’t be used to treat them as lithium side effects
References:
A competent 50-year-old male has hypertension, diabetes, and end-stage renal disease. He has been on dialysis for a year. While hospitalized for a foot wound infection, he decides to stop dialysis. The patient does not appear to be depressed, and his foot wound infection is resolving after several days of antibiotic therapy. He states he is tired of “being tied to a dialysis machine.” What is the appropriate response?
A. Declare the patient incompetent and obtain a court order to dialyze.
B. Stop dialysis.
C. Tell the patient that life-sustaining care cannot be withdrawn at this point.
D. Ask the patient’s family for consent for dialysis.
Correct Answer: B.
Stop dialysis.
When a competent adult decides to stop life-sustaining treatment, the provider must comply with the patient’s decision.
Incorrect Answers:
A. If the patient is competent, the provider cannot declare that patient incompetent by virtue of a decision to stop treatment.
C. With respect to patient autonomy, withdrawing care is no different from withholding care.
D. Family members cannot give consent for treatment when a competent patient refuses a medical intervention. If the decision appears to be the result of depression, a psychiatric consultation should be obtained.
Vital Concepts:
When a competent adult decides to stop life-sustaining treatment, the provider must comply with the patient’s decision.
References:
In a therapy session, the patient states that “I get turned down for every job because I’m overqualified.” The therapist replies, “There are fewer jobs that call for your high level of education.” What strategy does the therapist’s reply demonstrate?
A. Rationalizing
B. Reframing
C. Minimization
D. Projection
Correct Answer: B.
Reframing
The therapist has reframed the situation to make the patient feel better. The statement by the therapist might also be considered “reducing distortion,” as the therapist changes the patient’s statement from “every job” to “fewer jobs.”
Incorrect Answers:
A. Rationalizing by a therapist involves a comment that explains unacceptable impulses or behavior.
C. Minimization is a strategy the therapist uses to allow the patient to acknowledge some affect.
D. Projection is a primitive defense mechanism put forth by ego psychology.
References:
A range of psychiatric disorders may occur in childhood. Which of the following statements about the incidence and prevalence of psychiatric disorders in childhood is true?
A. Depression is the most prevalent childhood psychiatric disorder.
B. Generalized anxiety disorder occurs more frequently in boys than girls.
C. Major depression increases in prevalence with age.
D. Autism spectrum disorder is more prevalent in girls than in boys
Correct Answer: C.
Major depression increases in prevalence with age.
Major depression increases in prevalence with age. Based on epidemiological studies and estimates, the prevalence is about 0.3% in preschoolers, 2% in children, and 1.5-9% in adolescents.
Incorrect Answers:
A. ADHD is the most prevalent childhood psychiatric disorder. Data collected from a variety of sources from 2005-2011 show that children ages 3-17 years had ADHD (6.8%), behavioral or conduct problems (3.5%), anxiety (3.0%), depression (2.1%), autism spectrum disorders (1.1%), and Tourette syndrome (0.2% among children ages 6-17 years). Adolescents ages 12-17 years demonstrated illicit drug use disorder in the past year (4.7%), alcohol use disorder in the past year (4.2%), and cigarette dependence in the past month (2.8%).
B. Generalized anxiety disorder is seen more frequently in girls than boys.
D. Autism spectrum disorder is three to four times more common in boys than girls.
References:
A 54-year-old man presents after a recent myocardial infarction and requests pharmacological aid in smoking cessation. He has tried nicotine patches and gums to no effect and reports that bupropion did not help him when he tried it briefly several years prior. After discussion of possible psychiatric side effects, he is prescribed varenicline. What is the mechanism of action of this medication?
A. Partial agonist at alpha4beta2 and partial agonist at alpha7
B. Full agonist at alpha4beta2 and full agonist at alpha7
C. Partial agonist at alpha4beta2 and full agonist at alpha7
D. Dopamine receptor antagonist
Correct Answer: C.
Partial agonist at alpha4beta2 and full agonist at alpha7
Varenicline (Chantix) is a partial agonist at alpha4beta2 and a full agonist at alpha7. It binds to receptors avidly but stimulates receptors more weakly than nicotine, reducing cravings but also limiting the pleasurable effects of smoking.
Incorrect Answers:
A. Bupropion (Wellbutrin, Zyban) is a norepinephrine/dopamine reuptake inhibitor that also acts as a nicotinic acetylcholine antagonist.
B. Nicotine, the addictive substance found in cigarettes, is a nicotine receptor agonist. Nicotine replacement therapies (gums, patches, etc.) use this route to aid in smoking cessation.
D. Dopamine receptor antagonists include neuroleptics as well as antiemetic drugs such as metoclopramide.
References:
A certain behavior change model predicts that a person takes a health action to avoid negative consequences, believing that the action prevents a negative outcome. What is this model?
A. Theory of reasoned action
B. Theory of planned behavior
C. Stress appraisal and coping theory
D. Health belief model
Correct Answer: D.
Health belief model
The Health Belief model predicts health behavior with the understanding that a person takes a health action to avoid negative consequences, if the person believes the action prevents negative outcomes.
Incorrect Answers:
A, B, and C. These options are incorrect.
Vital Concepts:
The Health Belief model predicts health behavior with the understanding that a person takes a health action to avoid negative consequences, if the person believes the action prevents negative outcomes.
References:
A 38-year-old woman is struggling with the balance between work life and home. She has always been driven and associates her identity with her career. She has a high-paying marketing job that requires a lot of her time and energy. She has three children who are school age and feels that she isn’t there enough for them. She is thinking of quitting her job to focus on her children. Her family is financially sound and could afford for her not to work. What developmental task would Erikson consider this to be?
A. Generativity vs. Stagnation
B. Intimacy vs. Isolation
C. Integrity vs. Role confusion
D. Industry vs. Stagnation
Correct Answer: A.
Generativity vs. Stagnation
This woman is struggling with generativity vs. stagnation, she wishes to be able to give her children more. Although her career has always defined her she is growing and changing with her life shown by the desire to quit working to provide more of her time to her children. Intimacy vs. isolation has to do with committed relationships and the capacity to love which does not pertain to her. (C) (D) These are not developmental tasks developed by Erikson.
Incorrect Answers:
B. This is related to committed relationships and the capacity to love, which doesn’t apply to this woman, and she has a good home life and family.
C, D. These aren’t developmental tasks developed by Erikson.
References:
Which of the following scenarios is generally considered unethical?
A. Having notes on a therapy patient, keeping them out of the medical record, and denying the patient access to them
B. Charging patients a fee for missed appointments
C. Hiring a patient to do lawn maintenance
D. Declining a box of chocolates from a patient
A nurse practitioner is named as a defendant in a medical malpractice lawsuit that was filed by a patient she saw two years ago, when she was working in another state. She had occurrence insurance at the time. Which of the following is true?
A. The claim will not be covered
B. The claim will be covered only if she purchased “tail coverage”
C. The claim will be covered
D. The NP cannot be sued in another state
Correct Answer: C.
The claim will be covered
Occurrence-based malpractice coverage is a type of malpractice insurance that is not affected by retirement or job changes. Under this type of policy, all claims derived from the period the policy is in effect will be covered.
If she had a claims-made policy, the claim would not be covered after she left that employment and policy, unless she purchased “tail coverage.” A nurse can be sued in another state over actions that took place in that state.
Incorrect Answers:
A. The claim will be covered
B. Tail coverage isn’t needed for those with occurrence-based coverage
D. A nurse can be sued in another state over actions that took place in that state
Vital Concepts:
Occurrence-based malpractice coverage is a type of malpractice insurance that is not affected by retirement or job changes. Under this type of policy, all claims derived from the period the policy is in effect will be covered.
If she had a claims-made policy, the claim would not be covered after she left that employment and policy, unless she purchased “tail coverage.” A nurse can be sued in another state over actions that took place in that state.
References:
A 65-year-old woman presents to your office and throughout the appointment, the patient expresses quite a bit of anger and bitterness. She is regretful of her past and has alienated herself. According to Erikson, which developmental stage is this patient struggling with?
A. Integrity vs. Despair
B. Intimacy vs. Isolation
C. Generativity vs. Stagnation
D. Industry vs. Inferiority
Correct Answer: A.
Integrity vs. Despair
This patient is struggling with integrity vs. despair which is evidenced by her anger, bitterness, regret, and alienation.
Incorrect Answers:
B. A patient struggling with this would be emotionally isolated. However, this stage doesn’t explain the patient’s anger, bitterness, and regret.
C. This relates to work and family. It doesn’t relate to this patient’s current state.
D. This relates to learning the pleasure of applying oneself to tasks. It doesn’t relate to this patient’s current state.
Vital Concept:
Fulfillment of integrity vs despair is evidenced by comfort with life, willingness to face death, insight and balanced perspective on life’s events. Those struggling with the developmental task of Intimacy vs. isolation results in emotional isolation, this is not the correct answer because of the other attributes of this patient including, anger, bitterness, and regret. Choices C and D are not appropriate for this patient.
References:
You have been providing therapy to a 41-year-old female with a history of PTSD, suicidal ideations and severe depression for about six months. Knowing how important it is, you were sure to have discussed the termination process with her throughout treatment. As the last month approaches to end therapy, your client begins to have exacerbations of symptoms she had in the beginning of treatment and a decrease in overall function.
What is the next step you should take for her termination process if all her goals were met?
A. Extend therapy three more months and then re-evaluate her progress due to high risk of her becoming suicidal again.
B. Recidivism commonly occurs in the termination process of therapy and she should be allowed to express her thoughts and feelings while continuing your plan to terminate as previously discussed.
C. You should refer her for EMDR due to the poor progress she has made.
D. She has borderline personality disorder and this behavior is common at termination of treatments, which means to continue with discharge.
Correct Answer: B.
Recidivism commonly occurs in the termination process of therapy and she should be allowed to express her thoughts and feelings while continuing your plan to terminate as previously discussed.
This is a common occurrence due to the bond and trust one may have built with the therapist. This is also a time where the patient is challenged to use the new skills they have developed in treatment. The literature states some psychotherapist use PRN follow-ups every 3 months and some state this may be harmful. Overall, this is not a reason to stop the discharge plan.
Incorrect Answers:
A. This answer is incorrect because it is common for patients to develop these feelings at the end of therapy, especially if there were dependent/abandonment issues. Extending the therapy will just continue this cyclic process. The risk of suicide may always be there with her history, so it is important not to let this be the sole reason to change her discharge plan.
C. EDMR has nothing to do with this question. This is a treatment usually used for PTSD and has a set number of visits. The patient developing these symptoms at the end of treatment is not uncommon and could be part of the natural termination process.
D. Nothing in this question reveals and clues that this may be her diagnosis. Although these patients may be a challenge to work with, this would not be a reason to change her treatment and discharge plan.
References:
Billy is a 10th grader on the varsity football team. During practice, the coach talks strategy and Billy starts to understand how the team can be better by focusing on individual strengths. This makes Billy think about his own strengths and he realizes how his strengths can aid him in other parts of his life. Which of Jean Piaget’s development stage is Billy in?
A. Preoperational
B. Formal operations
C. Sensorimotor
D. Concrete operations
Correct Answer: B.
Formal operations
Billy is demonstrating abstract thinking by making analogies which is developed during Piaget’s formal operations stage. Preoperational, Sensorimotor, and concrete operations, are not applicable in this scenario.
Incorrect Answers:
A. This is ages 2-7 and involves perception-dominated thinking; he’s too old for this stage.
C. This is ages 0-2 years and involves interacting with the environment; he’s too old for this stage.
D. This is ages 7-11.5 years and involves only applying logical reasoning to objects that are real or can be seen; he’s too old for this stage.
References:
A nurse practitioner is considering the feasibility of an independent practice vs. a practice under supervision of a physician in an area that is heavily populated with Medicare and Medicaid recipients. Which of the following is correct?
A. Billing under a physician practice allows tracking and evaluation of NP care
B. The public is better able to see quality indicators for providers who delivered care when NP care is billed under a physician practice
C. The physician must be present in the physical location for immediate assistance to allow the physician to bill for NP care under “incident to” model
D. NPs billing under a physician practice under their own provider number can bill 100% for services
Correct Answer: C.
The physician must be present in the physical location for immediate assistance to allow the physician to bill for NP care under “incident to” model
Medicaid may pay from 75% to 100% of the physician fee with additional payment for rural areas. Medicare will only reimburse a physician practice 85% of the physician fee if an NP bills using her own Medicare provider ID. As a result, this encourages the use of billing “incident to” for NP services, for which the practice will receive 100% of the physician fee schedule. However, billing “incident to” has several requirements, including the provision that services must be initiated and performed under physician supervision and the physician must be present in the physical location for immediate assistance. This type of billing system does not allow tracking and evaluation of NP care and it limits the public’s right to see quality care indicators for providers who actually deliver the care.
Incorrect Answers:
A. B. Billing this way doesn’t allow NP care evaluation
D. Medicaid pays 75-100%
Vital Concepts:
Medicaid may pay from 75% to 100% of the physician fee with additional payment for rural areas. Medicare will only reimburse a physician practice 85% of the physician fee if an NP bills using her own Medicare provider ID. As a result, this encourages the use of billing “incident to” for NP services, for which the practice will receive 100% of the physician fee schedule.
References:
Patients with body dysmorphic disorder attempt suicide at a different rate from the general population. What is the lifetime suicide-attempt rate in patients with body dysmorphic disorder?
A. 2-5%
B. 12-15%
C. 22-25%
D. 32-35%
Correct Answer: C.
22-25%
The lifetime suicide attempt rate in patients with body dysmorphic disorder has been estimated at 25%. The annual suicide attempt rate is approximately 3-12 times greater than in the general population.
Incorrect Answers:
A. Patients with body dysmorphic disorder do not attempt suicide at a rate of 2-5% annually.
B. Patients with body dysmorphic disorder do not attempt suicide at a rate of 12-15% annually.
D. Patients with body dysmorphic disorder do not attempt suicide at a rate of 32-35% annually.
Vital Concept:
Patients with body dysmorphic disorder are at increased risk for suicidal ideations and attempts, often attempting suicide multiple times.
References:
Some drugs auto-induce their own metabolism. These types of drugs contain two auto-inducers. Which of the following drug pairs consists of two auto-inducers?
A. Cyclophosphamide and valproic acid
B. Rifampicin and isoniazid
C. Ethanol and carbamazepine
D. Cyclophosphamide and carbamazepine
Correct Answer: D.
Cyclophosphamide and carbamazepine
Both Cyclophosphamide and carbamazepine auto-induce their own respective metabolisms. Other drugs that induce their own metabolism are theophylline, ifosfamide, and aminoglutethimide.
Incorrect Answers:
A. Cyclophosphamide is an auto-inducer, but valproic acid is an INHIBITOR.
B. Although rifampicin is an inducer, but it is not an auto-inducer. Isoniazid is an INHIBITOR.
C. Although carbamazepine is an auto-inducer, ethanol is not auto-inducer. Remember in chronic dosing of ethanol = induction, but in acute, binge doses = inhibition.
References:
The mother of a 10-year-old boy is concerned that his performance at school has been declining over the last 6 months. Teachers complain he frequently gets out of his seat, fidgets constantly, and disrupts other students. At home, the patient cannot sit still at family meals and often loses his shoes and toys. Which of the following is the best way to pursue a diagnosis of attention-deficit/hyperactivity disorder (ADHD) in this child?
A. Order a complete blood count
B. Interview the mother
C. Order genetic screening
D. Ask the parents and teachers to complete the Vanderbilt ADHD Rating Scale
Correct Answer: D.
Ask the parents and teachers to complete the Vanderbilt ADHD Rating Scale
Attention-deficit/hyperactivity disorder (ADHD) is a neurodevelopmental disorder that appears in early childhood. It is characterized by inattention, hyperactivity, and impulsivity that is developmentally abnormal. Boys are more commonly diagnosed than girls. There is no single test that can establish the diagnosis of ADHD. Information should be gathered directly from both parents and teachers on how the child’s behavior impacts the ability to function at home, at school, and with other children. The inattentive, hyperactive, and impulsive behaviors must cause significant impairment in two separate environments to make a diagnosis. Validated rating measures such as the Conners-3 and the Vanderbilt ADHD Rating Scale (VARS) can help make the diagnosis.
Incorrect Answers:
A. A complete blood count (CBC) is a good screening test for anemia. Anemia should be in the differential diagnosis of disorders that can cause symptoms like ADHD. No specific values on the CBC would confirm a diagnosis of ADHD.
B. Interviewing the parents is a great first step toward diagnosing ADHD, but input is needed from multiple adults in a minimum of two situations to confirm the diagnosis.
C. This child is more likely to have ADHD if a parent or sibling also has the diagnosis. There is a high concordance of ADHD in identical twins, and multiple different genes contribute to the disorder. There is no specific genetic test currently for ADHD.
Vital Concept:
ADHD is characterized by the onset of limited attention span or impulse control that starts before the age of 12 and last for more than 6 months. There must be impairment in more than one setting (e.g., at school and at home). Symptoms often persist into adulthood. Treatment includes stimulant medications or non-stimulant medications and cognitive behavioral therapy.
References:
While being sued by a former patient, you are also seeing a good friend of that patient who is unaware that the former patient is suing you. During your sessions, your current patient often talks about this friend and discloses details about the person’s lifestyle, habits, and litigious past that might help your case. What is the right action in this situation?
A. Have your lawyer subpoena the current patient’s records to include them in the case.
B. Obtain written consent from the current patient to use as testimony in the case.
C. The information can’t be used in the case.
D. Use the information but state that it is from an anonymous source.
Correct Answer: C.
The information can’t be used in the case.
You cannot ethically use any information obtained from your patient in the legal case. The information was discussed confidentially, and using or acquiring information for such use is unethical.
Incorrect Answers:
A, B. and D. You can’t ethically use any information obtained from your patient in the legal case. This information was disclosed confidentially, and using or acquiring information for your personal use in the lawsuit is unethical.
Health care systems that use patient electronic medical records (EMR) need to ensure compliance with applicable hospital privacy regulations and policies. Which of the following is a true statement about these policies and regulations?
A. Logs that reveal who had access to a patient health record are protected information under federal law and can only be reviewed by a designated privacy officer
B. If off-site vendors request information, there are no safeguards for patient protection
C. If a healthcare facility has a policy in place about privacy of electronic health records, legal action cannot be taken against the facility
D. Routine random audits should be conducted to identify potentially inappropriate access
Correct Answer: D.
Routine random audits should be conducted to identify potentially inappropriate access
Compliance with hospital privacy regulations and policies should be enforced by routine random audits. If potentially inappropriate access is identified, an EMR system can identify the name of the person who gained access, and information about the time, date, screens, and duration of access, which can determine if the access was an error or if it was an unauthorized review of protected health information.
Incorrect Answers:
A. Patients have the right to a log of all individuals who have accessed their personal health record.
B. Although the use of EMR by creates special issues with respect to privacy, there are safeguards to limit the content and recipients of released data, such as a telefax report.
C. Legal action can be the outcome of any privacy and confidentiality violations, so in addition to creating a policy, the facility must be diligent in enforcement of the policy through monitoring and follow-up.
Vital Concepts:
Health care providers should only be accessing necessary information about their patients in order to continue providing care for them. Any additional access can be potentially inappropriate and in violation of the patient’s privacy.
How long must symptoms be present in children and adolescents to be diagnosed with cyclothymic disorder?
A. 1 month
B. 6 months
C. 1 year
D. 2 years
Correct Answer: C.
1 year
Symptoms must be present for a year in children and adolescents, as opposed to two years in adults. According to the DSM-5-TR, symptoms in adults must persist for 24 months or more (12 months in children and adolescents) and must remain present during at least half of that time; there are no asymptomatic periods that last longer than 8 consecutive weeks. About 15-50% of these patients later develop bipolar disorder (I or II).
Incorrect Answers:
A. Symptoms must be present for 12 months, not one month, in children and adolescents to be diagnosed with cyclothymic disorder.
B. Symptoms must be present for 12 months, not six months, in children and adolescents to be diagnosed with cyclothymic disorder.
D. Symptoms must be present for one year in children and adolescents to be diagnosed with cyclothymic disorder, as opposed to two years in adults.
Vital Concept:
The minimum duration of symptoms in children and adolescents to meet the diagnostic criteria for cyclothymic disorder is 12 months.
Different support systems offer varying levels of support. What type of support system in the community includes organizations and agencies which provide individual access to goods and services?
A. Formal
B. Semiformal
C. Informal
D. Family
Correct Answer: B.
Semiformal
Semiformal support systems provide physical and emotional assistance. They include organizations and agencies in the community that provide goods and services.
Incorrect Answers:
A. Formal support is regulated by laws or statutes. Formal social support is provided by social workers, financial support by Social Security, and medical support provided by Medicare.
C and D. Informal support derive from the social network, includes family, and friends, but only those who actually provide assistance in some way.
Vital Concept:
Semiformal support systems provide physical and emotional assistance. They include organizations and agencies in the community that provide goods and services.
Conduct disorder is a serious behavioral and emotional disorder that can occur in children and teens. Which of the following behaviors most strongly indicates conduct disorder in a child?
A. Chronic arguing with parents
B. Disrespecting teachers, parents, and police
C. Killing neighborhood cats
D. Teasing/bullying classmates
Correct Answer: C.
Killing neighborhood cats
Cruelty to or killing of animals is strongly tied to the diagnosis of conduct disorder. Childhood animal cruelty is one of the earliest and most specific symptoms of conduct disorder. Other criteria include other aggression towards animals or people, destruction of property, deceitfulness or theft, and serious rule/law violations. Many children diagnosed with childhood-onset conduct disorder also have comorbid ADHD or other neurodevelopmental diagnoses, and those with these comorbidities have poorer outcomes. Patients diagnosed in childhood as opposed to adolescence are more likely to have persistent conduct disorder into adulthood. Substance misuse and suicidal ideation are also more common in those diagnosed with conduct disorder. The prevalence of conduct disorder ranges from 2-10% (median is 4%). Risk factors include neglect, physical or sexual abuse, large family size, parental criminality, and a family history of substance use disorders.
Incorrect Answers:
A. While familial arguments are unpleasant, they don’t support a conduct disorder diagnosis in a child.
B. While disrespectful behavior is rude, it doesn’t support a conduct disorder diagnosis in a child.
D. Bullying classmates, while unkind, does not rise to the level of criminality or cruelty to support a conduct disorder diagnosis in a child.
Vital Concept:
Conduct disorder is a diagnosis provided in children and adolescents and is defined by incessant or repetitive actions that infringe on the fundamental rights of others or go against what is considered a societal norm or established law. Animal cruelty in childhood is an early and specific symptom of conduct disorder.
The CYP3A4 enzyme can be inhibited by a wide range of drugs. Which of the following drugs is a CYP3A4 inhibitor?
A. Barbiturate
B. Carbamazepine
C. Ketoconazole
D. Phenytoin
Correct Answer: C.
Ketoconazole
Drugs that induce CYP3A4 enzyme are carbamazepine, dexamethasone, ethosuximide, glucocorticoids, griseofulvin, phenytoin, primidone, progesterone, rifabutin, rifampin, nafcillin, nelfinavir, nevirapine, oxcarbazepine, phenobarbital, phenylbutazone, rofecoxib (mild), St. John’s wort, sulfadimidine, sulfinpyrazone, and troglitazone.
Drugs that inhibit CYP3A4 enzyme are amiodarone, anastrozole, azithromycin, cannabinoids, cimetidine, clarithromycin, clotrimazole, cyclosporine, danazol, delavirdine, diethyldithiocarbamate, diltiazem, dirithromycin, disulfiram, entacapone (high dose), erythromycin, ethinyl estradiol, fluconazole, fluoxetine, fluvoxamine, gestodene, grapefruit juice, indinavir, isoniazid, ketoconazole, metronidazole, mibefradil, miconazole, nefazodone, nelfinavir, nevirapine, norfloxacin, norfluoxetine, omeprazole, oxiconazole, paroxetine (weak), propoxyphene, quinidine, quinine, quinupristin and dalfopristin, ranitidine, ritonavir, saquinavir, sertindole, sertraline, troglitazone, troleandomycin, and valproic acid.
Incorrect Answers:
A. B. and D. These drugs are all CYP3A4 inducers.
Drugs of abuse have “street” or colloquial names by which users frequently refer to them. Which of the following drugs is correctly paired with one of its street names?
A. MDMA - cubes
B. Cocaine - Adam
C. Methamphetamine - Chinese tobacco
D. Ketamine - cat valium
Correct Answer: D.
Ketamine - cat valium
Ketamine is commonly referred to on the street as “cat valium,” “special K,” “vitamin K,” “kit kat,” and “honey oil.” Ketamine is a derivative of PCP that was first developed in 1965. It is less potent and shorter-acting and is used as a dissociative anesthetic in humans.
Incorrect Answers:
A. MDMA (methylenedioxymethamphetamine) is referred as “ecstasy,” “Adam,” “club drug,” “disco biscuits,” and “love drug.”
B. Cocaine is referred as “crack,” “coca,” “freeze girl” and “happy dust” on the streets. Crack is the hardened form of cocaine.
C. Methamphetamine is referred as “crank,” “crystal,” “ice,” “speed”, and “crystal meth” on the streets.
With the publication of the 2003 report, Achieving the Promise: Transforming Mental Health Care in America, mental health professionals were first sensitized to the need for a specific improvement to the health care system. What was this improvement?
A. Quality care for individuals with mental and substance abuse disorders
B. Expanded healthcare coverage and an assessment of the current system’s capacity to address anticipated demands of healthcare coverage
C. Awareness, acceptance, and adaptation of recovery based practices in the delivery of mental health services
D. A recovery-oriented mental health system
Correct Answer: D.
A recovery-oriented mental health system
In 2003, a recovery-oriented mental health system was published in the landmark report: Achieving the Promise: Transforming Mental Health Care in America. The report was an Executive Summary with the vision statement: “We envision a future when everyone with a mental illness will recover, a future when mental illnesses are detected early, and a future when everyone with a mental illness at any stage of life has access to effective treatment and supports-essentials for living, working, learning, and participating fully in the community”.
Incorrect Answers:
A. Quality care for individuals with mental and substance abuse disorders is addressed in the Paul Wellstone and Pete Domenici Mental Health Parity and Addiction Equal Act of 2008.
B. Expanded healthcare coverage is the goal of the Patient Protection and Affordable Care Act and was signed into law in 2010.
C. In 2010, The Substance Abuse and Mental Health Services Administration contributed mental health initiatives in support of the PPACA, to promote awareness, acceptance and adaptation of recovery-based practices; however, this was in support of the recovery model, not the initial sensitization.
A 16-year-old patient is treated with an antiepileptic drug for a seizure disorder. They decide to stop taking the medication because of weight gain, although it is the only medication that has successfully controlled the seizures. The patient and the caregivers understand the potential risk of increased seizure activity as a result of stopping this medication. What is the correct action?
A. Obtain a court order to continue therapy
B. Report parental refusal of continued therapy to child protective services
C. Discontinue treatment
D. Dismiss the patient from the practice for noncompliance
Correct Answer: C.
Discontinue treatment
For discontinuation of therapy that does not result in an immediate or imminent threat to the life of a minor, the caregivers’ wishes should be respected. The caregivers of a child have the right to refuse treatment, including treatments strongly recommended by the healthcare provider, unless refusal of treatment results in a serious and immediate risk to the life or health of the child. In this case, although it would be inappropriate (and illegal) to try to force the use of antiepileptic drugs, it would be appropriate to intervene if the child was in status epilepticus, which is an immediate threat.
Incorrect Answers:
A. This case is a good example of the difference between “immediate threat to life and limb” and a refusal of maintenance therapy to prevent seizures at some distant point in the future. Given that there is not an immediate risk to the patient, obtaining a court order would not be appropriate.
B. There must be an immediately compelling reason to override the wishes of the patient and/or the patient’s caregivers. In this situation, there is not imminent danger to the patient so reporting the caregivers to child protective services would not be appropriate.
D. Although a healthcare provider has the right to terminate a relationship with a patient, they must not abandon the patient. In general, the patient and/or parents must be given adequate opportunity to arrange a continuation of care with another healthcare provider.
Vital Concept:
For discontinuation of therapy that does not result in an immediate or imminent threat to the life of a minor, the parents’ wishes should be respected. The parents of a young child have the right to refuse treatment, including treatments strongly recommended by the healthcare provider, unless refusal of treatment results in a serious and immediate risk to the life or health of the child. In this case, although it would be inappropriate (and illegal) to try to force the use of antiepileptic drugs, it would be appropriate to intervene if the child was in status epilepticus, which is an immediate threat.
Theory and practice can be bridged by consciously thinking through experience to develop a greater understanding. Which of the following terms describes this process?
A. Modeled learning
B. Reflective practice
C. Espoused theories
D. Transitional space
Correct Answer: B.
Reflective practice
Reflective practice is the process of thinking through an experience to develop a greater understanding of the link between theory and practice by consideration of the process of thinking, including preconceptions, beliefs, and evidence underlying the thought process.
Incorrect Answers:
A. Modeled learning is a form of non-reflective learning in which the practitioner does not reflexively consider their actions or their knowledge or the patient, but imitates the actions carried out by the teacher or supervisor.
C. Espoused theories are theories enshrined within protocols and procedures of how practice should proceed, as opposed to theories in use, developed from application and reflection.
D. Transitional space refers to a space in which a practitioner is able to explore, develop, and grow, using reflective practices.
Vital Concepts:
Reflective practice is the process of thinking through an experience to develop a greater understanding of the link between theory and practice by consideration of the process of thinking, including preconceptions, beliefs, and evidence underlying the thought process.
Sumatriptan is used to treat migraine and cluster headaches. What is its mechanism of action?
A. 5-HT1A agonist
B. 5-HT1D and 5-HT1B agonist
C. 5-HT2C agonist
D. 5-HT7 agonist
Correct Answer: B.
5-HT1D and 5-HT1B agonist
Sumatriptan is a 5-HT1D and 5-HT1B agonist.
Incorrect Answers:
A. Sumatriptan is a 5-HT1D, not 5-HT1A, agonist. It’s also a 5-HT1B agonist.
C. D. Aripiprazole is a 5-HT2C agonist and a weak partial agonist at 5-HT7.
A patient presents for evaluation after receiving a diagnosis of acute stress disorder. Their symptoms began shortly after a traumatic event. What is the maximum length of time a patient can experience acute stress disorder symptoms before a diagnosis of posttraumatic stress disorder (PTSD) is considered?
A. 1 week
B. 2 weeks
C. 1 month
D. 6 months
Correct Answer: C.
1 month
Several criteria must be met to receive a diagnosis of acute stress disorder. They include:
The symptoms are related to a prior experience of potential or genuine fatality, severe harm, or sexual assault. The patient may have been a participant, victim, or witness/bystander. Alternately, in the case of violent acts, the patient may have learned about a loved one’s experience as a victim or participant or had recurrent professional experiences with unsettling or unpleasant details (e.g., law enforcement officers, social workers, first responders, etc.). Professional incidents typically occur first-hand, not through images, videos, etc.
The patient must endorse the onset of symptoms following the initial traumatizing experience, with at least nine of the following potential effects categorized as follows:
· Affective/mood
a consistent paucity of desirable feelings (e.g., joy, love, contentment)
· Intrusive
repetitious nightmares associated with the initial traumatizing experience
significant psychological reactions or physical responses to environmental or inner triggers associated with the initial traumatizing experience
recollections of the initial traumatizing experience that are repeated, compulsory, and anguishing/unpleasant (in older pediatric patients, this may include play scenarios resembling the experience in some aspect)
flashbacks, during which the patient temporarily mentally disconnects from their current environment and reality and has the sensation that the initial traumatizing experience is happening again
· Dissociative
a lack of memory regarding a crucial portion of the initial traumatizing experience
a distorted perception of oneself or one’s environment (e.g., a lack of attachment to their own body or thoughts as if watching from a distance, belief that the patient’s experienced environment is not real)
· Arousal
cantankerous disposition with frequent gratuitous fits of rage exhibited as loud outbursts or outwardly aggressive behavior
challenges maintaining focus
poor ability to initiate or maintain adequate sleep
behavior that is overly cautious and alert
extreme edginess or jumpiness
· Avoidance
Preventing exposure to things (e.g., locations, items, persons) that may provoke thinking, feeling, or remembering the initial traumatizing experience
Not allowing oneself to think, feel, or remember the initial traumatizing experience or things related to the experience
The symptoms are not directly related to the use of a substance, medication, or pre-existing medical diagnosis or health concern
The symptoms cause substantial anguish or drastically affect the patient’s ability to function professionally, socially, or otherwise
The symptoms must persist for 3 or more days but not more than 30. For patients with symptoms persisting longer than 30 days, please see the diagnostic criteria for posttraumatic stress disorder (PTSD).
Incorrect Answers:
A. Symptoms for acute stress disorder are present for at least 3 days and up to a month, not only one week.
B. Symptoms for acute stress disorder are present for at least 3 days and up to a month, not only two weeks.
D. If symptoms persist for longer than one month (e.g. 6 months) then consider a PTSD diagnosis.
Vital Concept:
In acute stress disorder, 9 or more symptoms occur for at least 3 days and up to a month after a traumatic experience. If symptoms persist for longer than one month then consider a PTSD diagnosis.
A mother is complaining about her 8-year-old child talking in a baby voice at times. The PMHNP learns that there was recently a new addition to the family. The 8-year-old talks in a baby voice only when her mother is breastfeeding the infant. What defense mechanism is the 8-year-old using?
A. Denial
B. Projection
C. Regression
D. Repression
Correct Answer: C.
Regression
The 8-year-old is showing regression by talking in a baby voice when her mother is breastfeeding the infant. Denial, projection, and repression are not appropriate answers for this question.
Incorrect Answers:
A. The 8-year-old isn’t denying the baby’s presence or pretending it doesn’t exist.
B. The 8-year-old isn’t seeking to attribute a quality in themselves to someone else.
D. The 8-year-old isn’t trying to ignore distressing memories, thoughts, or feelings related to the baby.
While working in a rural part of South Dakota, you receive a subpoena from a judge regarding your treatment of a 13-year-old male with a history of neglect and sexual abuse. After consulting an attorney, it is understood that you are being called as a fact witness. What is the definition of a fact witness?
A. Witness with specialized knowledge who assists a court in deciding a case
B. Witness who has knowledge about details of a case
C. Witness whose focus is to fulfill the basic and developmental needs of the child
D. Witness whose information is not allowed in open court
Correct Answer: B.
Witness who has knowledge about details of a case
A fact witness’s testimony consists of facts or events as opposed to an expert witness, whose testimony consists of the presentation of an opinion or a diagnosis.
Incorrect Answers:
A. An expert witness has specialized knowledge and assists a court in making a decision in a case.
C. The focus of the best interests of the child standard is to fulfill the basic and developmental needs of the child.
D. Privilege is the legal rule that protects certain information from being shared in open court. The individual possesses privilege, not the physician.
A family friend calls for advice about verified, effective treatments for maintenance of sobriety, as her loved one recently made an attempt to stop drinking. He was offered naltrexone or acamprosate. Which of the following is correct regarding these medications?
A. Naltrexone cannot be taken if the patient is still drinking.
B. Naltrexone should be avoided in a patient with kidney disease.
C. Patients taking acamprosate tend to have better adherence.
D. Naltrexone is available as once daily oral dosing or once monthly injection.
Correct Answer: D.
Naltrexone is available as once daily oral dosing or once monthly injection.
If a patient is not achieving abstinence on oral naltrexone, they may be switched to monthly injectable naltrexone for better adherence.
Incorrect Answers:
A. Naltrexone may be started while the patient is still drinking.
B. Naltrexone is metabolized in the liver and should be avoided in patients with hepatitis or liver failure.
C. Acamprosate is taken three times daily, but naltrexone is once daily.
A 16-year-old girl attended a rave with her friends. She now presents with hypertension and hyperthermia, and clinical examination indicates brisk reflexes. Her presentation is consistent with which of the following conditions?
A. Neuroleptic malignant syndrome
B. Serotonin syndrome
C. Schizophrenia
D. Alcohol withdrawal
Correct Answer: B.
Serotonin syndrome
This patient’s symptoms indicate serotonin syndrome, which is characterized by brisk reflexes. NMS is characterized by rigidity. She likely used MDMA (ecstasy) at the party, which resulted in serotonin syndrome. This is a known risk of intoxication with MDMA because the mechanism of action involves the release of serotonin.
Incorrect Answers:
A. The patient doesn’t display the symptoms of high fever, confusion, rigid muscles, variable blood pressure, sweating, and fast heart rate that would raise suspicion of this.
C. The patient doesn’t display the symptoms of delusions, hallucinations, disorganized speech, disorganized behavior or lack of normal behaviors (the “negative” symptoms, including lack of emotional expression, interest, or speech) that would raise suspicion of this.
D. The patient doesn’t display the symptoms of headaches, nausea, tremors, anxiety, hallucinations, or seizure that would raise suspicion of this.
Incorrect Answers:
A. This is characterized by high fever and confusion — neither of which is present
C. Serotonin is an amino acid that helps regulate blood pressure, not a condition
D. Anxiety, headaches, nausea, or shaking hands would happen if this were the case.
Which of the following statements is correct regarding psychosis-induced polydipsia?
A. Polydipsia and water intoxication are associated with heavy cigarette use.
B. Polydipsia is not observed prior to starting antipsychotic medications.
C. Polydipsia is associated with severe hypernatremia.
D. Management involves sodium restriction and water replacement to prevent seizures.
Correct Answer: A.
Polydipsia and water intoxication are associated with heavy cigarette use.
Polydipsia (water intake > 3L/day) and water intoxication are also associated with high antipsychotic dosages, moderate anticholinergic dosages, and extended hospitalization. Nicotine causes the release of antidiuretic hormone (ADH), which fuels thirst. Up to 70% of people with psychogenic polydipsia are cigarette smokers.
Incorrect Answers:
B. Polydipsia can occur before antipsychotic medications are used.
C. Polydipsia is associated with severe hypernatremia. Polydipsia is associated with severe HYPOnatremia (serum sodium < 130 mmol/L), not hypernatremia. Hyponatremia is also associated with calcium antagonists, diuretics, selective serotonin reuptake-inhibitors, tricyclic antidepressants, and venlafaxine.
D. Management involves sodium restriction and water replacement to prevent seizures. Polydipsia is associated with severe hyponatremia requiring sodium replacement not restriction. Polydipsia is a result of water intoxication and management would include water restriction, not replacement. Untreated polydipsia and water intoxication results in severe hyponatremia and associated with cerebral edema, delirium, seizures, coma, and death. Medical differential diagnoses of polydipsia include cancer, diabetes mellitus, diabetes insipidus, end-stage renal disease, hypocalcemia, hypokalemia, and lung disease.
A 32-year-old male suffers from a traumatic brain injury and recovers well after rehabilitation. He has been suffering from tonic-clonic seizures over the past month, however. You start him on Topiramate. He stops having seizures but he presents with a flank pain that is suspicious for a kidney stone. What could have triggered this adverse effect of topiramate?
A. Acute alcohol ingestion may be the culprit
B. Patient may have started using St. John’s wort
C. Phenytoin may have been added to his regimen
D. Phenobarbital may have been added to his regimen
Correct Answer: A.
Acute alcohol ingestion may be the culprit
Acute alcohol ingestion is known to inhibit CYP450 activity, which leads to substrate accumulation and an increase in the incidence of the adverse effect profile of CYP450-dependent drugs. Topiramate is one such drug, and one of its side effects is kidney stone formation. It is likely that the patient may have acutely increased his alcohol intake, leading to CYP450 inhibiting and topiramate accumulation. Consequently, the patient developed kidney stones.
Incorrect Answers:
B. St. John’s wort induces CYP450 activity, thus leading to increased clearance of Topiramate, and decreases the chance of developing kidney stones. This is not the case with our patient in this clinical vignette.
C. D. Phenytoin, Phenobarbital, and Carbamazepine are all anti-epileptics that induce CYP450 activity. Initiating either one would have caused an increase in Topiramate clearance, thus decreasing its concentration in the blood and decreasing the adverse effect profile. This patient developed kidney stones due to a sudden increase in Topiramate concentration.
A certain drug is effective in improving sexual adverse effects in men experiencing antidepressant related sexual dysfunction. It can also be anxiogenic in the same population. What is this drug?
A. Cyproheptadine
B. Sildenafil
C. Yohimbine
D. Vardenafil
Correct Answer: C.
Yohimbine
The adrenergic agonist yohimbine has been reported to help SSRI-induced sexual dysfunction. Unfortunately, yohimbine is also an anxiogenic for some patients, and this effect tends to be counterproductive. A study also showed that switching to or adding bupropion at 75-150mg/day to the SSRI regimen was useful in some cases. However, a dose of 300mg or more is also effective.
Incorrect Answers:
A. Cyproheptadine at dosages of 4-12mg/day may reverse some dysfunction, but unfortunately, cyproheptadine may also reverse the antidepressant or anti-obsessive effects of SSRIs.
B. The use of sildenafil (Viagra) has been more effective than placebo in men with SSRI-induced sexual dysfunction. Sildenafil has been reported to increase overall sexual satisfaction in both men and women.
A nurse practitioner in a large hospital system has worked with employees to identify several quality concerns. A quality improvement team will be formed. Which of the following is a primary responsibility of team members on a healthcare quality improvement team?
A. To stay objective and neutral and uninvolved in content
B. To contribute information and experience
C. To provide logical and analytic thinking
D. To introduce problem-solving tools and show the team how to use them
Correct Answer: B.
To contribute information and experience
Each team member responsible for the work involved in improving a specific process or outcome has the responsibility of focusing on the team objective, contributing information, sharing experiences, perspective, and ideas, and making decisions to develop an improvement plan. Team members should be involved in the process that is being improved. The team leader facilitates team members through a process of building consensus for decisions. The leader should have a positive attitude and enthusiasm, confidence in the team’s work, appreciation for the contribution of all team members, and logical, analytic thinking. A facilitator often assists the team leader, but is not a member of the team. The facilitator is a person outside the group who serves as a coach or consultant and assists the team leader to provide instruction on use of tools and methods in the QI process. Facilitators should not be involved in the process the team is working to improve, but should be selected on the basis of analytical abilities and problem-solving skills. They should stay objective and neutral, should help coordinate ideas and introduce problem-solving tools; guide the team in data collection; and help the team follow a logical problem-solving model while staying on track. They are uninvolved in content but help facilitate the process.
Incorrect Answers:
A. Team members should be involved in the process that’s being improved (not objective/neutral)
C. A facilitator outside the team provides analytical abilities and problem-solving skills
D. The facilitator should introduce problem-solving tools
Vital Concepts:
Each team member responsible for the work involved in improving a specific process or outcome has the responsibility of focusing on the team objective, contributing information, sharing experiences, perspective, and ideas, and making decisions to develop an improvement plan. Team members should be involved in the process that is being improved. The team leader facilitates team members through a process of building consensus for decisions.
A 55-year-old male with a history of alcohol use disorder, hypertension, and cirrhosis presents to the ER requesting a detox admission. The patient has been drinking 12 beers daily for the past 2 weeks. He has experienced withdrawal seizures when attempting to detox at home from alcohol. His initial lab work demonstrates a GFR >60, an AST of 210, and an ALT of 152. He is admitted for a medically managed detox. Which of the following agents would be the best choice to use for this patient’s alcohol detox?
A. Chlordiazepoxide
B. Diazepam
C. Clorazepate
D. Lorazepam
Correct Answer: D.
Lorazepam
Lorazepam is a benzodiazepine that can be used for alcohol detox. In patients requiring alcohol detox who have evidence of liver disease or impairment, as evidenced in this patient by his elevated liver transaminases, lorazepam would be a good option. Lorazepam metabolism does not produce active compounds, and it is cleared from the patient’s system more rapidly than other benzodiazepines, making it a preferable choice in patients with liver dysfunction. The other medications are benzodiazepines and can protect against alcohol withdrawal symptoms; however, in patients with liver dysfunction, lorazepam is preferred for alcohol detox.
Incorrect Answers:
A, B, and C. These benzodiazepine medications can protect against alcohol withdrawal syndromes. However, for patients with liver dysfunction, lorazepam is preferred for alcohol detox.
A 2-year-old girl is admitted to the ED for evaluation of what the mother says was a seizure. The patient appears lethargic and difficult to rouse. The mother states that the girl had just eaten dinner when she began to shake uncontrollably. This mother has brought her daughter to the ED 6 times in the last year, each time for an unrelated complaint. The patient’s lab results show low blood glucose and increased exogenous insulin. When asked if her daughter has ever received treatment for diabetes, her mother denies it. Records show that the mother had gestational diabetes that required insulin administration. When interviewing the mother to determine if the diagnosis is correct, which of the following precautions should be taken?
A. Mother should be restrained.
B. At least 2 people should be present, 1 to interview the mother and the other to observe interactions.
C. Child should be taken out of the room immediately.
D. Only 1 physician should interview the mother to prevent suspicion.
Correct Answer: B.
At least 2 people should be present, 1 to interview the mother and the other to observe interactions.
It is suggested that 2 people are present to achieve a more objective view.
Incorrect Answers:
A. and C. Restraining the mother or taking the child away could cause the woman to panic and leave. It is important to keep the woman calm so that the true diagnosis can be reached and the child can be protected.
D. Two people should be present to give more insight into the situation.
The NP identifies that infection is a problem in the nursing unit where they practice. The NP asks that a process improvement project be launched. The NP knows that the process improvement initiative:
A. Should be conducted by the Quality department
B. Must be approved by the Institutional Review Board
C. Can be done concurrently or retrospectively
D. Usually has a fixed endpoint
Correct Answer: C.
Can be done concurrently or retrospectively
The timing of the PI process will depend on when the issue is identified. Therefore, the initiative can be carried out concurrently or retrospectively.
Incorrect Answers:
A. The PI project should include all of the people involved in the process, including Quality, unit managers, and clinicians. It is important that all groups participate to ensure buy-in on the end product.
B. Unless specific patients will be identified, IRB approval is NOT required. Since PI results are meant to improve internal processes, this is rarely required.
D. Typically, there is no fixed endpoint. Instead, processes are implemented and tested, issues with the new process are identified, and new processes are put into place. This makes the PI process a continuous, cyclical process.
Vital Concept:
The timing of the PI process will depend on when the issue is identified. Therefore, the initiative can be carried out concurrently or retrospectively.
A 16-year-old male expresses resentment about having to care for his young child to his therapist. When the patient is with friends, he consistently reports how wonderful it is to be a father and that he is proud to be a good provider, and he spends much of his time planning special outings and activities for his son. He also volunteers to chaperone his son’s preschool (daycare) class trips every week. Which defense mechanism describes this behavior?
A. Identification
B. Reaction formation
C. Repression
D. Displacement
Correct Answer: B.
Reaction formation
Reaction-formation involves the conversion of unconscious wishes into their opposites. The behavior is the opposite of what one really desires or feels.
Incorrect Answers:
A. Identification is the unconscious modeling of one’s self on another person’s character and behavior.
C. Repression involves avoiding pleasurable instincts or drives due to the threat of suffering if the drive is satisfied.
D. Displacement is the shift of a sexual or aggressive drive to a more acceptable target.
A nurse practitioner has formed a team to implement a program that will increase the number of women in a population who receive cervical cancer screening in accordance with evidence-based standards. At the first few meetings of the team, which of the following should be considered?
A. Team members in a newly formed team usually show signs of reluctance
B. The expectations about participation should not be addressed by ground rules until the team has established trust
C. Meetings should be 1 to 2 hours weekly or every other week, but may run over if issues arise
D. The team leader should summarize key decisions and actions at the close of the team meeting
Correct Answer: D.
The team leader should summarize key decisions and actions at the close of the team meeting
Before the close of each team meeting, the team leader should summarize key decisions and actions taken, and action items that need to be taken. Time, place, and agenda for next meeting should be addressed as well.
Incorrect Answers:
A. During the initial phases of team development, typical behaviors include being polite and masking feelings of reluctance or insecurity. Groups of individuals who initially gather as a team often need time and effort to gain acceptance and build trust, which will allow progression to team functioning. Team members may be cautious in communication until they gain a clearer understanding of their roles on the team and a sense of trust with other team members. Trust-building activities include asking team members to share something about themselves or asking members to share what strengths they bring to the team.
B. In the first few meetings, a set of ground rules that govern how meetings are run and how members interact with each other should be established, and the rules can be changed as the work of the team progresses and new issues arise. Some issues addressed in ground rules include attendance; meeting location, time and frequency; expectation of participation in the meeting, including basic conversational courtesy; and expectation for timely completion of any tasks that are assigned for completion outside of the meeting time.
C. Productive meetings are ideally team meetings that last for 30 minutes to one hour weekly or every other week and are scheduled at a regular day and time. Meetings should have an agenda and should start and end on time.
Vital Concepts:
Before the close of each team meeting, the team leader should summarize key decisions and actions taken, and action items that need to be taken. Time, place, and agenda for next meeting should be addressed as well.
Antidepressants may cause sedation (depression of a patient’s awareness to their environment and reduction of their responsiveness to external stimulation). Which of the following antidepressants is least likely to cause this effect?
A. Protriptyline
B. Amitriptyline
C. Trimipramine
D. Doxepin
Correct Answer: A.
Protriptyline
Protriptyline does not cause sedation like the other listed drugs. Other sedating antidepressants include imipramine, trazodone, nefazodone, mirtazapine, and amoxapine. Unique among the TCAs, protriptyline tends to be energizing instead of sedating and is sometimes used for narcolepsy to achieve a wakefulness-promoting effect.
Incorrect Answers:
B, C, and D. These antidepressants all have sedation effects.
An individual supportive psychotherapist is attempting to build a therapeutic alliance with a patient. Which of the following strategies might they employ?
A. Disinterest
B. Disagreeing with the patient’s feelings
C. Expression of understanding
D. Maximization
Correct Answer: C.
Expression of understanding
Expression of understanding helps to assure the patient that the therapist is listening (e.g. seeking clarification when a patient’s statement is unclear).
Incorrect Answers:
A. Expression of interest, such as accurately recalling details from previous sessions, helps to build trust
B. Expression of empathy, which could involve agreeing with the patient’s feelings, is a way to assure the patient that the patient is listening
D. Maximization is a cognitive error where a part of a situation is picked out and exaggerated; it’s not a technique for building a therapeutic alliance
The DSM-5-TR has 11 criteria for a substance use disorder. How many need to be met to meet the “severe” subclassification?
A. 2
B. 3
C. 4
D. 6
Correct Answer: D.
6
To qualify for the “severe” subclassification of a substance use disorder, 6 of the 11 criteria must be met. A minimum of 2-3 criteria must be present for “mild,” and 4-5 criteria are required for “moderate” substance use disorders. The criteria include:
A destructive repetition or habit of ingesting/administering an intoxicating substance that causes substantial anguish or drastically affects the patient’s ability to function professionally, socially, or otherwise. These effects are evidenced by two or more of the conditions listed here within one year:
· ineffective attempts to reduce the use of the substance or a wish to do so
· an intense need or impulse to use the substance
· a persistent ingestion/administration of the substance even though they have experienced repeated relational challenges (i.e., with surrounding friends and family members) related to its use
· a persistent ingestion/administration of the substance in environments where it is unsafe
· the development of tolerance, which is a gradual reduction in the physical impact/effect of a given substance when administered at a consistent dose or amount, requiring an increase in dose or amount to achieve the prior effect
· a persistent ingestion/administration of the substance for a longer time and at a higher dose or amount than planned
· a considerable investment of time related to the substance, procuring it, ingesting/administering it, or recuperating from the consequences of its use.
· a persistent ingestion/administration of the substance, interfering with significant responsibilities and commitments (i.e., academic, professional, or familial)
· a decrease in attendance or participation in significant events at work, at home, or with friends/family due to the use of the substance
· a persistent ingestion/administration of the substance even though they are aware of a significant challenge directly related to the substance use
· the development of withdrawal as evidenced by the signs and symptoms of withdrawal syndrome for that particular substance or the use of the substance to prevent these symptoms.
This disorder may be graded as mild (two or three symptoms), moderate (four or five), or severe (at least six symptoms).
The disorder may be considered in early (3 or more months) or sustained (12 or more months) remission when all of the above symptoms have resolved, except for an intense need or impulse to use the substance. This may also be qualified in a controlled environment if the patient is currently without access to the substance.
Incorrect Answers:
A. 2-3 is considered mild.
B. 2-3 is considered “mild” classification
C. Meeting 4-5 criteria meets the “moderate” classification
Vital Concept:
Substance use disorders are defined by issues with impaired control of use, risky use, tolerance and withdrawal, and social impairments. The frequency and intensity of substance use are not independent criteria for diagnosing a substance use disorder. Meeting 6 of the 11 criteria suggests that the condition is severe.
A number of factors have been identified as safety hazards for medical errors in the healthcare workplace. Which of the following is one of those hazards?
A. Workarounds
B. Having the patient’s family participate in rounds
C. Having observers for procedures
D. Updated technology
Correct Answer: A.
Workarounds
Workarounds occur when clinicians encounter problems in delivering care and devise a quick way to solve the problem. Workarounds leave systems problems untreated and may cause errors. An example would be bypassing a safety system like the barcoding procedure for medication because it is perceived to have too many steps.
Incorrect Answers:
B. Having the patient’s family involved may improve care.
C. Having observers for procedures is an important part of medical teaching and doesn’t affect safety.
D. Updated technology should improve patient safety.
Vital Concepts:
Workarounds occur when clinicians encounter problems in delivering care and devise a quick way to solve the problem. Workarounds leave systems problems untreated and may cause errors. An example would be bypassing a safety system like the barcoding procedure for medication because it is perceived to have too many steps.
References:
The Patient Protection and Affordable Care Act (PPACA) passed in 2010. Which of the following is one of its provisions?
A. Nurse practitioners are to be reimbursed at 100% by Medicare
B. Nurse-managed health centers (NMHC) were funded in the first year
C. Medicare was set to reimburse a 15% bonus for services for NPs practicing in health professional shortage areas through January 1, 2016
D. Full implementation of the ACA was expected to decrease the demand for primary care
Correct Answer: B.
Nurse-managed health centers (NMHC) were funded in the first year
The Patient Protection and Affordable Care Act (PPACA) was passed in 2010 in an attempt to reform the health care system in the United States, with an emphasis on effectiveness, access, and care quality. With full implementation, the demand for primary care was expected to increase, offering an opportunity for nurse practitioners to contribute significantly to delivery of primary care services. The PPACA legislation created more leadership opportunities for nurse practitioners, in part by provision of funding of $50 million for Nurse-Managed Health Centers in the first year of the project. However, although there is increasing support and leadership opportunities for nurse practitioners, there was no mandate for 100% reimbursement by Medicare for advanced nurse practitioners. Medicare was to reimburse a 10% bonus for services through January 1, 2016, for NPs practicing in professional shortage areas.
Incorrect Answers:
A. There was NO mandate for 100% reimbursement by Medicare for advanced nurse practitioners.
C. Medicare was to reimburse a 10% bonus for services through January 1, 2016, for NPs practicing in professional shortage areas, not 15%.
D. With full implementation of the PPACA, the demand for primary care was expected to increase, not decrease.
Vital Concept:
The Patient Protection and Affordable Care Act (PPACA) was passed in 2010 and created more leadership opportunities for nurse practitioners, in part by provision of funding of $50 million for Nurse-Managed Health Centers in the first year of the project.
References:
Tricyclic antidepressants (TCAs) are a class of drugs used to treat depression, anxiety, and certain kinds of pain. Which of the following statements about the effects associated with TCA use is true?
A. TCA and quinidine taken concomitantly have no ECG effects.
B. Withdrawal syndrome following abrupt discontinuation of TCA results from cholinergic rebound.
C. Anticholinergic delirium is part of TCA withdrawal syndrome.
D. Adverse effects with TCA overdose include hypertension.
Correct Answer: B.
Withdrawal syndrome following abrupt discontinuation of TCA results from cholinergic rebound.
TCAs can cause withdrawal syndrome following abrupt discontinuation. This is the result of cholinergic rebound.
Incorrect Answers:
A. Quinidine also prolongs cardiac intervals. When taken concomitantly with a TCA, it can result in significant cardiac interval prolongation. Serotonin syndrome occurs when SSRIs are used in combination with other pro-serotonergic agents such as MAOIs and TCAs. This syndrome includes confusion, agitation, neuromuscular excitability, and seizures.
C. Withdrawal syndrome associated with abrupt discontinuation of TCAs includes malaise, nausea, muscle aches, chills, diaphoresis, and anxiety.
D. Adverse effects of TCA overdose include severe sedation, hypotension, and anticholinergic delirium.
References:
A 66-year-old man presents for evaluation after undergoing head and neck surgery for obstructive sleep apnea. This is his third surgery in 3 months, and the patient has recently been feeling anxious when he is getting ready to fall asleep, which he feels makes his sleep poor. He has been in and out of the hospital for complications from his ENT surgeries. The patient has no history of somatic or psychiatric complaints besides OSA and does not look exceedingly tired.
Today, he feels like there is something in the back of his throat that is making it difficult for him to sleep and is swallowing repeatedly to “make it go away.” He was scoped previously by his primary team of otolaryngologists, and they explain that there is no ear, nose, or throat cause for his current complaint. Physical exam is normal. What is a likely source of this man’s multiple complaints?
A. Brief psychotic episode
B. Cocaine withdrawal
C. Adjustment disorder
D. Illness anxiety disorder
Correct Answer: C.
Adjustment disorder
Head and neck surgery patients frequently (about 1 in 6) get adjustment disorder. This patient’s anxiety and frustrations with the surgeons seem to have created a maladaptive response. Adjustment disorder occurs when one develops an emotional or behavioral response to a stressor within 3 months. This response causes either marked distress or impairs functioning. In this patient it was causing marked distress.
Incorrect Answers:
A. None of the symptoms point toward a brief psychotic episode. Patient does not appear to be experiencing delusions, hallucinations, or disorganized speech/behavior.
B. Cocaine withdrawal would likely make this patient irritable and perhaps somewhat depressed. There is no reason to suspect cocaine use from the history.
D. The patient’s symptoms do not point toward illness anxiety disorder. Somatic symptoms are not typically present in this disorder. The patient is not performing excessive health-related behaviors. Also illness preoccupation has to be present for at least 6 months for a diagnosis of illness anxiety disorder to be made.
References:
What is the minimum duration of symptoms required to diagnose a hypomanic episode according to the DSM-5-TR?
A. 4 days
B. 1 week
C. 2 weeks
D. 4 weeks
Correct Answer: A.
4 days
According to the Diagnostic and Statistical Manual of Mental Disorders, Fifth Edition, Text Revision (DSM-5-TR), a hypomanic episode is a specific stage of consistent and significant alteration in:
a mood that is elated, capacious, or cantankerous
behavior that is energetic, dynamic, and lively
At least three of the following must occur to a substantial level, concurrently with the mood and behavior symptoms described above, and indicate a significant deviation from the patient’s typical comportment (if the mood is cantankerous, four of the following is required)
the ability to function on three hours of sleep without feeling tired/fatigued
thoughts that move at an accelerated pace without the ability to sustain focus
a significant elevation in movement or to become more active, with or without purpose
an exaggerated self-regard or concept of self-worth
speaking more than baseline or is typical for the individual
difficulty focusing due to frequent distractions, either subjective or objective
consistent participation in behaviors with an increased risk for negative results or effects
A hypomanic episode is defined as lasting 4 or more days, with symptoms obvious nearly all day, every day.
The symptoms must affect the patient’s ability to function professionally or socially but do not result in an inpatient admission, and psychosis is NOT present.
A hypomanic episode is distinguished from a manic episode in that hypomania does not have any psychotic features. Also, a hypomanic episode is severe enough to cause impairment in social or job functions but does require hospitalization. To make a diagnosis of bipolar II disorder, a single hypomanic episode is not sufficient; the diagnosis requires at least one hypomanic and at least one depressive episode. If a patient has a manic episode, a diagnosis of bipolar I disorder is made. Importantly, a single manic episode is sufficient to make the diagnosis of bipolar I disorder.
Incorrect Answers:
B. 1 week is the minimum duration of symptoms for diagnosing a manic episode.
C. 2 weeks is the minimum duration of symptoms for diagnosing a major depressive episode.
D. 4 weeks is the minimum duration of symptoms for diagnosing a delusional disorder.
Vital Concept:
To diagnose a hypomanic episode the patient must experience symptoms for at least 4 consecutive days.
References:
A therapist says to her patient, “You seem to be so hardworking because you want to humiliate your father.” Which of the following best describes her statement?
A. Exhortation
B. Interpretation
C. Countertransference reaction
D. Observation
Correct Answer: B.
Interpretation
An interpretation is a statement made to promote an insight, which is the conscious realization of unconsciousness conflict.
Incorrect Answers:
A. Exhortation is a therapeutic technique from interpersonal individual psychotherapy.
C. A countertransference reaction is the therapist’s reaction to a patient or the patient’s behavior.
D. An observation offers a connection between statements or behaviors but does not explain motivation.
References:
Success in implementing a quality improvement strategy in healthcare relies on a handful of strategies. Which of the following is an important strategy for ensuring quality improvement success?
A. When forming a team, choose team members who are not leaders in order to obtain a variety of new viewpoints
B. When setting aims, it is critical to avoid specificity in order to achieve a creative result
C. When establishing measures, use a variety of tools through each cycle of plan-do-study-act in order to obtain a broad viewpoint
D. When selecting changes, use ideas from team members who work within the system, or ideas derived from change theories, brainstorming, or the success of others
Correct Answer: D.
When selecting changes, use ideas from team members who work within the system, or ideas derived from change theories, brainstorming, or the success of others
When implementing a quality improvement strategy and initiating a Plan-Do-Study-Act cycle, it is important to determine what change must be accomplished, who will work toward the goal, and what measures will be necessary to improve the organization. When establishing measures of quality improvement, after the determination of the most useful tools for measurement of change, the same tools should be used throughout each PDSA cycle to ensure continuity.
Incorrect Answers:
A. When forming the team, it is important to recruit people critical to the success of the project. Leaders should be involved, but each team member’s strengths (and weaknesses) should be identified. The team should be built to meet the specific needs of the organization or the project.
B. When setting goals, they should be specific and directed at the affected population.
C. When establishing measures, they should be targeted at measurable goals, with deadlines established. All team members should understand the aims and measures of the team.
Vital Concepts:
When establishing measures of quality improvement, after the determination of the most useful tools for measurement of change, the same tools should be used throughout each PDSA cycle to ensure continuity.
References:
A 35-year-old woman was diagnosed in the last 3 months with stage IV breast cancer. She is in the hospital recovering after a bilateral mastectomy. The patient states that she feels that she will never be free of breast cancer and is convinced that it has spread to other parts of her body. She has wondered openly to nurses if it would be better to “cut off all the non-essential parts.”
Physical exam is unremarkable except for her surgical scars, and she appears to be mentating normally otherwise. The patient has no history of depression but admits to feeling depressed for the last few days. She has no suicidal ideation but has a somewhat decreased appetite. The patient also has poor sleep (which she attributes to the chemotherapy), poor energy, but fairly good support in her mom and husband. However, she has little interest in things which previously made her happy. She reports no feelings of guilt, problems with concentration, or psychomotor retardation.
What is the most likely diagnosis?
A. Major depressive disorder
B. Adjustment disorder with depression
C. Adjustment disorder with dementia
D. Persistent depressive disorder
Correct Answer: B.
Adjustment disorder with depression
Adjustment disorder with depression is the most common psychiatric disorder among cancer patients, and up to 50% of all cancer patients have a comorbid psychiatric disorder. This condition often manifests with self-harm.
Adjustment disorder occurs within 3 months of a stressor and ceases within 6 months of a stressor ending. Has a change in emotional or behavioral symptoms that are out of proportion to severity of stressor and/or causes impairment. The symptoms should not be better explained by another psychiatric disorder. Specifiers of adjustment disorder include the following: with depressed mood, with anxiety, with mixed anxiety and depressed mood, with disturbance of conduct, with mixed disturbance of emotions and conduct, unspecified. The specifier “with depressed mood” consists of predominant tearfulness, low mood and hopelessness.
Incorrect Answers:
A. Major depressive disorder (MDD). This patient’s symptoms do not meet criteria for MDD. MDD would involve at least 2 weeks of depressive feelings most days; while she does feel depressed, she has no suicidal ideation reported and has an event that appears to correlate with the focus of symptoms.
C. Adjustment disorder with dementia. Dementia is unlikely, as this woman has otherwise normal mentation. Also “with dementia” is not a specifier of adjustment disorder.
D. Persistent depressive disorder. Her symptoms have not lasted long enough to qualify as persistent depressive disorder (previously dysthymia), as 2 years duration is needed for this diagnosis.
References:
Which of the following statements is true for transitional objects?
A. Transitional objects usually have some association with the father.
B. Transitional objects are hard objects placed over the crib to create security.
C. Children usually do not surrender transitional objects until the age of 10
D. Transitional objects are physical reminders of a mother’s presence.
Correct Answer: D.
Transitional objects are physical reminders of a mother’s presence.
Winnicott hypothesized that children choose a transitional object as a physical representation of their mother at a time when they have not yet internalized their mother.
Incorrect Answers:
A. Transitional objects usually have an association with the mother.
B. Transitional objects are usually soft items such as a stuffed animal or blanket that help soothe a toddler.
C. Children usually surrender their transitional object by age 5, although there is considerable variation.
References:
Some SSRIs can be stopped abruptly without a tapering period. Of the following medications, which is an SSRI that can be stopped this way?
A. Fluoxetine
B. Paroxetine
C. Sertraline
D. Fluvoxamine
Correct Answer: A.
Fluoxetine
Long-acting SSRIs like fluoxetine can decrease symptoms of discontinuation. The other options are shorter-acting SSRIs, which should be tapered over several weeks to decrease symptoms of discontinuation.
Incorrect Answers:
B, C, and D. These are all shorter-acting SSRIs. They should be tapered over several weeks to decrease symptoms of discontinuation.
References:
Which of the following is the name of the program that the Institute for Healthcare Improvement uses to direct quality-improvement programs by employing a scientific model to facilitate change?
A. Meaningful use
B. Scientific method
C. Plan-do-study-act cycle
D. Reflexive thinking
Correct Answer: C.
Plan-do-study-act cycle
The plan-do-study-act cycle is a scientific method used in the model by the Institute for Healthcare Improvement to accelerate the process of bringing quality care to patients.
Incorrect Answers:
A. Meaningful use refers to utilizing electronic health record technology to improve quality, safety, and efficiency and to reduce health disparities. It is intended to engage patients and families and to improve care coordination and public health.
B. The scientific method consists of systematic observation, measurement, and experimentation. Using the scientific method, hypotheses are formulated, tested, and modified.
D. Reflexive thinking refers to active thinking that involves the use of higher-order thinking skills, which encourages learners to think in both abstract and conceptual terms, apply specific strategies to problems, relate new knowledge to prior understanding, and examine their own thinking, learning strategies, and assumptions.
Vital Concepts:
The plan-do-study-act cycle is a scientific method used in the model by the Institute for Healthcare Improvement to accelerate the process of bringing quality care to patients.
References:
A high percentage of schizophrenic patients relapse within five years of discontinuation of treatment following their initial episode. What is this percentage?
A. 45-55%
B. 55-65%
C. 65-75%
D. >75%
Correct Answer: D.
>75%
Most patients will relapse within a year, and almost all will relapse within 2 years. Around 80% will relapse by 5 years after antipsychotics are discontinued. Interestingly, a longer treatment period prior to discontinuation does not reduce the relapse rate. Even with re-introduction of antipsychotic treatment after relapse, treatment failure may emerge in up to 1 out of 6 patients. Recent evidence suggests that the very high relapse rates and abrupt re-emergence of psychotic symptoms indicate a reduced threshold for psychotic decompensation once a first episode has occurred.
Incorrect Answers:
A. B. C. These percentage bands are all too low. Around 80% of schizophrenic patients relapse within five years of discontinuation of treatment following their initial episode.
References:
A nurse practitioner is treating an elderly male patient. He is alert but does have periods of forgetfulness. The patient develops metastatic prostate cancer and after discussing treatment options with the oncologist, he decides to undergo treatment with Lupron and Taxotere. His daughter tells the nurse that she does not want her father to undergo treatment because the side effects are potentially too severe for a man his age. Which of the following is true?
A. The daughter’s wishes should be respected since the father is forgetful
B. The daughter should get a durable healthcare power of attorney for her father
C. The elderly man should get the treatment he has decided upon
D. The nurse practitioner should suggest the daughter seek a second opinion
Correct Answer: C.
The elderly man should get the treatment he has decided upon
An elderly patient who is competent mentally has the right to make decisions on his or her own healthcare preferences. Competence is a legal term, but in practice, if the patient can understand the risks and benefits of a treatment, they can make the decision. This is the ethical principle of autonomy. If a patient is incapacitated mentally, such as a patient in a coma or a patient with dementia, then a surrogate will be designated. An individual may appoint a healthcare surrogate while they are well, in which case that person can make decisions in the patient’s best interests.
Incorrect Answers:
A. The father is competent his wishes should be respected
B. While getting a power of attorney may be useful, and an individual can designate this while well, they don’t have to do so
D. The father is responsible for this decision, not the daughter
Vital Concepts:
An elderly patient who is competent mentally has the right to make decisions on his or her own healthcare preferences. Competence is a legal term, but in practice, if the patient can understand the risks and benefits of a treatment, they can make the decision. This is the ethical principle of autonomy. If a patient is incapacitated mentally, such as a patient in a coma or a patient with dementia, then a surrogate will be designated.
References:
A nurse practitioner is researching the percentage of patients in a health plan with an inpatient admission during the prior twelve months. What type of quality measure best describes her research?
A. Management
B. Use of service
C. Efficiency
D. Patient experience
Correct Answer: B.
Use of service
Use of service refers to the provision of a service to, on behalf of, or by a group of persons identified by enrollment in a health plan or through use of clinical services. These measures can assess encounters, tests, or interventions that are not supported by evidence for appropriateness of the service for the specified individuals.
Incorrect Answers:
A. Management quality measures refer to measures that assess administrative activities that are important to health care but not part of a direct interaction between individual patients and health care professionals.
C. Efficiency measures are measures of the relationship between a specific level of quality of health care service and the resources used to provide that care, typically the cost of care.
D. Patient experience refers to a measure characterized by a patient or enrollee’s report of observations and participation in health care, or any assessment of the resulting change in their health.
Vital Concepts:
Use of service refers to the provision of a service to, on behalf of, or by a group of persons identified by enrollment in a health plan or through use of clinical services. These measures can assess encounters, tests, or interventions that are not supported by evidence for appropriateness of the service for the specified individuals.
References:
The nurse practitioner’s scope of practice is the full range of practice privileges allowed by certification and licensures. Which is a true statement about the nurse practitioner’s scope of practice?
A. The Nurse Practice Act of each state will help define the NP scope of practice based on state legislation
B. Scope of practice refers to the clinical aspects of patient care
C. Scope of practice refers to national standards of care for clinical practice
D. All states require the signature of the NP and the collaborating physician for prescription of a controlled substance
Correct Answer: A.
The Nurse Practice Act of each state will help define the NP scope of practice based on state legislation
The nurse practitioner’s scope of practice represents the full range of practice privileges allowed by certification and licensure. Each state’s Nurse Practice Act will help to define an individual NP’s scope of practice based upon the legislation in that state.
Incorrect Answers:
B. Standards of practice refer to the clinical aspects of patient care.
C. National guidelines for clinical care may be incorporated into the standard of practice for a nurse practitioner. They are often recommendations by government agencies or professional organizations that are made after a review of research and the harms and benefits of a particular treatment.
D. NPs have the authority to prescribe Schedule II controlled medications in many states, but other states limit prescribing of controlled substances by NPs or may require cosignature by a collaborating physician.
Vital Concepts:
The nurse practitioner’s scope of practice represents the full range of practice privileges allowed by certification and licensure. Each state’s Nurse Practice Act will help to define an individual NP’s scope of practice based upon the legislation in that state.
References:
A 48-year-old male from a rural area develops progressive pruritus of the left arm, followed by paresthesias. He presents to the local emergency department complaining of chest and back pain. The patient receives a cardiovascular evaluation and is discharged home. He returns the following day with worsening chest pain, agitation, and mental confusion. At this visit, his fiancée reports that the patient was bitten by a bat on his left arm 4-5 weeks prior to symptom onset. He is given human rabies immune globulin (HRIG) and the first dose of a rabies vaccine. What should the patient be told regarding disease evolution?
A. Acute neurological phase (paralytic rabies) occurs in 40% of patients and presents with flaccid paralysis.
B. 80% of patients may develop an aggressive course with a fast progression.
C. Vocal cord paralysis is rare.
D. Nonneural tissues can be affected.
Correct Answer: D.
Nonneural tissues can be affected.
Variant forms of rabies may have a different course, requiring minimal inoculums and causing infection in nonneural tissues. The virus travels inwards from the peripheral nerves to infect the CNS. The time course of the onset of symptoms is dependent on the bite location, inoculum, and host immunity. Once the virus reaches the CNS, the patient will experience physical agitation, episodic confusion, psychosis, and combativeness.
Incorrect Answers:
A. Paralytic rabies occurs in 20% of patients.
B. The more aggressive and rapidly progressing form is furious rabies, and it is determined by psychosis, bizarre behavior, biting, anxiety, agitation, hydrophobia, and autonomic dysfunction.
C. Vocal cord paralysis often occurs.
References:
A physician has asked a nurse practitioner in another department to write orders for his patients. The nurse practitioner is not in practice with the physician and resents the request. Initially, she writes orders for several patients, but the requests become more frequent and she begins to ignore the physician’s telephone calls. Which of the following approaches to conflict resolution is the nurse practitioner now using?
A. Competition
B. Collaboration
C. Avoidance
D. Accommodation
Correct Answer: C.
Avoidance
There are emotional as well as rational reactions to conflict resolution. Emotional reactions include avoidance, competition, and giving up. Rational reactions include accommodation, compromise, and collaboration. The avoidance approach to conflict resolution results in both sides “losing.” The collaborative approach gives greater weight to achieving the goal of optimal workplace results than to effective concerns and results in a win-win situation.
Incorrect Answers:
A. Avoiding the physician means competition can’t be the approach
B. Not working with the physician means collaboration isn’t the approach
D. Not giving in to the physician’s contacts means accommodation isn’t the approach
Vital Concepts:
The avoidance approach to conflict resolution results in both sides “losing.” The collaborative approach gives greater weight to achieving the goal of optimal workplace results than to effective concerns and results in a win-win situation.
References:
A client presents to your office questioning the treatment of MAOI medications. What is true about using the selegiline patch?
A. It is transferred through the skin and is partially absorbed in the GI tract causing restrictions on tyramine in food choices
B. It is a MAOI medication and needs to have restrictions on tyramine in diet
C. Higher doses of selegiline patch can be used to treat Parkinson’s disease
D. It bypasses the gastric tract, so no need for dietary restriction
Correct Answer: D.
It bypasses the gastric tract, so no need for dietary restriction
MAOI users usually need strict dietary guidelines, but because of the delivery system in the patch, the digestive system is bypassed. For doses greater than 9 mg/24 hr, tyramine may need to be avoided in the diet due to limited safety data.
Incorrect Answers:
A. This route will bypass the GI tract, allowing no limitations at lower doses to food options, unlike the oral tablets.
B. It is an MAOI medication and needs to have restrictions on tyramine in the diet is incorrect as the patch bypasses GI tract.
C. Higher doses of selegiline patch can be used to treat Parkinson’s disease is incorrect. At lower doses, it is considered an MAOB and is used for Parkinson’s disease but at higher doses, it acts as an MAOI.
References:
An adult patient with schizophrenia is brought in for an evaluation in anticipation of involuntary commitment. Past records show that they have been hospitalized six times over the past year. The patient has been tried on haloperidol (Haldol) and several second-generation antipsychotics, including risperidone, aripiprazole, quetiapine, and olanzapine. He did not respond to a trial of depot risperidone and haldol decanoate. Which of the following would be an appropriate medication to trial next?
A. Clozapine (Clozaril)
B. Risperidone (Risperdal)
C. Thioridazine (Mellaril)
D. Chlorpromazine (Thorazine)
Correct Answer: A.
Clozapine (Clozaril)
The patient is showing frequent relapse due to treatment resistance to medications. Clozapine is an oral atypical antipsychotic that has strong evidence as being helpful for treatment-resistant psychosis. Alternatives include lamotrigine, topiramate, and minocycline. Nonpharmacological options include ECT and repetitive transcranial magnetic stimulation (rTMS).
Incorrect Answers:
B. This patient has already attempted risperidone orally and IM without success.
C. D. Another first-generation antipsychotic is unlikely to be helpful, as haloperidol has been tried twice with poor response.
Vital Concept:
Treatment-resistant schizophrenia is challenging. Clozapine is the medication that has proven to be most effective in these cases.
References:
Family therapy is a form of psychotherapy that seeks to reduce distress and conflict by improving interactions between family members. Which of the following is a technique used in family therapy?
A. Grief
B. Role transition
C. Role dispute
D. Role reversal
Correct Answer: D.
Role reversal
Role reversal is used in some forms of family therapy.
Incorrect Answers:
A. Grief or complicated bereavement is 1 of the 4 specific problem areas declared by individual interpersonal psychotherapy.
B. Role transition is 1 of the 4 specific problem areas declared by individual interpersonal psychotherapy.
C. Role dispute is 1 of the 4 specific problem areas declared by individual interpersonal psychotherapy.
References:
A 7-year-old child is brought in for evaluation due to impulsive behavior. During the chart review, the clinician sees that the child has an atrial septal defect. On examination, the child appears to have shortened fifth digits, microcephaly, and facial features such as a smooth philtrum and short palpebral fissures. They also appear to have a very thin upper lip and their height and weight are both below the 5th percentile. What is the most likely diagnosis?
A. Down syndrome
B. Williams syndrome
C. Prader-Willi syndrome
D. Fetal alcohol syndrome
Correct Answer: D.
Fetal alcohol syndrome
The key is to identify the facial anomalies in the premaxillary zone, which are characteristic of fetal alcohol syndrome. These anomalies include a flat upper lip, flattened philtrum, and flat midface. Fetal alcohol syndrome can also be associated with multiple other birth defects (cardiac, skeletal, renal, ocular, and auditory), as well as behavioral and learning problems.
Incorrect Answers:
A. Down syndrome facial features include upslanting palpebral fissures, a suggestion of epicanthal folds, and a flat nasal bridge.
B. Williams syndrome facial features include a broad forehead, a short nose with a broad tip, full cheeks, and a wide mouth with full lips.
C. Prader-Willi syndrome facial features include a narrow forehead, almond-shaped eyes, and a triangular mouth.
Vital Concept:
The physical and facial features described here are characteristic of fetal alcohol syndrome, which classically presents with short palpebral fissures, thin vermillion border, and smooth philtrum.
References:
A woman in her first trimester of pregnancy is experiencing an acute psychotic episode. Generally speaking, the latest studies have shown that both typical and atypical antipsychotics are safe to use in pregnancy. However, one antipsychotic still requires further study due to a small increase in risk to pregnant women. Which of the following medications is it?
A. Risperidone
B. Olanzapine
C. Aripiprazole
D. Quetiapine fumarate
Correct Answer: A.
Risperidone
The latest studies have found no increase in the risk of typical and atypical antipsychotics with any form of birth defects, except for risperidone. The small increase in risk for risperidone should be studied further. There is a likely possibility that this increase in risk is simply the result of chance alone and will require confirmation. Typical and atypical antipsychotics may be used in pregnancy, and withdrawing these medications during this period is not necessary.
Incorrect Answers:
B, C, and D. The latest studies have found that these are all safe to use in pregnant women.
References:
A patient writes a negative online review of his healthcare provider and refuses to pay the balance of the bill. A few months later, he develops acute influenza symptoms and calls the office to schedule an urgent appointment with his provider. Which of the following is an appropriate response by the healthcare provider?
A. Transfer care to another healthcare provider in the same group.
B. Ask a hospitalist to see the patient.
C. Inform the patient in writing that care will be provided only until an alternative provider is identified.
D. The provider cannot terminate the doctor-patient relationship without the patient’s agreement.
Correct Answer: C.
Inform the patient in writing that care will be provided only until an alternative provider is identified.
After accepting a patient into a medical practice, healthcare providers are under legal and ethical obligation to provide services to the patient as long as the patient requires those services. If circumstances render the healthcare provider unable to continue the relationship, he or she must give the patient reasonable notice and sufficient opportunity to make alternative arrangements for care. Failure to pay a bill does not end the healthcare provider-patient relationship, which is based on fiduciary responsibility, not financial responsibility. Written notice should be given, preferably by certified mail, with a brief explanation for termination of the relationship and an agreement to continue to provide treatment and access to services for a reasonable length of time (30 days) to allow the patient to secure care from another provider. The healthcare provider should also provide resources to help the patient locate another healthcare provider and offer to transfer the patient’s medical records after execution of a signed authorization.
Incorrect Answers:
A. and B. The patient needs to be given adequate notice and opportunity to make alternate arrangements for care — so these aren’t viable in the current situation.
D. False; the provider can terminate the relationship in writing.
Vital Concepts:
After accepting a patient into medical practice, healthcare providers are under legal and ethical obligation to provide services to the patient as long as the patient requires those services. If circumstances render the healthcare provider unable to continue the relationship, he or she must give the patient reasonable notice and sufficient opportunity to make alternative arrangements for care. Failure to pay a bill does not end the healthcare provider-patient relationship, which is based on fiduciary responsibility, not financial responsibility.
References:
Bupropion is a form of medication with multiple uses. Which is a true statement about bupropion?
A. Bupropion is a tricyclic antidepressant.
B. Bupropion modulates only norepinephrine.
C. Bupropion is associated with a lower rate of drug-induced seizures than other antidepressants.
D. Bupropion is often used for patients with ADHD and other comorbidities.
Correct Answer: D.
Bupropion is often used for patients with ADHD and other comorbidities.
Bupropion has been studied in small groups of adolescents with ADHD and substance use disorders, nicotine dependence, conduct disorder, and depression.
Incorrect Answers:
A. Bupropion is a unicyclic aminoketone and an antidepressant. It is also used in smoking cessation.
B. Bupropion modulates both norepinephrine and dopamine.
C. Bupropion is associated with a higher rate of drug-induced seizures than other antidepressants.
References:
Some nurses may choose to act as advocates at the state and federal level regarding healthcare-related policy. Which of the following is true regarding NPs functioning in this regard?
A. Nurse practitioners should remain focused on their patients and address issues within their own organizations.
B. Nurse practitioners should not confuse political advocacy with their professional role. Advocacy is not supported by professional nursing organizations.
C. Advocacy offers the nurse practitioner the chance to have broader control over patient care and outcomes.
D. Nurse practitioners should limit involvement in political advocacy for issues that are popular within their organization.
Correct Answer: C.
Advocacy offers the nurse practitioner the chance to have broader control over patient care and outcomes.
Nurses and NPs are often aware of the challenges faced by patients and providers in the quest for quality healthcare and improved outcomes. Issues that nurses often see include problems with access to services, health disparities, patient safety and satisfaction, and clinical outcomes. When NPs choose to become a policy advocate for change, they may be required to move out of their comfort zone of their practice area. However, this element of professional practice can reward them with the opportunity to make a positive change and participate in the creation of a better healthcare system.
Incorrect Answers:
A. Political advocacy often requires that nurse practitioners move out of their comfort zone and look beyond their own patients and their own organization.
B. Nurse practitioners often find political advocacy adds a new dimension to their professional identities. The ANA states that advocacy is “a pillar of nursing”.
D. Nurse practitioners may become involved in political advocacy for change that will result in better outcomes but should be aware that change often presents challenges. The nurse practitioner should approach any potential conflict thoughtfully and should communicate honestly within the healthcare organization.
Vital Concept:
The ANA considers legislative and political advocacy “a pillar of nursing” and is no less important than advocating for a patient.
References:
Patients receiving medical treatment have an expectation of confidentiality. Which of the following would constitute a breach of patient confidentiality?
A. Releasing medical records that have been subpoenaed
B. Releasing information to a patient’s spouse
C. Reporting an infectious disease to the health department
D. Releasing a patient diagnosis to his insurance company
Correct Answer: B.
Releasing information to a patient’s spouse
Patient confidentiality is breached when medical information is given to any person, including a spouse, without the patient’s permission. Patient consent is not required to release medical information to a health plan or insurance company that is paying for medical care; a third party or business associated hired by the insurance company or doctor’s office to assist in billing or collections; collection agencies for unpaid bills; health care operations; victims of abuse, neglect, or domestic violence; required reporting; legal subpoenas.
Incorrect Answers:
A. Patient consent is not required to release medical information in response to a legal subpoena.
C. Patient consent is not required to release medical information to a health department for purposes of required reporting.
D. Patient consent is not required to release medical information to a health plan or insurance company that is paying for medical care.
Vital Concept:
Patient consent is not required to release medical information to a health plan or insurance company that is paying for medical care
References:
Quetiapine is a second-generation antipsychotic agent that has been approved for a range of conditions. Which of the following is an off-label use?
A. Bipolar depression
B. Schizophrenia
C. Acute mania
D. OCD
Correct Answer: D.
OCD
Use of quetiapine to treat OCD is an off-label use.
Incorrect Answers:
A, B, and C. Quetiapine is FDA-approved for the treatment of these conditions.
References:
A nurse practitioner would like to study the effects of solitary confinement on prisoner suicide attempts. Which of the following is true?
A. No consent is necessary when studying prisoner behavior
B. Special protections and additional informed consent requirements are necessary when studying prisoners
C. A study of a small population, e.g., prisoners, is confounded by selection bias
D. A randomized controlled study is the most appropriate study
Correct Answer: B.
Special protections and additional informed consent requirements are necessary when studying prisoners
Informed consent is required for almost all types of biomedical and behavioral research in the United States. Some groups are considered “vulnerable populations” and have additional requirements for consent. Additional paperwork is typically required. These groups include infants and children under the age of 18 years; pregnant women; prisoners; persons at risk of suicide; and persons with impaired decision-making capacity (those with cognitive delay; the demented elderly). When performing research on human subjects, nurse practitioners should follow the principles in the Belmont report, a paper that outlines important ethical principles that should be followed when performing research involving human subjects. This paper was issued by a national commission of experts in 1978.
Incorrect Answers:
A. Informed consent is needed for almost all biomedical and behavioral research in the U.S., and because the prison population is a “vulnerable population” additional paperwork is needed
C. This isn’t true, as the U.S. prison population is large enough to create a large, random sample
D. Appropriate study design depends on the study aims
Vital Concepts:
Some groups are considered “vulnerable populations” and have additional requirements for consent. Additional paperwork is typically required. These groups include infants and children under the age of 18 years; pregnant women; prisoners; persons at risk of suicide; and persons with impaired decision-making capacity (the mentally retarded; the demented elderly).
References:
A physician is tired of patients arriving late for appointments and institutes a rule: patients who are late will only be seen for 10 minutes instead of the usual 15. The number of patients arriving late subsequently goes down. What type of learning has taken place?
A. Trial and error learning
B. Classical conditioning
C. Operant conditioning
D. Coincidental learning
Correct Answer: C.
Operant conditioning
Operant conditioning takes place when reinforcement or punishment is used to modulate the frequency of a particular behavior. In this case, timely arrival has been reinforced by a negative reinforcer (taking away time with a physician).
Incorrect Answers:
A. Trial and error learning refers to the type of action, not the mechanism by which learning occurs. A rat may pick the exit to the maze through trial and error, but when the rat is rewarded with freedom, this learning is reinforced through operant conditioning.
B. In classical conditioning, a conditioned stimulus elicits a conditioned response due to its proximity to an unconditioned stimulus and unconditioned response.
D. Much learning takes place secondary to the unexpected consequences of behavior, but coincidental learning is not a learning modality.
Vital Concept:
Conditioning in behavioral psychology is a theory that the reaction, called a response, to an object or event, called a stimulus, can be modified through learning, or conditioning.
References:
Persistent depressive disorder symptoms must be present for at least how long to meet the DSM-5-TR diagnostic criteria in adults?
A. At least 1 month
B. At least 12 months
C. At least 24 months
D. At least 5 years
E. At least 10 years
Correct Answer: C.
At least 24 months
What was referred to as dysthymia in DSM-IV is now termed persistent depressive disorder, which includes both chronic major depressive disorder and the previous dysthymic disorder. The DSM-5-TR diagnostic criteria include:
· A poor disposition characterizes this disorder, present most days for most of the day for 24 months or more in adults.
· This should not include any asymptomatic periods lasting longer than 8 weeks.
· The disposition may be cantankerous and crabby in teens and pediatric patients for 12 months or more.
· In addition to the poor disposition, at least two of the following co-occur:
changes in sleep, resulting in poor sleep or excessive sleep
a decreased sense of self-worth and self-regard
a sense that nothing good will happen and that the current situation will never improve
changes in food intake or interest, resulting in no desire to eat or eating significantly more than typical
a sense of being tired or worn out despite adequate rest or sleep, without much drive or vigor
poor decision-making or an inability to maintain focus
· The patient should not exhibit symptoms meeting the diagnostic criteria for a manic or hypomanic episode.
· The patient may exhibit symptoms meeting the diagnostic criteria for a major depressive disorder for 24 months. A separate diagnosis of MDE or MDD should also be documented in this case. A specifier should be added to the persistent depressive disorder of with persistent major depressive episode or with intermittent major depressive episodes, with/without current episode.
· The symptoms are not more appropriately due to another condition, such as a psychotic disorder (e.g., schizophrenia, schizoaffective DO, delusional disorder).
· The symptoms are not directly related to the use of a substance, medication, or pre-existing medical diagnosis or health concern
· The symptoms cause substantial anguish or drastically affect the patient’s ability to function professionally, socially, or otherwise
· The following specifiers may be added:
Mild/moderate/severe
With anxious distress
With atypical features
Early onset (prior to age 21)
Late onset (after age 21)
In partial/full remission
Incorrect Answers:
A. At least one month is incorrect as this does not meet the “At least 2-year requirement” of s/s that is required to have dx of PDD.
B. At least 12 months is incorrect as this does not meet the “At least 2-year requirement” of s/s that is required to have dx of PDD.
D. At least 5 years exceeds the amount of “At least 2-year requirement” of s/s that is required to have dx of PDD. The question asks for the “least” amount of required time.
E. At least 10 years exceeds the amount of “At least 2-year requirement” of s/s that is required to have dx of PDD. The question asks for the “least” amount of required time.
Vital Concept:
The condition involves a depressed mood that occurs for most of the day, for more days than not, and for at least 2 years (at least 1 year for children and adolescents). Major depression may precede persistent depressive disorder, and major depressive episodes may occur during persistent depressive disorder. Individuals whose symptoms meet major depressive disorder criteria for 2 years should be given a diagnosis of persistent depressive disorder with persistent major depressive episode.
References:
Schizophrenia risk increases when relatives have it. The closer the occurrence is in the family tree, the greater the risk of schizophrenia. When one parent is schizophrenic, what is the child’s risk of schizophrenia?
A. 40%
B. 50%
C. 13%
D. 6.5%
Correct Answer: C.
13%
Schizophrenia has a polygenetic mode of inheritance. The risk of schizophrenia increases in occurrence with greater closeness in the family tree. The lifetime risk in the general population is just below 1%. It is 6.5% in first-degree relatives of patients and rises to >40% in monozygotic twins of affected people. With a parent affected, the risk is about 13%. If both parents are affected, the risk is nearly 50%.
Incorrect Answers:
A. This is the risk of schizophrenia in monozygotic twins of schizophrenics
B. This is the risk of schizophrenia when both parents have schizophrenia
D. This is the risk of schizophrenia when a first-degree relative has schizophrenia
References:
The self-efficacy theory is part of the social-cognitive learning theory. Which of the following is true regarding self-efficacy theory?
A. People who are able to care for themselves are less likely to suffer from mental illness or to suffer from disorders in response to trauma.
B. People who feel they have control over their coping in response to traumatic events are less likely to suffer from disorders in response to trauma.
C. People who are independent are more efficient at dealing with stress.
D. People who live independently are less efficient at dealing with stress.
Correct Answer: B.
People who feel they have control over their coping in response to traumatic events are less likely to suffer from disorders in response to trauma.
Albert Bandura developed social Cognitive Therapy (SCT) in the 1960s. SCT emphasizes learning from social context with a reciprocal relationship between the person, behavior and environment. For example, how the patient’s social environment responded to a past behavior influences this patient’s reinforcement expectations for future behaviors. SCT is composed of six components including self-efficacy, expectations, reinforcements, observational learning, behavioral capability, and reciprocal determinism. Self-efficacy theory discusses the effects of patients’ confidence in their ability to do a behavior successfully. In order to change “self-efficacy,” people must give themselves the credit for coping well rather than giving credit to other people or factors. People who feel they have control over their coping in response to traumatic events are less likely to suffer from disorders in response to trauma.
Incorrect Answers:
A. People who are able to care for themselves are less likely to suffer from mental illness or to suffer from disorders in response to trauma is not a part of the self-efficacy theory
C. People who are independent are more efficient at dealing with stress is not a part of the self-efficacy theory.
D. People who live independently are less efficient at dealing with stress is not a part of the self-efficacy theory.
Vital Concept:
Social Cognitive Therapy (SCT) emphasizes learning from social context with a reciprocal relationship between the person, behavior and environment. Self-efficacy theory is part of SCT that discusses the effects of patients’ confidence in their ability to do a behavior successfully.
References:
An observational study is designed to examine the number of ambulatory patients who experience different degrees of orthostatic hypotension with quetiapine therapy versus those who are not on alpha-blocking therapies. Which statistical test is most appropriate for comparing these groups?
A. Student’s t-test
B. Paired t-test
C. Z-test
D. Chi-square test
Correct Answer: A.
Student’s t-test
A student’s t-test is used to compare the means of 2 independent sample populations.
Incorrect Answers:
B. A paired t-test is used to compare the means of 2 related (“paired”) sample populations. An example would be a twin study.
C. A z-test is similar to a t-test, but the z-test requires either a large sample or a known population variance.
D. A chi-square test is used to compare samples with non-measurable nominal/categorical variables (as opposed to ordinal or interval measurements).
References:
Patients taking selegiline may test urine positive for which drug?
A. Lysergic acid diethylamide (LSD)
B. Methamphetamine
C. Phencyclidine (PCP)
D. Methadone
Correct Answer: B.
Methamphetamine
Selegiline is metabolized by the body into l-methamphetamine and l-amphetamine. The drug’s package insert is meant to alert users that this is a likely side effect of being on this drug. If they test positive for the other drugs on the list, it is more likely a positive drug test. Selegiline is a substituted phenethylamine used for the treatment of early-stage Parkinson’s disease, depression, and dementia. In normal clinical doses, it is a selective irreversible MAO-B inhibitor. However, in larger doses it loses its specificity and also inhibits MAO-A. Other psychotropics that can cause a false positive for amphetamines are promethazine, chlorpromazine, bupropion, fluoxetine and trazodone.
Incorrect Answers:
A. Lysergic acid diethylamide (LSD). Amitriptyline, fluoxetine, sertraline can lead to false positives for LSD.
C. Phencyclidine (PCP). Thioridazine, mesoridazine, and venlafaxine can lead to false positives for PCP.
D. Methadone. Quetiapine, chlorpromazine, thioidazine, clomipramine, and diphenhydramine can lead to false positives for methadone.
Vital Concept:
Psychotropics that can lead to a false positive for amphetamines are selegiline, promethazine, chlorpromazine, bupropion, fluoxetine and trazodone.
References:
Gamma Hydroxybutyrate (GHB) withdrawal is rare. However, when it does occur, what are its earlier symptoms?
A. Anxiety, insomnia, tremor, confusion, nausea, and vomiting
B. Diaphoresis, extreme hypertension, and delirium
C. Hypothermia and seizures with delirium tremens-type picture
D. Fatigue, dysphoria, depression with loss of appetite, and trouble concentrating
Correct Answer: A.
Anxiety, insomnia, tremor, confusion, nausea, and vomiting
Gamma Hydroxybutyrate (GHB) withdrawal shares features of both alcohol withdrawal and benzodiazepine withdrawal and is rare due to the rapid elimination of the drug. Withdrawal may take place in certain chronic GHB users who consume the drug every 3-4 hours. Earlier symptoms may include anxiety, insomnia, tremor, confusion, nausea, and vomiting. Later symptoms include disorientation, hallucinations, agitation, and possibly combative behavior.
Incorrect Answers:
B. There is an associated autonomic instability that is similar to alcohol withdrawal, manifesting as tachycardia, hypertension, tremor, and diaphoresis.
C. Hypothermia and seizures with a delirium tremens-type picture are more consistent with ethanol withdrawal.
D. Fatigue, dysphoria, depression with loss of appetite, and trouble concentrating describes the withdrawal picture for MDMA.
References:
Group therapy can be used to encourage better adaptation to one’s environment. What form of group therapy is used in the pursuit of this goal?
A. Psychodynamic group
B. Cognitive-behavioral group
C. Supportive group
D. Day hospital group
Correct Answer: C.
Supportive group
This describes supportive group therapy.
Incorrect Answers:
A. The goal of a psychodynamic group is to reconstruct personality dynamics.
B. The goal of a cognitive-behavioral group is to relieve specific psychiatric symptoms.
D. The goal of a day hospital group is to reconstitute defenses.
References:
What lab would you order on a reproductive aged female patient prior to beginning Depakote?
A. Human chorionic gonadotropin
B. Creatinine
C. Potassium
D. Sodium
Correct Answer: A.
Human chorionic gonadotropin
Human chorionic gonadotropin is a hormone produced by the placenta after implantation, and HCG is tested to determine if a patient is pregnant. Before starting a medication that can have negative effects on a fetus, it is best to rule out pregnancy and have a discussion about pregnancy prevention during the period of time the medication is taken. Depakote (valproate) can cause major congenital malformations, particularly neural tube defects. Also before starting treatment should order a complete blood counts, liver function tests and coagulation tests.
Incorrect Answers:
B. Creatinine. Depakote is metabolized by the liver not the kidneys. It is not necessary to order a comprehensive metabolic panel (CMP) prior to starting Depakote.
C. Potassium. Depakote does not affect potassium levels. It is not necessary to order a comprehensive metabolic panel (CMP) prior to starting Depakote.
D. Sodium. Depakote does not affect sodium levels. It is not necessary to order a comprehensive metabolic panel (CMP) prior to starting Depakote.
Nursing practice has a theoretical basis. Which of the following is a true statement about nursing’s theoretic basis?
A. A relatively new approach to nursing care
B. Borrowed from other professions such as medicine
C. As old as formal nursing and began with Florence Nightingale
D. Unrelated to the conduct of nursing research studies
Correct Answer: C.
As old as formal nursing and began with Florence Nightingale
The theoretical basis for nursing practice began with Florence Nightingale and has been used for practice and research for over a century.
Incorrect Answers:
A. B. D. Nursing theories are specifically developed for nursing. These options are not correct.
Clomipramine is a tricyclic antidepressant (TCA) used to treat OCD, panic disorder, major depressive disorder, and chronic pain. Which of the following symptoms is typical of clomipramine toxicity?
A. Convulsions and coma
B. Acute dystonia, akinesia, and late tardive dyskinesia
C. Hypothyroidism and polyuria
D. Increased BP
Correct Answer: A.
Convulsions and coma
Clomipramine is a TCA and blocks the reuptake of norepinephrine or 5-HT.
Incorrect Answers:
B, C, and D. These symptoms aren’t characteristic of clomipramine toxicity.
The eclectic brief therapy of Budman and Gurman rests on the interpersonal, developmental, and existential (IDE) focus. Which of the following is not included in this method?
A. Losses
B. Developmental dyssynchronies
C. Interpersonal conflicts
D. Role models
Correct Answer: D.
Role models
The IDE perspective doesn’t include role models. It includes losses, developmental dyssynchronies, interpersonal conflicts, symptomatic presentations, and personality disorders.
Incorrect Answers:
A, B, and C. These are all included in the IDE perspective. The major focus of the IDE perspective includes losses, developmental dyssynchronies, interpersonal conflicts, symptomatic presentations, and personality disorders. The eclectic brief therapy of Budman and Gurman focuses on the reason that the patient seeks therapy now. A characteristic feature of their therapy is the belief that maximal benefit occurs early and is the opportune time for change.
A group of 2,000 people affected by a toxic chemical spill in 1965 was initially examined at a baseline for leukemia associated with the toxin exposure. They are examined every year for new cases of the disease. The group has been divided into subgroups based on the amount and type of exposure, and incidence is calculated for each year. What type of study design is this?
A. Prospective cohort study
B. Clinical trial
C. Case-control study
D. Retrospective cohort study
Correct Answer: A.
Prospective cohort study
This is a prospective cohort study, which follows similar individuals over time with respect to certain variables to determine how those variables will impact the rate of a certain outcome. Prospective cohort studies may have errors that result from patients who are lost to follow-up, but a prospective study generally has fewer sources of bias when compared to a retrospective cohort design. All cohort studies measure outcomes after exposure. They require large numbers of subjects to yield statistically significant rates of true incidence and relative risk.
Incorrect Answers:
B. A clinical trial is a study based on therapeutic intervention. It is designed to answer a specific question about the intervention and is always a prospective study.
C. A case-control study is usually retrospective and compares patients with a disease or condition to patients who do not have the disease or condition. Subjects are identified by known outcomes, and investigators look back to identify risk factors and exposures.
D. Retrospective cohort studies identify patients with a certain outcome and look back at exposure or risk factors to identify associations.
Vital Concepts:
This is a prospective cohort study, which follows similar individuals over time with respect to certain variables to determine how those variables will impact the rate of a certain outcome. Prospective cohort studies may have errors that result from patients who are lost to follow-up, but a prospective study generally has fewer sources of bias when compared to a retrospective cohort design.
A 60-year-old patient with schizophrenia is admitted to the intensive care unit (ICU) after experiencing the first seizure of his lifetime. He was recently discharged to a nursing home after spending over 20 years in a state mental hospital.
After consultation with the neurology and psychiatry services, the treating physician has decided the patient’s seizure was due to a supratherapeutic level of clozapine.
Which of the following changes in the patient’s lifestyle would most likely account for this seizure?
A. Recently starting St. John’s wort for depressive symptoms
B. Increased consumption of cruciferous vegetables as part of healthy living program
C. Smoking ban at nursing home
D. Underlying medical problem leading to seizure (not lifestyle change)
Correct Answer: C.
Smoking ban at nursing home
Tobacco use and smoking are potent inducers of CYP1A2 enzymes, which processes clozapine. At the state hospital, surprisingly, many individuals can smoke even while taking antipsychotics. The patient likely had a stable amount of clozapine in his system and was utilizing a higher amount of medication than what he would normally need if he had not used tobacco daily at the hospital.
Daily tobacco use induced enzymes, causing clozapine to be cleared more quickly and necessitating a higher dose of clozapine. New restrictions at his nursing home stopped his inducing agent (tobacco) and led to supratherapeutic levels of clozapine, causing his seizure. Clinicians must be very cautious when patients start or stop smoking, as it can drastically affect medication levels in the body.
Incorrect Answers:
A. St. John’s wort (Hypericum perforatum) is an inducing agent for CYP3A4 and CYP3A5. Substrates affected by this herbal supplement include alprazolam, amlodipine, atorvastatin, diazepam, cyclosporine, and many other commonly prescribed medications.
When St. John’s wort is taken with these medications, they will be metabolized more quickly and be less effective. In other words, there will be lower levels of the alprazolam, amlodipine, and atorvastatin when they are co-administered with St. John’s wort. However, this inducing agent does not affect the metabolism of clozapine.
B. Cruciferous vegetables induce the metabolism of CYP1A2. Increasing consumption of this inducing agent would lower clozapine levels in the patient’s system (i.e. clozapine is metabolized more quickly). This lower level of clozapine would make the patient more psychotic and not increase his risk for a seizure.
D. This is a distractor.
A 6-year-old boy is being treated for ADHD, combined type. His mother asks if he will always have these symptoms or if some of them will go away as he ages. Which of the following ADHD symptoms is most likely to persist into adulthood?
A. Impulsivity
B. Disorganization
C. Inattention
D. Hyperactivity
Correct Answer: C.
Inattention
Children diagnosed with ADHD, combined type, tend to exhibit fewer impulsive-hyperactive symptoms as they grow older. Inattention is the symptom which may persist (disorganization is considered a symptom of inattention). Oppositional behavior is not a symptom of ADHD.
Incorrect Answers:
A. Impulsivity in ADHD-diagnosed children tends to go down as they grow older
B. Disorganization is considered a symptom of inattention, not a symptom in itself
D. Hyperactivity in ADHD-diagnosed children tends to go down as they grow older
A 40-year-old AIDS patient has been taking ritonavir 600mg twice daily for the past 2 years since she was diagnosed. Since the patient’s diagnosis, she has felt isolated and depressed. She does not enjoy the same social activities as before her diagnosis and has felt the need to hide her condition from her family. Which of the following antidepressants should be used with caution in this patient?
A. Nefazodone
B. Tranylcypromine
C. Phenelzine
D. Sertraline
Correct Answer: A.
Nefazodone
Caution should be used in prescribing nefazodone for AIDS patients taking protease inhibitors (e.g. ritonavir, saquinavir), as nefazodone may increase the serum concentration of ritonavir. . Tranylcypromine, phenelzine, sertraline, and bupropion are not known to cause interactions with protease inhibitors.
Incorrect Answers:
B, C, and D. These medications aren’t known to cause interactions when taken with protease inhibitors.
The NP is caring for a patient in alcohol withdrawal. The NP knows the best way to interact with a patient experiencing withdrawals includes:
A. A cheerful tone and humor.
B. Short sentences and a matter-of-fact manner.
C. A loud voice to ensure that the patient can hear.
D. Full explanations in a soft voice.
Correct Answer: B.
Short sentences and a matter-of-fact manner.
The NP must ensure they communicate with a patient in withdrawal in a matter-of-fact manner using short sentences. This patient may be disoriented and anxious, so sentences must be kept short and to the point.
Incorrect Answers:
A. An overly cheerful tone and humor may lead to the patient feeling like the staff member is laughing at themm, which may lead to dysfunctional communication.
C. A loud voice may make the patient feel as though they’re being yelled at or chastised. Alcohol withdrawal does not affect a patient’s ability to hear.
D. A patient in withdrawal will typically not be able to follow extensive explanations, so sentences should be kept short and to the point.
Vital Concept:
he NP must ensure they communicate with a patient in withdrawal in a matter-of-fact manner using short sentences. This patient may be disoriented and anxious, so sentences must be kept short and to the point.
Although the onset of panic disorder (the first panic attack) usually occurs “out of the blue” during everyday activities, which of the following is one of the circumstances that has been correlated?
A. Prior fasting
B. Recently return from vacation
C. Sleep-deprivation
D. Separation from family
Correct Answer: D.
Separation from family
Panic disorders tend to begin with a panic attack that is seemingly out of the blue. This usually occurs when people are young adults (average is 30s) but can appear later. Although the person thinks the first panic attack is completely out of the blue and unrelated to the activity he or she was engaging in, there is often a correlation with some type of life event (including serious illness of a family/friend, an accident, loss or major change in relationship, or separation from family in some way like moving for a new job). It is also not uncommon for the first panic attack to occur postpartum or while abusing substances.
Incorrect Answers:
(A) Prior fasting. Prior fasting has not been correlated with onset of first panic attack. There is often a correlation with some type of life event; including serious illness of a family/friend, an accident, loss or major change in relationship, or separation from family.
(B) Recently return from vacation. Prior fasting has not been correlated with onset of first panic attack. There is often a correlation with some type of life event; including serious illness of a family/friend, an accident, loss or major change in relationship, or separation from family.
(C) Sleep-deprivation. Sleep-deprivation has not been correlated with onset of first panic attack. There is often a correlation with some type of life event; including serious illness of a family/friend, an accident, loss or major change in relationship, or separation from family.
Opiate use may have a range of consequences. Which of the following is a potential consequence of opiate use?
A. Irritability
B. Onset of hallucinations and paranoia
C. Significant appearance of lights in central visual field
D. Respiratory depression
Correct Answer: D.
Respiratory depression
Opiate use may result in respiratory depression. The remaining options describe the sequela of cocaine use, including hallucinations, paranoia, euphoria, increased energy, hypersexuality, and irritability. With heavy cocaine use, patients can experience a shower of lights in their central vision, as well as visual hallucinations of black dots on their skin and in the environment (coke bugs).
Incorrect Answers:
A. B. C. These are all potential consequences of cocaine use. Significant appearance of lights in the central vision field is a potential consequence of heavy cocaine use.
A 38-year-old female presents to your clinic wanting to become pregnant. She is concerned about conceiving while being on valproic acid. In addition to advising the patient to stop smoking and refrain from alcohol consumption, what is the most likely cause to ask the patient to possibly consider stopping valproate?
A. It can cause spina bifida.
B. It is the most common etiological agent for fetal hydantoin syndrome.
C. It may worsen seizures during pregnancy.
D. It is completely safe during pregnancy as long as the mother takes 0.4 mg of folate daily.
Correct Answer: A.
It can cause spina bifida.
Valproate and carbamazepine are known for being teratogenic. Specifically, they have been shown to be associated with neural tube defects, such as spina bifida. It is thought that the mechanism of the neural tube defects is related to folate antagonism. Therefore, it is advised to abstain from valproate during pregnancy. However, if that was not possible, it would be recommended to give the mother a significantly higher dose of folate daily during pregnancy.
Incorrect Answers:
B. Fetal hydantoin syndrome is characterized by short limbs and phalangeal hypoplasia, which has been associated with maternal use of Phenytoin during pregnancy. Valproate has not been shown to cause this syndrome.
C. Valproate is an antiepileptic, which means that it functions to dampen seizure activity. It is a known teratogen, which maybe the reason to terminate it during pregnancy. However, continuing Valproate should not worsen seizure activity during pregnancy.
D. Even though increasing folate supplementation during pregnancy when valproate is used has been shown to decrease the incidence of neural tube defects, it is still not completely safe to use it during pregnancy.
Plasma levels of lithium should be measured while patients are on lithium therapy. When is the best time to measure plasma levels of lithium during therapy?
A. 12 hours after last dose
B. 24 hours after last dose
C. 36 hours after last dose
D. 72 hours after last dose
Correct Answer: A.
12 hours after last dose
For an accurate plasma level of lithium during therapy, obtain the blood sample 12 hours after the last dose. This ensures that the steady-state concentration has been achieved and that absorption variations have been surpassed.
Incorrect Answers:
B. This is 2x too long to wait.
C. This is 3x too long to wait.
D. This is 6x too long to wait.
Vital Concepts:
For an accurate plasma level of lithium during therapy, obtain the blood sample 12 hours after the last dose.
Which of the following drugs is used to treat seasonal affective disorder (SAD) in a patient who wishes to avoid sexual dysfunction?
A. Fluoxetine
B. Clonidine
C. Buspirone
D. Bupropion
Correct Answer: D.
Bupropion
Bupropion is a better choice than an SSRI for a patient diagnosed with SAD who wishes to avoid certain side effects, such as sexual dysfunction.
Incorrect Answers:
A. Fluoxetine is an SSRI. It is used to treat SAD, but SSRIs do have side effects, including sexual dysfunction and weight gain.
B and C. Clonidine and buspirone are not first-line for treatment of SAD.
Vital Concept:
SSRIs are first-line for treatment of SAD. However, buproprion is a good choice for patients wishing to avoid sexual dysfunction associated with SSRIs.
A patient with schizophrenia is pregnant. What should she and her healthcare provider know about the interactions between schizophrenia and pregnancy?
A. Antipsychotics are more closely associated with fetal malformations and neonatal behavioral effects than mood stabilizers and benzodiazepines.
B. High rates of smoking, substance use disorders, obesity, and low socioeconomic status are related to poor prenatal care and outcomes in pregnant patients with schizophrenia.
C. Periods of high risk for fetus include third trimester of pregnancy and time of delivery.
D. Pregnancy involves increased symptoms for women with schizophrenia.
Correct Answer: B.
High rates of smoking, substance use disorders, obesity, and low socioeconomic status are related to poor prenatal care and outcomes in pregnant patients with schizophrenia.
High rates of smoking, substance use disorders, obesity, and low socioeconomic status are independently related to poor outcomes such as low birth rate and increased stillbirth.
Incorrect Answers:
A. Mood stabilizers and benzodiazepines are more closely associated with fetal malformations and neonatal behavioral effects than antipsychotics. There is a secondary effect of low folate intake with increased risk of neural tube defects with second-generation antipsychotics. Antipsychotics are not generally associated with fetal malformations.
C. Periods of high risk for the fetus include the first trimester of pregnancy and time of delivery. Pregnant women with schizophrenia rarely have early entry to prenatal care, so the first time of risk cannot readily be managed by coordinated obstetric and psychiatric care; drug withdrawal risk is highest at the time of delivery but can be managed.
D. Although pregnancy is a time of decreased symptoms, pregnant women with schizophrenia need coordinated obstetric and psychiatric care.
A 19-year-old college freshman presents to the emergency room after his parents came for a surprise visit but found him drowsy and disoriented. On exam, the patient has prominent miosis, bradycardia, slurred speech, impaired attention and memory, and drowsiness. What should be quickly administered?
A. Naltrexone
B. Methadone
C. Naloxone
D. Buprenorphine
Correct Answer: C.
Naloxone
Naloxone is the drug of choice for the acute treatment of opiate overdose. Patients addicted to opioids should be withdrawn gradually over time. Drugs often used for this include methadone, naltrexone, buprenorphine, and clonidine.
Incorrect Answers:
A. This is used to help prevent relapses into alcohol or drug abuse, but doesn’t reverse overdoses.
B. This is used to treat narcotic drug addiction through medication-assisted treatment, but doesn’t reverse overdoses.
D. This can be used to treat addiction to opioids, but doesn’t reverse overdoses.
Pediatric population use of psychotropic medications is closely regulated. Which of the following is true about this practice?
A. FDA only allows strict label usages of medications. If a medication is not approved for a specific age group, it cannot be prescribed.
B. Pharmacotherapy is an appropriate alternative to individual and family psychotherapy and educational and behavioral interventions.
C. Large doses of an individual medication for a single disorder is preferred over combined pharmacotherapy.
D. Standard usage of psychotropics for children in state custody may require a legal hearing, although emergency use is permitted.
Correct Answer: D.
Standard usage of psychotropics for children in state custody may require a legal hearing, although emergency use is permitted.
Consent to use psychotropic medications should be obtained from a parent or legal guardian after a thorough discussion of the indications, risks, and benefits of the medication with the patient and parent or legal guardian. If a child is in state custody, permission to utilize standard doses of psychotropic medications must be obtained from the state. In some cases, this may require a legal hearing.
Incorrect Answers:
A. The FDA does allow off-label use of medication and use of medication in age groups not studied.
B. Pharmacotherapy is part of a comprehensive treatment plan, including individual and family psychotherapy and educational and behavioral interventions.
C. Combined pharmacotherapy often achieves more targeted treatment and efficacy with fewer side effects and lower doses.
Vital Concept:
Consent for children’s psychotropic medications should be obtained from the parent, legal guardian, or from the state (if in state custody).
References:
Schizophrenia is characterized by distorted and bizarre thoughts, perceptions, emotions, movements, and behavior with positive and negative symptoms. Which of the following epidemiological statements regarding schizophrenia is true?
A. Females are often diagnosed in their late 20s.
B. More women than men are diagnosed.
C. Negative and deficit symptoms are predominant in females.
D. Social functioning is better in men.
Correct Answer: A.
Females are often diagnosed in their late 20s.
Peak ages of diagnosis are early to mid-20s (18-25) for males and late 20s/early 30s (25-35) for females. An episodic course is more common in females. Long-term prognosis is better for women than it is for men. Men have poorer premorbid functioning, social functioning, educational achievements, and outcomes. Men also have more negative symptoms and cognitive decline, while women tend to display more mood and psychotic symptoms. Women also tend to experience a second peak of symptoms (especially psychotic) later in life that may be related to hormonal shifts during menopause.
Incorrect Answers:
B. Schizophrenia has roughly equal prevalence in males and females, but early onset is seen in males with poor prognosis, and late onset is seen in females with good prognosis. The prevalence is 1.4:1 (men:women) worldwide.
C. Negative and deficit symptoms are predominant in males.
D. Social functioning is better in females.
Vital Concept:
Men have an earlier schizophrenia onset than women and a poorer prognosis.
References:
A nurse practitioner is beginning a new practice and considering the purchase of malpractice insurance. The insurer tells the NP they can get a better price on a “claims-made policy.” Which of the following is true of this type of malpractice insurance?
A. If the NP drops her coverage and is sued for an event that happened during the period of coverage, she will be covered
B. The NP will be covered for all “claims made” by any patient seen during the period of coverage
C. A “tail” policy is recommended if the NP changes coverage
D. This policy is unaffected by job changes and retirement
Correct Answer: C.
A “tail” policy is recommended if the NP changes coverage
“Claims made” coverage refers to malpractice insurance that only covers claims if the NP is enrolled with the same insurance company at the time the claim is filed in court. If an NP drops coverage and is sued for an event that happened during the period of coverage, the NP will not be covered unless he or she has purchased a “tail coverage.” If the NP has claims-made coverage and decides to change insurers or to retire, it is advisable to buy “tail coverage” insurance that will cover the NP for any future claims filed against him or her that result from patient encounters during the period of coverage. The other type of malpractice insurance is “occurrence” insurance. An occurrence-based policy is unaffected by job changes or retirement. If an NP had an occurrence-based policy during the time of a patient encounter that later resulted in a lawsuit, the policy would cover that claim. No tail is necessary with occurrence coverage.
Incorrect Answers:
A. The NP won’t be covered if she drops her coverage, unless she has purchase a “tail coverage”
B. The NP won’t be covered for future claims made with this type of coverage
D. An occurrence-based policy is unaffected by job changes and retirement; but this policy is affected
Vital Concepts:
If an NP drops coverage and is sued for an event that happened during the period of coverage, the NP will not be covered unless he or she has purchased a “tail coverage.” If the NP has claims-made coverage and decides to change insurers or to retire, it is advisable to buy “tail coverage” insurance that will cover the NP for any future claims filed against him or her that result from patient encounters during the period of coverage.
References:
A 17-year-old female patient is intellectually disabled. Her mother brings her to your clinic and demands Norplant contraceptives be placed. The patient refuses. Which of the following should you do in this situation?
A. Implant the Norplant with the mother’s consent
B. Respect the patient’s wishes
C. Obtain a court order
D. Call child/adult protective services
Correct Answer: B.
Respect the patient’s wishes
Explain the risks and benefits of long-acting contraceptives and the risks of engaging in unprotected sex. If the patient still declines the contraceptive, her wishes should be respected. There are several areas in which minors have rights to treatment or to refuse treatment. These areas include contraception, abortion, prenatal care, drug, alcohol, and mental health services. Intellectual disability and other limitations of decision making capacity, are not an absolute indication to deny the patient her reproductive rights, particularly when there is no substantial evidence to suggest that harm will come to the patient or to her child.
Incorrect Answers:
A. The patient’s wishes should be respected — minors have the right to treatment and to refuse treatment in contraception
C, D. Neither is warranted without substantial evidence to suggest harm will come to the patient
Vital Concepts:
Explain the risks and benefits of long-acting contraceptives and the risks of engaging in unprotected sex. If the patient still declines the contraceptive, her wishes should be respected.
References:
A nurse practitioner designs a study of the effect of different exercise programs on mood. She selects 100 clients who sign up at a local wellness clinic with 1000 members for one of three exercise programs. The nurse plans to survey the participants before the start of the program and again at the end of the program. Which of the following factors is most likely to influence the results of this study in a way that does not reflect the actual difference in the exercise programs?
A. Anonymity
B. Randomization
C. Bias
D. Sample size
Correct Answer: C.
Bias
The clients are not randomized to different samples, so bias can occur, primarily as a result of self-selection. As an example, clients who are more gregarious may choose a group exercise class. The difference seen in their moods may be a result of differences in their personalities, not the result of the specific effects of the type of exercise.
Incorrect Answers:
A. Classes aren’t anonymous.
B. Classes aren’t randomized.
D. Sample size is sufficient at 10% of the population.
Vital Concepts:
The clients are not randomized to different samples, so bias can occur, primarily as a result of self-selection.
References:
Certain medications affect the efficacy of oral contraceptives.
Which drug only affects oral contraceptives at high doses of 200 milligrams or more?
A. Carbamazepine
B. Topiramate
C. Lithium
D. Oxcarbazepine
Correct Answer: B.
Topiramate
At dosages < 200mg, topiramate as daily monotherapy in healthy volunteers did not have a significant interaction with oral contraceptives but did at 200 mg or more.
Incorrect Answers:
A. Carbamazepine is associated with neural tube defects (1% of cases) and neurodevelopmental problems. Carbamazepine is associated with decreased levels of oral contraceptives.
C. Lithium does not decrease the efficacy of oral contraceptives, but should be avoided in patients who are trying to get pregnant due to the risk posed to the developing fetus (see chart below). Lithium is associated with Ebstein’s abnormality and other cardiovascular defects, as well as neonatal goiter. Lithium should be avoided in the first trimester if possible.
D. Oxcarbazepine may decrease the serum concentration of oral contraceptives leading to contraceptive failure.
References:
A 20-year-old college student presents to the emergency room with a 2-week history of auditory hallucinations and paranoia. They have no prior psychiatric history, no family psychiatric history, and no known medical problems. The patient admits to heavy daily marijuana use over the past 6 weeks, including immediately prior to presentation at the emergency room. They deny any other alcohol or substance use. They grow the marijuana themself in their dorm room. What is the most likely DSM-5-TR diagnosis?
A. Cannabis-induced psychotic disorder
B. Psychotic disorder NOS
C. Cannabis use disorder, in withdrawal
D. Schizophreniform disorder
Correct Answer: A.
Cannabis-induced psychotic disorder
In the DSM-5-TR, substance-induced mental disorders are named by the causative agent. The diagnostic criteria for psychosis related to a substance include that the patient must have at least one of the following symptoms:
illusions or perceived experiences that do not exist (e.g., sounds, voices, smells, visions, feelings, etc.)
a misconception, belief, or thought that is firmly held despite not being grounded in reality
The patient’s symptoms must become apparent in the midst of or following withdrawal or intoxication related to a substance or following the administration of a medicine.
The implicated medicine/substance has been known to cause the patient’s signs and symptoms
Symptoms cause substantial dysfunction in at least one environment (work, home, social settings)
The patient’s symptoms are not due to a more appropriate psychiatric condition, as evidenced by the satisfaction of the following features:
initial symptom presentation prior to substance exposure
symptom duration for longer than expected (e.g., 4 weeks)
a past episode of similar symptoms not associated with substance use
Symptoms are not only present with acute delirium
If the hallucinations/delusions are mild and clinically insignificant, a diagnosis of substance intoxication or withdrawal may be more clinically appropriate
Incorrect Answers:
B. The DSM-5-TR does not include the psychotic disorder NOS diagnosis.
C. Withdrawal from cannabis is not typically associated with psychotic symptoms.
D. Schizophreniform disorder is not diagnosed when the cause of symptoms is related to a substance.
Vital Concept:
The diagnosis of psychosis related to a substance requires that the symptoms present during intoxication or withdrawal of the implicated substance.
References:
More than 234,000 women receive a diagnosis of breast cancer annually in the United States. Research has identified risk factors that can be modified, including post-menopausal obesity. Some women with certain risk factors appear to benefit from the use of medications known as aromatase inhibitors, with some studies demonstrating almost a 50% reduction in the incidence of invasive breast cancer in high-risk women who use these medications. Which of the following is a widespread secondary prevention strategy for breast cancer?
A. Increased exercise
B. Weight loss
C. Radical mastectomy
D. Screening mammograms
Correct Answer: D.
Screening mammograms
Secondary prevention strategies refer primarily to detection of a disease early in its course, leading to early treatment and improved outcomes. Secondary prevention strategies are used to identify and treat asymptomatic individuals who do not yet show clinical signs or symptoms of a disease. Secondary strategies also refer to identification and treatment of individuals with certain risk factors. Screening mammography is an example of a secondary prevention strategy for breast cancer.
Primary prevention strategies are designed to prevent the onset of a particular condition or disease. Examples of primary preventive strategies include weight loss in obese women to reduce the risk of development of breast cancer. Tertiary prevention strategies are strategies of management of existing diseases or conditions that are designed to restore the individual with the disease to optimal function, to reduce disease related complications, and to eliminate or reduce negative consequences of a disease.
Radical mastectomy, when indicated, is a tertiary prevention measure against spread or recurrence of breast cancer.
Incorrect Answers:
A. This is a healthy lifestyle habit.
B. A primary prevention strategy only for obese women.
C. Tertiary prevention measure.
Vital Concepts:
Secondary prevention strategies refer primarily to detection of a disease early in its course, leading to early treatment and improved outcomes. Secondary prevention strategies are used to identify and treat asymptomatic individuals who do not yet show clinical signs or symptoms of a disease.
References:
A 4-year-old presents with a foster parent. The child was initially placed into foster care due to severe neglect at age 2. The child has been in five different foster families since that time. The current foster parent describes the child as “emotionally cold,” stating that the child avoids all physical contact with them and is extremely resistant to being comforted when upset. The child has been in the current foster home for 4 months. What is the correct diagnosis?
A. Autism spectrum disorder
B. Selective mutism
C. Reactive attachment disorder
D. Disinhibited social engagement disorder
Correct Answer: C.
Reactive attachment disorder
This child has a classic history for and displays several features of reactive attachment disorder (formerly reactive attachment disorder, inhibited type), including repeated changes in caregiver, history of neglect, avoidance of social interaction, and resistance to comforting from others. The diagnostic criteria for reactive attachment disorder:
The symptoms described must be obvious prior to age 5, and the patient’s development age must be 9 months or older
Actions that indicate a lack of attachment to caregiving adults, as evidenced by both of the following:
· the child does not consistently want to be comforted by the caregiver when upset
· the child does not react to being comforted by the caregiver when upset
A consistent interpersonal or psychological dysfunction, as evidenced by two or more of the following:
· minimal display of positive emotional response
· periods of cantankerous mood, unhappiness, or fear when around caregiving adults without provocation
· limited response to people on a social or psychological level
Historically, the child has endured poor care that preceded the symptoms described, such as one or more of the following:
· an inability to form a solid relationship with caregiver(s) due to recurrent switches in primary caregivers
· caregiving adults that failed to meet the child’s emotional needs (e.g., love, consolation, encouragement)
· being raised in an institutional or otherwise non-conventional environment that significantly prevents selective attachment (e.g., too few caregivers available for the number of children
The child does not meet the established criteria for autism spectrum disorder (ASD)
If symptoms have persisted for at least a year, the disorder is considered persistent
If all symptoms can be identified in one patient, the disorder is considered severe
Incorrect Answers:
A. There are no indications of developmental delay related to communication, obsessive interests, or repetitive behaviors that would raise suspicion of autism.
B. There’s no mention of difficulty speaking in select social settings, which is what selective mutism is.
D. Disinhibited social engagement disorder presents with excessive indiscriminate sociability and lack of selectivity in attachment figures.
Vital Concept:
Reactive attachment disorder is characterized by actions that indicate a lack of attachment to caregiving adults, as evidenced by the child not consistently wanting to be comforted or not reacting to being comforted by the caregiver when upset.
References:
A 65-year-old woman presents to your office and throughout the appointment, the patient expresses quite a bit of anger and bitterness. She is regretful of her past and has alienated herself. According to Erikson, which developmental stage is this patient struggling with?
A. Integrity vs. Despair
B. Intimacy vs. Isolation
C. Generativity vs. Stagnation
D. Industry vs. Inferiority
Correct Answer: A.
Integrity vs. Despair
This patient is struggling with integrity vs. despair which is evidenced by her anger, bitterness, regret, and alienation.
Incorrect Answers:
B. A patient struggling with this would be emotionally isolated. However, this stage doesn’t explain the patient’s anger, bitterness, and regret.
C. This relates to work and family. It doesn’t relate to this patient’s current state.
D. This relates to learning the pleasure of applying oneself to tasks. It doesn’t relate to this patient’s current state.
Vital Concept:
Fulfillment of integrity vs despair is evidenced by comfort with life, willingness to face death, insight and balanced perspective on life’s events. Those struggling with the developmental task of Intimacy vs. isolation results in emotional isolation, this is not the correct answer because of the other attributes of this patient including, anger, bitterness, and regret. Choices C and D are not appropriate for this patient.
References:
A patient has recently admitted to being an alcoholic and has been attending the same group therapy weekly for about six weeks. Several of the group members frequently discuss their family genetics and generational patterns involved in substance abuse. The patient has never known much about their family history and has spent dozens of hours in the library and online over the last month researching their family’s lineage and historical background. They proudly announce a recent discovery that they likely descend from Egyptian royalty. This patient is demonstrating which of the following higher-level defense mechanisms?
A. Denial
B. Rationalization
C. Intellectualization
D. Sublimation
Correct Answer: C.
Intellectualization
This patient is demonstrating intellectualization. They’re attempting to avoid dealing with their alcoholism by studying their family heritage. Intellectualization is characterized by excessive thinking and analyzing as a way of avoiding discussing and dealing with emotions and feelings.
Incorrect Answers:
A. The patient isn’t trying to ignore a factual reality by focusing internally, which would be denial. Denial is also considered a primitive defense mechanism, not higher level.
B. The patient isn’t trying to use faulty logic to explain their behavior or reactions, which would be rationalization.
D. The patient isn’t demonstrating sublimation by trying to transform a socially unacceptable impulse (drinking alcohol) into a socially acceptable action or behavior (focusing on hydration).
Vital Concept:
This patient is using thinking and analyzing as a defense mechanism to distance himself from his feelings and emotions, termed intellectualization.
References:
A patient with a history of liver disease and chronic alcohol use disorder requires treatment with a benzodiazepine for withdrawal. What benzodiazepine is preferred in a patient with this type of medical history?
A. Chlordiazepoxide
B. Clonazepam
C. Carbamazepine
D. Oxazepam
Correct Answer: D.
Oxazepam
In general, long-acting benzodiazepines with active metabolites (e.g. diazepam and clonazepam) are preferred due to a smoother course with less chance of recurrent withdrawal or seizures. However, their metabolites stay longer in the liver. These drugs should not be used in patients with liver disease. Intermediate- and short-acting benzodiazepines, including lorazepam and oxazepam, are often used in elderly patients or those with liver damage, as they are renal excreted and lack metabolites. Lorazepam or oxazepam is preferred for the treatment of patients with advanced cirrhosis or acute alcoholic hepatitis due to the shorter half-life of lorazepam and the absence of active metabolites with oxazepam.
Incorrect Answers:
A. Chlordiazepoxide has a long half-life and is a cost-effective medication for treating alcohol withdrawal, but it is metabolized by the liver. As a result, it is used rarely in patients with impaired liver function or in the elderly.
B. Clonazepam should be avoided in this patient.
C. Carbamazepine’s role in treating alcohol withdrawal has yet to be delineated fully.
References:
A 70-year-old female undergoes a biopsy of a nodule. Her daughter asks about the results of the biopsy so she can “prepare” her mother for bad news in the event of a malignancy. She is concerned that her mother will become depressed, and she plans to enlist her mother’s pastor to “provide some inspiration.” What is the appropriate response?
A. Consider the patient’s spiritual beliefs when determining how to share the news of a serious illness.
B. Take advice from concerned family members when determining how to deliver concerning test results to an elderly patient.
C. Tell the patient about her biopsy results unless she has requested otherwise and has provided written consent for release of information.
D. Tell the daughter that you must inform her mother first, but you will be happy to discuss the results with her afterward.
Correct Answer: C.
Tell the patient about her biopsy results unless she has requested otherwise and has provided written consent for release of information.
All medical information (like results of laboratory testing or a biopsy) must be delivered to the patient, not to the family. Unless the patient has provided specific instructions to the contrary, the family should never receive the patient’s confidential medical information. The only exceptions are when a patient is not competent or when the patient has a psychiatric disturbance that may be exacerbated by news of a serious medical condition, resulting in a suicide attempt. Therapeutic privilege refers to withholding relevant health information from a patient if disclosure of the information is medically contraindicated, but the therapeutic privilege is rarely invoked, and consideration of therapeutic nondisclosure should only occur after consultation with a colleague. A thorough review of the risks and benefits to the patient and an ethical justification of non-disclosure are required.
Incorrect Answers:
A. A patient’s spiritual beliefs have no bearing on the obligation to deliver all medical information to the patient unless the patient has provided specific instructions to the contrary, is incompetent, or has a psychiatric disturbance that might be exacerbated by news of a serious medical condition.
B. and D. All medical information (such as biopsy results) must be delivered to the patient, not the family unless the patient has provided specific instructions to the contrary, is incompetent or has a psychiatric disturbance that might be exacerbated by news of a serious medical condition.
Vital Concepts:
All medical information (like results of laboratory testing or a biopsy) must be delivered to the patient, not to the family. Unless the patient has provided specific instructions to the contrary, the family should never receive the patient’s confidential medical information. The only exceptions are when a patient is not competent or when the patient has a psychiatric disturbance that may be exacerbated by news of a serious medical condition, resulting in a suicide attempt.
References:
A nurse practitioner is treating an adolescent patient with depression. During the encounter, the patient confides that his girlfriend has been cheating and he is going to kill both his girlfriend and her lover. He tells you he has been carrying his gun in his car for three days, driving by their home. What should the NP do?
A. Talk to the patient about the potential consequences for him and his family if he carries out his plan
B. Call clinic risk management and ask them to intervene
C. Do not break the patient’s confidentiality, but you can suggest a cooling-off period with a follow-up appointment the next day
D. Keep the patient in your office and have a nurse call the police and the intended targets; the police will arrive and take over
Correct Answer: D.
Keep the patient in your office and have a nurse call the police and the intended targets; the police will arrive and take over
Although the right to confidentiality is a critical ethical concept in medicine, it is not an absolute right. When the possibility of harm to an innocent party exists, the health care provider must report the threat to the person and to the authorities. The duty is not only to report but to act to prevent harm. This may include keeping the patient in the office until authorities arrive, or warning the threatened person.
If a clinician obtains information from a patient that an identified victim is at risk, it is the duty of the healthcare personnel to warn the intended victim, even if it will break confidentiality (Tarasoff I). Tarasoff II extended this mandate to a duty to include a duty to protect that supersedes the duty to warn, which means that a warning may not by itself be sufficient to protect the patient. The healthcare staff should call the police and should also try in every way to notify the potential victim of danger. The patient who is making the threat should be detained, then the police must be called. Finally, notification and warning of the potential victim should occur after these protective measures are in place. All three actions should be attempted.
Incorrect Answers:
A. This is inadequate to protect the intended victim.
B. The intended victim AND the law enforcement authorities should be notified.
C. Confidentiality is not absolute if an innocent victim is in harm’s way.
Vital Concepts:
Although the right to confidentiality is a critical ethical concept in medicine, it is not an absolute right. When the possibility of harm to an innocent party exists, the health care provider must report the threat to the person and to the authorities.
References:
A specific brain system is primarily responsible for sensory integration. Which part is it?
A. Medulla
B. Occipital lobe
C. Frontal lobe
D. Parietal lobe
Correct Answer: D.
Parietal lobe
The temporal lobes are the key areas of the brain for hearing, naming, and visual recognition. The frontal lobe is primary in executive function, social conduct, judgment, insight, and some motor function. The occipital lobe is vital for visual functions. The parietal lobes are key areas for sensory integration and somatosensory function.
Incorrect Answers:
A. Medulla - Responsible for autonomic (involuntary) functions
B. Occipital lobe- Responsible for visual functions
C. Frontal lobe-Responsible for executive function, social conduct, judgment, insight, and some motor function
References:
To estimate the prevalence of major depression in a large metropolitan area, a representative sample of people is selected and surveyed about their psychiatric history. What is this type of study?
A. Case-control study
B. Cross-sectional study
C. Randomized controlled trial
D. Cohort study
Correct Answer: B.
Cross-sectional study
Cross-sectional studies analyze a population (or representative subset) at a single point in time (a cross-section) to assess disease incidence and prevalence.
Incorrect Answers:
A. Case-control studies analyze a known outcome by looking back in time to assess exposure.
C. An RCT is an experiment/clinical trial that studies the effect of an intervention by comparing the treatment to a placebo in 2 equivalent groups separated by randomization.
D. A cohort study follows a group of similar individuals (a cohort) without a disease over time to determine the risk of developing the disease.
References:
Which of the following is a recognized subtype of specific phobia in the DSM-5-TR?
A. Food-related
B. Social
C. Clown
D. Natural environment
Correct Answer: D.
Natural environment
Specific phobia is diagnosed when there is fear about a specific situation/object and that stimulus always provokes an immediate response. Criteria for diagnosis include
· Terror or significant concern about a certain condition or item (small spaces, spiders, heights).
· The terror or concern is consistent for at least 6 months.
· The patient evades the condition or item.
· The terror/concern or active evasion of the condition or item leads to dysfunction (academic, professional, social, or otherwise) or considerable anguish.
· The patient reports sudden terror or significant concern almost every time they are presented with the certain condition or item.
· The actual risk or threat posed by the condition or item is insignificant compared to the patient’s emotional response and concern.
· The patient’s symptoms are not due to a more appropriate psychiatric condition such as panic attacks (i.e., panic disorder), past trauma (i.e., post-traumatic stress disorder), gaining weight (i.e., eating disorders), social interactions (i.e., social anxiety disorder), separation from a loved one (i.e., separation anxiety disorder), or a recurrent thought (i.e., obsessive-compulsive disorder).
· In pediatric patients, phobias may manifest as dependence, immobility, outbursts, or fits.
Specific subtypes include:
animal (fear that relates to animals or insects)
natural environment (fears of things like heights, lightning, or bodies of water)
blood-injection injury (fear of blood, medical procedures, needles, etc.)
situational (fear of situations such as being in an elevator, tight spaces, driving, or airplanes.)
other (fear of choking or vomiting, loud sounds, costumed characters)
Incorrect Answers:
A. Food-related is not a subtype of specific phobia.
B. Social is not a subtype of specific phobia. Social anxiety disorder is a separate diagnosis.
C. Clown is not a subtype of specific phobia, but could be considered an ‘other’ subtype.
Vital Concept:
Subtypes of specific phobia in the DSM-5-TR include animal, natural environment, blood-injection injury, situation, and other.
References:
Which of the following age groups constitutes the most rapidly growing segment of the US population?
A. 0-5 years
B. Under 18 years
C. 18 to 25 years
D. Greater than 65 years
Correct Answer: D.
Greater than 65 years
The 2010 census showed that the population 65 and older is the largest in terms of size and percent of the population. There were 40.3 million people over the age of 65, representing an increase of 5.3 million since the 2000 census. The population aged 65 and older grew faster than the total population, growing 15.1 percent, compared to total population growth in the US of 9.7 percent. Within the older population, 85- to 94-year-olds experienced the fastest growth between 2000 and 2010, 29.9 percent, representing an increase from 3.9 million to 5.1 million.
Incorrect Answers:
A. B. and C. These groups are growing more slowly than the 65+ population.
References:
Which of the following patients has the highest risk of death by suicide?
A. 17-year-old Hispanic male
B. 80-year-old White male
C. 24-year-old African American female
D. 75-year-old White female
Correct Answer: B.
80-year-old White male
Older people, particularly Caucasian men >75, have the highest rates of suicide in the U.S. In fact, white males ages 85 and older have the highest suicide rate of any age and ethnic group. Nearly 70% of older suicide victims visit their primary care physician within the month of their death, and many did not tell their doctors they were depressed and were not screened. Men are more likely to die by suicide than women, but women are more likely to attempt suicide. Without looking at age, American Indians and Alaska Natives tend to have the highest rate of suicides, followed by non-Hispanic Whites. Hispanics have the lowest rate of suicide, and African Americans have the second lowest rate.
Incorrect answers:
(A) 17-year-old Hispanic male. Elderly people are at greater risk for suicide versus a 17-year-old. Hispanics have the lowest rate of suicide among ethnic groups.
(C) 24-year-old African American female. African Americans have the second lowest suicide rate among ethnic groups. Men have a greater risk than women. Elderly people are at greater risk for suicide versus a 24-year-old.
(D) 75-year-old White female. Men are more likely to die by suicide than women.
References:
Which of the following is true about receiving gifts from patients?
A. Do not accept a gift from a patient under any circumstances.
B. Regardless of the gift or its cost, accept the gift to avoid damaging the therapeutic alliance.
C. Never accept a cash gift.
D. Avoid discussing gifts, as this may lead to resistance.
Correct Answer: C.
Never accept a cash gift.
A therapist should never accept money from a patient (except as previously agreed upon in a payment schedule).
Incorrect Answers:
A. If the gift is inexpensive or handmade, it is generally appropriate to accept it.
B. Expensive gifts or inappropriate gifts should not be accepted. A small gift card may be acceptable. Fifty dollars is typically the limit, but it depends on the institution’s policy.
D. Gifts should always be discussed to explore the motivation of the patient and any significance with respect to transference.
References:
Lithium is associated with a risk of Ebstein’s anomaly. What structure if malformed in this condition?
A. Aortic valve
B. Tricuspid valve
C. Mitral valve
D. Pulmonary valve
Correct Answer: B.
Tricuspid valve
Ebstein’s anomaly is a malformation of the tricuspid valve.
Incorrect Answers:
A, C, and D. These structures aren’t malformed in Ebstein’s anomaly.
References:
A patient presents to the emergency room with complaints of chest pain. While the emergency provider is assessing her he determines the patient was experiencing a panic attack. The PMHNP is consulted, while interviewing this patient it is determined she has been extremely anxious because her parents have learned about her high number of sexual partners. The patient further discloses that she has had abnormal vaginal discharge. What should be the PMHNP next step?
A. Complete a pelvic exam
B. Have her parents come to the ER
C. Have her notify her sexual partners
D. Notify the emergency provider of the new findings
Correct Answer: D.
Notify the emergency provider of the new findings
The PMHNP should avoid highly personal or intrusive procedures that may make the formation of a therapeutic alliance more difficult. They should be able to recognize between normal and abnormal exams. The age of the patient is not discussed and therefore it is uncertain the legal implications of notifying the parents. Having her notify her sexual partners is not necessary yet as a diagnosis of an STD has not been established. Notifying the emergency provider of the new findings would be the next best step as they would be the one to complete the pelvic exam.
Incorrect Answers:
A. The emergency provider will complete the pelvic exam.
B. It’s not clear what the patient’s age is, so not clear if parents need to be notified.
C. Notifying sexual partners isn’t needed unless an STD diagnosis is established, which it hasn’t been yet.
References:
A 50-year-old man with chronic alcohol use disorder ran out of alcohol and drank methyl alcohol from the factory where he is employed. Ingestion occurred 18 hours prior to presentation. He was brought to the ER by family members with encephalopathy, ataxia, seizures, and bilateral vision loss. Imaging reveals an intracranial hemorrhage. What is the likely location of his hemorrhage?
A. Cerebellum
B. Occipital lobe
C. Temporal lobe
D. Putamen
Correct Answer: D.
Putamen
Methyl alcohol, an industrial solvent, is metabolized to formaldehyde and formate, leading to neurotoxicity. Symptoms begin 12-24 hours after exposure and include encephalopathy (often with disinhibition), ataxia, visual loss, seizures, headache, GI complaints, and tachypnea. The mechanism of visual loss is not known, although it may relate to formic acid accumulation within the optic nerve. Edema and hyperemia of the fundus and optic nerve may be evident, and patients may report blurring, scintillations, scotoma, diminished acuity, or frank blindness. The classic MRI finding is an infarction or hemorrhage of the putamen, as formic acid also tends to accumulate in this region. Lesions in the cortex and cerebellum are rare. Long-term survivors may experience Parkinsonism. Chronic exposure may result in an axonal polyneuropathy.
Incorrect Answers:
A. Lesions in the cerebellum due to chronic alcohol use disorder are rare.
B, C. These are located in the cortex, where it’s rare to see lesions.
References:
Mr. Mathews presents for evaluation of his out-of-control aggressive impulses. Usually, he is “a regular guy with a decent temperament” but sometimes becomes disproportionately aggressive to a precipitating stressor. Which of the following medications may worsen the symptoms of intermittent explosive disorder long-term?
A. Fluoxetine
B. Oxcarbazepine
C. Propranolol
D. Diazepam
Correct Answer: D.
Diazepam
The SSRI fluoxetine, mood stabilizers such as oxcarbazepine, propranolol, or a second-generation antipsychotic may benefit patients with intermittent explosive disorder. In particular, fluoxetine and oxcarbazepine have been superior to placebo in reducing impulsive aggression in these patients. Diazepam and other benzodiazepines may worsen the symptoms of intermittent explosive disorder long-term. Beta-blockers might help control explosive rage. Other new and unverified treatments also include decreasing testosterone levels in men or neurosurgery.
Incorrect Answers:
A. Will reduce impulse aggression
B. Will reduce impulse aggression
C. Treats high blood pressure, irregular heartbeats, and shaking (tremors)
References:
Guanfacine is a treatment for ADHD. Which of the following statements about its use is true?
A. Guanfacine is a less selective alpha-adrenergic compound than clonidine.
B. Guanfacine is associated with more sedation and shorter duration of action than clonidine.
C. Guanfacine is associated with tachycardia and hypertension.
D. School-aged children with ADHD and comorbid tic disorder treated with guanfacine show reduced both tics and ADHD symptoms.
Correct Answer: D.
School-aged children with ADHD and comorbid tic disorder treated with guanfacine show reduced both tics and ADHD symptoms.
The reduction in tics and ADHD is associated with a guanfacine at 0.5mg bid to 1mg tid. Guanfacine is FDA approved to treat ADHD in children ages 6 to 17-years-old. Not FDA approved for ADHD treatment in adults. It is also used off-label for conduct disorder, Tourette’s and motor tics, migraine prophylaxis and opioid withdrawal. Is a centrally-acting selective alpha 2 adrenergic agonist. Guanfacine comes in IR tablets (Tenex) and ER tablets (Intuniv). Side effects include dry mouth, sedation, rare hypotension, and rare orthostatsis.
Incorrect Answers:
A. Guanfacine is a MORE selective alpha-adrenergic compound than clonidine.
B. Guanfacine is associated with LESS sedation and a longer duration of action than clonidine.
C. Guanfacine is associated with minor bradycardia and hypotension, not tachycardia/hypertension.
Vital Concept:
Guanfacine has been shown to reduce both tics and ADHD symptoms in children. Only FDA approved for ADHD for 6-17 year-olds.
References:
A patient who is on PCP is admitted to the hospital for suspected overdose. Why are handcuffs for an extended period of time contraindicated?
A. Restraints violate individual rights.
B. Restraints increase risk of rhabdomyolysis.
C. Restraints must be ordered by medical personnel.
D. This is generally not an accepted practice.
Correct Answer: B.
Restraints increase risk of rhabdomyolysis.
The restrained patient may need to be sedated to avoid rhabdomyolysis associated with continued combativeness. Rhabdomyolysis occurs in approximately 2.5% of patients with phencyclidine (PCP) intoxication. Rhabdomyolysis can lead to kidney damage or failure. PCP overdose, in some cases, can cause muscle rigidity, which may evolve into rhabdomyolysis that mimics neuroleptic malignant syndrome.
Incorrect Answers:
A. This isn’t a concern if medically appropriate.
C. This isn’t a reason for it to be medically inappropriate to keep a patient in handcuffs while on PCP.
D. This is insufficient as reasoning either way.
References:
A test for screening an aggressive familial variant of colon cancer is available. The cancer is invariably fatal if not detected and treated early. The serum concentration of the marker in patients with and without the particular variant of colon cancer is depicted in the graph below. The x-axis represents the serum concentrations of the marker. At which of the following points would the test have the highest sensitivity?
A. A
B. B
C. C
D. D
E. E
Correct Answer: A.
A
The double-hump curve can be used to infer the negative and positive predictive value of a screening test. The sensitivity of the test is greatest at point A, where a low serum concentration of the marker is the cutoff for a positive test. Choosing point A will result in more false-positive tests. The negative predictive value of the test will be greater at point A. The specificity of the test will be greater if the cutoff for a positive test is at a higher concentration of the marker.
The specificity of the test will be greatest at point E, but this will result in false-negative tests. The positive predictive value at point E is greatest, however, since people with a positive test are more likely to have the disease. The figure below illustrates this concept in terms of false positive and false negative results. If asked about where the cutoff point should be for a test, it usually depends on the consequences of missing a diagnosis or, in the case of research, the desire to avoid including subjects in a study who do not have the condition. In the first case, the vertical line for the cutoff for a test would be moved to the left. In the second case, to avoid including a false positive in a study, the cutoff point for a positive test would be higher, so the vertical line would be moved to the right. The World Health Organization has 2 sets of diagnostic criteria in acknowledgment of the utility of 2 different cutoff points, 1 for research and 1 for clinical care. Increased sensitivity leads to over-diagnosis and increased specificity leads to under-diagnosis.
Incorrect Answers:
B. Point B has increased sensitivity but point A has the highest sensitivity.
C. Point C is considered the most accurate point.
D. Point D has increased specificity but point E has the highest specificity.
E. Specificity of the test is greatest at point E.
Vital Concept:
The double-hump curve can be used to infer the negative and positive predictive value of a screening test.
References:
A 5-year-old female patient presents with a recurrent UTI. The NP suspects possible sexual abuse and discusses these concerns with the child’s caregiver. They live with the caregiver’s boyfriend, but they’re certain that he has not sexually abused the child. The caregiver requests that the NP keeps their suspicion confidential and states they will not leave the child alone with the boyfriend in the future. What is the best course of action?
A. Keep your suspicion confidential and follow-up soon to verify compliance with your recommendation.
B. Report this case immediately to Child Protective Services.
C. Without clear evidence of abuse, do not report this incident to authorities.
D. Tell the caregiver you are uncomfortable with the situation and suggest they find another healthcare provider.
Correct Answer: B.
Report this case immediately to Child Protective Services.
Healthcare providers are mandatory reporters of suspected child abuse and have a legal and ethical duty to report any valid suspicion to child protective services. The duty to report a reasonable suspicion of child abuse supersedes any duty to maintain confidentiality. There is no breach of discretion in reporting either documented abuse or suspected abuse. Reporting should be immediate so intervention can be initiated to prevent further abuse. Healthcare providers are protected from liability when reporting suspected abuse in good faith.
Incorrect Answers:
A. Healthcare providers are mandatory reporters of suspected child abuse and have a legal and ethical duty to report any valid suspicion to child protective services.
C. Healthcare providers are protected from liability when reporting suspected abuse in good faith.
D. Suggesting the caregiver find another provider doesn’t absolve the provider of the duty to report.
Vital Concept:
Signs of child maltreatment should be a part of the NP’s evaluation during clinic visits and knowing how to report and refer to the appropriate authority is a part of their scope of practice.
References:
A patient has had excessive and disruptive thoughts and anxiety about pain symptoms they’re experiencing (including muscle, abdominal, and head pain) for the past 6 months despite a thorough negative medical workup. This patient meets the criteria for which of the following disorders?
A. Functional neurological symptom disorder (FNSD)
B. Factitious disorder
C. Illness anxiety disorder
D. Somatic symptom disorder (SSD)
Correct Answer: D.
Somatic symptom disorder (SSD)
Correct Answer: D. Somatic symptom disorder (SSD).
Formerly pain disorder, the DSM-5-TR describes SSD as a condition with somatic symptoms that must be significantly distressing or disruptive to daily life and must be accompanied by excessive thoughts, feelings, or behaviors. The diagnostic criteria include:
At least one somatic symptom that substantially impacts the patient’s daily functioning or leads to anguish
The patient then develops extraordinary actions, emotions, or ruminations regarding the symptom(s) or related medical condition(s), as evidenced by one or more of the following:
· consistent worry or concern about their physical wellbeing
· extraordinary devotion of resources (e.g., energy, time) to the symptom(s) or related medical condition(s)
· ruminations that are consistent and unbalanced regarding the gravity and meaning underlying these symptoms
These conditions persist for at least 6 months, although the specific symptom of focus may vary or shift during that time
Specifiers include:
· with prominent pain if the symptoms involve physical discomfort
· persistent if the symptoms are severe, enduring (greater than 6 months), and cause substantial dysfunction
· mild/moderate/severe based on the number of symptoms listed above that are present (1/2/2+ multiple symptoms)
Incorrect Answers:
A. Functional Neurological symptom disorder (FNSD) requires the presence of one or more voluntary motor or sensory symptoms that are incompatible with known neurological or medical conditions.
B. There’s no evidence that the patient is faking their condition to assume the role of the patient, which is the underlying motivation in factitious disorder.
C. Illness anxiety disorder is characterized by a preoccupation with having or acquiring an illness and the presence of a high level of anxiety regarding one’s health despite no somatic symptoms present (or only mild in intensity).
Vital Concept:
Somatic symptom disorder (SSD) is a disease with somatic symptoms that must be significantly distressing or disruptive to daily life and must be accompanied by excessive thoughts, feelings, or behaviors.
References:
Tetracyclic antidepressants (TeCAs) are a class of antidepressants first introduced in the 1970s. Which of the following is a TeCA?
A. Atomoxetine
B. Imipramine
C. Doxepin
D. Mirtazapine
Correct Answer: D.
Mirtazapine
In terms of structure, mirtazapine is classified as a tetracyclic antidepressant. It is a noradrenergic and specific serotonergic antidepressant. In addition, it is commonly used as an anxiolytic, hypnotic, antiemetic, and appetite stimulant. Atomoxetine is a norepinephrine reuptake inhibitor failed as an antidepressant but gets approved for the ADHD. Imipramine, doxepin, and clomipramine are tricyclic antidepressants.
Incorrect Answers:
A. This is a norepinephrine reuptake inhibitor (NRI, NERI)
B, C. These are tricyclic antidepressants (TCAs)
References:
According to the DSM-5-TR, which of the following features distinguishes grief from major depressive disorder?
A. Grief is dominated by sensations of having lost something or feeling empty inside
B. Persistent feelings of sadness, and despair, without a visible end in sight, are characteristic of grief.
C. The pain of grief rarely occurs with laughing and positive feelings.
D. Emotions of grief are not typically associated with memories of the deceased and self-regard is poor
Correct Answer: A.
Grief is dominated by sensations of having lost something or feeling empty inside
Distinguishing grief from a major depressive episode per DSM 5-TR
Grief
dominated by sensations of having lost something or feeling empty inside
symptoms decrease within weeks and fluctuate with reminders of the deceased
can occur with laughing and positive feelings
associated with memories of the deceased but self-regard is largely intact
Major depressive episode
persistent feelings of sadness, and despair, without a visible end in sight
emotional state is not related to specific thoughts/ruminations
feeling self-critical and degrading is common, with poor sense of self-worth
Incorrect Answers:
B. Persistent feelings of sadness, and despair, without a visible end in sight are characteristic of a major depressive episode (MDE), not grief.
C. Grief can occur with laughing and positive feelings as memories of the deceased are recalled. This is not seen in MDE
D. Grief may be associated with memories of the deceased but self-regard is largely intact. In MDE, self-critical feelings thoughts are commonplace.
Vital Concept:
Grief and depression are dominated by sensations of having lost something or feeling empty inside, which typically resolve within weeks to months, and may co-occur with periods of levity and laughter when remembering good times with the deceased.
References:
It’s important for practitioners to understand how acute stress disorder, PTSD, and comorbid conditions may interact. Which of the following statements about acute stress disorder, PTSD, and comorbid conditions’ interactions is not true?
A. Associated PTSD in patients with psychotic disorders and borderline personality disorders is often unrecognized.
B. Patients with serious mental illness experience high rates of sexual abuse and physical assault.
C. Patients who have experienced major physical trauma (e.g. burns, serious accidents) rarely have ASD or PTSD.
D. PTSD can be associated with bereavement after witnessing traumatic loss of family member.
Correct Answer: C.
Patients who have experienced major physical trauma (e.g. burns, serious accidents) rarely have ASD or PTSD.
This patient population often experiences PTSD and ASD.
Incorrect Answers:
A. Patients with borderline personality disorder and psychotic disorders are particularly likely to have experienced victimization as a child and/or adult.
B. These patients often have PTSD, which is usually overlooked in clinical assessment.
D. Traumatic death of a family member (especially a spouse) may result in PTSD associated with bereavement.
References:
You are asked to assess the competency to stand trial of a 22-year-old man who broke into a local celebrity’s house. The patient claims that the local celebrity has been communicating with him through coded messages on her news weather show, telling him that she was in love with him. He broke into her house at night and was waiting for her when she came home with her husband. The news meteorologist claims never to have met the man. He confirms this but claims that he has “watched her for years” and that she began to speak specifically to him through the television about a year ago. He is not currently employed, despite graduating from high school with good grades. According to his parents, he has been staying alone in his room for days. Which type of delusions is this patient experiencing?
A. Capgras syndrome and erotomania
B. Delusions of reference and de Clerambault syndrome
C. Erotomania and delusions of control
D. Delusions of grandeur and de Clerambault syndrome
Correct Answer: B.
Delusions of reference and de Clerambault syndrome
This patient is experiencing delusions of reference and de Clerambault syndrome, also known as erotomania.
Incorrect Answers:
A. Capgras syndrome is the belief that a family member or close friend is not actually the same person and has been replaced by a body double.
C. The patient does not report delusions of control.
D. A delusion of grandeur is an unrealistic belief in one’s powers and abilities.
References:
What statement is the most accurate in patients diagnosed with unipolar major depressive diorder?
A. Dialectical Behavior Therapy (DBT) is one of the most common therapy types used in clients with unipolar major depressive d/o
B. A score of greater than 10 indicates Unipolar major depressive d/o on the PHQ-9 screening
C. The most common side effect to SSRIs to treat depression is weight gain
D. The best treatment is a combination of psychotherapy and pharmacotherapy.
Correct Answer: D.
The best treatment is a combination of psychotherapy and pharmacotherapy.
The suggested treatment is both psychotherapy and pharmacotherapy as it has been shown to be more effective than either treatment alone.
Incorrect Answers:
Dialectical Behavior Therapy (DBT) is one of the most common therapy types used in clients with unipolar major depressive d/o is incorrect as CBT and interpersonal therapy are most common.
A score of greater than 10 indicates Unipolar major depressive d/o on the PHQ-9 screening is incorrect as as score of greater than 10 suggests moderate depression.
The most common side effect to SSRIs to treat depression is weight gain is incorrect as it more common to have GI upset and sexual related side effects than weight gain (paxil is the exception).
References:
A 50-year-old man presents and relates a story consistent with alcohol dependence. When he is asked further about his habits, the patient gets defensive, saying “I didn’t come here to talk about that.” Which of the DiClemente and Prochaska’s stages of change describes this man?
A. Denial
B. Precontemplation
C. Contemplation
D. Preparation
Correct Answer: B.
Precontemplation
People at Stage 1: Precontemplation (not ready) do not intend to start the healthy behavior in the near future (within 6 months) and may be unaware of their need to change. These patients need to learn more about healthy behavior and are encouraged to think about the pros of changing their behavior and to feel emotions about the effects of their negative behavior on others.
In Stage 2: Contemplation (getting ready), participants intend to start the healthy behavior within the next 6 months. While they are usually more aware of the pros of changing, their cons are about equal. This ambivalence about changing can cause them to avoid taking action.
In Stage 3: Preparation (ready), people are ready to start taking action within the next 30 days. They take small steps that they believe can help them make the healthy behavior a part of their lives. For example, they tell their friends and family that they want to change their behavior.
In Stage 4: Action, people have changed their behavior within the last 6 months and need to work hard to keep moving ahead. These participants need to learn how to strengthen their commitments to change and to fight urges to slip.
In Stage 5: Maintenance, people at this stage changed their behavior more than 6 months ago. It is important for people in this stage to be aware of situations that may tempt them to slip back into the unhealthy behavior, particularly stressful situations.
Incorrect Answers:
A. Denial isn’t a stage of DiClemente and Prochaska’s stages of change.
C. In this stage, the patient intends to start the healthy behavior in the near future (next six months) — this patient isn’t there yet, although it’s the next step.
D. In this stage, the patient is ready to start the healthy behavior in the next 30 days — this patient isn’t there yet, as there’s still the Contemplation stage between his current stage (Precontemplation) and this.
References:
Olanzapine is used to treat certain mental and mood conditions, such as schizophrenia or bipolar disorder. However, it has some potentially problematic side effects. Which of the following is the most problematic side effect of olanzapine?
A. Weight gain
B. Weight loss
C. Hypertension
D. Sexual dysfunction
Correct Answer: A.
Weight gain
Weight gain is typically the most problematic side effect of olanzapine.
Incorrect Answers:
B. Olanzapine may cause weight gain, not weight loss
C. Olanzapine causes orthostatic hypotension.
D. Sexual dysfunction does occur but is rarely problematic.
References:
A nurse practitioner notices that many of her patients who have breast cancer are obese. She decides to perform a study of the characteristics of obese women and women who are not obese, recording the incidence of breast cancer. Which of the following describes this type of research?
A. Randomized controlled study
B. Descriptive study
C. Correlational study
D. Case study
Correct Answer: C.
Correlational study
A correlational study describes and measures the relationships between two or more variables or interrelationships. Data are numerical and measurable, subject to statistical testing.
Incorrect Answers:
A. A randomized controlled study selects two or more groups in a population with a certain condition and randomly assigns patients to different treatment groups to measure the effects of treatment.
B. A descriptive study describes and measures the characteristics of a group or phenomenon.
D. A case study is a study of one person with a condition or illness.
Vital Concepts:
A correlational study describes and measures the relationships between two or more variables or interrelationships. Data are numerical and measurable, subject to statistical testing.
References:
A patient is projecting their feelings and identifies onto their therapist. What term describes this phenomenon?
A. Adaptability
B. Connectedness
C. Transference
D. Interpersonal and generational boundaries
Correct Answer: C.
Transference
Transference is a phenomenon in which a patient projects feelings and identities on a therapist.
Incorrect Answers:
A. Adaptability is a combination of flexibility and stability. Healthy families balance the two.
B. Connectedness is a key part of family functioning and defined as a balance between closeness and separateness.
D. Interpersonal and generational boundaries are part of the family system.
References:
Schizophrenia is caused by a combination of biological, psychological, and social factors. Which of the following nonheritable or environmental factors is associated with an increased risk for schizophrenia?
A. Cold temperature
B. Radiation in third trimester
C. Rural birth
D. Spring birth
Correct Answer: D.
Spring birth
Studies show that seasonal variation in the prevalence of births leading to schizophrenia has been identified, with an excess of births in winter and spring months. Other environmental factors that predispose to schizophrenia development are ambient temperature, exposure to infectious agents, nutritional deficiencies, infections, maternal rubella, late-winter/early-spring birth, urban birth, irradiation in the first trimester, prolonged labor, perinatal complications, and family and social stressors.
Incorrect Answers:
A. AMBIENT temperatures are associated with an increased incidence of schizophrenia in offspring, not cold temperature.
B. There is an increased incidence of schizophrenia in the offspring of women who were subjected to radiation in their FIRST trimester, not third trimester.
C. With URBAN birth there is an increased incidence of schizophrenia, not rural birth.
Vital Concept:
There is a seasonal variation in the prevalence of births leading to schizophrenia with an excess of births in winter and spring months.
References:
A nurse practitioner wants to perform a study on women and attitudes towards alcohol. She plans to send a survey to 100 women in a large family practice and question them about their attitudes and habits. Which of the following is true?
A. A descriptive study that measures the characteristics of a group cannot be evaluated statistically
B. Data in a descriptive study are not measurable
C. Data in a descriptive study are not numerical
D. Descriptive studies do not prove causation
Correct Answer: D.
Descriptive studies do not prove causation
A descriptive study’s data are measurable, numerical, and are evaluated by standard statistical testing. It is not possible to prove causation with a descriptive study, although the correlation between variables may provide a direction for further research.
Incorrect Answers:
A. A descriptive study’s data are evaluated by standard statistical testing.
B. A descriptive study’s data are measurable.
C. A descriptive study’s data are numerical.
Vital Concepts:
A descriptive study’s data are measurable, numerical, and are evaluated by standard statistical testing. It is not possible to prove causation with a descriptive study, although the correlation between variables may provide a direction for further research.
References:
A young nursing student is interviewing a nurse practitioner about her profession. Which of the following is true?
A. The first nurse practitioners were family practitioners
B. The first nurse practitioner program was started in 2001
C. A nurse practitioner derives the legal right to practice from federal law
D. NP practice is not regulated by the Department of Health and Human Services
Correct Answer: D.
NP practice is not regulated by the Department of Health and Human Services
Nurse practitioners are regulated by the Nurse Practice Act of the state where he or she plans to practice. The legal right to practice is derived from state law, which contains regulations that mandate educational requirements, responsibilities, and scope of practice for nurse practitioners (and other nurses in the state). NP practice is not regulated by the federal government, the Department of Health and Human Services, or the American Medical Association. Enforcement of each state’s Nurse Practice Act is the responsibility of the state board of nursing, a formal governmental agency with statutory authority to regulate nursing practice. The board of nursing in each state has legal authority to license, monitor, and discipline nurses. The board of nursing also has the authority to revoke a nurse’s license, after formal hearing.
Incorrect Answers:
A. The first NPs were pediatric NPs who practiced in rural areas with a shortage of physicians.
B. Dr. Loretta Ford and Dr. Henry Silver developed the first Nurse Practitioner (NP) program at the University of Colorado in 1965.
C. NP practice is not regulated by the federal government, the Department of Health and Human Services, or the American Medical Association.
Vital Concepts:
Nurse practitioners are regulated by the Nurse Practice Act of the state where he or she plans to practice. The legal right to practice is derived from state law, which contains regulations that mandate educational requirements, responsibilities, and scope of practice for nurse practitioners (and other nurses in the state).
References:
A certain SSRI has been shown to increase infants’ risk of developing cardiovascular malformations by 1.5-2x as compared to other SSRIs. Which of the following drugs is the SSRI in question?
A. Sertraline
B. Citalopram
C. Fluoxetine
D. Paroxetine
Correct Answer: D.
Paroxetine
Paroxetine is known to cause cardiovascular malformations after first-trimester exposure, compared to exposure to other antidepressant medications. Infants exposed to paroxetine during the first trimester have a 1.5-2x increased risk of developing cardiovascular malformations compared to other SSRIs.
Incorrect Answers:
A, B, and C. While these are SSRIs, they haven’t been shown to increase infants’ risk of developing cardiovascular malformations to the same degree as Paroxetine.
References:
The DSM-5-TR sets forth the diagnostic criteria for schizoaffective disorder. Which of the following is included in these criteria?
A. Symptoms of psychosis and a major mood disorder must be present, either concurrently or separately
B. Delusions or hallucinations must be present for 1 week.
C. Two or more primary criteria for Schizophrenia in addition to criteria for mood disorder (mania or depression)
D. Disturbance can be attributable to effects of substance or another medical condition.
Correct Answer: C.
Two or more primary criteria for Schizophrenia in addition to criteria for mood disorder (mania or depression)
A major mood episode must be present for the majority of the disorder’s total duration after Criterion A has been met. DSM-5-TR criteria for schizoaffective disorder include:
The patient must have two sets of symptoms simultaneously:
· Symptoms of a major depressive episode (MDE) or mania- the patient must meet the criteria, including poor or sad mood for the majority of the day most days
· At least two primary symptoms of schizophrenia:
o speaking incoherently without logical organization*
o a misconception, belief, or thought that is firmly held despite not being grounded in reality*
o illusions or perceived experiences that do not exist (e.g., sounds, voices, smells, visions, feelings, etc.)*
o actions that are chaotic or confused, repetitive, purposeless, or significantly reduced (or absent) movement and speech (with catatonia)
o decreased display of emotion or a lack of motivation
· The symptoms of schizophrenia must be present for at least two weeks alone, without the mood symptoms listed above
· Mood symptoms are then present for the majority of the episode/illness
· The patient’s symptoms are not directly related to the use of a substance, medication, or pre-existing medical diagnosis or health concern and are not due to a more appropriate psychiatric condition
· The disorder should be classified as Bipolar type if mania is present or Depressive type if ONLY depressive symptoms are present.
Incorrect Answers:
A. The patient must experience an uninterrupted period of illness during which there is a mood episode (major depressive episode or mania) concurrent with Criterion A of schizophrenia.
B. Delusions, or hallucinations for 2 or more weeks in the absence of a mood episode (major depressive episode or mania) during the lifetime duration of the illness.
D. Disturbance is not attributable to the effects of a substance or another medical condition.
Vital Concept:
The patient must have two sets of symptoms simultaneously, including a mood disorder (major depressive episode or mania) and at least two primary symptoms of schizophrenia.
References:
A Latino family from the Caribbean presents with concerns regarding their adolescent child. The caregiver reports that the adolescent is a “drama queen” and seems to have ataque de nervios (attack of the nerves). The caregiver is exhausted by the adolescent, who is extremely labile and irritable. What nursing theory will best assist the practitioner in caring for these patients without providing nursing interventions?
A. Theory of Cultural Care (Madeline Leininger)
B. Theory of Self-care (Dorothy Orem)
C. Human Becoming Theory (Rosemarie Parse)
D. Health Promotion Theory (Nola Pender)
Correct Answer: A.
Theory of Cultural Care (Madeline Leininger)
People from diverse cultural groups often experience, understand, and communicate suffering, behavioral problems, or troubling thoughts and emotions in differing manners.
Incorrect Answers:
B. The provider’s self-care isn’t in play in this situation; this theory isn’t helpful in this case.
C. This theory concerns the patient becoming a unitary being in continuous interaction with his environment; this theory isn’t helpful in this case.
D. This theory concerns healthy living; this theory isn’t helpful in this case.
Vital Concept:
Commonly in Latino culture, the expression’ ataque de nervios’ is used to describe screaming uncontrollably, crying, or verbal/physical aggression. Madeline Leininger developed the Theory of Cultural Care, recommending using cultural-based knowledge to provide comprehensive care to patients. Nursing Interventions are not included in this theory as they are usually imposed practices that may offend certain cultures.
References:
Joseph is a 32-year-old software engineer who presents with complaints of anxiety at work. He feels uncomfortable eating lunch with others at business meetings because he thinks that everyone can hear him chew and that he has food on his face or in his teeth. He is scared to speak in meetings because he does not know how others will respond and feels embarrassed. The patient is even afraid to use the bathroom at work for fear others will be in the room with him. After this initial visit, Joseph’s new therapist instructs him to enter the situations that are associated with anxiety until the anxiety associated with the exposure subsides. What is the name of this type of therapy?
A. Habit-reversal therapy
B. In vivo exposure
C. Guided discovery
D. Psychodynamic therapy
Correct Answer: B.
In vivo exposure
In vivo exposure, sometimes called flooding, is a method of psychotherapy in which patients are instructed to enter (in vivo) the situation causing them anxiety to decrease the anxiety associated with the exposure. Systemic desensitization may include flooding (in vivo exposure) or may utilize imagined exposure to increasing severity of feared scenarios as well as progressive muscle relaxation.
Incorrect Answers:
(A) Habit-reversal therapy. Habit-reversal therapy used to treat repetitive behavior disorders (e.g. Tourette’s syndrome or tic disorders). Consists of awareness training, competing response training, contingency management, relaxation training and generalization training.
(C) Guided discovery. Guided discovery is used in teaching to guide students to learn on their own.
(D) Psychodynamic therapy. Psychodynamic therapy is long term talk therapy focusing on discovering unconscious processes that effect patient’s current behaviors.
References:
Reflective thinking is one way for people to think through a problem. According to Taggart and Wilson (2005), which of the following is an element of the reflective thinking process?
A. It is an internal process only
B. It does not involve consideration of moral principles
C. It takes into consideration similar past situations
D. Routine approaches should be ignored
Correct Answer: C.
It takes into consideration similar past situations
Reflective thinking, according to the model developed by Taggart and Wilson (2005), is a process that includes identification of a problem and consideration of the problem or situation from the perspective of a third person in order to frame it, which involves observation, collection of data, reflection, and consideration of moral principles. This consideration should provide context or a way to frame the situation, taking into account similar past events and any solutions you may have used successfully in similar situations. After identifying possible approaches, reflective thinking includes observation and testing, with a final review of any action taken and the consequences of that action.
Incorrect Answers:
A. Reflective thinking is both an internal and an external process.
B. When considering how to frame or reframe a problem or situation, an objective third-person viewpoint is useful. When defining the context of the situation, reflective thinkers will observe, gather data, reflect, and consider moral principles.
D. Routine approaches within your experience may provide possible solutions to a problem.
Vital Concepts:
Reflective thinking is a process that includes identification of a problem and consideration of the problem or situation from the perspective of a third person in order to frame it, which involves observation, collection of data, reflection, and consideration of moral principles.
References:
Modafinil is a medication that promotes wakefulness.
Which of the following statements about it is true?
A. Its proposed mechanism of action is that of a dopamine reuptake inhibitor.
B. It can often cause Stevens-Johnson syndrome.
C. Common side effects include bradycardia and hypotension.
D. It is FDA-approved for treating depression.
Correct Answer: A.
Its proposed mechanism of action is that of a dopamine reuptake inhibitor.
Modafinil is a schedule 4 controlled substance and has a proposed mechanism of action as a dopamine reuptake inhibitor. It is FDA-approved for use in narcolepsy, shift-work disorder, and excessive tiredness from obstructive sleep apnea.
Used off-label in attention deficit and hyperactivity disorder (ADHD), fatigue, and treatment-resistant depression. Common side effects include increased blood pressure/heart rate, diarrhea, headache, and insomnia. Rare side effects include Stevens-Johnson syndrome, angioedema, multi-organ hypersensitivity reaction, and psychosis/mania/agitation.
Incorrect Answers:
B. Modafinil can rarely, not often, cause Stevens-Johnson syndrome. This is why although clinical efficacy in children was demonstrated, modafinil is not approved for use in children due to increased risk of Stevens-Johnson syndrome even a rare risk.
C. Common side effects include tachycardia and hypertension, not bradycardia and hypotension.
D. Modafinil is not FDA-approved for depression; it has been used off-label to in treatment-resistant depression as an adjunct to antidepressants.
Vital Concept:
Modafinil is a schedule 4 controlled substance, dopamine reuptake inhibitor and is FDA approved for narcolepsy, shift work disorder, and excessive tiredness from obstructive sleep apnea.
References:
A consulting psychiatrist shows a patient three 1-dollar bills and hides the bills while she watches. The patient is unable to locate the hidden dollar bills after several minutes. A problem in what area best explains her deficits?
A. Procedural memory
B. Remote memory
C. Recall
D. Episodic memory
Correct Answer: C.
Recall
Recent memory operates on a scale of minutes to days. This bedside demonstration enhances the patient’s affective engagement with the task.
Incorrect Answers:
A. Procedural memory is not conscious, is automatically utilized, and includes complex motor activities like riding a bike and cognitive skills like reading.
B. Remote memory operates on a scale of days to years.
D. Episodic memory involves autobiographical events that can be consciously accessed and explicitly stated.
Vital Concept:
Problems of encoding, consolidation, and retrieval will cause errors of recall. Errors of recall might trigger other bedside tests, such as hiding objects around the patient’s room and asking the patient to locate them after several minutes.
References:
Williams Syndrome is a genetic condition that is present at birth. Which of the following statements about it is true?
A. Common comorbidities include bipolar disorder and schizophrenia.
B. It is often associated with atrial septal defects and susceptibility to renal vein thrombosis.
C. It is associated with genetic deletion at chromosome 17.
D. Patients often have elfin facies and starburst irises.
Correct Answer: D.
Patients often have elfin facies and starburst irises.
Elfin facies and starburst irises are characteristic dysmorphic features of Williams syndrome.
Incorrect Answers:
A. Common comorbidities are anxiety disorders and depression.
B. Patients typically have supravalvular aortic stenosis, renal artery stenosis, and hypertension.
C. A genetic deletion occurs at chromosome 7.
VitalConcept:
Williams syndrome occurs due to a genetic deletion at chromosome 7. This typically leads to elfin facies, starburst irises, supravalvular aortic stenosis, renal artery stenosis, hypertension, anxiety, and depression.
References:
You are a resident on-call at a local psychiatric emergency room. During a slow period, the overworked and sleep-deprived intern confides that he is worried he may have schizophrenia. He recounts numerous episodes of seeing things, most recently last night. He describes lying in bed last night and was on the verge of going to sleep when he suddenly saw a figure standing at the edge of his bed leaning over him. This startled him initially, and he then became fully awake and sat up in his bed. He quickly realized that the figure was not there, but it was still startling at the time. He notes that this happened previously when he was in his third year in med school with particularly time-consuming rotations. However, he is concerned because these episodes have been happening more frequently since he started residency. Which of the following terms best describes the intern’s symptoms?
A. Command hallucination
B. Illusion
C. Hypnopompic hallucination
D. Hypnagogic hallucination
Correct Answer: D.
Hypnagogic hallucination
Hypnagogic hallucinations are experienced when an individual is going to sleep. They also resolve quickly with full awakening and are known by the individual to have been non-existent.
Incorrect Answers:
A. Command hallucinations are a subtype of auditory hallucinations. The voices give commands to the person experiencing the hallucination. This patient is not reporting auditory hallucinations or command type hallucinations.
B. Illusions are misinterpretations of actual stimuli. The individual will experience an actual sight or sound but will misinterpret the stimulus and manifest an inaccurate interpretation. For example, someone hears a dog barking but interprets it as a voice telling them to kill themselves. The real stimulus, a dog barking, is misinterpreted as a command to kill oneself. Hallucinations do not have a real stimulus that precipitates the hallucination.
C. Hypnopompic hallucinations are hallucinations experienced when waking from sleep. These typically are brief and, once the individual is completely awake, resolve quickly and are known by the individual to have been non-existent. This patient has symptoms when going to sleep rather than when awakening.
References:
A civil commitment requires a standard of proof. What is the definition of this term?
A. Clear and convincing evidence
B. Satisfactory evidence
C. Preponderance of evidence
D. Paucity of evidence
Correct Answer: A.
Clear and convincing evidence
The standard of proof in a civil commitment, per the landmark US Supreme Court case Addington v. Texas (1979), is clear and convincing evidence. A preponderance of evidence is the standard of proof in a malpractice case. Guilt beyond a reasonable doubt is the standard of proof in a criminal case.
Incorrect Answers:
B, D. This is the isn’t the standard of proof in any type of case
C. This is the standard of proof in a malpractice case
References:
A new marker is used to create a screening test for the bedside diagnosis of myocardial infarction. The test is performed on 2,000 people. The test is positive in 750 people, but of the 750 people with a positive test, only 600 have a myocardial infarction, and 150 people with a positive test have non-cardiac chest pain. In total, 1250 people have a negative test, but 250 of those people are diagnosed with acute myocardial infarction. What is the accuracy of the screening test?
A. 600/750
B. 1000/1150
C. 1600/2000
D. 600/850
Correct Answer: C.
1600/2000
Accuracy is the number of true positives and true negatives divided by the entire population tested. For clarity, it may be helpful to make a 2x2 table as shown below. Taking the number of true positives and true negatives divided by the total population gives 1600/2000. The accuracy of a binary classification test is the measure of how well the test correctly identifies or excludes a condition. It refers to the degree to which an estimate is immune from systematic error or bias. It also reflects the degree to which a measurement or test reflects a true value. Accuracy may be determined when the sensitivity, specificity, and prevalence are known. When the term is used to refer to a psychometric instrument of measurement, it is often used interchangeably with the term validity.
In contrast, 600/750
Incorrect Answers:
A. 600/750. Reflects the precision of the test or the proportion of true positives divided by all positive tests (true and false positives). Precision is the degree to which repeated measurements under unchanged conditions show the same results. A measurement or test is valid when it is both accurate and precise.
B. 1000/1150. This is the specificity of the test. Specificity= (true negatives)/ (false positives+ true negatives). Specificity is the probability the test will be negative among those who do not have the disease.
C. 600/850. This is the sensitivity of the test. Sensitivity= (true positives)/ (true positives + false negatives). Sensitivity is the probability that the test will be positive among those who have the disease.
Vital Concepts:
Accuracy is the number of true positives and true negatives divided by the entire population tested.
References:
Benzodiazepines carry side effects that patients and practitioners need to be aware of. Which of the following statements about benzodiazepines’ side effects and drug interactions is true?
A. Cognitive effects are most significant in young patients.
B. Initial side effects are worse with short-acting benzodiazepines than with long-acting benzodiazepines.
C. Patients should not use alcohol or sedating antihistamines (e.g. diphenhydramine) while on benzodiazepines due to increased risk of accidents.
D. Increased irritability, hostility, and paradoxical disinhibition is common with benzodiazepine use.
Correct Answer: C.
Patients should not use alcohol or sedating antihistamines (e.g. diphenhydramine) while on benzodiazepines due to increased risk of accidents.
Even without concomitant use of alcohol and sedating antihistamines, there is a higher risk for motor vehicle accidents and for accidental injuries in patients taking benzodiazepines. The greatest risk of accidental injuries is during the first 2-4 weeks of therapy.
Incorrect Answers:
A. Cognitive effects are greatest in the elderly. Patients with delirium and dementia are especially at risk.
B. Initial side effects can persist and worsen in patients with liver disease and those taking longer-acting agents (e.g. diazepam).
D. These psychological effects are uncommon.
References:
Fluoxetine is used to treat OCD, bulimia nervosa, and panic disorder. Which of the following is a common side effect of this medication?
A. Diarrhea and cramps
B. Hypertension
C. Constipation
D. Hyponatremia
Correct Answer: A.
Diarrhea and cramps
The gut is lined with 5-HT receptors. By blocking 5-HT reuptake, fluoxetine causes gastrointestinal problems like nausea, diarrhea, cramps, and heartburn.
Incorrect Answers:
B. Fluoxetine is not associated with hypertension.
C. The anticholinergic activity of fluoxetine is low. Constipation is not a commonly reported side effect.
D. Hyponatremia is a serious but uncommon side effect of fluoxetine, occurring more often in the elderly population. The mechanism of action is likely SSRI-induced SIADH.
References:
It’s important for health care professionals to understand the burden of mental illness. Which of the following is true?
A. Mental illness does not affect overall physical health in most people.
B. Effective treatment for mental disorders is not yet available but is on the horizon.
C. Stigma is a significant barrier for people who are seeking treatment for mental health.
D. Mental illness affects the adult population but occurs in less than 1% of children.
Correct Answer: C.
Stigma is a significant barrier for people who are seeking treatment for mental health.
There are many barriers to treatment for individuals with diagnosed mental illness. These include health disparity, cost, lack of available services, and stigma. Stigma is a significant barrier that prevents many persons with mental illness from receiving treatment.
Incorrect Answers:
A. Mental illness can influence the onset, progression, and outcome of physical illness and overall health and well-being. It should be treated with the same urgency that physical illness is treated. Many individuals with mental illness have health risk behaviors that include substance abuse, tobacco use, and physical inactivity.
B. Effective treatment of mental disorders is available.
D. One in ten children in the United States are estimated to have a mental disorder.
References:
Which of the following is not correct about traditional behavioral couples therapy?
A. Therapeutic goal is to increase reinforcing behaviors.
B. Therapeutic goal is to decrease punishing behaviors.
C. Therapeutic goal is to identify negative cognitions.
D. Therapeutic goal is to replace arguing with problem-solving.
Correct Answer: C.
Therapeutic goal is to identify negative cognitions.
Behavioral couples therapy is distinct from subsequent therapies. Cognitive-behavioral couples therapy does not emphasize cognition.
Incorrect Answers:
A. Behavioral couples therapy is based on operant conditioning in which the increase of social reinforcers decreases dyadic stress. This is accomplished through behavioral exchange.
B. Behavioral couples therapy is based on operant conditioning, in which decreases in social punishers decrease dyadic stress.
D. Couples behavioral therapy seeks to replace conflict with a problem-solving approach consisting of problem definition and problem solution.
References:
A healthcare provider is evaluating a 30-year-old male for dysuria. When the patient steps out of the room to produce a urine specimen, the patient’s wife asks you in confidence to test the patient’s urine specimen for alcohol, as she suspects her husband (a former patient with chronic alcohol use disorder) has been surreptitiously drinking. Which of the following actions is correct?
A. Send the specimen for testing for alcohol but only inform the patient of the results.
B. Tell the patient about his wife’s request and ask him about alcohol use.
C. Do not perform a test for alcohol without the patient’s permission.
D. Ask the patient’s wife for further information before deciding whether to send the specimen for alcohol testing.
Correct Answer: C.
Do not perform a test for alcohol without the patient’s permission.
A healthcare provider cannot perform any procedure or test on a patient unless the patient has agreed, even when the treatment will be beneficial to the patient. A healthcare provider can only violate patient autonomy if there is a risk of harm to others. In most cases, questions about drug or alcohol use can be asked routinely during an office visit. If a competent adult does not acknowledge a problem or ask for assistance, the healthcare provider cannot act on the request of a family member, even if the healthcare provider feels it would be in the patient’s best interest.
Incorrect Answers:
A. This would be a violation of patient autonomy
B. and D. If a competent adult doesn’t acknowledge the problem or ask for assistance, the healthcare provider can’t act on the request of a family member
Vital Concepts:
A healthcare provider cannot perform any procedure or test on a patient unless the patient has agreed, even when the treatment will be beneficial to the patient. A healthcare provider can only violate patient autonomy if there is a risk of harm to others.
References:
When assessing the community health needs for implementation of a community health program, which of the following is true of the target population?
A. It should include the entire population in the defined geographical area
B. The target population includes people in the defined service area who have access to preventive health care
C. The target population is the only population the community health center will serve
D. Barriers to healthcare that may be encountered by the target population should be considered
Correct Answer: D.
Barriers to healthcare that may be encountered by the target population should be considered
When identifying a target population in the assessment of community health needs for implementation of a community health program, a narrow definition is used for planning purposes. Community health programs provide care for all who request it, but they are designed specifically to meet the needs of the target population.
Incorrect Answers:
(A) The target population is a group of people in the defined service area who are unserved or underserved and are experiencing health disparities as a result.
(B) The target population is a group of people in the defined service area who are unserved or underserved and are experiencing health disparities as a result.
(C) It is usually a subset of the total population and can be identified by obtaining demographic and health status information about the community, the amount of accessible health care available, and barriers faced by people in the target population.
Vital Concept:
When identifying a target population in the assessment of community health needs for implementation of a community health program, a narrow definition is used for planning purposes. Community health programs provide care for all who request it, but they are designed specifically to meet the needs of the target population.
References:
The NP is conducting a medication follow-up appointment with a patient who has been stable on Clozaril, which was prescribed for Schizophrenia that had been unresponsive to other medication, for six months. The patient stated she was recently diagnosed with psoriasis by her primary care manager. The PMH NP completes a medication reconciliation and is concerned to find the patient’s primary care manager has prescribed this patient which medication?
A. Coal Tar Shampoo
B. Methotrexate
C. Tazarotene (Tazorac)
D. Calcipotriene (Dovonex)
Correct Answer: B.
Methotrexate
Taken orally, methotrexate helps psoriasis by decreasing the production of skin cells and suppressing inflammation. When methotrexate is used for long periods, it can cause a number of serious side effects, including severe liver damage and decreased production of red and white blood cells and platelets. Because Clozaril puts the patient at risk for leukopenia, adding methotrexate places the patient at increased risk for the reduction of white blood cells. An alternative treatment for psoriasis should be discussed with the patient and primary care manager and the methotrexate be discontinued.
Incorrect Answers:
A. Coal Tar is a thick, black byproduct of the manufacture of petroleum products and coal. It reduces scaling, itching and inflammation. Coal tar has few side effects, but it is messy and has a strong odor.
C. Tazarotene (Tazorac) is a Vitamin derivative that normalizes DNA activity in skin cells and my decrease inflammation. The most common side effect is skin irritation.
D. Calcipotriene (Dovonex) is a prescription cream or solution containing vitamin D analogue that may be used alone to treat mild to moderate psoriasis or in combination with other topical medications. The side effect of calcipotriene is skin irritation.
References:
A mother comes to the clinic for a psychiatric evaluation of her 18-month-old child. Her pediatrician has told her that he is developing slowly and may have autism. The child does make eye contact when spoken to, and even has been pointing with his index finger. With limited history, one could say he is developing within his age appropriate milestones if:
A. He is not yet aware of strange situations.
B. He sat upright at 22 weeks.
C. He does not yet understand peek a boo.
D. He can throw a ball and try to scribble.
Correct Answer: D.
He can throw a ball and try to scribble.
This is within his 18-month milestones and would be considered a normal development, if the child could not yet throw a ball or attempt to scribble, this may be an indication there are more delays involved. It will be important on the exam to be aware of any information that may lead you to believe the patient is or is not meeting their milestones. Milestones help diagnose many normal or abnormal developments in children and is a basic assessment skill.
Incorrect Answers:
A. At 16 weeks a baby should be aware of a strange situation and if they are not other delayed milestones need to be evaluated.
B. He should have been sitting up many months before. 4 months is the average age for this milestone.
C. At 40 weeks a baby should understand peek-a-boo and other simple games.
References:
Imaging the activity of specific brain regions in awake and behaving subjects has been one of the most important developments in the history of psychiatric and schizophrenia research. What is revealed by studies of hallucinations in individuals with schizophrenia?
A. There is decreased activity in the Heschl’s gyrus during auditory hallucinations.
B. Hallucinations have no relation to primary auditory cortex.
C. Hallucinations reflect cognitive disturbance.
D. Auditory hallucinations may be due to overactivation in the temporal-parietal cortex.
Correct Answer: D.
Auditory hallucinations may be due to overactivation in the temporal-parietal cortex.
Auditory hallucinations appear to be the result of abnormal activation of the neural system serving auditory sensory processing. In a study of patients with schizophrenia with auditory hallucinations, onset and offset of hallucinations correlated with engagement and disengagement of the primary auditory cortex. There is activation of Heschl’s gyrus during auditory hallucinations and in response to acoustical stimulation. Functional neuroimaging studies have shown overactivation in the temporal-parietal cortex during auditory hallucinations. Consequently, treatment of auditory hallucinations could be effected through the deactivation of this region.
Incorrect Answers:
A. There is activation of the Heschl’s gyrus during auditory hallucinations.
B. Hallucinations are related to the primary auditory cortex.
C. Hallucinations are a positive symptom of schizophrenia and are related to sensory disturbances.
Vital Concept:
Functional neuroimaging studies have shown overactivation in the temporal-parietal cortex during auditory hallucinations.
References:
An adult patient is referred for a psychiatric consultation by their primary care physician. The patient comes into the office or hospital on a regular basis, each time with different complaints that, after workup, have no organic cause. Although the patient has been referred to psychiatry several times in the past, they have not followed this suggestion. Records indicate that the patient has been to their primary care doctor once a month on average since they first moved to the area 10 years ago. On interview, the patient states that since they were a child, they have always worried about developing a severe medical illness. They go to the doctor so frequently because they often develop new symptoms and want to make sure that they have not developed a serious condition. The patient previously tried to get routine screening tests like a colonoscopy more regularly than suggested by their doctor. Their exams have always been normal, and other than suffering from the occasional cold, they have had a clean bill of health. Which of the following is the most likely DSM diagnosis?
A. Illness anxiety disorder
B. Body dysmorphic disorder
C. Generalized anxiety disorder
D. Somatization disorder
Correct Answer: A.
Illness anxiety disorder
Due to the patient’s preoccupation with health concerns despite good overall health, they likely have illness anxiety disorder. They want exams done more frequently than recommended, as they fear developing a disease. The diagnostic criteria for IAD include:
An obsession with having an illness or developing an illness
Usually, there are no physical symptoms of illness present; if present, symptoms are mild. If there is a medical condition present or a strong family history that puts the patient at an increased risk of developing a medical condition, the preoccupation with developing the condition is unequal and excessive compared to the individual’s risk
The individual exhibits extreme anxiety surrounding their health.
The individual performs unnecessary or excessively frequent health checks to monitor the development of illness and seeks medical treatment and screening more frequently than recommended or indicated based on presentation or health history (care-seeking type); may also avoid medical care, including routine check-ups with their provider or visits to the hospital when indicated due to fear of being diagnosed with an illness (care-avoiding type).
The preoccupation with health status has persisted for greater than 6 months. The illness that the patient fears developing may change over the 6-month period
The obsession with the development of an illness cannot be attributed to the presence of another mental health disorder.
Incorrect Answers:
B. Body dysmorphic disorder occurs when a person has an unrealistic belief that a body part is somehow malformed.
C. While illness anxiety disorder may cause the patient anxiety, generalized anxiety disorder is not the best diagnosis. The patient’s concerns and worries are not pervasive and present in multiple locations or in multiple situations. They are limited only to their physical health.
D. Although this syndrome may at first appear to be malingering, somatization, or even conversion disorder, the best diagnosis is illness anxiety disorder as there is no evidence of underlying secondary gain or conscious intentionality to the patient’s actions.
Vital Concept:
Illness anxiety disorder is characterized by consistent concern or worry about illness .
References:
A broad range of people may suffer from body dysmorphic disorder. Of the following four people, which is the most likely to be diagnosed with body dysmorphic disorder?
A. A 25-year-old woman who is concerned with breast asymmetry and consults a plastic surgeon for breast augmentation
B. A 30-year-old man who frequently visits his physician for evaluation of musculoskeletal pain after returning from military duty in Iraq
C. A 20-year-old woman who continues to visit multiple dermatologists for treatment of her “severe acne,” but physical evaluation reveals no blemishes
D. A 50-year-old woman who refuses to leave her home due to fear of outside world
Correct Answer: C.
A 20-year-old woman who continues to visit multiple dermatologists for treatment of her “severe acne,” but physical evaluation reveals no blemishes
This patient perceives that she has severe acne, although the objective views of her doctors reveal no problem. She has body dysmorphic disorder, which should be managed with psychological treatment.
Incorrect Answers:
A. Although plastic surgeons are likely to encounter patients with body dysmorphic disorder, due to their desire to fix their perceived physical flaws, there is nothing to suggest that this patient does not have uneven breasts. It is possible that this woman is completely warranted in her perception and would gain some benefit from surgery. If the patient had no physical asymmetry, then this could be diagnosed as body dysmorphic disorder.
B. This describes a pain disorder and would more likely be a somatization disorder or PTSD syndrome.
D. This describes agoraphobia.
References:
The PMH-NP is evaluating a 27 year old female diagnosed with Rheumatoid Arthritis (RA) three weeks ago. Today, during the appointment with the NP the patient states “I am feeling so down and am not sure if it is depression or the pain of my joints. I had labs repeated by my Primary Care Manager yesterday. Can you check to see if that new medicine she put me on last week has been effective to decrease my inflammation?” Which laboratory value would the NP review that is an indicator of inflammation?
A. Methotrexate
B. C-Reactive Protein
C. Uric Acid
D. B-type Natriuretic Peptide (BNP)
Correct Answer: B.
C-Reactive Protein
CRP, an acute phase reactant, is a protein made by the liver and released into the blood within a few hours after tissue injury, the start of an infection, or other cause of inflammation. Because the patient was recently put on medication to decrease inflammation, the recently drawn CRP should be lower than it was with when the rheumatoid arthritis was initially diagnosed if the medication the patient was started on is effective.
Incorrect Answers:
A. Methotrexate is a medication to treat (decrease or suppress) inflammation, not a lab test to detect inflammation.
C. Uric Acid is used to measure the level of uric acid in the blood and is indicative of gout. Uric acid lab value is not used to monitor rheumatoid arthritis.
D. B-type Natriuretic Peptide (BNP). When the BNP level is elevated, that is suggestive of heart failure. BNP is not used to monitor rheumatoid arthritis.
References:
An established patient agrees to pay the Nurse Practitioner directly for providing a specific service, which is a:
A. Fee for service system
B. Third party system
C. Contracted service system
D. Second party system
Correct Answer: A.
Fee for service system
Incorrect Answers:
B. Third party reimbursement includes private insurance and is the most common type of reimbursement.
C. The NP signs a contract and agrees to provide health care for a certain population, such as the homeless.
D. Second party reimbursement occurs when a legal guardian/guarantor makes a direct payment for services.
Vital Concepts:
Fee for service reimbursement occurs when the patient pays the nurse directly for specific services provided.
References:
Vagus nerve stimulation (VNS) uses a battery-powered device similar to a cardiac pacemaker.
What are FDA indications for VNS in the United States? Select all that apply.
A. Fibromyalgia
B. Refractory epilepsy
C. Movement disorders
D. Treatment-resistant depression in adults
E. Treatment-resistant depression in children in ages 12-17
Correct Answers:
B. Refractory epilepsy
D. Treatment-resistant depression in adults
E. Treatment-resistant depression in children in ages 12-17
Vagus nerve stimulation (VNS) is FDA indicated for treatment-resistant depression in adults and refractory epilepsy (ages 12 and up). In 2017 in the United States a noninvasive VNS “gammaCore” was FDA approved for cluster headaches in adults. Other areas of current study for the use of VNS are anxiety, obesity, and pain.
VNS is a battery-powered device similar to a cardiac pacemaker. In most models, stimulating leads are surgically placed around the left vagus nerve in the carotid sheath and are connected to an infraclavicular subcutaneous programmable pacemaker. Pulses stimulate the vagus nerve at regular intervals, typically for 30 seconds every 5 minutes.
Side effects include voice hoarseness, cough, neck pain, shortness of breath, infection, and nerve damage. Mechanism of action of VNS to treat depression is unclear but it is thought to be related to the vagus nerve stimulating the locus coeruleus and median raphe nucleus which could potentially stimulate serotonin and norepinephrine release. VNS is not typically covered by insurance.
Because the right vagus nerve provides more innervation to the cardiac atria than the left vagus nerve, electrical stimulation of the left vagus nerve is generally used in clinical practice to avoid adverse cardiac effects.
Incorrect Answers:
A. The FDA has approved the antidepressants duloxetine (Cymbalta) and milnacipran (Savella) as well as the anti-seizure medicine pregabalin (Lyrica).
C. A large, multisite clinical trial yielded the FDA approval of deep brain stimulation (DBS) in the subthalamic nucleus (STN) or globus pallidus internus (GPi) to treat motor symptoms in advanced Parkinson’s disease (PD) in 2002. Istradefylline (Nourianz) is a new drug approved by the FDA for pharmacologic management of off episodes in adults with PD.
Vital Concept:
Vagus nerve stimulation (VNS) is only FDA indicated for treatment-resistant depression ages 12 and older and refractory epilepsy (ages 12 and up) at this time.
References:
Which of the following neurobiological abnormalities is associated with autism spectrum disorders?
A. Delayed brain growth in first 6 months of life
B. Decreased formation of Purkinje cells in cerebellum
C. Delayed head growth in infancy
D. Abnormal structure of brainstem
Correct Answer: B.
Decreased formation of Purkinje cells in cerebellum
Brains of individuals with autism spectrum disorders (ASD) have decreased formation of Purkinje cells in the cerebellum. ASD represents a diversity of conditions that vary in severity from Asperger’s disease (mild symptoms, less cognitive impairment) to Rett’s disease (severe symptoms, profound impairment), with considerable variation between types. Although a variety of brain anomalies occur in patients with ASD, a single unifying brain process has not yet been identified that is common to all patients with ASD. Also, 7% of patients with ASD have been identified with abnormalities at the mitochondrial level. Abnormal brain connections or organizational and biochemical changes may contribute to ASD.
Incorrect Answers:
A. Patients with ASD have increased overall brain size.
C. Patients with ASD experience accelerated growth of the head during infancy.
D. Although other portions of the brain are thought to demonstrate abnormal structure in patients with ASD, the brainstem is not affected.
References:
Patients with intellectual disability may also have anxiety disorders. Which of the following statements is true of anxiety in patients with intellectual developmental disorder?
A. Anxiety disorders are rare in patients with intellectual disability.
B. If there is an acute change from baseline behavior, developmental transition is impending.
C. Sensory-integration interventions can be tried under guidance of a physical therapist.
D. Individuals with OCD and intellectual disability have increased anxiety with response-blocking intervention.
Correct Answer: D.
Individuals with OCD and intellectual disability have increased anxiety with response-blocking intervention.
Patients with stereotypy (repetitive, simple movements that can be voluntarily suppressed) will have increased anxiety with response-blocking intervention.
Incorrect Answers:
A. Anxiety disorders are common in patients with intellectual disability, and anxiety around transitions (daily and developmental) is often seen.
B. If there is an acute change from baseline behavior, trauma and posttraumatic stress needs to be considered. This population is particularly vulnerable to exploitation by others.
C. Sensory-integration interventions are employed under the guidance of a qualified occupational therapist.
Vital Concept:
Patients with stereotypy will have increased anxiety with response-blocking intervention.
References:
A 30-year-old man presents to his primary care physician for an annual check-up. When asked if he ever feels depressed, the patient initially says no, then admits that he was “previously having a really hard time in my marriage, but right now it is slightly better.” These symptoms have been going on for 3 months now, most days. He is having trouble getting enough sleep at night and is frequently waking at 4:00am and is not able to get back to sleep. His appetite has declined, and he has lost 10lbs since his last check-up 6 weeks ago. The patient previously contemplated death, and currently admits occasional suicidal permeations, yet has not formed a plan of carrying it out. He is also having problems concentrating at work. He talks about his kids (he and his wife have 2) and his worries for them. He worries what their lives will be like if they “turn out like” their dad, a “meaningless, no-consequence kind of person.” If this patient has a partial response to fluoxetine, which was initiated during his prior checkup, would augmentation be plausible?
A. Yes, augmentation is clinically indicated with a partial response.
B. No, the patient is unlikely to achieve remission with this agent.
C. No, a change to another antidepressant class is needed.
D. No, a full response is imminent, and the dose does not need to be changed.
Correct Answer: A.
Yes, augmentation is clinically indicated with a partial response.
According to the Texas STAR-D algorithm, an agent with a partial response can be augmented or changed. Augmentation is indeed plausible. Can also consider maximizing the primary agent before adjunct. This question asked if augmentation was plausible and it is a reasonable option. In the STAR-D trial at level one patient’s were trialed on citalopram, a selective serotonin reuptake inhibitor, and if did not reach remission advanced to level two. In level two patient’s where either switched to sertraline or venlafaxine; or an adjunct (bupropion or buspirone) was added to citalopram.
Incorrect Answers:
B. No, the patient is unlikely to achieve remission with this agent. This is a false statement. It is common for a patient to be able to achieve remission with an agent that patient did have not have full therapeutic effect to once an adjunct is added.
C. No, a change to another antidepressant class is needed. Changing to another antidepressant class is an option. However this answer also states that it is not reasonable to augment, of which it is.
D. No, a full response is imminent, and the dose does not need to be changed. Patient’s last check up was 6 weeks ago when the patient was started on a selective serotonin reuptake inhibitor (SSRI). It is reasonable to considering increasing the dose or adding an adjunct after a 6-week trial on a SSRI without therapeutic effect.
References:
A deletion on a certain chromosome increases schizophrenia risk. What is that chromosome?
A. 5
B. 7
C. 15
D. 22
Correct Answer: D.
22
Microdeletions in chromosomal region 22q11.2, including DiGeorge syndrome (DGS) and velocardiofacial syndrome, are associated with a 20-30x increased risk of schizophrenia. Studies provide various rates of 22q11.2 deletion syndrome in schizophrenia, ranging from 0.5-2.0% compared to the overall estimated 0.025% risk of the 22q11.2 deletion syndrome in the general population. About 30% of adults with DiGeorge syndrome have at least 1 incident of psychosis, and some studies suggest about 25% of these individuals develop schizophrenia.
Incorrect Answers:
A. 5. Microdeletion of 5p can lead to Cri-du-chat that consists of “cat like” cry, intellectually disabled, microcephaly, micrognathia and low set ears.
B. 7. Microdeletion of 7q can lead to Williams with a loss of the elastin gene leading to elf like face, overly friendly and intellectually disabled.
C. 15. Deletion of 15q11-q13 in mother (paternal imprinting) can lead to Angelman and deletion in father (maternal imprinting) can lead to Prader-Willi. Traits seen with Angelman include smiling, hyperactive, hypotonic, intellectually disabled, dysmorphic facial features and ataxic. Traits associated with Prader-Willi are short, obese, hyperphagia, intellectually disabled and dysmorphic facial features.
Vital Concept:
Microdeletions in chromosomal region 22q11 are associated with a 20-30x increased risk of schizophrenia.
References:
Children who have been sexually abused may manifest their trauma in a range of ways. Which of the following is a potential symptom of sexual abuse?
A. Functional somatic symptoms
B. Normal eating patterns
C. Reluctance to discuss sexual acts
D. Sleepwalking
Correct Answer: A.
Functional somatic symptoms
Medically not well-explained or functional somatic symptoms can be a symptom of sexual abuse in children. Children of sexual abuse can also have injuries, infections, pain, or bleeding in their genital area. Other possible symptoms include regression (bedwetting, thumb sucking, etc.), excessive talk about sexual acts or inappropriate knowledge of sexual acts, depression, anxiety, eating disorders and changes in school performance. Recognizing that “it’s normal to have a normal exam” is important in the evaluation of possible sexual abuse.
Incorrect Answers:
B. Children who have been sexually abused may have eating disorders and having normal eating patterns would not be an indication of sexual abuse.
C. Children who have been sexually abused are more likely to talk excessively and inappropriately about sexual acts.
D. Although sleep disturbances such as bedwetting and not wanting to sleep alone can be signs of sexual abuse, sleepwalking isn’t known to be a symptom of sexual abuse in children.
Vital Concept:
Given that primary care providers assess pediatric patients on a regular basis they are at an advantage of being able to assess the family structure as well as interactions between child and caregiver routinely. Signs of child maltreatment should be a part of the primary care NPs evaluation during clinic visits and knowing how to report and refer to the appropriate authority is a part of the NPs scope of practice.
References:
Ziprasidone an atypical antipsychotic used to treat certain mental and mood disorders, such as schizophrenia and bipolar disorder. Which of the following is true regarding its use?
A. Tardive dyskinesia can occur at lower rates with ziprasidone compared to other atypical antipsychotics.
B. QTc prolongation occurs often, and ECG monitoring is necessary.
C. Orthostasis due to alpha-1 blockade occurs commonly.
D. Weight gain and diabetes occur moderately with ziprasidone.
Correct Answer: A.
Tardive dyskinesia can occur at lower rates with ziprasidone compared to other atypical antipsychotics.
Although ziprasidone is associated with QTc prolongation, it is not clinically significant. There has been only 1 reported case of death secondary to cardiac arrhythmia. ECG monitoring is not advised, but the FDA warns that ziprasidone prolongs QTc and should not be used with other medications that prolong QTc. Tardive dyskinesia does occur, although rarely (see chart below).
Incorrect Answers:
B. Although ziprasidone is associated with QTc prolongation, the connection isn’t clinically significant, and EGC monitoring isn’t necessary
C. This does not occur due to ziprasidone
D. Very low weight gain is occasionally a rare side effect with ziprasidone use, but diabetes isn’t a side effect
References:
Cocaine and amphetamines act on the body differently. Which of the following statements accurately describes the distinction?
A. Cocaine prevents dopamine reuptake, but amphetamine both slows reuptake of dopamine and induces dopamine release.
B. Amphetamine prevents dopamine reuptake, but cocaine both slows reuptake of dopamine and induces dopamine release.
C. Both cocaine and amphetamine slow dopamine reuptake and induce dopamine release.
D. Cocaine prevents dopamine reuptake, but amphetamine induces dopamine release.
Correct Answer: A.
Cocaine prevents dopamine reuptake, but amphetamine both slows reuptake of dopamine and induces dopamine release.
Neurotransmitter activity is modulated by several mechanisms, including diffusion of the neurotransmitter out of the synaptic cleft, enzymatic degradation, and reuptake mechanisms. Monoamine neurotransmitter transporters mediate the reuptake process and are the sites of action of many drugs and neurotoxins. Cocaine prevents the reuptake of both dopamine and serotonin, while amphetamines not only slow the reuptake of dopamine and serotonin but also induce the efflux of dopamine by the presynaptic neuron.
Incorrect Answers:
B. These definitions are switched. Amphetamine slows reuptake of dopamine and induces dopamine release; it doesn’t entirely prevent dopamine uptake. Cocaine prevents dopamine reuptake; it neither slows reuptake of dopamine nor induces dopamine release.
C. It’s true that amphetamine slows reuptake of dopamine and induces dopamine release; however, cocaine doesn’t do this. It prevents dopamine reuptake.
D. These statements are both true, but the statement of how amphetamine acts is incomplete. In addition to inducing dopamine release, amphetamine also slows reuptake of dopamine.
References:
An elderly male patient with hypertension requires 4 medications for control of his blood pressure. He says he has Medicare Part D. Which of the following is true of Medicare Part D?
A. All Medicare patients are not eligible for Part D
B. All prescription drugs are included in Part D
C. Medicare Part D is a prescription drug benefit
D. Non-formulary drugs are never covered under Medicare Part D
Correct Answer: C.
Medicare Part D is a prescription drug benefit
Medicare Part D is also known as the Medicare prescription drug benefit. Only individuals enrolled or eligible for Medicare Part A or Part B are eligible. The Medicare Advantage plan is one type of Part D coverage. All prescription drug plans have a formulary of preferred drugs. If a non-formulary drug is used, it may not be covered and the patient will have to pay for it out-of-pocket.
Incorrect Answers:
A. Medicare Part A and B patients are enrolled and eligible patients are eligible for Medicare Part D
B. Not all prescription drugs are covered by Medicare Part D
D. Some non-formulary drugs may be covered
Vital Concepts:
Medicare Part D is also known as the Medicare prescription drug benefit. Only individuals enrolled or eligible for Medicare Part A or Part B are eligible. The Medicare Advantage plan is one type of Part D coverage. All prescription drug plans have a formulary of preferred drugs. If a non-formulary drug is used, it may not be covered and the patient will have to pay for it out-of-pocket.
References:
In behavioral couples therapy, the initial sessions are spent doing a functional analysis of behavior (FAB). Which of the following is true about FAB?
A. FAB is based on operant conditioning.
B. FAB is based on classical conditioning.
C. FAB focuses on understanding cognitions associated with behavior.
D. FAB is based on Gestalt conditioning.
Correct Answer: A.
FAB is based on operant conditioning.
FAB is based on identifying antecedent stimuli, operant behaviors, and consequent stimuli. FAB may suggest treatment strategies such as stimulus control.
Incorrect Answers:
B. While FAB does identify stimuli that are possibly conditioned, the goal of FAB is modifying willful operant behaviors.
C. FAB is concerned only with observable stimuli connected to operant behavior.
D. Gestalt theory deals with perceptual psychology, not behavior. There is no theory of Gestalt conditioning.
References:
L-carnitine can treat methamphetamine toxicity. What is the mechanism by which it acts to treat methamphetamine toxicity?
A. Increasing both free radicals and ATP formation
B. Preventing formation of free radicals and peroxynitrite in neurons
C. Increasing vitamin E
D. Increasing ATP
Correct Answer: B.
Preventing formation of free radicals and peroxynitrite in neurons
Neurotoxicity is mediated by the formation of peroxynitrite, which can be reduced by antioxidants or L-carnitine. L-carnitine is needed to transport long-chain fatty acids into the mitochondria for fatty acid oxidation, preventing the generation of free radicals and peroxynitrite.
Incorrect Answers:
A. It decreases free radicals in the neuron.
C. Vitamin E acts as antioxidant, but L-carnitine has no effect on it.
D. L-carnitine is needed to transport long-chain fatty acids into the mitochondria for fatty acid oxidation.
References:
In order to bill for services provided to Medicare beneficiaries, the care provided and NPs must meet certain criteria. Which of the following is one of the criteria when billing services for Medicare beneficiaries?
A. The care provided must comply with state restrictions and supervision requirements
B. The care must be provided under direct supervision
C. Care must be provided in a rural health clinic (RHC) or federally qualified health center (FQHC)
D. Charges must be billed through a physician-directed clinic, health agency, or hospital
Correct Answer: A.
The care provided must comply with state restrictions and supervision requirements
Nurse Practitioners may bill Medicare for services in accordance with state restrictions and supervision requirements.
Incorrect Answers:
B. Some states require direct supervision by a physician on the premises while others require indirect or periodical supervision.
C and D. The Centers for Medicare and Medicaid Services (CMS) pay for NP services that are: Medically necessary; equivalent to physician services; accurately documented on medical records; and billed correctly. Medicare may directly reimburse the NP, if state law allows it.
Vital Concepts:
Nurse Practitioners may bill Medicare for services in accordance with state restrictions and supervision requirements.
References:
Sumatriptan is used to treat migraine and cluster headaches. What is its mechanism of action?
A. 5-HT1A agonist
B. 5-HT1D and 5-HT1B agonist
C. 5-HT2C agonist
D. 5-HT7 agonist
Correct Answer: B.
5-HT1D and 5-HT1B agonist
Sumatriptan is a 5-HT1D and 5-HT1B agonist.
Incorrect Answers:
A. Sumatriptan is a 5-HT1D, not 5-HT1A, agonist. It’s also a 5-HT1B agonist.
C. D. Aripiprazole is a 5-HT2C agonist and a weak partial agonist at 5-HT7.
Tricyclic antidepressants (TCAs) may be used to treat ADHD. Which of the following statements is true about TCA use for ADHD?
A. TCAs are renally metabolized.
B. TCA plasma levels have been shown to be related to efficacy.
C. Special supervision of administration or storage may be necessary.
D. TCAs have more robust evidence in treating ADHD than stimulants.
Correct Answer: C.
Special supervision of administration or storage may be necessary.
Overdose is a significant adverse event associated with TCA use. Special supervision of administration and storage is important. TCAs have some evidence to be efficacious in adult ADHD but less effective than stimulants. TCAs have a wide range of side effects and are poorly tolerated compared to stimulants and other non-stimulant medications utilized for ADHD. Common side effects include dry mouth, constipation, weight gain, urinary hesitation, blurred vision and sexual dysfunction. Rare cardiac side effects (e.g. arrhythmias, AV block, QT prolongation, orthostasis) are the primary concern with TCAs especially in overdoses. Monitor EKG at baseline, at dosage increase, and every 3-4 months if on maintenance treatment. TCA plasma levels have not been shown to be related to efficacy but should be monitored at higher doses.
Incorrect Answers:
A. TCAs are metabolized primarily through the liver, not renally.
B. TCA plasma levels have NOT been shown to be related to efficacy.
D. TCAs have LESS robust evidence in treating ADHD than stimulants.
Vital Concept:
TCAs have some evidence to be efficacious in adult ADHD but less effective than stimulants. Due to poor tolerable and overdose toxicity risk, TCAs are rarely used for ADHD. If utilized special supervision of administration and storage is important due to overdose toxicity.
Which neurotransmitter has the most evidence for its association with obsessive-compulsive disorder (OCD)?
A. Norepinephrine
B. Serotonin
C. Dopamine
D. Glutamate
Correct Answer: B.
Serotonin
Serotonin is the most studied neurotransmitter in OCD and is the basis for the well-established efficacy of potent serotonin reuptake inhibitors in OCD treatment. Serotonin is thought to play a fundamental role in the pathogenesis of obsessions and compulsions. Patients with OCD have dopaminergic hyperfunction in the prefrontal cortex (mesocortical dopamine pathway) and serotonergic hypofunction in the basal ganglia. Recent evidence also implicates dysregulation in glutamate as well.
Incorrect Answers:
A. Norepinephrine works on the beta 1 adrenergic receptors as well as the alpha-adrenergic receptors that can increase heart rate and blood pressure. There are some drug classes such as SNRIs and NDRIs that are used to help treat depression related disorders.
C. Dopamine is a neurotransmitter within the brain. Most often we see dopamine affected with ADHD. In schizophrenia medications are used to block dopamine (antipsychotics). Patients with OCD have dopaminergic hyperfunction in the prefrontal cortex but serotonin is the neurotransmitter that has the most evidence for its association with OCD.
D. Glutamate is an excitatory CNS neurotransmitter. Decrease in function of both glutamate and NMDA have been associated with Schizophrenia. Other common neurotransmitters involved with Schizophrenia include GABA, Acetylcholine, and dopamine. There is supporting research that glutamate modulating drugs can be used with the treatment of OCD but serotonin is the neurotransmitter that has the most evidence for its association with OCD.
Vital Concept:
Serotonin is the most studied neurotransmitter in OCD and is the basis for the well-established efficacy of potent serotonin reuptake inhibitors in OCD treatment.
The DSM-5-TR has four alcohol-related disorders. Which of them involves craving as a criterion?
A. Alcohol use disorder
B. Alcohol withdrawal
C. Other alcohol-induced disorder
D. Unspecified alcohol-related disorder
Correct Answer: A.
Alcohol use disorder
Alcohol use disorder, as defined in the DSM-5-TR, includes craving as a criterion for diagnosis.
Incorrect Answers:
B. Alcohol withdrawal does not involve craving as a criterion for diagnosis.
C. Other alcohol-induced disorder does not involve craving as a criterion for diagnosis.
D. Unspecified alcohol-related disorder does not involve craving as a criterion for diagnosis.
Vital Concept:
Alcohol use disorder includes the criterion of an intense need or impulse to drink alcohol.
A school-age child presents for evaluation due to an irrational fear of burglars. The child checks all of the locks in the house repeatedly, even in the middle of the night. The child acknowledges that the symptoms are disruptive but explains that they “can’t help it.” They need to check every lock 7 times, “or else the burglars are gonna get in.” They admit to waking in the middle of the night on several occasions and describe an intense need to perform the “lock check.” When the child is prevented from doing this, they become upset and inconsolable. Eventually, the caregiver says, “I have to let them finish.” Both caregiver and child agree that the symptoms are interfering with their ability to meet both family and school-related responsibilities. What other symptoms are they at risk for developing?
A. Narcolepsy
B. Below-average intelligence
C. Eye blinking and throat clearing
D. Fear of animals
Correct Answer: C.
Eye blinking and throat clearing
This child is presenting with symptoms of obsessive-compulsive disorder (OCD). Tourette’s syndrome is associated with OCD, showing substantial comorbidity (up to 30%) in family studies and suggesting a shared genetic diathesis. Eye blinking and throat clearing are the most common initial motor and vocal tics, respectively. OCD is also comorbid with anxiety disorders, depressive disorder, and bipolar disorders. In children there can be a triad seen between OCD, tic disorder and attention-deficient/hyperactivity disorder (ADHD). There are also associations between OCD and body dysmorphic disorder, trichotillomia, excoriation disorder, and oppositional defiant disorder. Patients with OCD are also commonly diagnosed with anxiety disorders (76%), mood disorders (63%), and substance use disorders (39%).
Incorrect Answers:
(A) There is no correlation between OCD and sleep/wake disorders.
(B) There is no correlation between OCD and below-average intelligence.
(D) There is no correlation between OCD and increased risk of fear of animals.
Vital Concept:
OCD and Tourette’s syndrome
Mr. Jones is a 54-year-old man who is undergoing treatment for depression. The patient presents complaining that since starting his medication, he has been unable to fall asleep at night and feels “wired.” What antidepressant is he most likely taking?
A. Citalopram
B. Fluoxetine
C. Paroxetine
D. Mirtazapine
Correct Answer: B.
Fluoxetine
Fluoxetine (selective serotonin reuptake inhibitor) and bupropion (norepinephrine and dopamine reuptake inhibitor) are antidepressants that can increase insomnia and activation. When starting a patient on an SSRI and the patient could benefit from activation then consider fluoxetine (Prozac). Fluoxetine is FDA approved for major depressive disorder, obsessive-compulsive disorder, panic disorder, bulimia, and premenstrual dysphoric disorder. Also used off-label in anxiety disorders and posttraumatic stress disorder. Dose range is 20 to 80 mg daily and has a long half-life of 4-6 days.
Incorrect Answers:
A. Citalopram. Citalopram can be associated with dose dependent insomnia but also sedation. Fluoxetine is the SSRI that is most associated with activation and is the best answer choice.
C. Paroxetine. Paroxetine is associated with sedation, not insomnia.
D. Mirtazapine. Mirtazapine is an antidepressant with noradrenergic and serotonergic activity. It is sedating and is utilized to help patients with insomnia and appetite stimulation. Lower doses are more sedating because at higher doses there is increased noradrenergic effect compared to antihistaminergic effect.
Vital Concept:
Fluoxetine (selective serotonin reuptake inhibitor) and bupropion (norepinephrine and dopamine reuptake inhibitor) are antidepressants that can increase insomnia and activation.
Otto F. Kernberg’s theories on narcissistic personality disorder are often contrasted with those of Heinz Kohut. Which statement accurately describes Kernberg’s approach to treating a patient with narcissistic personality disorder?
A. Narcissist and borderline patients share core features of borderline personality organization.
B. Narcissist patients use self-objects to regulate self-esteem due to deficiencies in their internal structure.
C. Emphasis of therapeutic work is on internalizing missing functions.
D. Diagnosis of narcissistic personality disorder is based on predominant transference established (i.e. self-object transferences involving mirroring and idealization).
Correct Answer: A.
Narcissist and borderline patients share core features of borderline personality organization.
Kernberg asserts that the narcissist and borderline patient both share a borderline personality organization.
Incorrect Answers:
B. Borderline personality organization consists of pathological internalized object relations.
C. D. These accurately depict how Kohut conceptualized and treated people with narcissistic personality disorder. Overall, Kohut tended to be more supportive and introspective, while Kernberg gently confronted defenses more actively.
Legislation is an important part of how health care is shaped. Which of the following statements about the legislative arena is true?
A. The most intense consideration of proposed legislation occurs when it arrives on the floor for a vote
B. All proposed legislation from a session is considered by the appropriate committee
C. The agenda for the committee is established by the Speaker of the House
D. Nurses can request an opportunity to testify in a committee
Correct Answer: D.
Nurses can request an opportunity to testify in a committee
The centers of policy making in a legislative body at the state or federal level are committees, which is where proposed legislation receives the greatest scrutiny. After legislation is proposed, the committee chairman sets an agenda for consideration of a policy or bill by negotiating with members of the committee. The procedures of a committee serve the function of gatekeeping, by determining which pieces of legislation will be considered, out of the many proposed actions that arrive from the members of a legislative body. Meetings of a committee are held to educate members of the committee and the public about the bill that is being considered. During a hearing, individuals can request the opportunity to testify, although representatives of large organized groups are more likely to get those opportunities. Individuals may still influence the committee by speaking privately to staffers or members. Individual nurses may request the opportunity to speak to a committee, but may be more successful as part of a large nursing organization with shared vision.
Incorrect Answers:
A. The most intense consideration of proposed legislation occurs within a committee.
B. The committee has a gatekeeper function and determines which of the many pieces of proposed legislation will be considered and sent for a vote.
C. The committee chair sets the agenda for the committee by negotiation with committee members.
Vital Concepts:
During a hearing, individuals can request the opportunity to testify, although representatives of large organized groups are more likely to get those opportunities. Individuals may still influence the committee by speaking privately to staffers or members. Individual nurses may request the opportunity to speak to a committee but may be more successful as part of a large nursing organization with a shared vision.
There are three main components to traditional behavioral couples therapy. What are these three components?
A. Behavioral exchange, communication training, and problem-solving
B. Behavioral exchange, communication training, problem-solving, and cognitive restructuring
C. Behavioral exchange, imaginal exposure, and problem-solving
D. Successive approximation, communication training, and problem-solving
Correct Answer: A.
Behavioral exchange, communication training, and problem-solving
Behavioral couples therapy involves behavioral exchange, communication training, and problem-solving.
Incorrect Answers:
B. Cognitive restructuring is a component of cognitive-behavioral therapy.
C. Imaginal exposure is a component of cognitive-behavioral therapy.
D. Successive approximation is an operant conditioning technique used to develop complex behaviors, often in animals.
Professional nursing associations may play a role in political advocacy. Which of the following is true of such organizations’ political advocacy?
A. Nursing associations may sponsor state legislative days
B. Nursing associations should be politically neutral with respect to election campaigns
C. A few phone calls by a nursing association generally can change healthcare policies
D. Nursing associations are prohibited by law from forming Political Action Committees (PACs)
Correct Answer: A.
Nursing associations may sponsor state legislative days
Membership in a professional nursing association can help nurses become familiar with political advocacy. Professional associations have the resources to monitor and disseminate important policy information that affects healthcare. Many state nursing and specialty nursing societies sponsor annual state legislative days and may also conduct policy workshops to educate members about the current issues in health care and about the legislative process. Some organizations provide experienced mentors for members who would like to become involved in political advocacy for the first time. There are many opportunities for a nurse interested in influencing the formation of health policy.
Incorrect Answers:
B. Professional associations often support candidates who are supportive of nursing and healthcare issues.
C. Complex issues usually require the concerted efforts of many individuals who work together to reach a common goal.
D. Some professional associations have formed political action committees that determine where candidates stand on important issues and how their membership can help those candidates who support nursing and health care improvement.
Vital Concepts:
Professional associations have the resources to monitor and disseminate important policy information that affects healthcare. Many state nursing and specialty nursing societies sponsor annual state legislative days and may also conduct policy workshops to educate members about the current issues in health care and about the legislative process.
A 36-year-old male with a 7-year history of catatonia has had a poor response to several trials of typical antipsychotics, benzodiazepines, risperidone, and olanzapine. His catatonic symptoms and agitated behavior have prevented him from returning to his group home. What is the best treatment at this point?
A. Benzodiazepines
B. Beta-blockers
C. ECT
D. Lithium
Correct Answer: C.
ECT
Electroconvulsive therapy (ECT) may continue to have a role in the rapid treatment of acute and subacute states that are refractory to pharmacological intervention, particularly catatonia. ECT is quite safely administered, with minimal short-term adverse events and no evidence for long-term morbidity. Short-term cognitive dysfunction is common. For this patient, ECT is recommended as an adjunctive therapy to control behavioral symptoms as well as agitation with the continual of antipsychotic medications. Use of lithium is a relative contraindication given its association with post-ECT delirium. Use of anti-epileptics and benzodiazepines may reduce seizure time and increase seizure threshold.
Antipsychotics are the mainstay of treatment for schizophrenia. First-generation antipsychotics, such as haloperidol, bind the dopamine-2 receptors with high affinity and antagonize them, blocking dopamine release. In general, a blockade of 80% or more of the receptors is associated with increased extrapyramidal effects, while a lower blockade (60-70%) is safer and still clinically effective. Second-generation antipsychotics also block dopamine-2 receptors, but each second-generation drug has a different binding affinity (e.g. risperidone is high, and clozapine and quetiapine are lower). Both first- and second-generation antipsychotics bind with muscarinic, histaminergic, and adrenergic receptors as well, which may also contribute to their clinical effects. Second-generation drugs are more effective for treating negative symptoms. First-generation antipsychotics are also used to treat acute psychosis (at higher than maintenance dosage). Beta-blockers and lithium can be used for aggression and impulsivity in schizophrenics, and antidepressants can be used to treat their depression or anxiety disorders. ECT can also be used in schizophrenia for catatonia. Early studies also suggest that TMS might be useful in treating hallucinations. Case management, vocational rehab, family therapy, social work, and CBT are important in the treatment success of schizophrenia.
Incorrect Answers:
A. Benzodiazepines can be used in conjunction with other antipsychotics for sedation. In this refractory case, ECT is an ideal recommendation.
B. Beta-blockers are used for the treatment of akathisia, aggression, restlessness, and impulsivity in conjunction with the treatment of schizophrenia.
D. Lithium is used for managing aggressive and impulsive behaviors in conjunction with other treatment modalities in schizophrenia. For catatonic symptoms, ECT is a better conjunction therapy for behavioral management.
TCAs may have cardiac side effects. Which of the following statements regarding TCAS’ cardiac side effects is true?
A. TCAs are associated with decreased risk of myocardial infarction (MI) when compared to SSRIs.
B. Cardiac conduction defects are most severe with nortriptyline.
C. TCAs are structurally similar to class II antiarrhythmics.
D. Ventricular arrhythmia can result from TCA overdose.
Correct Answer: D.
Ventricular arrhythmia can result from TCA overdose.
Ventricular arrhythmia and seizures are seen with TCA overdose; cardiovascular effects are a frequent cause of death in overdoses.
Incorrect Answers:
A. TCAs are associated with an increased risk of MI when compared to SSRIs. It is unclear whether SSRIs reduce a depression-mediated decreased MI risk or whether TCAs increase the risk of MI associated with depression.
B. Cardiac conduction defects are most severe with desipramine.
C. TCAs are structurally similar to class I antiarrhythmics that are actually proarrhythmic in 20% of the population; approximately 20% of patients with preexisting conduction disturbances have cardiac complications while taking TCAs.
Somatic symptom disorder (SSD) diagnostic criteria are defined in DSM-5-TR. Which of the following is true of how the DSM defines SSD?
A. Patients with SSD feel reassured by medical visits.
B. Symptoms must be present for at least 1 week.
C. Symptoms must be medically unexplained.
D. Symptoms can be associated with another medical condition.
Correct Answer: D.
Symptoms can be associated with another medical condition.
SSD diagnosis does not require that somatic symptoms are medically unexplained. In other words, symptoms may or may not be associated with another medical condition or diagnosis. To meet the criteria for SSD, patients must have:
At least one somatic symptom that substantially impacts the patient’s daily functioning or leads to anguish
The patient then develops extraordinary actions, emotions, or ruminations regarding the symptom(s) or related medical condition(s), as evidenced by one or more of the following:
· consistent worry or concern about their physical wellbeing
· extraordinary devotion of resources (e.g., energy, time) to the symptom(s) or related medical condition(s)
· ruminations that are consistent and unbalanced regarding the gravity and meaning underlying these symptoms
These conditions persist for at least 6 months, although the specific symptom of focus may vary or shift during that time
Specifiers include:
· with prominent pain if the symptoms involve physical discomfort
· persistent if the symptoms are severe, enduring (greater than 6 months), and cause substantial dysfunction
· mild/moderate/severe based on the number of symptoms listed above that are present (1/2/2+ multiple symptoms)
Incorrect Answers:
A. SSD patients are rarely reassured by medical visits and may feel that their medical care is insufficient even with frequent visits.
B. Symptoms need to persist for six or more months to be diagnosed.
C. SSD diagnosis doesn’t require that somatic symptoms be medically unexplained. Symptoms may or may not be associated with another medical condition or diagnosis. However, to meet the criteria for SSD, patients must have one or more chronic somatic symptoms and meet other criteria.
Vital Concept:
Somatic symptom disorder (SSD) is a disease with somatic symptoms that must be significantly distressing or disruptive to daily life and must be accompanied by excessive thoughts, feelings, or behaviors.
John wants to graduate from high school and his grades suggest he will. He currently has no plans for attending college. He spends all his time playing video games and struggles with socializing. He doesn’t feel like he fits in with any of his other peers and doesn’t really know who he is. Which of Erikson’s developmental stage is John struggling with?
A. Industry vs. Inferiority
B. Identity vs. Role confusion
C. Intimacy vs. Isolation
D. Generativity vs. Self-absorption or stagnation
Correct Answer: B.
Identity vs. Role confusion
John is struggling with the developmental task of Identity vs. role confusion. This is evidenced by no plans for the future and no sense of identity or belonging. Industry vs. inferiority is more related to sense of accomplishment and the ability to work, which John is not struggling with. Intimacy vs. isolation could have been considered due to the lack of social relationships, however identify vs. role confusion was a better answer due to the lack of self-identity. Generativity vs. self-absorption or stagnation is experienced during middle adulthood.
Incorrect Answers:
A. This stage involves learning to read and create.. Presumably, John is beyond this stage (which should be ages 5-12).
C. This stage is about seeking intimacy to avoid feeling isolated. John is too young for this stage (which should be ages 18-40).
D. This stage involves establishing careers and families. John is too young for this stage (which should be ages 40-65).
The Health Insurance Portability and Accountability Act (HIPAA) was passed by Congress in 1996. Which of the following is covered under HIPPA?
A. Mandates industry-wide standards for paper billing
B. Requires the protection and confidential handling of protected health information
C. Requires health insurers to cover pre-existing conditions
D. Makes children up to age 25 eligible to stay on their parents’ health insurance plans
Correct Answer: B.
Requires the protection and confidential handling of protected health information
HIPAA addresses the rights of the individual and family to continue insurance after a job loss or change and to maintain the privacy of their health information. HIPAA is the acronym for the Health Insurance Portability and Accountability Act that was passed by Congress in 1996. HIPAA does the following: - Protects the transfer of health information for continuity of care - Reduces health care fraud and abuse - Mandates industry-wide standards for health care information on electronic billing and other processes - Requires the protection and confidential handling of protected health information
Incorrect Answers:
A. HIPAA mandates industry-wide standards for health care information on electronic billing, not paper billing
C. This is a mandate of the Affordable Care Act.
D. This is a mandate of the Affordable Care Act.
Vital Concept:
HIPPA was created to “improve the portability and accountability of health insurance coverage” for employees between jobs. Once it was established, the Privacy Rule was enacted for Protected Health Information (PHI) giving patients permission the right to privacy of their health information and the ability to withhold it in certain circumstances. Under HIPPA, patients have a right to privacy of their PHI.
In a healthcare institution, certain people are responsible for identifying performance improvement projects. Who are these people?
A. Administrative staff
B. Nursing team leaders
C. All staff
D. Physicians
Correct Answer: C.
All staff
All staff members are responsible for identifying performance improvement projects.
Incorrect Answers:
A. Continuous Quality Improvement (CQI) is a management philosophy that emphasizes the organization structure, and system processes, rather than individuals.
B and D. Total Quality Management (TQM) is a management philosophy to meet the needs of customers (patients and staff) at all levels.
Vital Concept:
All staff members are responsible for identifying performance improvement projects.
A new study describes a rapid screening test for autism. The test has a positive predictive value of 2.5% and a negative predictive value of 99.9%. Which of the following is the correct interpretation of this data?
A. Expect that a child testing positive would have a 2.5% chance of not having the condition.
B. Expect that a child testing positive would have a 97.5% chance of having the condition.
C. Expect that a child testing negative would have a 99.9% chance of not having the condition.
D. Expect that a child testing negative would have a 0. 1% chance of not having the condition.
Correct Answer: C.
Expect that a child testing negative would have a 99.9% chance of not having the condition.
Positive predictive value (PPV) is the probability that patients with a positive screening test have the condition. In this question, for a patient who has a positive screening test, there is a 2.5% probability that he or she has autism (see figure below). This test has a poor PPV because only 2.5%, or 1 out of 40, of the patients with a positive screen, will ultimately be diagnosed with the condition.
Negative predictive value (NPV) is the probability that patients with a negative screening test do not have the condition. If a child screens negative, there is a 99.9% probability that he or she does not have the condition. This test has a good NPV given the fact if the child’s screen is negative there is a 99.9% probability that he or she does not have the condition (the test was correct).
Positive Predictive Value = (Condition / Screen Positive) = a/(a+b).
Negative Predictive Value = (Condition-Free / Screen Negative) = d/(c+d).
Has the condition Does not have the condition Total
Screen Positive a b a+b
Screen Negative c d c+d
a+c b+d N
Incorrect Answers:
A. A positive predictive value of 2.5% indicates that a child who tests positive would only have a 2.5% chance of having the condition.
B. Since only 2.5% of patients who screen positive will ultimately have the condition, the remaining 97.5% of children testing positive will not have the condition. This means that a child testing positive would have a 97.5% chance of not having the condition.
D. Since 99.9% of patients who have a negative screening do not have the condition, this means the remaining 0.1% of patients who screen positive do have the condition. Therefore, a negative predictive value of 99.9% indicates that the probability of still having the condition with a negative screening is only 0.1%.
Vital Concept:
Positive predictive value (PPV) is the probability that patients with a positive screening test have the condition. Negative predictive value (NPV) is the probability that patients with a negative screening test do not have the disease.
Kleptomania is a compulsion to commit theft. Which of the following is a characteristic of kleptomania?
A. Items stolen are of great value
B. The patient feels great pleasure/relief while committing the theft
C. Items stolen are always the same thing
D. Patients work in pairs
Correct Answer: B.
The patient feels great pleasure/relief while committing the theft
The DSM-5-TR defines kleptomania as:
The repetitive urge to steal objects, even when they have no financial value or are not required or essential. The patient is unable to ignore these urges.
The individual experiences increased strain and pressure directly before committing the crime.
Once the crime is committed, the individual feels relaxed, satisfied, and fulfilled.
The theft is not related to the presence of hallucinations or delusions and is not committed due to animosity or the need for retribution.
Actions are not better explained by the presence of another mental health disorder, including conduct disorder, antisocial personality disorder, or a manic episode.
Incorrect Answers:
A. Items stolen are not of great value.
C. Items stolen are not always the same thing
D. Patients may or may not work in pairs
Vital Concept:
A defining characteristic of kleptomania is the elevated strain leading up to the act of stealing, followed by fulfillment and joy at the time of stealing.
Valproic acid is used to treat seizures and bipolar disorder and prevent migraine headaches. Which of the following statements regarding poisoning of valproic acid is true?
A. Valproate overdose can lead to CNS depression and is often fatal.
B. Severe poisoning that involves a coma is typically above a level of 850mg/L.
C. Hemodialysis is never used to treat valproate overdose.
D. Thrombocytosis and cerebral edema commonly occur.
Correct Answer: B.
Severe poisoning that involves a coma is typically above a level of 850mg/L.
Incorrect Answers:
A. Valproic acid overdose can lead to CNS depression but is rarely fatal.
C. Hemodialysis can also aid the removal of valproate.
D. Thrombocytopenia and cerebral edema are associated with valproate poisoning but are rare
Vital Concept:
Key signs and symptoms of VPA toxicity include: CNS depression, lethargy and potential encephalopathy, respiratory depression, nausea/vomiting, and myoclonus. Lab abnormalities often include, in addition to elevated serum VPA levels, hypernatremia, elevated anion gap metabolic acidosis, hyperosmolality, hypocalcemia, and hyperammonemia with a transaminitis. A non-contrast head CT may demonstrate cerebral edema, the peak occurrence of which is between 12 hours and 4 days after ingestion. Activated charcoal can be given early if the timing of overdose is known. Serial head CT scans should be ordered to monitor the possible delayed side effect of cerebral edema.
A man presents complaining that when he leaves his house, he always returns several times to check if the door is locked. He also checks the locks on his car. The patient is always delayed at work because of these actions. What is the most appropriate treatment?
A. Buspirone
B. Chlorpromazine
C. Clomipramine
D. Imipramine
Correct Answer: C.
Clomipramine
This patient is suffering from obsessive-compulsive disorder. Clomipramine (a tricyclic antidepressant) and the selective serotonin reuptake inhibitors (SSRIs) are effective in treating this disorder.
Incorrect Answers:
A. This is a non-benzodiazepine anxiolytic that does not have marked sedative or euphoric effects. Unlike the benzodiazepines, buspirone does not have hypnotic, anticonvulsant, or muscle relaxant properties.
B. This is an antipsychotic, and is not indicated to manage OCD.
D. This is a tricyclic antidepressant, but it is not used to manage OCD.
In addition to implementing the Electronic Health Record, Title XII of the American Recovery and Investment Act of 2009 (known as the HITECH Act) contains provisions to promote the use of health information technology to improve health care quality, safety, and efficiency. What is the name of the standards that EMRs were required to meet in order to be eligible for reimbursement through the HITECH act?
A. Privacy rule
B. Meaningful use
C. Electronic health systems
D. Efficiency rule
Correct Answer: B.
Meaningful use
The HITECH Act contains provisions for “meaningful use” of health information technology. Meaningful use is described in Section 4101 of the HITECH Act as e-prescribing, engaging in health information exchange, and submitting information regarding quality measures. The goal of the provision is to increase the use and reporting of outcome measures and to increase the exchange of electronic patient information in order to achieve changes in provider behaviors. The Centers for Medicare and Medicaid Services have defined objectives for meaningful use that are categorized under major policy initiatives, which include promoting patient and family engagement; reducing health care disparities and improving quality, safety, and efficiency; improving care coordination and population and public health; and maintaining privacy and security of patient health information.
The objectives are to evolve over a 5-year period in 3 stages:
- Stage 1: Data sharing and capture
- Stage 2: Advance clinical process
- Stage 3: Improve outcomes
Incorrect Answers:
A. The privacy rule — which ensures the confidentiality of patient health information — is set forth in HIPAA
C. This term simply describes the system used to store digital versions of patients’ health records
D. This merely describes the pursuit of efficiency in healthcare
Vital Concepts:
The HITECH Act contains provisions for the “meaningful use” of health information technology. Meaningful use is described in Section 4101 of the HITECH Act as e-prescribing, engaging in health information exchange, and submitting information regarding quality measures.
A nurse practitioner notices a trend of longer length of stay among patients admitted to a particular unit in the local hospital. A review of charts shows that patients who have undergone surgical procedures and are later admitted to this unit have an average length of stay (LOS) of 1.4 additional hospital days as a result of wound infection. The investigation identifies practices in the unit that are impediments to hand washing and infection containment. They decide to implement changes to existing practices to improve outcomes. Which of the following programs will they most likely implement?
A. Risk management
B. Quality improvement
C. Case management
D. Accreditation
Correct Answer: B.
Quality improvement
Quality improvement programs use patient outcome measures as surrogate markers of the quality and health of the program. Goals include improvement in quality of care, decreased complications, decreased hospitalizations, and improved patient satisfaction and outcomes. Risk management programs identify risky practices to identify and minimize adverse patient outcomes and corporate liability. These may include things like falls, iatrogenic infection, and medication errors. Accreditation refers to voluntary compliance in a program of evaluation and recognition that the providers have complete special additional training in a field, through education, experiment, and evaluation (examination) to indicate they are performing by meeting these requirements and have achieved advanced knowledge in the field.
Incorrect Answers:
A. Identifies risky practices to identify and minimize adverse outcomes and corporate liability
C. Collaborative process of assessment, planning, facilitation, care coordination, evaluation and advocacy for options and services to meet an individual’s and family’s comprehensive health needs
D. Refers to voluntary compliance in an evaluation and recognition program
Vital Concepts:
Quality improvement programs use patient outcome measures as surrogate markers of the quality and health of the program. Goals include improvement in quality of care, decreased complications, decreased hospitalizations, and improved patient satisfaction and outcomes.
Various drugs have different side effects. Which of the following drug/side effect pairings is correct?
A. MAOI - pyridoxine deficiency
B. MAOI and morphine - seizures and delirium
C. Bupropion - pyridoxine deficiency
D. Nefazodone - weight loss
Correct Answer: A.
MAOI - pyridoxine deficiency
MAOIs can cause pyridoxine deficiency.
Incorrect Answers:
B. An MAOI in combination with meperidine can result in seizures and delirium.
C. Bupropion can lower the seizure threshold and should be avoided in patients with a seizure disorder or an eating disorder.
D. Nefazodone is not known to cause weight loss (however, it’s known to increase appetite, which may lead to weight gain)
Certain treatment strategies can help adolescents diagnosed with psychotic depression recover more rapidly. Which of the following strategies can help adolescents diagnosed with psychotic depression recover more rapidly?
A. Antidepressants combined with antipsychotics
B. Antidepressants combined with mood stabilizers
C. Antipsychotics used alone
D. Mood stabilizers used alone
Correct Answer: A.
Antidepressants combined with antipsychotics
Patients with psychotic depression appear to recover more rapidly when antidepressants are combined with antipsychotics. Antipsychotics carry the risk of tardive dyskinesia and should be tapered after remission of the psychosis. Atypical antipsychotics are generally used as an alternative to typical antipsychotics but carry an increased risk of type 2 diabetes and dyslipidemia; they should also be tapered after remission of psychosis. Anecdotal reports and literature reviews suggest that ECT may be efficacious for psychotic, depressed adolescents. Currently there are a limited number of studies accessing combinations of antidepressants and antipsychotics for this population and more research need to be done.
Incorrect Answers:
B. Antidepressants combined with antipsychotics, not mood stabilizers, have shown to help adolescents diagnosed with psychotic depression recover more rapidly.
C. Antidepressants combined with antipsychotics have shown to improve response verses antipsychotics alone in adolescents diagnosed with psychotic depression.
D. Antidepressants combined with antipsychotics have shown to improve response verses mood stabilizers alone in adolescents diagnosed with psychotic depression.
Vital Concept:
Patients with psychotic depression appear to recover more rapidly when antidepressants are combined with antipsychotics. Antipsychotics should be tapered after remission of the psychosis.
A 32-year-old pregnant female with a history of bipolar I disorder presents to the ER in labor. She is at 39 weeks gestational age. Her water soon breaks, and she gives birth to a healthy baby girl; however, the baby has small outpouching on her back directly in the midline. After initial workup, the protuberance is diagnosed to be a myelomeningocele. Which of the following medications was the child most likely exposed to in utero?
A. Valproate
B. Lithium
C. Lamotrigine
D. Levetiracetam
Correct Answer: A.
Valproate
Valproate is commonly used for mood stabilization in bipolar disorder. Exposure during pregnancy has been associated with an increased risk of neural tube defects such as myelomeningocele, meningocele, or spina bifida. Carbamazepine and oxcarbazepine are other mood stabilizers commonly associated with an increased risk of neural tube defects.
Incorrect Answers:
B. Lithium is a mood stabilizer that has been associated with cardiac defects, such as Ebstein’s anomaly, in newborns exposed to this medication in utero.
C. Lamotrigine is a mood stabilizer, and in utero exposure during the first trimester has been associated with a higher risk of a cleft lip or palate.
D. Levetiracetam is not a preferred medication for bipolar disorder and is typically reserved for those who have failed other, more efficacious, medications. In utero exposure has not been studied significantly in humans, so there is no common anomaly associated with this medication at this time.
In couples therapy, the therapist looks for ways to understand how the marriage is “stuck.” Which approach is the therapist using?
A. Structural-strategic approach
B. Experiential humanistic approach
C. Transgenerational approach
D. Behavioral approach
In the method of Aaron Beck, the therapist seeks to bring the patient’s “automatic” thoughts into awareness and to demonstrate how these thoughts affect behavior and feelings. What do automatic thoughts mediate?
A. Events and affective and behavioral responses
B. Event and emotion
C. Emotion and lack of response
D. Action and subsequent emotional response
Correct Answer: A.
Events and affective and behavioral responses
Negative automatic thoughts mediate between experience and action, causing feelings and actions associated with depression. CBT is a data-based short-term therapy that focuses on relapse prevention. The underlying principle of CBT is cognitive interventions. Open-ended questions are used to evaluate the accuracy of a patient’s assumptions and perceptions. Patients are taught how to examine the evidence that they distort into automatic thoughts. Other topics that are addressed are catastrophic thinking, mind-reading, and personalization. Interventions used in CBT include imaginal exposure (exposure to feared thoughts or memories), interoceptive exposure (exposure to bodily sensations), gradual exposure (as opposed to flooding), response prevention, and systematic desensitization. Behavioral assignments and skill rehearsal are used to test beliefs and to ensure that patients can utilize certain skills.
Incorrect Answers:
B. Automatic thoughts mediate events, but not emotion.
C. Automatic thoughts mediate response, but not emotion.
D. Automatic thoughts mediate neither action nor subsequent emotional response.
References:
Rumination is focused attention on the symptoms of one’s distress and its possible causes and consequences (as opposed to its solutions). Which of the following statements about this is true?
A. It is never associated with gastrointestinal diseases.
B. It’s always associated with behavior problems
C. It isn’t a form of self-stimulation behavior
D. It’s associated with overstimulation when associated with anxiety
Correct Answer: D.
It’s associated with overstimulation when associated with anxiety
Incorrect Answers:
A, B. These are both partially true. Rumination is associated with both gastrointestinal diseases and behavior problems
C. Rumination may be part of self-stimulation behavior and anxiety.
References:
Pharmacokinetics is the branch of pharmacology concerned with the movement of drugs within the body. Which of the following pairings of concept and definition is correct?
A. First-pass elimination: extensive drug extraction and metabolism in liver immediately after absorption from gut
B. Bioavailability: serum concentration of drug
C. Cmax: time after ingestion to maximum plasma concentration of drug
D. Tmax: maximum plasma concentration of drug
Correct Answer: A.
First-pass elimination: extensive drug extraction and metabolism in liver immediately after absorption from gut
First-pass elimination is extensive drug extraction and metabolism in the liver immediately after absorption from the gut.
Incorrect Answers:
B. Bioavailability is the fraction of the drug that reaches the systemic circulation and is available to exert a biological effect on target tissues.
C. Cmax is the maximum plasma concentration of the drug.
D. Tmax is not a term typically used in pharmacokinetics. Key time related measure are the time to peak plasma or half-life.
References:
Prader-Willi syndrome has three key features. Which of the following is one of these features?
A. Genetic abnormality occurs at chromosome 7.
B. Patients have short stature with hypogonadism, hyperphagia, and obesity.
C. Generalized anxiety disorder is the most associated psychiatric disorder.
D. Level of insight with regard to obsessions and compulsions is low.
Correct Answer: B.
Patients have short stature with hypogonadism, hyperphagia, and obesity.
In this disorder, patients have short stature with hypogonadism, hyperphagia, and obesity.
Incorrect Answers:
A. The genetic abnormality in 70% of patients is a deletion at chromosome 15.
C. Obsessive-compulsive disorder is the most associated psychiatric disorder.
D. Level of insight with regard to obsessions and compulsions may vary, but there is often an ego-dystonic aspect. Verbalizations for help may also occur in patients with higher cognitive function.
References:
Christopher Peterson and Martin Seligman have a “positive psychology” definition of mental health. Which of the following item(s) is from Peterson and Seligman’s definition?
A. Efficient self-perception
B. Realistic self-esteem and acceptance
C. Voluntary control of behavior
D. Generosity and nurturance
Correct Answer: D.
Generosity and nurturance
Generosity and nurturance are part of Christopher Peterson and Martin Seligman’s definition of mental health “positive psychology,” which also includes kindness, love, intimacy, social intelligence, forgiveness, modesty, curiosity, courage, humor, leadership, hope, spirituality, honesty, and open-mindedness.
The other answer choice options are a part of Marie Jahoda’s definition of normal mental health (“mental health” or “normality”). In 1958, Marie Jahoda defined normal mental health as including the following six criteria: efficient self-perception, realistic self-esteem and acceptance, voluntary control of behavior, true perception of the world, sustaining relationships, and self-direction/productivity.
Incorrect Answers:
A. Efficient self-perception is a part of Marie Jahoda’s definition of normal mental health, not Christopher Peterson and Martin Seligman’s definition.
B. Realistic self-esteem and acceptance are a part of Marie Jahoda’s definition of normal mental health, not Christopher Peterson and Martin Seligman’s definition.
C. Voluntary control of behavior is a part of Marie Jahoda’s definition of normal mental health, not Christopher Peterson and Martin Seligman’s definition.
Vital Concept:
Christopher Peterson and Martin Seligman’s definition of mental health is referred to as “positive psychology” and includes kindness, generosity, nurturance, love, intimacy, social intelligence, forgiveness, modesty, curiosity, courage, humor, leadership, hope, spirituality, honesty, and open-mindedness.
References:
A nurse practitioner in psychiatric practice is interested in mental health advocacy. Which of the following is true?
A. Personal experiences with mental illness should be kept confidential.
B. When referring to someone with schizophrenia, the nurse should use the term “schizophrenic.”
C. Nurses should avoid casual use of terms of phrases that could be offensive, such as “I’m so bored with this class that I am going to slit my wrists.”
D. Advocacy is a personal decision, not a professional call to action.
Correct Answer: C.
Nurses should avoid casual use of terms of phrases that could be offensive, such as “I’m so bored with this class that I am going to slit my wrists.”
Policy and legislation impact the practice of health care providers. It’s important for practitioners to be knowledgeable about current policies and legislation that affects mental health and substance use care since these problems contribute significantly to the burden of illness and disability in the population. When advocating for mental health, it’s critical to be aware of the power of words and the way they can stigmatize people with mental illness. When referring to someone with a mental illness, it is better to use a phrase like “persons with mental illness” compared to “mentally ill persons.” Another sphere in which someone can inadvertently be offensive is the use of a casual phrase, such as “psycho” or “I’m going to slit my wrists.” You can never be certain what experience others may have had.
Incorrect Answers:
A. Sharing the personal experience with mental health issues with friends and colleagues may normalize these issues and empower others.
B. To call someone “schizophrenic” is similar to calling patients with cancer “cancers.” An NP should refer to these patients as persons with schizophrenia.
D. Advocacy is both a personal and professional call to action and all nurses should be responsible for advocacy for the health and quality of life of their patients.
References:
Acute stress disorder (ASD) and PTSD may interact in ways that are important for health care providers to understand. Which of the following statements about ASD and PTSD is true?
A. Estimated lifetime prevalence of ASD is 7.8%.
B. Most people exposed to traumatic events develop ASD or PTSD.
C. Lifetime prevalence of PTSD is higher in men than in women.
D. Some patients develop PTSD without prior having ASD.
Correct Answer: D.
Some patients develop PTSD without prior having ASD.
Some patients develop PTSD without prior ASD, although some patients with ASD do not develop PTSD.
Incorrect Answers:
A. Estimated lifetime prevalence of PTSD is 7.8%. The lifetime prevalence of ASD is unknown.
B. About 50-90% of people experience traumatic events, but most do not develop ASD or PTSD.
C. Lifetime prevalence of PTSD is higher in women than men.
References:
Which of the following statements about acute intermittent porphyria (AIP) is true?
A. Usual onset of disease is 10-20 years.
B. Abdominal pain, psychiatric symptoms, and seizures that are highly responsive to barbiturates are common physical findings.
C. Phenothiazines are safe to treat psychosis.
D.
Eating a diet with high amounts of protein is recommended during the AIP attack.
Correct Answer: C.
Phenothiazines are safe to treat psychosis.
Most seizure medications exacerbate this condition. Medicines known to be safe include the phenothiazines, glucocorticoids, narcotic analgesics, and bromides. Gabapentin and levetiracetam (Keppra) are generally safe, but phenytoin, barbiturates, or valproic acid must be avoided. Olanzapine can be used for psychosis and nausea.
Incorrect Answers:
A. The onset is between ages 18-40 years of life.
B. Acute intermittent porphyria may manifest as: (1) episodic, acute, abdominal pain; (2) motor polyneuropathy; and (3) psychosis. However, it may cause purely psychiatric symptoms, including insomnia, anxiety, mood lability, and psychosis.
D. Eating a diet with high amounts of protein is recommended during the AIP attack is incorrect as it is recommended during an attack the diet should consist of high carbohydrates.
References:
Patient education can be an effective form of intervention. Which of the following metrics best determines the effectiveness of an educational intervention?
A. Patient satisfaction
B. Patient’s ability to report understanding
C. Patient’s ability to explain and demonstrate understanding
D. Patient’s behavior modification and compliance rates
Correct Answer: D.
Patient’s behavior modification and compliance rates
Behavior modification and compliance rates are the best determinants of the effectiveness of patient education.
Incorrect Answers:
A, B, and C. The patient may be satisfied, may understand, and may be able to provide a demonstration. But, if the patient does not use what he learned, your education of the patient was ineffective.
Vital Concept:
Behavior modification and compliance rates are the best determinants of the effectiveness of patient education.
References:
Which process describes the mechanism by which the hippocampus stores memories?
A. Neurogenesis
B. Apoptosis
C. Long-term depression
D. Long-term potentiation
Correct Answer: D.
Long-term potentiation
Brief high-frequency stimulation of hippocampal pathways leads to long-term potentiation (LTP) of the synaptic connections and is the crucial process underlying memory formation. LTP is composed of an early phase that lasts for the first 3 hours after induction and does not require protein synthesis and a late phase that lasts for several hours and requires both gene transcription and protein translation. LTP can also stimulate the growth of new synapses and increase the rate of synaptic transmission via the release of additional synaptic vesicles. Short-term forms of learning generally involve the modification of existing proteins in response to sensitizing stimuli.
Long-term memory, however, requires changes in gene transcription and the synthesis of new proteins to support the structural changes necessary to strengthen the synapse. Learning and memory require both short- and long-term changes that lead to dramatic increases in the range of responses that a neuron shows to synaptic input. Over time, activation of a single receptor may trigger a coordinated cellular response involving several systems.
Incorrect Answers:
A. Neurogenesis is the formation of neurons.
B. Apoptosis is programmed cell death.
C. Long-term depression involves similar mechanisms as long-term potentiation and eliminates irrelevant memories while fine-tuning lasting memories.
References:
Which of the following is a factor that would indicate the need for maintenance therapy of an antidepressant in a patient with major depression?
A. No prior episodes
B. Later age of onset
C. Family history of mood disorders
D. Substance abuse
Correct Answer: C.
Family history of mood disorders
Maintenance therapy is generally recommended for patients with risk factors for recurrence such as the presence of residual symptoms, ongoing psychosocial stressors, early age at onset, and family history of mood disorders. Patients who have had ≥3 prior major depressive episodes should receive maintenance treatment.
Incorrect Answers:
A. Recommended that patients who have had ≥3 prior major depressive episodes should receive maintenance treatment.
B. EARLIER age of onset is a factor for continuing maintenance therapy of an antidepressant in a patient with major depression.
D. History of substance abuse is not considered a factor for continuing maintenance therapy of an antidepressant in a patient with major depression.
Vital Concept:
Maintenance therapy is recommended with risk factors for recurrence such previous depressive episodes, current residual symptoms, ongoing psychosocial stressors, early age at onset, and family history of mood disorders.
References:
The cytochrome p-450 enzymes are very important for drug metabolism and can be affected by enzyme inducers and inhibitors. Which of the following statements regarding the cytochrome P450 enzyme system is true?
A. The cytochrome P-450 enzymes only function to metabolize foreign drugs and chemicals that enter the body.
B. Phase I metabolism in the liver is characterized by glucuronidation.
C. The majority of cytochrome p-450 enzymes are found in equal amounts in the liver and kidney.
D. CYP1A2, 2C9, 2C19, 2D6, 3A4, and 3A5 enzymes are responsible for metabolizing over 90% of drugs processed in the liver.
Correct Answer: D.
CYP1A2, 2C9, 2C19, 2D6, 3A4, and 3A5 enzymes are responsible for metabolizing over 90% of drugs processed in the liver.
Of the more than 50 Cytochrome p450 enzymes, CYP1A2, 2C9, 2C19, 2D6, 3A4, and 3A5 enzymes are responsible for metabolizing over 90% of drugs processed in the liver.
Incorrect Answers:
A. Cytochrome P-450 enzymes function both to metabolize foreign drugs and chemicals AND produce synthetic substances for the body such as cholesterol and steroids.
B. Phase I metabolism involving the CYP 450 system is characterized by oxidation, reduction, and hydrolysis.
C. The majority of cytochrome p-450 enzymes are found in the liver and small intestine. Smaller amounts of the enzymes can be found in the kidney, lung, brain, and placenta.
References:
Once an individual has detoxified successfully from alcohol, maintenance of the abstinence is the next goal, which is a difficult goal to achieve. It has been estimated that approximately 50% of patients with chronic alcohol use disorder relapse within 3 months after completing treatment. Which of the following is not a recommended medication for relapse prevention in detoxified patients with chronic alcohol use disorder?
A. Disulfiram
B. Acamprosate
C. Chlordiazepoxide
D. Topiramate
Correct Answer: C.
Chlordiazepoxide
All these drugs are used for the maintenance treatment of alcohol dependence except chlordiazepoxide, which is used in acute detoxification of alcohol It is not favored for relapse prevention in patients with chronic alcohol use disorder. Psychosocial support remains the cornerstone in achieving prevention of relapse.
Incorrect Answers:
A. Disulfiram serves as an alcohol deterrent drug by interrupting the metabolism of alcohol. The patient’s fear of this disulfiram–alcohol reaction is meant to serve as reinforcement for abstinence.
B. Acamprosate is an anti-craving drug and helps the maintenance of abstinence and decreases negative symptoms associated with the acute post-withdrawal period in recently detoxified alcohol-dependent individuals.
D. Topiramate is an anti-craving drug used to prevent alcohol relapse.
References:
A 64-year-old man with a history of major depressive disorder is admitted to the geriatric psychiatry inpatient unit after 2 months of worsening depression. His symptoms include depressed mood, social withdrawal, and a steady decline in self-care. The patient is a widower with a history of alcohol dependence and has been sober for the last 13 years. He has a history of 7 psychiatric hospitalizations for depressive episodes dating back to his early 20s. His past medical history includes high cholesterol treated with atorvastatin. His psychiatric medications include sertraline 100mg and lithium 300mg daily. The patient saw his psychiatrist 6 weeks ago and stopped taking all his medications 1 week prior to admission. On exam, he has poor eye contact, significantly delayed response time, and psychomotor slowing. He also expresses a wish to die and see his wife again. Since admission, he has refused all treatment, including routine blood work, vitals, and medication. He has refused to eat for the last few days. The patient achieved remission from a depressive episode in his late 50s after receiving ECT. The inpatient psychiatric team is granted a court order for treatment over his objection, including blood work, medication administration, and ECT. What is the most commonly reported side effect of ECT treatment?
A. Anterograde amnesia
B. Retrograde amnesia
C. Transient global amnesia
D. Expressive aphasia
Correct Answer: B.
Retrograde amnesia
Amnesia is the most commonly discussed side effect of ECT. Severe amnesia during the ECT course can be managed by increasing the interval between treatments (from 3 per week to 2 or even 1 per week), changing ECT type (e.g. bilateral to unilateral electrode placement), or, if necessary, ending treatment. Retrograde amnesia (i.e. difficulty remembering information learned before the ECT course) is more prominent with ECT use. This deficit is greatest for more recent memories, particularly those occurring several months before the ECT. Retrograde amnesia is more marked for information of an impersonal nature. The proportion of patients with persistent retrograde amnesia following ECT is unknown; patient surveys suggest that this may occur in a sizable minority.
Pharmacological attempts to ameliorate ECT-induced amnesia (e.g. using nootropics, hormones, stimulants, and peptides) have not been reliably demonstrated. Amnesic effects with ECT are more prominent and may last longer in patients with preexisting cerebral disease, a greater number of treatments in a course, or bilateral stimulus electrode placement, particularly when treatment frequency is maintained at 3x weekly. Ironically, greater amnesia is generally associated with a greater antidepressant response.
Incorrect Answers:
A. and C. Retrograde, not anterograde or transient, amnesia is the most commonly discussed side effect of ECT
D. No connection between ECT and expressive aphasia
References:
Which statement regarding tricyclic antidepressants (TCAs) and anticholinergic side effects is true?
A. Anticholinergic effects result from blockade of nicotinic cholinergic receptors.
B. Sedation and hypertension are examples of anticholinergic side effects.
C. TCAs can be dangerous to patients with pre-existing glaucoma and lead to narrow-angle glaucoma.
D. Patients with dementia have minimal side effects on TCAs.
Correct Answer: C.
TCAs can be dangerous to patients with pre-existing glaucoma and lead to narrow-angle glaucoma.
The anticholinergic side effects of TCAs include dry mouth, blurry vision, constipation, tachycardia, and delirium. In those with a history of benign prostatic hypertrophy this may result in acute urinary retention.
Incorrect Answers:
A. Anticholinergic effects result from the blockade of muscarinic cholinergic receptors.
B. Hypotension is due to alpha-1 receptor blockade, while sedation is most likely related to histamine receptor blocking.
D. Patients with dementia can develop acute confusional states due to the anticholinergic side effects of TCAs.
Vital Concept:
The side effects associated with TCAs, including anticholinergic effects, are largely responsible for their infrequent use since the introduction of newer antidepressants such as SSRIs and SNRIs.
References:
Family therapy may have a here-and-now focus that encourages change through growth experiences. Which of the following schools of family therapy uses this approach?
A. Psychodynamic
B. Experiential
C. Structural
D. Strategic
Correct Answer: B.
Experiential
The experiential approach focuses on the present and encourages change through growth experiences.
Incorrect Answers:
A, The psychodynamic approach focuses on the past that is causing present problems. It also addresses multigenerational transmission and encourages change through insight.
C. The structural approach focuses on the blueprint of a healthy family with well-defined subsystems, clear boundaries, and parents in charge.
D. The strategic approach requires a change that interrupts maladaptive behavior sequences.
References:
A 40-year-old male presents to the emergency room with flushing, sedation, and confusion. His vitals are BP 80/55, RR 12/min, and pulse 60/min irregular with pupillary constriction, clouded sensorium, and poor coordination. Which medication is the appropriate antidote in this emergency?
A. Acamprosate
B. Bupropion
C. Buprenorphine
D. Naloxone
Correct Answer: D.
Naloxone
Naloxone is a drug of choice for opioid intoxication. This patient is suffering from opioid overdose. Nausea, vomiting, and severe itching can also occur as part of opioid intoxication. Fatal respiratory depression can occur due to direct suppression of respiratory centers in the midbrain and medulla. Opioid intoxication is a medical emergency that is treated by adequate hydration, mechanical ventilation, and naloxone.
Incorrect Answers:
A. Acamprosate is an anti-craving drug used for maintenance therapy of alcohol abstinence.
B. Bupropion is used for nicotine withdrawal and acts on dopamine and noradrenaline receptors.
C. Buprenorphine is used for detoxification and as a part of maintenance therapy in opioid withdrawal.
References:
What percentage of individuals with specific learning disorder with impairment in reading are male?
A. 36-45%
B. 46-55%
C. 56-65%
D. 66-75%
Correct Answer: D.
66-75%
About 66-75% of patients with specific learning disorder are male. The ratio of males to females is 2-3:1. Specific learning disorder is a DSM-5-TR diagnosis that combines the prior diagnoses of reading disorder, mathematics disorder, disorder of written expression, and learning disorder not otherwise specified.
Specific learning disorder is defined as
Challenges acquiring or utilizing new mental skills or capabilities, as evidenced by one or more of the following for a period lasting 6 months or more despite attempts to address those challenges:
· challenges with reading comprehension (reading to understand)
· challenges writing with proper punctuation, grammar, organization, and intelligibility
· challenges with numerical facts/calculation (math skills) OR reasoning (concepts)
· challenges reading individual words (may guess, read words incorrectly, unable to sound them out, or read extremely slow)
· challenges with spelling words correctly
These challenges are not as expected given the patient’s age and substantially hinder educational, professional, or daily functioning as evidenced by standardized tests or clinical evaluation.
In patients over 17, historical evidence of challenges may replace standardized testing.
While these challenges emerge while in primary school, the patient may be capable of compensating, and thus delay their observation until the requirements exceed the patient’s abilities (e.g., a significant deadline, increased education responsibilities, time-based testing)
These challenges are not more accurately attributed to a poor teaching/access to education, pervasive intellectual disability, decreased language proficiency, vision or hearing deficit, or another mental or medical condition.
Specifiers:
· r/t reading: word reading accuracy; reading rate/fluency; reading comprehension
· r/t writing: spelling accuracy; grammar and punctuation accuracy; clarity or organization of written expression
· r/t mathematics: number sense; memorization of arithmetic facts; accurate or fluent calculation; accurate math reasoning
· mild: one or two domains affected but able to complete tasks with accommodations
· moderate: substantial challenges in at least one domain requiring intense support/assistance
· severe: several domains affected requiring intense support/assistance continuously and accommodations. Despite this, patient may still not complete all work.
Incorrect Answers:
A. B. and C. These percentages are all lower than the correct response of 66-75%.
Vital Concept:
Specific learning disorder is more common in males than females, with ratios that range from 2:1 to 3:1.
References:
The use of antidepressants during pregnancy can carry risks to the fetus. Which of the following is a potential neonatal risk associated with antidepressant use during the third trimester of pregnancy?
A. Pregnancy loss or miscarriage
B. Organ malformation or teratogenesis
C. Gestational diabetes
D. Poor neonatal adaptation
Correct Answer: D.
Poor neonatal adaptation
Correct Answer: D. Poor neonatal adaptation
Adverse effects in the newborn following SSRI/SNRI exposure in the third trimester include agitation/restlessness, respiratory distress, cyanosis, apnea, seizures, temperature instability, feeding difficulty, vomiting, hypoglycemia, hypo- or hypertonia, hyperreflexia, jitteriness, irritability, constant crying, and tremor. Symptoms may be due to SSRIs/SNRIs toxicity or the result of poor neonatal adaptation (or neonatal behavioral syndrome). The symptoms may be similar to those seen in serotonin syndrome associated with SSRI treatment. Discontinuation syndrome is seen at higher rates with paroxetine, compared to other SSRI/SNRI antidepressants. Incidence varies in studies from 5-85%. Symptoms are typically mild and self-limiting, beginning within 72 hours of birth and largely resolving within 2 weeks. Discontinuation syndrome is more likely to occur when paroxetine is used during the late third trimester.
At this time SSRIs have been found to pose minimal risk of teratogenicity. They do not appear to increase the risk of spontaneous abortion, gestational diabetes, or perinatal death. They may increase the risk of cardiac malformation (especially paroxetine), hypertension during pregnancy, postpartum hemorrhage, preterm birth (before 37 weeks), and persistent pulmonary hypertension in the newborn. The data regarding the effect of SSRIs on birth weight and Apgar scores are conflicting.
Incorrect Answers:
A. Pregnancy loss or miscarriage is not a potential neonatal risk associated with continuing antidepressants during the third trimester of pregnancy.
B. Organ malformation or teratogenesis is not a potential neonatal risk associated with continuing antidepressants during the third trimester of pregnancy.
C. Gestational diabetes is not a potential neonatal risk associated with continuing antidepressants during the third trimester of pregnancy.
Vital Concept:
The decision to continue antidepressants through the third trimester should be made with extensive patient education via shared decision-making between the patient and the prescriber. There is a risk of transient mild symptoms of poor neonatal adaptation following delivery.
References:
Which of the following is a reason to use motivational interviewing with children and adolescents?
A. Children and adolescents are already motivated to change.
B. Children and adolescents are relieved to get a diagnostic label.
C. Children and adolescents prefer to be told what to do by a parental figure.
D. Child and adolescent resistance can be managed with less resistance and hostility.
Correct Answer: D.
Child and adolescent resistance can be managed with less resistance and hostility.
MI is well-suited to treatment with children and adolescents. Of note, it is often used in substance abuse disorders to assess readiness to change and to help patients progress in their commitment when ready.
Incorrect Answers:
A. Children and adolescents are likely to be resistant to change.
B. Children and adolescents dislike diagnostic labels.
C. Children and adolescents most likely to resist the advice of parental figures.
References:
Which of the following scenarios is generally considered unethical?
A. Having notes on a therapy patient, keeping them out of the medical record, and denying the patient access to them
B. Charging patients a fee for missed appointments
C. Hiring a patient to do lawn maintenance
D. Declining a box of chocolates from a patient
Correct Answer: C.
Hiring a patient to do lawn maintenance
Having any type of financial or business relationship with a patient is considered an exploitation of the physician’s authority and the therapeutic relationship. This would include buying or selling anything to/from a patient, investing in or accepting an investment from a patient, or hiring a patient. This involves the beneficence principal as providers need to do the best treatment that is best for the patient, not the provider.
Incorrect Answers:
A. Having separate therapy notes and denying patient access to those notes is not considered unethical.
B. Charging fees for missed appointments or late arrivals are not considered unethical.
D. Declining gifts is not unethical; in fact, accepting a gift is generally unethical except in certain situations where the gift’s value is negligible (e.g. homemade goods or a bottle of water).
Vital Concepts:
Having any type of financial or business relationship with a patient is considered an exploitation of the physician’s authority and the therapeutic relationship. This would include buying or selling anything to/from a patient, investing in or accepting an investment from a patient, or hiring a patient. This involves the beneficence principal as providers need to do the best treatment that is best for the patient, not the provider.
References:
A 37-year-old patient is discharged from the inpatient psychiatric unit on sertraline, risperidone, benztropine, and clonazepam after treatment for an acute worsening of schizoaffective disorder. One week later they present to the emergency room with a fever and clouded sensorium but no diaphoresis. Labs are drawn, but results are not yet available. The patient is given a trial of physostigmine preemptively, and the symptoms temporarily improve. What is the most likely diagnosis?
A. Neuroleptic malignant syndrome secondary to antipsychotic overdose
B. Anticholinergic delirium secondary to benztropine overdose
C. Serotonin syndrome secondary to sertraline
D. Malignant hyperthermia
Correct Answer: B.
Anticholinergic delirium secondary to benztropine overdose
Anticholinergic delirium is distinguished from neuroleptic malignant syndrome (NMS) by a lack of diaphoresis and a reduction in symptoms after a trial of physostigmine. NMS, serotonin syndrome, and malignant hyperthermia all present with elevated temperature and clouded sensorium.
Incorrect Answers:
A. NMS is diagnosed by severe muscle rigidity and fever as well as two or more of the following: hypertension or labile blood pressure, change in the level of consciousness, leukocytosis, elevated CPK, elevated liver enzymes, tremor, incontinence, mutism, dysphasia, or tachycardia. Treatment includes immediate discontinuation of antipsychotics, IV hydration, temperature control, and in some cases, ventilator support or dialysis. In severe cases, admission to the ICU is warranted. NMS is fatal in about 10% of cases. Treatment is with dopaminergic agonists, including levodopa/carbidopa, dantrolene, amantadine, and bromocriptine, as well as benzodiazepines and ECT.
C. In serotonin syndrome, the tremor is peripheral and is less likely to have fever or lab abnormalities common to NMS. Serotonin syndrome presents with tremors, confusion, restlessness, myoclonus, autonomic changes, and hyperreflexia. The most common interaction between drugs causing the reaction involves SSRIs and MAOIs. Treatment is the discontinuation of the offending agent and supportive treatment. Symptoms usually resolve within 24 hours. Labs are drawn to rule out NMS (chemistry, blood count, and CPK).
D. Malignant hyperthermia is caused by a genetic predisposition to hyperthermia, rhabdomyolysis, and possible kidney failure secondary to exposure to general anesthesia. This patient has had no exposure to general anesthetics.
Vital Concept:
Fever and clouded sensorium may represent benztropine overdose (anticholinergic delirium), neuroleptic malignant syndrome, or serotonin syndrome. Benztropine overdose will respond to physostigmine administration.
References:
Schizophrenia and homelessness interact in important ways that are essential for health care providers to understand. Which of the following statements regarding the relationship between homelessness and schizophrenia is true?
A. Schizophrenia has no correlation with homelessness.
B. Deinstitutionalization has no correlation with increased numbers of homeless persons with schizophrenia.
C. Substance use disorders contribute to homelessness in patients with schizophrenia.
D. Only 10% of homeless patients with schizophrenia receive mental health treatment.
Correct Answer: C.
Substance use disorders contribute to homelessness in patients with schizophrenia.
Substance use disorders, including alcohol, contribute to homelessness in patients with schizophrenia.
Incorrect Answers:
A. Other risk factors for homelessness include social isolation, infectious disease. and substance use.
B. Other factors include lack of affordable housing, fragmented services, and limited public funding.
D. Slightly less than 50% of homeless people with schizophrenia receive psychiatric services.
References:
Lamotrigine is an anticonvulsant medication used to treat seizures and bipolar disorder. Which of the following statements regarding its side effects and overdose risk is true?
A. Toxic epidermal necrolysis is the only serious dermatological side effect.
B. A rash with facial involvement, mucus membrane involvement, or fever and lymphadenopathy requires immediate emergency evaluation.
C. Initial side effects (neurological, nausea, vomiting, and rash) are common.
D. Lamotrigine overdose is treated (in part) with dialysis.
Correct Answer: B.
A rash with facial involvement, mucus membrane involvement, or fever and lymphadenopathy requires immediate emergency evaluation.
Mucus membrane involvement includes dysuria and tongue/mouth/eye lesions. This can be life-threatening.
Incorrect Answers:
A. Both Stevens-Johnson syndrome and toxic epidermal necrolysis are associated with lamotrigine use.
C. Initial side effects (dizziness, ataxia, diplopia, blurred vision, sedation, headache, nausea, vomiting, and simple rash) are rare due to the slow dose escalation required to reduce dangerous dermatologic effects.
D. Lamotrigine overdose can be fatal. Common symptoms include stupor, convulsions, and intraventricular delay. Treatment is largely supportive.
References:
Which of the following describes organizations that have cultures of safety, foster a learning environment and evidence-based care, promote positive working environments for nurses, and are committed to improving the safety and quality of care?
A. Preferred Provider Organizations
B. Health Maintenance Organizations
C. High Reliability Organizations
D. Accredited Health Organizations
Correct Answer: C.
High Reliability Organizations
Organizations with a culture of safety, that foster a learning environment and evidence-based care, promote positive working environments for nurses, and are committed to improvements in safety and quality care are considered to be high reliability organizations (HROs). In their culture centered on safety and quality, there is direct involvement of top and middle leadership, and safety and quality efforts are aligned with the strategic plan. There is an established infrastructure for safety with active engagement of staff and continuous improvement.
Incorrect Answers:
A. Preferred provider organizations (PPOs) are managed care organizations composed of doctors, hospitals, and other healthcare providers who have agreed with a third party or insurance company to provide care at reduced rates to participants in the plan.
B. Health maintenance organizations are organizations that provide or arrange managed care for insurance or benefit plans, individuals, and other entities.
D. An accredited health care organization has been reviewed and has met predetermined standards set by an accrediting body, such as the Joint Commission.
Vital Concepts:
Organizations with a culture of safety, that foster a learning environment and evidence-based care, promote positive working environments for nurses, and are committed to improvements in safety and quality care are considered to be high reliability organizations (HROs.)
References:
Chronic pain may be treated with pharmacologic agents. When treating chronic pain with medication, which of the following is not true?
A. NSAIDs canot be safely combined with lithium.
B. TCAs analgesic effects occur at lower doses than those used for depression.
C. TCA serum levels help establish appropriate doses.
D. Tramadol and venlafaxine are functionally similar.
Correct Answer: D.
Tramadol and venlafaxine are functionally similar.
Tramadol is an opiate agonist with weak mu properties, though it should be noted that Tramadol does have some SSRI properties, and therefore its combination with an SSRI could increase the risk for serotonin syndrome. Venlafaxine and duloxetine are SNRIs, and duloxetine has recently been FDA-approved for treating chronic pain (especially neuropathic pain).
Incorrect Answers:
A. NSAIDs are useful for acute and chronic pain, but alternatives are necessary for patients on lithium due to the risk of potentiating lithium toxicity.
B. TCAs are analgesia at lower doses than those used for depression, although some patients may require higher doses for effective analgesia. They are often used in conjunction with other agents such as anticonvulsants.
C. Serum TCA levels can help establish the appropriate dose, particularly in patients who are rapid or slow metabolizers. Following TCA serum levels are very important when multiple pharmacologic agents are used, as many medications will increase TCA serum levels. The only TCA levels that can be reliably followed are desipramine, imipramine, and nortriptyline.
References:
Some patients with schizophrenia may exhibit aggressive behavior. Which of the following statements about schizophrenia and aggressive behavior is not true?
A. Schizophrenia is associated with increased risk of aggressive behavior.
B. Risk for aggressive behavior increases with comorbid antisocial personality.
C. Risk for aggressive behavior decreases with neurological impairment.
D. Violent patients with schizophrenia have more bizarre behaviors and may act on their delusions, especially if delusions are distressing.
Correct Answer: C.
Risk for aggressive behavior decreases with neurological impairment.
Risk for aggressive behavior increases with neurological impairment.
Incorrect Answers:
A. This is true, although only a minority of patients with schizophrenia are violent.
B. Risk for aggressive behavior also increases with comorbid alcohol abuse and substance abuse.
D. Patients with schizophrenia are also more likely to act on their delusions if they can find evidence to support them. Patients who experience command hallucinations to harm others are also more likely to be violent.
References:
Factitious disorder and somatic symptom disorder can have seemingly similar symptoms. How are the two differentiated?
A. Somatic symptom disorder involves intentional feigning of symptoms, but factitious disorder does not.
B. Symptoms are unconsciously produced in factitious disorder but are intentionally produced or feigned in somatic symptom disorder.
C. There is no motivation for producing symptoms in factitious disorder, but in somatic symptom disorder, the patient wants to assume a “sick role” for secondary gain
D. Factitious disorder involves deceptively reporting false symptoms, whereas symptoms in somatic symptom disorder are produced unconsciously.
Correct Answer: D.
Factitious disorder involves deceptively reporting false symptoms, whereas symptoms in somatic symptom disorder are produced unconsciously.
Patients are diagnosed with a factitious disorder or malingering because they are “faking” or even creating symptoms that have no real pathological basis. Somatic symptom disorder is difficult to differentiate from other psychiatric disorders, but it is characterized by physical symptoms - one or more current somatic symptoms that are long-standing and cause distress or psychosocial impairment. The patient experiences excessive thoughts, feelings, or behaviors related to the symptom or symptoms. In some cases, these symptoms are related to a patient’s known medical history or illness.
Differentiation between factitious disorder and malingering involves determining the patient’s motivation. In malingering, there is some secondary gain (e.g., getting out of a crime by “pleading insanity,” making money). In factitious disorder imposed on self or another, the patient wants to assume the “sick role” (such as in Munchausen syndrome). The two disorders require different treatments. Both are diagnoses of exclusion, and clinicians should take care to rule out any other causes for these symptoms before coming to a diagnosis of factitious or somatic symptom disorder. Factitious disorder imposed on another is considered child maltreatment and must be reported to the appropriate authorities. The DSM-5-TR diagnostic criteria for factitious disorder include:
Exaggerating or fabricating physical or phycological manifestations of an injury or medical condition or the intentional self-infliction of an injury.
The individual manipulates themselves to appear to others as unwell, disabled, or wounded.
The false and misleading behavior persists despite the lack of an apparent benefit to the individual.
The behavior cannot be attributed to another mental health disorder.
Incorrect Answers:
A. Somatic symptom disorder does NOT involve faking symptoms, and factitious disorder DOES involve faking symptoms.
B. Symptoms are unconsciously produced in SOMATIC SYMPTOM disorder but are intentionally produced or feigned in FACTITIOUS disorder.
C. Malingering, not somatic symptom disorder, involves the patient wanting to assume the “sick role” for secondary gain. Factitious disorder does not involve secondary gain.
Vital Concept:
Factitious disorder involves deceptively reporting false symptoms to play the “sick role” without motivation related to secondary gain, whereas symptoms in somatic symptom disorder is produced unconsciously.
References:
Nurse practitioners are held to certain standards of practice. Which of the following is one of those standards?
A. “Quality standards of care for nurse practitioners” refers to regulations that are determined by the state board of nursing for all nurse practitioners within that state
B. Identification of expected outcomes is a standard of care for psychiatric mental health registered nurse practitioners with at least three years’ experience
C. Time estimates for attainment of expected outcomes in psychiatric care are inappropriate
D. The standard of practice for psychiatric mental health registered nurse practitioners includes identification of consumer resources to assist consumers in appropriate use of mental health services
Correct Answer: D.
The standard of practice for psychiatric mental health registered nurse practitioners includes identification of consumer resources to assist consumers in appropriate use of mental health services
Standards of practice in nursing refer to authoritative statements of the duties that all nurses are expected to perform competently, regardless of role, specialty, or patient population. Among the standards of psychiatric mental health nursing standards is the development of expected outcomes of treatment, as measurable goals and to provide direction for continuity of care. Coordination of care delivery is another standard of care for advanced practice psychiatric mental health nurse practitioners.
Incorrect Answers:
A. Quality standards of care are determined by governmental agencies, professional societies, and licensing boards. They may be general in scope or specific.
B. Identification of expected outcomes is a standard of care for psychiatric mental health registered nurse practitioners in general; there’s no experience component to this expectation.
C. Time estimates for the attainment of expected outcomes in psychiatric care are a standard of care for psychiatric mental health registered nurse practitioners.
Vital Concepts:
Standards of practice in nursing refer to authoritative statements of the duties that all nurses are expected to perform competently, regardless of role, specialty, or patient population. Among the standards of psychiatric mental health nursing standards is the development of expected outcomes of treatment, as measurable goals and to provide direction for continuity of care.
References:
Health professionals may engage in Interprofessional collaborative practice training. Which of the following is true of this type of teamwork training?
A. Outcomes-based competency expectations for teamwork training and interprofessional collaborative practice are required for healthcare professions schools accreditation
B. Teamwork training for interprofessional collaborative practice in nurse practitioner training includes training with other nurses only
C. Teamwork training for interprofessional collaborative practice in health professions has lagged behind practice changes
D. The primary responsibility for developing core competencies is with the State Nursing Board
Correct Answer: C.
Teamwork training for interprofessional collaborative practice in health professions has lagged behind practice changes
Teamwork training for interprofessional collaborative practice in the education of health professions has lagged behind the realities of current practice, which has resulted in a gap between current training and actual practice.
Incorrect Answers:
A. Although accrediting standards often contain content about interdisciplinary teams, few are outcomes-based competency expectations.
B. Teamwork training for interprofessional collaborative practice includes all healthcare professions that form part of a healthcare team.
D. The primary responsibility for developing core competencies is with health professions schools.
Vital Concepts:
Teamwork training for interprofessional collaborative practice in the education of health professions has lagged behind the realities of current practice, which has resulted in a gap between current training and actual practice.
References:
When a nurse practitioner engages in clinical practice based on evidence, it is important that they can effectively interpret the data to weigh the effects of an intervention. Which of the following refers to the percentage of the benefit conferred by a given therapy in the treatment group compared with the control group?
A. Risk ratio
B. Relative risk reduction
C. Number needed to treat
D. Confidence interval
Correct Answer: B.
Relative risk reduction
The relative risk reduction is calculated on the basis of event rates and is a percentage of the benefit conferred by a specific therapy.
Incorrect Answers:
A. Risk ratio is the ratio of risk in the treated group to the risk in the control group, and can be helpful to determine if an intervention prevents or increases an outcome or disease.
C. The numbers needed to treat (NNT) defines the number of people who must receive a therapy for one person to benefit from it.
D. A confidence interval is certainty, given in percentage, that the true value lies within the range of values. If a 95% Confidence Interval is given, it means there is a 95% chance that the true value is within the range and a 5% chance that the true value is not within the range.
Vital Concepts:
The relative risk reduction is calculated on the basis of event rates and is a percentage of the benefit conferred by a specific therapy.
References:
An adult patient endorses a lack of appetite, lack of energy, inability to sleep, and feelings of hopelessness. They feel this way more days than not. They deny suicidal and homicidal ideology or poor concentration and have never been diagnosed with depression before. This has been going on for 2 years (starting after they broke up with their last partner). They would like to feel better and asks if there is anything to help. What is the correct diagnosis?
A. Major depressive disorder in partial remission
B. Major depressive disorder
C. Persistent depressive disorder
D. Hypomanic episode
Correct Answer: C.
Persistent depressive disorder
In the DSM-5-TR, persistent depressive disorder is characterized by at least two symptoms of depressed mood with a duration of 24 months or more.
Incorrect Answers:
A. Major depressive d/o has a list of nine diagnostic criteria, five of which need to be met within a 2-week period.
B. Major depressive d/o in partial remission is diagnosed when MDD was previously diagnosed, and now some of the s/s of depression remain but the patient no longer meets the full criteria of the dx.
D. Hypomanic episode displays symptoms of a mood disorder involving elevated mood and these symptoms are not within the question.
Vital Concept:
A patient must be symptomatic on most days for at least 2 years for a diagnosis of persistent depressive disorder in adults.
References:
An adult patient presents for evaluation. Every year around the end of January, they get “the blues”. The symptoms continue daily for weeks. During this period the patient has no energy, no drive to get up in the morning, decreased appetite, decreased sleep, difficulty concentrating, and feelings of sadness. The only things that really help are going on a trip to a sunny locale for a few days or taking 5-HTP, an herbal supplement from the local pharmacy.
These symptoms last from late January until the middle of April, or whenever the temperatures increase. Once the temperature is warmer, they experience “life in all its glorious beauty” and have a burst of energy for a few weeks when the cold has broken. The patient wants to avoid these symptoms this winter. What is the best diagnosis?
A. Persistent depressive disorder
B. Persistent major depressive disorder
C. Major depressive episode, seasonal pattern
D. Mixed episode
Correct Answer: C.
Persistent depressive disorder
In the DSM-5-TR, persistent depressive disorder is characterized by at least two symptoms of depressed mood with a duration of 24 months or more.
Incorrect Answers:
A. Major depressive d/o has a list of nine diagnostic criteria, five of which need to be met within a 2-week period.
B. Major depressive d/o in partial remission is diagnosed when MDD was previously diagnosed, and now some of the s/s of depression remain but the patient no longer meets the full criteria of the dx.
D. Hypomanic episode displays symptoms of a mood disorder involving elevated mood and these symptoms are not within the question.
Vital Concept:
A patient must be symptomatic on most days for at least 2 years for a diagnosis of persistent depressive disorder in adults.
References:
A 45-year-old male is brought to the ER after a car crash. X-ray of his leg shows a fracture of the right femur. After three days of his hospital stay, the patient reports hearing his father’s voice. He is diaphoretic and tachycardic. He has no previous history of psychosis. MSE reveals disorientation to time, place, and person, with visual and auditory hallucinations. What is the most likely diagnosis?
A. Alcoholic psychosis
B. Delirium tremens
C. Schizophrenia
D. Subdural bleed
Correct Answer: B.
Delirium tremens
The patient has developed delirium tremens. This is the most common cause of new-onset hallucinations. Delirium tremens (DT), or alcohol withdrawal delirium, is characterized by agitation and tremulousness, autonomic instability, fevers, auditory and visual hallucinations, and disorientation. DT usually develops 2-4 days from the person’s last drink, and the average duration is <1 week.
Incorrect Answers:
A. Alcoholic psychosis is a complicated form of alcohol withdrawal with auditory, visual, or tactile hallucinations in the presence of a clear sensorium.
C. The sudden onset of schizophrenia in a 45-year-old male with no previous psychosis is unlikely.
D. A subdural bleed usually does not cause diaphoresis and hallucinations. It mostly presents with headaches and confusion.
References:
A nurse practitioner is active in advocating for the future of nursing. When advocating for the profession of nursing, a nurse practitioner knows that responsibility for interpreting the scope of practice parameters rests with which of the following?
A. The State Medical Licensure Board
B. The State Board of Nursing
C. The State Legislature
D. The individual NP
Correct Answer: B.
The State Board of Nursing
The mission of the State Board of Nursing in each state and the District of Columbia is to protect the public from harm. The nursing board of each state is responsible for establishing initial licensure requirements and retention of licensure requirements; interpretation of scope of practice parameters; investigation of complaints against licensees; and taking appropriate action when complaints are found to be valid.
Incorrect Answers:
A. The State Board of Nursing in each state regulates the practice of nursing.
C. The state legislature may pass laws concerning scope of practice, but interpretation rests with the State Board of Nursing.
D. The individual NP must become aware of the scope of practice that is acceptable in the state.
Vital Concepts:
The nursing board of each state is responsible for establishing initial licensure requirements and retention of licensure requirements; interpretation of the scope of practice parameters; investigation of complaints against licensees; and taking appropriate action when complaints are found to be valid.
References:
A psychiatrist sees a child 3 times with his parents, who are going through a difficult divorce to cope with the family situation. Several months later, the father of the child reaches out to the psychiatrist and asks her out for dinner. She wants to go. The psychiatrist never treated the parents, only the child, and has not seen any of them for several months. According to the APA, which of the following responses is appropriate?
A. Proceed with the date, as this is not unethical.
B. Proceed with the date but sever all therapeutic ties with the family.
C. Proceed with the date but assign the child to a new therapist.
D. Decline, as this is considered unethical.
Correct Answer: D.
Decline, as this is considered unethical.
Dating a patient or any key third party (parent, spouse, ex-spouse, guardian, or surrogate) even months later is considered unethical. Although it may seem benign, the psychiatrist has treated this patient in the past, and the family members have disclosed personal information. The father may have developed a positive transference toward her that would be considered “exploiting the therapeutic relationship.”
Incorrect Answers:
A. This is unethical, and the therapist shouldn’t proceed with the date
B. It doesn’t matter if the therapist severs all therapeutic ties with the family — it’s still unethical to date a former patient’s father
C. It doesn’t matter if the therapist assigns the child a new therapist — it’s still unethical to date a former patient’s father
References:
A 24-year-old man presents to the ED escorted by police. He is thrashing in his handcuffs and very agitated. On exam, he is hypertensive and has vertical nystagmus. What substance did he use?
A. Cocaine
B. Benzodiazepines
C. Phencyclidine (PCP)
D. Heroin
Correct Answer: C.
Phencyclidine (PCP)
Phencyclidine (PCP, “angel dust”) causes violent behavior, nystagmus, tachycardia, hypertension, anesthesia, and analgesia. It is the only illicit drug that causes vertical nystagmus, but horizontal and rotatory nystagmus can also occur with PCP. Other substances that cause nystagmus include alcohol, amphetamines, barbiturates, benzos, ketamine, lithium, SSRIs, and salicylates. Benzodiazepines, marijuana, and heroin are unlikely to cause combativeness and agitation.
Incorrect Answers:
A. B. D. These medications are unlikely to cause combativeness and agitation. They also don’t cause vertical nystagmus.
References:
A 47-year-old man is admitted for evaluation of disorientation, nausea, vomiting, unsteady gait, and postural instability. On admission, neurological examinations reveal cerebellar ataxia, extrapyramidal signs including parkinsonism, and bone pain. Doctors also notice mental slowing with attention and memory problems. Laboratory findings reveal severe hypercalcemia and elevated serum parathyroid hormone. Brain computed tomography reveals severe calcification of basal ganglia and dentate nuclei. Which statement about the treatment and/or diagnosis of this disease is true?
A. 75% of patients have no recognizable symptoms.
B. Correcting the calcium level will prevent delirium.
C. Late in disease, hyperparathyroidism (associated with hypocalcaemia) may manifest as nonspecific mental slowing with attention and memory problems.
D. There is a moderate but conclusive correlation between symptoms and serum calcium levels.
Correct Answer: B.
Correcting the calcium level will prevent delirium.
Hyperparathyroidism and elevated calcium level can also cause delirium or personality change. Remember, “stones (kidney stones), groans (bone pain), and psychiatric overtones” for hypercalcemia.
Incorrect Answers:
A. As many as 50% of patients may have no recognizable symptoms.
C. Early hyperparathyroidism, associated with hypercalcemia, may manifest as nonspecific mental slowing with attention and memory problems.
D. Despite the lack of conclusive correlation with absolute serum calcium levels, symptoms of parathyroid dysfunction are likely associated with disturbances in calcium, phosphate, and bone metabolism.
References:
The NP knows that his or her legal right to practice is derived from:
A. The laws of the state where he or she practices.
B. The Nurse Practice Act of the state where he or she practices.
C. The federal Medicare laws.
D. The Board of Nursing in the state where he or she practices.
Correct Answer: B.
The Nurse Practice Act of the state where he or she practices.
The Nurse Practice Act of the state gives every nurse the legal right to practice nursing.
Incorrect Answers:
A. The laws of the state give the Board of Nursing the authority to enforce the Nurse Practice Act.
C. The federal Medicare laws provide funding for health care for older adults aged 65 years and older.
D. The Board of Nursing is the agency that is tasked with enforcing the Nurse Practice Act in the state.
Vital Concepts:
The Nurse Practice Act of the state gives every nurse the legal right to practice nursing.
References:
Research is typically conducted on a sample of subjects who are representative of a larger population. Which of the following statements is true about these studies in comparison to population surveys?
A. Samples are often less accurate than surveys of larger populations.
B. Samples usually increase heterogeneity compared to population surveys.
C. Samples may allow estimations of error in resulting statistics.
D. Samples are most representative of the larger population when using nonprobability sampling.
Correct Answer: C.
Samples may allow estimations of error in resulting statistics.
Research is typically conducted on a sample of subjects who represent a larger population. When selected using random sampling, the study sample is most representative of the overall population. The use of samples rather than populations provides many advantages. Samples are, by definition, smaller and can be studied more quickly and at a lower cost. The use of samples may be the only feasible approach when the study of populations is impossible or impractical. It would not be possible, for example, to obtain blood samples from every US resident. Often, results from samples are more accurate than those of a population, and more time and costly resources can be invested in each research staff member or subject.
An MRI is more accurate than an X-ray in detecting certain abnormalities, for example, but this would only be financially feasible to perform in a smaller sample. When samples are appropriately selected, methods can be used to estimate the error of the resulting statistics, which allows the calculation of probabilities regarding study observations. Samples may be selected in ways that reduce heterogeneity. If studying a certain aspect of migraines, for instance, an investigator may not want to study the entire population with migraines, as there is great variability in presentation. Instead, he or she may wish to select subjects with certain features that are appropriate for the study.
Incorrect Answers:
A. Sampling may produce more accurate results than a population survey.
B. Samples may be selected to reduce heterogeneity.
D. Nonprobability sampling techniques base sample selection on the subjective judgment of the researcher, which can result in samples that are not representative of the entire population.
References:
You are asked to obtain consent for a lumbar puncture from a 17-year-old patient with suspected meningitis in the ER. When you explain the procedure to the patient, they are disoriented and delirious. They are away from family, and you are unable to contact the caregivers, who live in a different town. Which of the following is the most appropriate action?
A. Have the patient sign a consent form.
B. Continue to call the patient’s parents to get consent.
C. Do the lumbar puncture and treat the patient since they requires emergency care for a condition that could endanger life and limb.
D. Call another close relative for consent if the parents cannot be reached.
Correct Answer: C.
Do the lumbar puncture and treat the patient since they requires emergency care for a condition that could endanger life and limb.
Consent for emergency care is implied in the ER. If the patient was not able to give informed consent due to her illness, then the NP should proceed with the procedure and treatment. The diagnosis will be made by a lumbar puncture, so it would be inappropriate to guess and subsequently make decisions about whether suspected meningitis is bacterial or viral. Bacterial meningitis is one type of life-threatening emergency that may be treated even if the patient is not able to understand the procedure unless there is a prior instruction or proxy appointed to make decisions when the patient is incapacitated. In those cases, the proxy is expected to make decisions based on the best knowledge of the patient’s healthcare preferences.
Incorrect Answers:
A. The patient is not able to give informed consent due to her disorientation and delirium, therefore should not have the patient sign a consent form.
B. Attempts have already been made to contact her parents and continuing to call them could delay diagnosis and treatment.
D. Close relatives that are not her parents/legal guardians cannot give consent for the patient’s procedure.
Vital Concept:
Consent for emergency care is implied in the emergency room.
References:
A patient presents to the office because she is tired of being consumed by the people who are following her, she wants to move on with her life. After she divorced her husband about 8 months ago she started to notice that she was being followed. She states, “A white car is parked outside of my apartment every morning when I go to work and it is there every night when I get home. It is only at my work on Tuesday or Thursday, or at least that is all I’ve noticed. I do realize that they are not going to hurt me now, but I just want to stop thinking about them.” Paranoia is linked to failure at which stage of Freud’s Psychosexual Stages of Development?
A. Latency
B. Phallic
C. Anal
D. Oral
Correct Answer: D.
Oral
Paranoia, schizophrenia, and substance abuse are all linked to the oral stage that occurs during 0-18 months of life. Failure of the latency stage is the inability to form social relationships, phallic stage is sexual identity disorders, and anal is depressive disorders.
Incorrect Answers:
A. This is linked to failure to form social relationships.
B. This is linked to sexual identity disorders.
C. This is linked to depressive disorders.
A researcher is studying the effect of a new medication on blood pressure. There are 100 subjects in the control group and 100 subjects in the treatment group. At the conclusion of the experiment, the researcher rejects the null hypothesis. Which of the following is true?
A. The p-value is greater than 5%
B. The differences seen in the study are most likely due to chance
C. The effects seen in the study are not due to chance
D. The drug is effective
Correct Answer: C.
The effects seen in the study are not due to chance
The null hypothesis states that there is no significant relationship between the variables in a study. In this case, the null hypothesis would be that any differences between the control group and the treatment group were due to chance. Since the researcher has rejected the null hypothesis, he or she has determined that the differences between the two groups were not the result of chance. This does not mean that the drug or treatment is effective – that determination depends upon the treatment goals. In general, if the null hypothesis is rejected, the significance level or p-value will be less than 0.05. This means there is a 5% chance the differences seen between the two groups are due to chance alone.
Incorrect Answers:
A. Rejection of the null hypothesis means the differences between the two groups weren’t due to chance
B. It’s not determined yet if the effects were due to chance
D. It’s still not determined if the drug is effective at meeting treatment goals
Vital Concepts:
The null hypothesis states that there is no significant relationship between the variables in a study. In this case, the null hypothesis would be that any differences between the control group and the treatment group were due to chance. Since the researcher has rejected the null hypothesis, he or she has determined that the differences between the two groups were not the result of chance. This does not mean that the drug or treatment is effective – that determination depends upon the treatment goals.
References:
A psychiatrist is looking to prescribe pharmacologic treatment to a patient with chronic alcohol use disorder with a major depressive episode. They’ve chosen one of the antidepressants listed, which has a tetracyclic structure that works by antagonism of presynaptic alpha-2 agonists and post-synaptic serotonergic receptors. Which of the following medications is it?
A. Amitriptyline
B. Chlorpromazine
C. Mirtazapine
D. Doxepin
Correct Answer: C.
Mirtazapine
Mirtazapine has a tetracyclic structure but is not considered a tricyclic because it works by alpha 2 antagonism at the presynaptic receptors and antagonism of postsynaptic serotonin 5-HT3 and 5-HT2 receptors, which may block some of the body’s mechanisms of reward for alcohol abuse. The other drugs listed are all tricyclic antidepressants except chlorpromazine, which is an antipsychotic.
Incorrect Answers:
A. D. These are tricyclic, not tetracyclic.
B. This is an antipsychotic, not an antidepressant.
References:
A man is the only survivor of an apartment fire that kills two of his roommates. During the appointment you ask him how he is doing, and he replies “I don’t remember a thing about the fire”. Which defense mechanism is the best explanation?
A. Projection
B. Humor
C. Regression
D. Repression
Correct Answer: D.
Repression
Repression is the defense mechanism the man is using to deal with the tragedy of losing his roommates in a fire. Repression is withholding from consciousness an idea or feeling, hence while the patient is unable to recall details about the fire. Humor, regression, or projection does not pertain to this patient.
Incorrect Answers:
A. The man isn’t blaming the fire or its consequences on others, which would be projection.
B. The man isn’t trying to make a joke of the situation, which would be humor.
C. The man hasn’t returned to a childlike state, which would be regression.
References:
In couples therapy, the therapist looks for ways to understand how the marriage is “stuck.” Which approach is the therapist using?
A. Structural-strategic approach
B. Experiential humanistic approach
C. Transgenerational approach
D. Behavioral approach
Correct Answer: A.
Structural-strategic approach
This is an example of the structural-strategic approach. A strategic approach focuses on methods to solve unwanted behaviors in the family and has been described as strategies used by the therapist to uncover varied motives of family members and elicit change. Structural therapy uses various strategies to change the structure of the family over time.
Incorrect Answers:
B. In the experiential humanistic approach, the therapist tries to help the patient to get out of a persistent narrow role by encouraging connective emotional experiences in the here-and-now.
C. In the transgenerational approach, the therapist focuses on the relationship between one’s role in the family of origin and the current couple’s difficulty.
D. In the behavioral approach, the therapist encourages cognitive restructuring to relieve marital tension.
References:
A study of patients with obsessive-compulsive traits splits participants into several groups based on their primary psychiatric issue: impulsivity, reward deficiency, and somaticism. Each group is then individually analyzed for conserved genetic polymorphisms. What is the term for this separation process?
A. Factor analysis
B. Cluster analysis
C. Dimension reduction
D. Discriminant analysis
Correct Answer: B.
Cluster analysis
Cluster analysis breaks a large group into smaller groups (clusters) so that each individual cluster is more homogenous by some measure.
Incorrect Answers:
A. Factor analysis is a statistical technique that seeks to explain the variance of multiple variables by utilizing a smaller number of unobserved factors.
C. Dimension reduction is the process of reducing the number of variables studied to focus on a smaller number.
D. Discriminant analysis is used to determine independent variables (X) that appear to affect a quantity of interest (Y).
References:
Lithium is used to treat major depressive disorder. Which of the following statements about its use is true?
A. Lithium is contraindicated in patients with all cardiac diseases.
B. Lithium has a high therapeutic index.
C. Gastrointestinal lithium-toxicity symptoms and side effects may overlap.
D. Lithium has a characteristic discontinuation syndrome.
Correct Answer: C.
Gastrointestinal lithium-toxicity symptoms and side effects may overlap.
Nausea and vomiting can occur as a side effect or as a sign of toxicity.
Incorrect Answers:
A. Lithium affects the sinoatrial node and atrial node transmission. Sick sinus syndrome is the only contraindication for lithium use.
B. Lithium has a low therapeutic index; lithium toxicity has many symptoms.
D. Lithium does not have a characteristic discontinuation syndrome, but rapid discontinuation of lithium is associated with higher rates of relapse than when lithium is tapered.
References:
Different comorbid conditions and drug combinations alter the best practices in the treatment of depression. Which of the following is a true statement regarding the treatment of depression?
A. Phenelzine is an MAOI and is safe and most effective when combined with an SSRI.
B. Duloxetine is an SSRI that exerts its antidepressant effect solely via norepinephrine.
C. Clomipramine is a TCA and is recommended for treating depression in patients with epilepsy.
D. Bupropion is an NDRI that should be avoided in patients with anorexia nervosa.
Correct Answer: D.
Bupropion is an NDRI that should be avoided in patients with anorexia nervosa.
Bupropion is an NDRI that has been demonstrated to lower the seizure threshold. It should be avoided in patients with comorbid conditions that increase seizure risk such as anorexia nervosa or heavy alcohol use.
Incorrect Answers:
A. Phenelzine is an MAOI; however, MAOIs should not be combined with SSRIs because of an increased risk of serotonin syndrome.
B. Duloxetine is an SNRI and exerts its action via serotonin and norepinephrine. Also, SSRIs do not have an effect on norepinephrine.
C. Clomipramine is a TCA; however, it lowers the seizure threshold and should be avoided in patients with epilepsy.
References:
Childhood bipolar disorder may be treated with pharmacologic treatment. Which of the following is a true statement about pharmacologic treatments for childhood bipolar disorder?
A. Anticonvulsants are used to treat the depressive phase of childhood bipolar disorder.
B. Early onset bipolar disorder is less responsive to lithium.
C. Therapeutic lithium levels are different in pediatric patients as compared to adult patients.
D. Stevens-Johnson syndrome occurs in 10% of children on lamotrigine.
Correct Answer: B.
Early onset bipolar disorder is less responsive to lithium.
Early onset bipolar disorder is less responsive to lithium, therefore, is used often in conjunction with other medications. FDA approved medications for bipolar mania in adolescents are lithium and the following second generation antipsychotics: aripiprazole, risperidone, quetiapine, olanzapine, and asenapine. FDA approved medications for bipolar depression for adolescents are lurasidone and olanzapine/fluoxetine. None of the FDA approved medications for bipolar disorder are approved for children <10 years old.
Incorrect Answers:
A. Anticonvulsants are used to treat and prevent MANIA, not the depressive phase.
C. Therapeutic lithium levels are the SAME in pediatric patients as compared to adult patients. Renal, thyroid, and cardiac function should also be monitored.
D. Stevens-Johnson syndrome occurs in 1% of children on lamotrigine.
Vital Concept:
Early onset bipolar disorder is less responsive to lithium, therefore, is used often in conjunction with other medications.
References:
A 55-year-old woman presents to her primary care provider with a 3-day history of being unable to lift her right arm. Just prior to the onset of this deficit, she had a verbal altercation with her adult son; she became so angry that she tried to slap him across the face but was unable to move her arm. What is the most likely DSM-5-TR diagnosis?
A. Somatic symptom disorder
B. Factitious disorder
C. Functional neurological symptom disorder (conversion disorder)
D. Illness anxiety disorder
Correct Answer: C.
Functional neurological symptom disorder (conversion disorder)
All these diagnoses are listed in the DSM-5-TR Chapter on Somatic Symptoms and Related Disorders. This patient’s presentation is consistent with functional neurological symptom disorder, which is also called conversion disorder.
Incorrect Answers:
A. Somatic symptom disorder occurs when a patient has a strong focus on physical symptoms that can impact their day-to-day functioning and cause distress.
B. Factitious disorder describes when a patient purposefully deceives others by creating illness, typically by intentionally getting sick or injuring themselves.
D. Illness anxiety disorder, previously called hypochondriasis, describes when a patient is overly concerned that they have a serious medical condition, despite having mild or no symptoms.
Vital Concept:
Functional neurological symptom disorder (FNSD), also called conversion disorder, describes nervous system symptoms affecting movement or senses that cannot be explained by a neurological disease or other medical condition.
References:
Individuals with good coping skills exhibit certain behaviors. Which of the following is a characteristic of those with good coping skills?
A. They are quite composed and vigilant in avoiding emotional extremes when they are inappropriate, in an effort to avoid impaired judgment.
B. They tend to be practical and emphasize past problems, issues, and obstacles that were not conquered.
C. They select from a narrow range of potential strategies and tactics.
D. They improve coping by being unaware of consequences.
Correct Answer: A.
They are quite composed and vigilant in avoiding emotional extremes when they are inappropriate, in an effort to avoid impaired judgment.
These individuals are are quite composed and vigilant in avoiding emotional extremes that could impair judgment.
Incorrect Answers:
B. Good copers tend to be practical and emphasize immediate problems, issues, and obstacles that must be conquered before visualizing a remote or ideal resolution.
C. Good copers select from a wide range of potential strategies.
D. Good copers improve their coping by being aware of consequences.
References:
Christopher attended his communication class to present his speech. Christopher received feedback from his peers as well as the teacher. Most of the class and his teacher had very positive things to say, however there was one student who had slightly more critical comments. For the remainder of the week Christopher obsessed over the critical comments, which cognitive distortion is Christopher struggling with?
A. Overgeneralization
B. Filtering
C. Emotional reasoning
D. Comparing
Correct Answer: B.
Filtering
Filtering is defined as the need to focus on the negative details while all of the positive aspects of the situation are ignored. Christopher appears to have only heard the negative feedback regarding his speech. Overgeneralization is more associated with playing the victim, everything always goes wrong for me. Emotional reasoning is always feeling inadequate which may apply to this situation, but this is regardless of feedback. There is no evidence of comparing occurring in the situation.
Incorrect Answers:
A. This is associated with playing the victim, such as thinking, “everything goes wrong for me.”
C. This is associated with always feeling inadequate regardless of feedback. It could apply here, but doesn’t account for the fact that Christopher is reacting to feedback.
D. There’s no evidence of comparing, which is basing social and personal worth on how oneself stacks up to others
References:
A 17-year-old girl presents with abdominal pain to the ED. Her abdominal pain comes and goes and often is present when she lies down. She often feels full after eating only a few bites of a meal. The patient is thin and worries about being “too thin” but can’t seem to eat much before she feels like she “just can’t eat any more.” On physical exam, she has some patches of thinner hair over her scalp. She admits to recent stress, stating that she and her boyfriend have been “going through a rough patch” but that she thinks he’ll be back. The patient’s abdomen is soft and non-tender, and the rest of the exam is benign. Near the end of the appointment, she is chewing on her hair. What test would be most appropriate to confirm the suspected diagnosis if all these findings are related?
A. Complete blood count
B. Complete metabolic panel
C. Plain film of abdomen
D. Plain film of chest
Correct Answer: C.
Plain film of abdomen
The primary objective at this time is to rule out a bezoar as the cause of her symptoms. The alopecia, hair chewing, and abdominal pain are concerning for this diagnosis.
Incorrect Answers:
A. A CBC would be appropriate if this were a purely infectious process, however it would not confirm nor rule out a bezoar.
B. D. These may be appropriate later in the investigation. These would not confirm or rule out a bezoar, but would be helpful to evaluate for other pathology.
References:
A 16-year-old male expresses resentment about having to care for his young child to his therapist. When the patient is with friends, he consistently reports how wonderful it is to be a father and that he is proud to be a good provider, and he spends much of his time planning special outings and activities for his son. He also volunteers to chaperone his son’s preschool (daycare) class trips every week. Which defense mechanism describes this behavior?
A. Identification
B. Reaction formation
C. Repression
D. Displacement
Correct Answer: B.
Reaction formation
Reaction-formation involves the conversion of unconscious wishes into their opposites. The behavior is the opposite of what one really desires or feels.
Incorrect Answers:
A. Identification is the unconscious modeling of one’s self on another person’s character and behavior.
C. Repression involves avoiding pleasurable instincts or drives due to the threat of suffering if the drive is satisfied.
D. Displacement is the shift of a sexual or aggressive drive to a more acceptable target.
References:
Compared to other medications in the same class, a certain TCA has been found in considerably higher concentrations in infant plasma (which means it’s present at a higher concentration in breast milk). Which TCA is this?
A. Doxepin
B. Amitriptyline
C. Clomipramine
D. Nortriptyline
Correct Answer: A.
Doxepin
Infant plasma concentrations are considerably higher for doxepin than for other TCAs.
Incorrect Answers:
B, C, and D. Infant plasma concentrations for these TCAs aren’t significantly higher.
References:
An older adolescent with no prior psychiatric history has been increasingly socially isolated over the past 3 months. Although they had been a good student with many friends throughout most of high school, they dropped out of school in the spring of their senior year. Over the past 2 months, their family members report that the patient has started talking to themself and has yelled at them to stop talking when no one else was actually speaking. Over this same time period, they have been increasingly concerned that aliens are living in their head. Urine toxicology and standard medical workup for secondary causes of psychosis are negative. The patient denies any changes in their mood. What is the correct DSM-5-TR diagnosis for this patient, with appropriate specifiers?
A. Unspecified Schizophrenia Spectrum Disorder
B. Schizophrenia, first episode, currently in acute episode
C. Schizophreniform disorder with good prognostic features, provisional
D. Brief psychotic disorder without marked stressor
Correct Answer: C.
Schizophreniform disorder with good prognostic features, provisional
This patient has had a duration of symptoms of hallucinations and paranoia for >1 month but <6 months. Even though some change in functioning started 4 months ago, symptoms consistent with a psychotic disorder did not appear until the past 3 months. This symptom pattern and the duration of symptoms are consistent with schizophreniform disorder. The diagnosis is provisional since it is made before recovery. Since the symptoms did not develop rapidly within 1 month of the first sign of disturbance, the full range of affect and presence of confusion or perplexity constitute two positive prognostic factors to meet the “with good prognostic features” specifier.
To confirm the diagnosis, at least two of the following symptoms must be present for a substantial period during a month or more timeframe:
· speaking incoherently without logical organization*
· a misconception, belief, or thought that is firmly held despite not being grounded in reality*
· illusions or perceived experiences that do not actually exist (e.g., sounds, voices, smells, visions, feelings, etc.)*
· actions that are chaotic or confused, repetitive, purposeless, or significantly reduced (or absent) movement and speech (with catatonia)
· decreased display of emotion or a lack of motivation
*at least one of the two symptoms displayed must be among the first three symptoms described above
· An incidence of this disorder should persist for 1-6 months
· There have been no (or minimal) concurrent periods of depressive or manic symptoms that would satisfy the requirements for schizoaffective DO, major depressive disorder, or bipolar disorder with psychotic features
· The patient’s symptoms are not directly related to the use of a substance, medication, or pre-existing medical diagnosis or health concern and are not due to a more appropriate psychiatric condition
· The prognosis is considered good if two or more of the following are true:
o A healthy or strong ability to function in professional or social environments prior to the incident
o A sense of bewilderment or puzzlement
o A consistent, observable, and appropriate emotional reaction or response to external stimuli
o Significant symptoms of psychosis within a month of the initial alteration in function or behavior
Incorrect Answers:
A. This presentation meets all the required criteria for schizophreniform disorder.
B. The duration of psychotic symptoms is <6 months which is required for a diagnosis of schizophrenia.
D. The duration of psychotic symptoms is >1 month which is the limit for a diagnosis of brief psychotic disorder.
Vital Concept:
The diagnosis of schizophreniform disorder mirrors the symptomatology of schizophrenia, but with a duration of just 1-6 months.
References:
Which of the following statements is true about Sigmund Freud’s developmental theories?
A. Penis envy occurs during the genital phase of development.
B. Castration anxiety is prominent in girls ages 3-6 years.
C. The anal phase occurs from birth to 12 months
D. Oedipal complex describe feelings that start during the phallic phase.
Correct Answer: D.
Oedipal complex describe feelings that start during the phallic phase.
Freud used “oedipal complex” to describe a boy’s unconscious desire to possess his mother sexually and to kill his father.
Incorrect Answers:
A. Penis envy refers to girls realizing that they do not have a penis known as the electra complex. This occurs during the phallic phase of development (age 3-6).
B. Castration anxiety refers to boys aged 3-6 years who have feelings of guilt regarding the oedipal complex.
C. The oral phase occurs from birth to 12 months. The anal phase occurs from ages 12 months to 3 years.
References:
An adult patient is referred for evaluation. They have seen dozens of doctors over the past 2 years to evaluate their many symptoms. They are constantly worried about their symptoms and sure that they have “something serious and really bad,” even though all medical tests have been negative, including extensive imaging, laboratory work, and consultations with most medical specialists. The patient does not meet the criteria for GAD, panic disorder, depression, or OCD. Their belief that they’re sick is not of delusional intensity. Still, it does interfere with some of their daily activities and relationships, as they spend many hours each day logging their bowel movements and urinary output in detail, including digital photos.
What is the estimated prevalence of the most likely diagnosis?
A. <1%
B. 1-5%
C. 6-10%
D. 11-15%
E. 16-20%
Correct Answer: B.
1-5%
Illness anxiety disorder (formerly hypochondriasis) is a preoccupation with having or getting a serious disease causing anxiety and performing frequent health-related behaviors or avoiding medical care. Patients with illness anxiety disorder may or may not have a medical condition but have heightened bodily sensations, are intensely anxious about the possibility of an undiagnosed illness, or devote excessive time and energy to health concerns, often obsessively researching them. Like people with somatic symptom disorder, they are not easily reassured. Findings with respect to age and gender prevalence are inconsistent, but the prevalence in the general population is approximately 1-5%. Onset is commonly in early adulthood and displays a chronic, waxing/waning course.
Illness Anxiety Disorder Diagnostic Criteria according to DSM-5-TR:
An obsession with having an illness or developing an illness
Usually, there are no physical symptoms of illness present; if present, symptoms are mild. If there is a medical condition present or a strong family history that puts the patient at an increased risk of developing a medical condition, the preoccupation with developing the condition is unequal and excessive compared to the individual’s risk
The individual exhibits extreme anxiety surrounding their health.
The individual performs unnecessary or excessively frequent health checks to monitor the development of illness and seeks medical treatment and screening more frequently than recommended or indicated based on presentation or health history (care-seeking type); may also avoid medical care, including routine check-ups with their provider or visits to the hospital when indicated due to fear of being diagnosed with an illness (care-avoiding type).
The preoccupation with health status has persisted for greater than 6 months, but the illness that the patient fears developing may change over the 6-month period
The obsession with the development of an illness cannot be attributed to the presence of another mental health disorder.
Incorrect Answers:
A. Prevalence of illness anxiety disorder is about 1-5%, not <1%.
C. Prevalence of illness anxiety disorder is about 1-5%, not 6-10%.
D. Prevalence of illness anxiety disorder is about 1-5%, not 11-15%.
E. Prevalence of illness anxiety disorder is about 1-5%, not 16-20%.
Vital Concept:
The prevalence of illness anxiety disorder is about 1-5%.
References:
Which of the following statements is true regarding adolescent moral development?
A. By early adolescence, most teenagers have a fully formed conscience and sense of right and wrong.
B. All adults and adolescents attain a high level of sound reasoning in moral development.
C. Adolescents make moral decisions based on rules only.
D. Adolescent moral decision-making is impacted by limited impulse control, self-absorption, and vulnerability to peer pressure.
Correct Answer: D.
Adolescent moral decision-making is impacted by limited impulse control, self-absorption, and vulnerability to peer pressure.
Adolescents may have the capacity for sound moral decision-making, but they are often impacted by limited impulse control, self-absorption, and vulnerability to peer pressure.
Incorrect Answers:
A. This process occurs by mid-adolescence.
B. Not all adolescents and adults develop a high level of reasoning in moral development.
C. Adolescents make moral decisions based on their beliefs in the context of the rules.
References:
Certain schizophrenia signs and symptoms suggest a poor prognosis. Which of these is one such sign or symptom?
A. Sudden onset of illness
B. Magnetic resonance imaging showing no gross changes in brain parenchyma
C. Catatonia
D. Extensive history of extreme isolation and social withdrawal
Correct Answer: D.
Extensive history of extreme isolation and social withdrawal
A premorbid history of social withdrawal and extreme independence is predictive of more long-lasting psychopathology that is often more severe as well. Factors like male gender, family history of schizophrenia, structural brain abnormalities, and tardive dyskinesia are also associated with poor outcome.
Incorrect Answers:
A. With schizophrenia, a sudden onset of illness has a more favorable prognosis than when the onset is insidious, especially if symptoms of agitation and psychosis are present.
B. MRI changes in brain morphology are associated with a severe clinical course of schizophrenia.
C. The presence of catatonic symptomatology is not associated with any particular clinical course.
References:
The American Association of Nurse Practitioners supports the implementation of team-based care. Which of the following is true of quality improvement in healthcare?
A. It can only be achieved by following guidelines established by the Institute of Medicine
B. Optimal outcomes can be achieved when specific steps are taken, but they are not necessarily tied to pertinent clinical guidelines
C. Patient satisfaction is a poor measure of quality
D. Quality improvement is a team process
Correct Answer: D.
Quality improvement is a team process
Quality improvement is a team process in almost every case, whether it is targeted at improvement in disease care or telephone service. Significant and lasting improvements can result from the work of a team.
Incorrect Answers:
A. Although studies demonstrate that specific steps tied to pertinent clinical guidelines are required to deliver optimal health care services, guidelines can be created by a consensus among any group of knowledgeable experts.
B. Optimal outcomes are achieved when specific steps tied to pertinent clinical guidelines are taken.
C. Patient satisfaction is an important measure of health care quality. Some services designed to meet patient needs and expectations include patient safety, support for patient engagement, and cultural competence.
References:
The Theory of Culture Care: Diversity and Universality is meant to provide “culturally congruent care. “ This theory describes health as a state of well-being that is culturally defined and valued by a designated culture. Who developed this theory?
A. Abraham Maslow
B. Elisabeth Kubler-Ross
C. Madeleine Leininger
D. Rosentock, Strecher, and Becker
Correct Answer: C.
Madeleine Leininger
Madeleine Leininger is the founder of the transcultural nursing movement in education research and practice. She defined transcultural nursing as nursing practice that is focused on comparative cultural care values, beliefs, and on the practices of individuals or groups with similar or different cultural perspectives. This type of practice is aimed at providing culture-specific and universal nursing care practices when promoting health or well-being. It is also aimed at helping people face unfavorable human conditions, including illness, or death, in ways that are culturally meaningful to them.
Leininger developed the Theory of Culture Care: Diversity and Universality to provide “culturally congruent care.”
Incorrect Answers:
A. Abraham Maslow developed the psychological theory based on a hierarchy of needs.
B. Kubler-Ross identified the stages of grieving.
D. Rosentock, Stretcher, and Becker developed the Health Belief Model.
Vital Concepts:
Madeleine Leininger is the founder of the transcultural nursing movement in education research and practice. She defined transcultural nursing as nursing practice that is focused on comparative cultural care values, beliefs, and on the practices of individuals or groups with similar or different cultural perspectives.
References:
Which of the following conditions has the lowest incidence of depressive symptoms?
A. Alzheimer’s disease
B. Parkinson’s disease
C. Huntington’s disease
D. Pick’s disease
Correct Answer: D.
Pick’s disease
Although depression can occur in patients with FTD, it is uncommon compared to the other options. Apathy, which is commonly seen in FTD, is often mistaken for depression by caregivers. Other common early symptoms include disinhibition, hyperorality, and compulsive behaviors it can often be mistaken as depression but it is less common.
Incorrect Answers:
A. Depression is a common finding with Alzheimers and can often be displayed with aggression due to the confuision. It is often treated with SSRI’s.
B. Depressive disorders are common in patients with Parkinson’s disease as it decreases quality of life. It can also be associated with apathy and anxiety,
C. Patients with Huntington’s disease often present with neuropsychiatric symptoms like depression and can occur at any point during the illness.
References:
Which of the following structures is not part of the brainstem?
A. Pons
B. Midbrain
C. Medulla
D. Corpus callosum
Correct Answer: D.
Corpus callosum
Corpus callosum is the nerve fibers that run across the brain’s midline connecting the right and left hemispheres.
Incorrect Answers:
The brainstem has 3 divisions:
Pons
Midbrain
Medulla
References:
Buspirone and benzodiazepines may both be used to treat generalized anxiety disorder (GAD), and they are similarly efficacious. If a patient is put on buspirone for GAD, how many weeks will it take to see the same anxiolytic effects as benzodiazepines?
A. 1 week
B. 2 weeks
C. 4 weeks
D. 8 weeks
Correct Answer: C.
4 weeks
It takes 4 weeks of good compliance with buspirone to achieve the same anxiolytic effects as benzodiazepines. The drawback, obviously, is that it takes 4 weeks to become as effective. However, patients can avoid dependence, tolerance, and sedation, which are problems with benzodiazepines.
Generalized anxiety disorder (GAD) is characterized by excessive anxiety and worry that causes impairment of at least 1 realm of the patient’s life without panic disorder lasting at least 6 months. Patients with GAD often worry about small things and have significant fear while expecting the worst in most situations. They also may present with muscle tension, sleep problems, fatigue, or irritability. The prevalence of GAD is highest among women, the elderly, and patients with low socioeconomic status.
Treatment of GAD includes the use of selective serotonin reuptake inhibitors (SSRIs) and serotonin-norepinephrine reuptake inhibitors (SNRIs) as well as buspirone. Benzodiazepines may be effective in reducing physical symptoms of anxiety, and beta-blockers like propranolol may help with palpitations and tremors. Pregabalin has shown efficacy for GAD compared to placebo in several randomized trials. A meta-analysis of 5 trials with 884 patients showed that hydroxyzine was efficacious for GAD as well, but it was also more sedating than benzodiazepines and buspirone.
Incorrect Answers:
A. 1 week. It takes 4 weeks on buspirone to see the same anxiolytic effects as benzodiazepines, not 1 week.
B. 2 weeks. It takes 4 weeks on buspirone to see the same anxiolytic effects as benzodiazepines, not 2 weeks.
D. 8 weeks. It takes 4 weeks on buspirone to see the same anxiolytic effects as benzodiazepines, not 8 weeks.
References:
Nefazodone is a phenylpiperazine derivative used to treat depression. Which of the following statements is true with regard to this drug?
A. It shows minimal drug interactions.
B. Like other antidepressants, it is associated with a high incidence of sexual dysfunction.
C. It causes sleep disturbances.
D. It is a potent inhibitor of cytochrome P450.
Correct Answer: D.
It is a potent inhibitor of cytochrome P450.
Nefazodone is a potent inhibitor of cytochrome P450.
Incorrect Answers:
A. Nefazodone is a potent enzyme inhibitor of cytochrome P450 enzyme 3A4. Blood levels of drugs metabolized by this enzyme increase with coadministration of nefazodone.
B. This medication is associated with a low incidence of sexual dysfunction. It may be used as an antidote to SSRI-induced impotence.
C. Unlike other antidepressants, it enhances rapid eye movement (REM) and may increase restful sleep in some patients.
References:
Certain indicators are predictors of good treatment response in patients with first episode of psychosis. Which is one of those indicators?
A. Male gender
B. High level of expressed emotion in the patient’s family
C. Shorter duration of untreated psychosis
D. Severe hallucinations/delusions
Correct Answer: C.
Shorter duration of untreated psychosis
Shorter duration of untreated psychosis and good premorbid function are associated with a better response. Predictors of poor treatment response include a high level of expressed emotion in the patient’s family, male gender, more severe hallucinations/delusions, and low premorbid function among patients with their first psychotic episode in adulthood.
Incorrect Answers:
A. B. D. These are predictors of poor, not good, treatment response.
References:
A 45-year-old school teacher presents at the urging of her peers. The patient says that she was sent here because she “just likes things a little cleaner” than her colleagues do. In the school restaurant, she brings her own utensils because she’s seen the kitchen crew grasp all of the forks with their bare hands. When she gets home, she must wash her hands and dry them using only a clean towel without touching the fingers of her other hand. She repeats this washing process at least 5 times before she can stop. Because of the patient’s belief that the world is dirty, she has stopped going to shows or the movies, which she once enjoyed immensely, according to her friends. What is the next step in management of this patient?
A. Mirtazapine
B. Clomipramine
C. Haloperidol
D. Lorazepam
Correct Answer: B.
Clomipramine
Clomipramine is a tricyclic antidepressant (TCA) that is FDA approved to treat obsessive-compulsive disorder. When treating OCD the onset of therapeutic action can be up to 6-12 weeks. When using Clomipramine to treat OCD patients usually need higher doses, 200-250mg/day. TCAs have a prominent side effect profile. Most common side effects include sedation, weight gain, sexual dysfunction, constipation, blurred vision, urinary retention, nausea, diarrhea, anxiety, fatigue and sweating. Dangerous rare side effects include lowering seizure threshold, QTc prolongation, hyperthermia, arrhythmias, and hepatic failure. Remember the 3 Cs of TCA overdose: Convulsions, Coma, Cardiac conduction.
The only other medications FDA approved to treat OCD are the following selective serotonin reuptake inhibitors (SSRIs): fluoxetine, paroxetine, fluvoxamine, and sertraline.
Incorrect Answers:
(A) Mirtazapine. Mirtazapine, a serotonin/norepinephrine receptor antagonist and alpha 2 antagonist, has not been shown to be efficacious to treat OCD.
(C) Haloperidol. Haloperidol, a first generation (typical) antipsychotic, has not been shown to be efficacious to treat OCD.
(D) Lorazepam. Lorazepam, a benzodiazepine, and has not been shown to be efficacious to treat OCD.
References:
Clozapine is an antipsychotic medication used to treat severe schizophrenia or reduce the risk of suicidal behavior in people with schizophrenia or similar disorders. Which of the following is a false statement about its potential risks and side effects?
A. Agranulocytosis occurs with clozapine.
B. Seizures are a possible side effect.
C. Weight gain is common in adolescents taking clozapine.
D. TD is commonly associated with clozapine.
Correct Answer: D.
TD is commonly associated with clozapine.
TD is not commonly associated with clozapine. Clozapine is associated with some tardive dyskinesia, although less than other antipsychotics. There are also reports of TD improving with a switch to clozapine.
Incorrect Answers:
A. Agranulocytosis is the most significant adverse effect of clozapine. It occurs mostly in the first 6 months of treatment; it is more likely in women, those of Ashkenazi Jewish descent, and those of advanced age. There are special guidelines to follow white cell count and manage dosage.
B. Seizures are a risk with clozapine treatment and increase with increasing dose; about 4-6% of patients on dosages >600mg/day will have seizures.
C. Weight gain and metabolic changes are significant and similar to olanzapine. Weight gain, development of diabetes, and hyperlipidemia are very prevalent; teens especially have weight gain.
References:
A patient states that “I’m a freak. I’m afraid of crowds sometimes.” The therapist replies, “At times, many people feel anxious in crowds.” Which of the following characterizes the therapist’s reply?
A. Normalizing
B. Exhortation
C. Inspiration
D. Encouragement
Correct Answer: A.
Normalizing
The therapist has offered a kind of reassurance (normalizing) to help the patient feel more normal.
Incorrect Answers:
B. Exhortation is a form of encouragement that is used to convey the therapist’s high level of confidence in a patient.
C. Inspiration is a form of encouragement in which a patient is encouraged to have a self-transcendent experience and to achieve a higher level of self-actualization.
D. Encouragement, such as exhortation and inspiration, is done to promote the patient’s sense of hope and capability.
References:
The Mental Health NP is completing rounds on an inpatient adolescent unit. A 17 year old female admitted for acute mania asks to speak to the NP privately. In private, the patient tells the NP she is sexually active and thinks “something is wrong with her down there” and points to her pubis. Upon further questioning, the patient states she has been “itching and her underwear looks like cottage cheese has spilled on her panties”. If the suspicion of diagnosis of the NP is confirmed, what medication will be ordered for this patient?
A. Miconazole (Monistat)
B. Metronidazole (Flagyl)
C. Doxycycline (Vibramycin)
D. Azithromycin (Zithromax)
Correct Answer: A.
Miconazole (Monistat)
The patient has described Candida albicans (most likely) but could be Candida glabrata which is a fungal or yeast infection. Clinical manifestations of yeast include pruritus (itching) and subsequent irritation. The discharge may be watery or thick, but usually has a white, cottage cheese-like appearance. Treatment for yeast or fungus is an anti-fungal, such as miconazole, nystatin (Mycostatin), clotrimazol (Gyne-Lotrimin) and terconazol (Terazol) cream.
Incorrect Answers:
B. Metronidazole (Flagyl) is used to treat bacterial vaginosis (BV) most commonly caused by Gardnerella vaginalis. BV may present as a heavier than normal and gray to yellowish discharge. More than half of all women with BV do not notice any symptoms. Flagyl may also be used to treat Trichomoniasis vaginalis, which is a flagellated protozoan that presents with clinical manifestations of a vaginal discharge that is thin (sometimes frothy), yellow to yellow-green, malodorous, and very irritating.
C. Doxycycline (Vibramycin) is used to chlamydia. Chlamydia infections of the cervix often produce no symptoms, but cervical discharge, dyspareunia (painful intercourse), dysuria (painful urination) and bleeding may occur.
D. Azithromycin (Zithromax) is an alternative treatment for chlamydia to use for a one time dose. This is a good choice for patients who may not be able to remember to take medication for 7-10 days.
References:
Exposure Responsive Prevention (ERP) is a form of therapy that exposes patients to their fears. What response is prevented with this type of therapy?
A. Anxiety
B. Compulsive behaviors
C. Anxious thoughts
D. Healthy coping behaviors (e.g. relaxation techniques)
Correct Answer: B.
Compulsive behaviors
Response prevention means refraining from compulsions, avoidance, or escape behaviors. The response most commonly prevented is a compulsion that acutely reduces anxiety but perpetuates the disorder by making the sufferer believe that the compulsion is needed to reduce anxiety or some feared consequence of not performing the compulsion.
Incorrect Answers:
A. By preventing avoidant behaviors, anxiety will actually increase.
C. By preventing avoidant behaviors, anxious thoughts will actually increase.
D. Coping behaviors such as relaxation are not prevented.
References:
Before planning a large project, a researcher makes a series of observations about interventions that appear to reduce community violence . Which of the following refers to this process?
A. Systematic review
B. Pilot study
C. Experimental study
D. Cohort study
Correct Answer: B.
Pilot study
A pilot study is a small-scale experiment or a set of observations that are undertaken to decide how and whether to launch a full-scale project.
Incorrect Answers:
A. A systematic review is a critical assessment and evaluation of all research studies about a particular topic, using an organized method to locate, assemble, and evaluate a body of literature on the topic, using a specific set of criteria. It may or may not include a meta-analysis, which is a quantitative pooling of data.
C. An experimental study is an evaluation of whether a program or intervention had the intended effect on participants or subjects.
D. A cohort study is a clinical research study in which one group of people with a certain condition or exposure are followed over time and compared to another similar group not affected by the condition or exposure of interest.
Vital Concept:
A pilot study is a small-scale experiment or a set of observations that are undertaken to decide how and whether to launch a full-scale project.
References:
Aggressive behavior and PTSD may occur concurrently in some patients. Which of the following statements about aggressive behavior and PTSD is true?
A. Comorbid substance use disorders may increase aggressive behavior only during intoxication.
B. Concurrent treatment of PTSD and Substance Use Disorder may increase aggressive behavior.
C. Child abuse and domestic violence rates decrease after disasters.
D. Sleep deprivation associated with PTSD may amplify aggressive behavior beyond level of provocation.
Correct Answer: D.
Sleep deprivation associated with PTSD may amplify aggressive behavior beyond level of provocation.
After sleep deprivation in PTSD, the resulting aggression is due to a reduced ability to tolerate mild or moderate slights.
Incorrect Answers:
A. Increased aggressive behavior can occur in a patient with comorbid PTSD and SUD during intoxication and withdrawal.
B. Concurrent treatment of PTSD and Substance Use Disorder should decrease aggressive behavior.
C. Rates of delinquency increase after disasters.
References:
A 16-year-old girl is brought to the hospital by her parents following what they describe as a seizure. According to her parents, the patient complained of dizziness and double vision throughout the day and later began to shake uncontrollably while working on homework at her desk. Since arriving at the hospital, the patient has been in and out of consciousness. A neurological exam is normal except for mild confusion when the patient is asked direct questions. She is sent for several imaging studies, all of which appear normal. After 8 hours in the hospital, the patient becomes lucid and has little memory from the prior day, claiming that the last thing she remembers is working on her homework. History reveals that she has never had a seizure before, but she frequently complains of vertigo and often needs to lean on her parents to steady her walking. After a 24-hour observation, she is released from the hospital and is free of symptoms. Which of the following is the most likely diagnosis?
A. Borderline personality disorder
B. Somatic symptom disorder
C. Depression
D. Conversion disorder
Correct Answer: D.
Conversion disorder
Conversion disorder presents as a pseudo-neurological syndrome. This patient most likely did not experience a true seizure, nor were her other symptoms (dizziness, unsteady gait) likely caused by a physiologic disorder. Normal labs and imaging rule out a neurological cause. Additionally, she recovers completely within 24 hours, which is characteristic of conversion disorder.
Incorrect Answers:
A. While those with personality disorders may also be more likely to show conversion disorder symptoms, this patient does not meet the criteria for borderline personality disorder.
B. To be diagnosed with somatic symptom disorder (SSD), the individual must be persistently symptomatic (typically at least for 6 months).
C. This patient may also be depressed, but there is not enough information to make this diagnosis.
References:
As the APN, you are aware that potential lawsuits may occur if the standards of care are not met. If a case were to go to trial in regards to malpractice, the most common way for the courts to establish a standard of care is?
A. An expert witnesses and peers in the field
B. The professional organization is contracted and they will send the experts
C. Reviewing the bylaws for the organization
D. These most always go for a jury trial
Correct Answer: A.
An expert witnesses and peers in the field
Expert witnesses and peers in the field are most commonly used in malpractice trials to determine if standards of care were met. This could also be determined by journal articles, textbooks, practice guidelines, and professional organizations. It is also common to poll peers in the same field to review their common practices, if there are no concrete guidelines for the subject area.
Incorrect Answers:
B. Professional organizations do not develop standards of care or treatment guidelines for clinical practice. They may review common standards and their qualifications to be part of the organization.
C. By laws are written facility specific. These are not the legal general guidelines for clinical practices. A facility may initiate a tort of action if the provider went against the by laws and was on staff.
D. While it is true that some cases may go to jury, this has nothing to do with establishing a standard of care. The jury should be aware of standards of care by each parties attorney’s plans.
References:
Drugs of abuse have “street” or colloquial names by which users frequently refer to them. Which of the following drugs is correctly paired with one of its street names?
A. MDMA - cubes
B. Cocaine - Adam
C. Methamphetamine - Chinese tobacco
D. Ketamine - cat valium
Correct Answer: D.
Ketamine - cat valium
Ketamine is commonly referred to on the street as “cat valium,” “special K,” “vitamin K,” “kit kat,” and “honey oil.” Ketamine is a derivative of PCP that was first developed in 1965. It is less potent and shorter-acting and is used as a dissociative anesthetic in humans.
Incorrect Answers:
A. MDMA (methylenedioxymethamphetamine) is referred as “ecstasy,” “Adam,” “club drug,” “disco biscuits,” and “love drug.”
B. Cocaine is referred as “crack,” “coca,” “freeze girl” and “happy dust” on the streets. Crack is the hardened form of cocaine.
C. Methamphetamine is referred as “crank,” “crystal,” “ice,” “speed”, and “crystal meth” on the streets.
References:
A culture of safety requires a few key elements. Which of the following is one of those elements?
A. Swift punishment for errors
B. Individual responsibility for learning about safe practices
C. Employees defer to leadership on safety matters
D. Shared goals
Correct Answer: D.
Shared goals
Key elements of an organization culture of safety include an organizational priority for safety, patient involvement, openness, and accountability. Shared core values and goals, non-punitive responses to errors or adverse events, and promotion of safety through education and training are also required to establish a culture of safety.
Incorrect Answers:
A. Non-punitive responses to error or adverse events are a key element of a strong culture of safety.
B. Goals and vision within an organization should be shared and the organization with a strong safety culture will promote safety through education and training.
C. Although strong leadership is important, all employees should be engaged and empowered.
Vital Concepts:
Shared core values and goals, non-punitive responses to errors or adverse events, and promotion of safety through education and training are also required to establish a culture of safety.
References:
As part of the body, the CYP-450 system is affected by alcohol interactions. Which of the following statements is true regarding the relationship between alcohol and the CYP-450 system?
A. Acute alcohol ingestion lowers the rate of drug metabolism.
B. Acute alcohol ingestion will result in decreased levels of most benzodiazepines.
C. Chronic alcohol ingestion is associated with a decreased binding of ethanol to cytochrome P-450.
D. Chronic alcohol ingestion neither causes enzyme induction nor inhibition.
Correct Answer: A.
Acute alcohol ingestion lowers the rate of drug metabolism.
Acute or binge use of alcohol lowers the rate of drug metabolism and alcohol serves as an enzyme inhibitor. When any enzyme works as an inhibitor of the CYP-450 system, it reduces the activity of the CYP-450 system. For example, say a patient is taking Drug-X. Without the ingestion of alcohol, CYP-450 activity is low. The CYP-450 system easily breaks down Drug-X and the metabolites are excreted. But after binge amounts of alcohol, the CYP-450 must work to break down Drug-X and the large amounts of alcohol. This competition reduces the metabolism of Drug-X. Drug-X is not broken down as quickly (and subsequently less is excreted) leaving large amounts of Drug-X in the body (potentially toxic).
Incorrect Answers:
B. Acute alcohol ingestion will result in higher levels of most benzodiazepines such as diazepam. Remember, when consumed acutely (or in binge form) alcohol is an inhibitor and will subsequently decrease the metabolism of drugs – in this case, benzodiazepines. Therefore, there will be HIGHER levels of benzodiazepines. For further explanation, see the full response to answer (A).
C. Chronic alcohol ingestion is associated with an increased binding of ethanol to cytochrome P-450. For chronic, heavy drinkers CYP-450 activity is enhanced/higher. This heighted activity of CYP-450 results in an increased metabolism of medications, subsequently leaving lower levels of medications in the body because they are metabolized and excreted so quickly.
D. Chronic alcohol ingestion enhances/increases the rate of drug metabolism in humans, serving as an inducer.
References:
Substance abusers may seek help through a range of group treatments. Which group treatment is used by the largest number of substance abusers?
A. Interpersonal group psychotherapy
B. Cognitive therapy groups
C. DBT groups
D. Self-help groups
Correct Answer: D.
Self-help groups
Self-help groups like AA comprise the largest number to affect substance abusers. These types of groups provide mutual support, offer relationships, and are usually free of charge.
Incorrect Answers:
A. B. C. These groups aren’t used by as many substance abusers as self-help groups like AA.
References:
A 55-year-old man with a past medical history of hypertension and coronary artery disease presents as a new patient to the primary care clinic requesting treatment for ADHD. He works as a lawyer. Over the past 3 months, he has become easily distracted at work and thinks stimulants will improve his focus and performance. He fails to pay close attention to details, makes mistakes at work, and frequently loses necessary items for work. Denies signs of impulsivity or hyperactivity. No previous diagnosis with ADHD. On social history review, he endorses weekly cocaine use.
After the physician explains the patient does not currently meet the criteria for ADHD and discouraging the use of stimulants given his cocaine use and cardiovascular history, the patient insists this is the only type of medication he will consider and it is vital for him to continue functioning at work. What is the most appropriate response to this patient’s request?
A. Prescribe stimulants for inattention as requested by the patient.
B. Prescribe a short course of stimulants to help the patient with his work deadline.
C. Discuss possible alternatives to stimulants.
D. Dismiss the patient from the practice.
Correct Answer: C.
Discuss possible alternatives to stimulants.
Nonmaleficence, or the principle of doing no harm, applies in this case. Physicians have no ethical obligation to fulfill a patient’s request for medication, especially when the requested medication is contraindicated and could adversely affect the patient. The correct course of action is to discuss alternatives to stimulants. A referral to psychiatry could be warranted in this case, but not as a means of deferring the patient’s request for stimulants. Dismissing the patient from the practice would be premature.
The ethical issues of prescribing “cognitive enhancers” or using medications for reasons other than treatment are debated at this time. A diagnosis of ADHD in adults is based on meeting 5 symptoms of inattention and/or hyperactivity and impulsivity for at least 6 months. This patient meets 3 criteria for inattention and has only had symptoms for 3 months. It would be atypical to develop ADHD so late in life. Most likely symptoms would have interfered during his school years.
Incorrect Answers:
A. Prescribe stimulants for inattention as requested by the patient. Physicians have no ethical obligation to fulfill a patient’s request for medication, especially when the requested medication is contraindicated and could adversely affect the patient.
B. Prescribe a short course of stimulants to help the patient with his work deadline. Should NOT prescribe stimulants, even a short course, given the patient’s cocaine use and cardiovascular history.
D. Dismiss the patient from the practice. Dismissing the patient from the practice would be premature.
Vital Concept:
Nonmaleficence, or the principle of doing no harm, is an essential tenet of the practice of medicine. When patients request medications that are contraindicated for them, a physician should not fulfill the request.
References:
The report “Healthy People 2020” includes goals for mental health in the U.S. Which of the following is one of the goals?
A. Reduce mental health treatment by primary care facilities.
B. Mandate mental health coverage by insurers.
C. Screen children in grade 3 for mental health disorders.
D. Reduce suicide rate and suicide attempts by adolescents.
Correct Answer: D.
Reduce suicide rate and suicide attempts by adolescents.
Healthy People 2020 is a milestone report issued by the United States Department of Health and Human Services in 2010 that identified a list of leading health indicators reflecting major public health concerns in the United States. Issues related to mental health included tobacco use, substance abuse, mental health, responsible sexual behavior, injury and violence, and access to healthcare. Objectives were derived from this report for each leading health indicator. For mental health, objectives were the reduction of the suicide rate and suicide attempts by adolescents, reduction of the proportion of adolescents engaging in disordered eating behaviors in an attempt to control their weight, and reduction of the proportion of persons who experience major depressive episodes.
Incorrect Answers:
A. Healthy People 2020 objectives for mental health outline a goal of increased primary care facilities that provide mental health treatment on-site or by paid referral.
B. The goals of treatment expansion in mental health include increasing depression screening by primary care providers.
C. Treatment expansion recommendations include increasing the proportion of children with mental health problems who receive treatment and increasing the proportion of juvenile residential facilities that screen admissions for mental health problems.
References:
A 30-year-old man is brought to the ER unconscious. His vitals are BP: 70/55, pulse: 60/min, and RR: 8/min. The patient’s friends report that he took several pills with alcohol. He has a history of anxiety and takes medication. The patient became drowsy and disinhibited, lost his balance, and had a brief episode of agitation before unconsciousness. Which of the following drugs was the most likely cause of this condition?
A. Cocaine
B. Alcohol
C. Heroin
D. Benzodiazepine
Correct Answer: D.
Benzodiazepine
Benzodiazepines, as well as other sedative-hypnotics, are commonly used in polysubstance abuse. They may enhance the “high” of other substances or may be used to help a person “come down” from the effects of stimulant drugs. Signs of intoxication are similar to those of alcohol intoxication and can include slurred speech, ataxia, and incoordination (see image below). At more severe levels of intoxication, stupor and coma may develop. An overdose on benzodiazepines alone virtually never leads to death. When they are ingested along with alcohol, major tranquilizers, or opioids, however, the polysubstance overdose can be fatal. Because of synergistic effects, a mixture of benzodiazepine and alcohol is about 4x as powerful as either drug used by itself. Benzodiazepines are frequently abused with opioids, and when detected, the benzodiazepine component of the overdose can be reversed with flumazenil. Treatment of an opioid overdose includes general supportive management in addition to naloxone, a pure opioid antagonist that can reverse the CNS effects of opioid intoxication and overdose.
Incorrect Answers:
A. Cocaine overdoses can produce high blood pressure with restlessness, insomnia, headaches, nausea, convulsions, tremors, hallucinations, delusions, and even sudden death due to respiratory or cardiovascular collapse.
B. Heavy alcohol consumption results in serious health sequelae over time, and many cases ultimately result in death. It elevates blood pressure and increases the risk of myocardial infarction.
C. Maladaptive behavior, pupillary constriction, hypotension, memory impairment, and dizziness are commonly associated with heroin intoxication.
References:
A teenage girl is smoking cigarettes. She is willing to quit and has a multistep plan to quit. This patient is most likely in which stage of change?
A. Maintenance
B. Precontemplation
C. Contemplation
D. Preparation
Correct Answer: D.
Preparation
In preparation, the patient is willing to quit and is making a plan to quit but is not yet actively trying to quit.
Incorrect Answers:
A. Maintenance stage is characterized by completed goals being maintained.
B. Precontemplation stage is characterized by not being cognizant of a need for change or resistance to change.
C. Contemplation stage is characterized by ambivalence with a willingness to consider but not to commit to change.
References:
An adverse event occurs at a workplace. If the workplace has a culture of safety, what is the focus after the event?
A. Who caused the problem?
B. What went wrong?
C. Financial consequences of the error or adverse event
D. Apportionment of blame for the error or adverse event
Correct Answer: B.
What went wrong?
In a culture of safety, when an error or adverse event occurs, the focus is on what went wrong, not on who should be blamed.
Incorrect Answers:
A. The focus in a culture of safety after an adverse event is on “what” went wrong, not on “who” caused the problem.
C. A culture of safety emphasizes determination of what went wrong, not the financial consequences of the event or error.
D. In a culture of safety, the focus is not on who is at fault, but instead it is a non-punitive culture that emphasizes excellence, accountability, integrity, honesty, and mutual respect.
Vital Concepts:
In a culture of safety, when an error or adverse event occurs, the focus is on what went wrong, not on who should be blamed.
References:
An adolescent begins to experiment with drugs and alcohol despite knowing the risks and being fearful of the consequences. What best describes this behavior?
A. Denial
B. Splitting
C. Acting out
D. Counterphobia
Correct Answer: D.
Counterphobia
Counterphobia is seeking out experiences that are consciously or unconsciously feared. For example, the attraction to horror movies may stem from a counterphobic impulse.
Incorrect Answers:
A. Denial is the unconscious process through which unpleasant feelings, thoughts, or impulses are avoided.
B. Splitting is a defense that is created by the failure of an individual to unite the positive and negative attributes of self and others into a realistic whole (also known as all-or-none thinking).
C. Acting out is the expression of unconscious conflicts in a setting other than the one in which they initially arose.
References:
A 23-year-old male presents to the ED with a history of hallucinations, delusions, internal preoccupation, and disorganized behavior for 7 months. For the last 2 days, the patient has been agitated, impulsive, and violent at times. He was admitted to the hospital 3 years ago for similar complaints with less severity; at the time, he developed abnormal postures, tremors, difficulty talking, and drooling after 4 days of treatment. What is the drug of choice for the treatment of acute psychosis in this patient?
A. Lorazepam
B. Risperidone
C. Haloperidol
D. Olanzapine
Correct Answer: D.
Olanzapine
Olanzapine is the drug of choice in patients with a history of severe extrapyramidal symptoms (EPS). These medications are as effective in acute psychosis as first-generation antipsychotics and cause fewer EPS but more metabolic symptoms. Anticholinergic medications are a mainstay of treatment, serving as effective prophylaxis for EPS found in response to first-generation antipsychotics and occasionally second-generation antipsychotics (especially risperidone). Anticholinergic medications contribute to increased side effects (e.g. cognitive dulling) when used with a first-generation antipsychotic like haloperidol.
Antipsychotics are the mainstay of treatment for schizophrenia. First-generation antipsychotics like haloperidol bind the dopamine-2 receptors with high affinity and antagonize them, blocking dopamine release. In general, a blockade of 80% or more of the receptors is associated with increased extrapyramidal effects, while a lower blockade (60-70%) is safer and still clinically effective. Second-generation antipsychotics also block dopamine-2 receptors, but each drug has a different binding affinity (e.g. risperidone is high, while clozapine and quetiapine are lower). Both first- and second-generation antipsychotics bind with muscarinic, histaminergic, and adrenergic receptors as well, which may also contribute to their clinical effects. Second-generation drugs are more effective in treating negative symptoms. First-generation antipsychotics are also used to treat acute psychosis at higher than maintenance doses. Beta-blockers and lithium can be used for aggression and impulsivity in schizophrenics, and antidepressants can be used to treat comorbid depression or anxiety disorders. ECT can also be used in schizophrenia, most often for catatonia. Early studies also suggest that transcranial magnetic stimulation (TMS) might be useful in treating hallucinations. Case management, vocational rehab, family therapy, social work, and CBT are important in the treatment success of schizophrenia.
Incorrect Answers:
A. Benzodiazepines can be used as an adjunctive medication for schizophrenia to sedate the patient and to permit the use of relatively lower doses of antipsychotics.
B. Risperidone is an atypical antipsychotic that causes fewer EPS symptoms compared to typical antipsychotics. However, it is associated with greater EPS than other atypical antipsychotics.
C. High-potency haloperidol is a drug of choice in the management of acute psychosis, but it is also linked to severe EPS, especially in younger patients.
References:
Antipsychotics have been associated with an increased risk of seizures. Which of the following drugs carries a black box warning from the FDA regarding seizure risk?
A. Clozapine (Clozaril)
B. Olanzapine (Zyprexa)
C. Lurasidone (Latuda)
D. Aripiprazole (Abilify)
Correct Answer: A.
Clozapine (Clozaril)
Clozapine is the only antipsychotic currently carrying a black box warning from the FDA regarding seizure risk, although almost all antipsychotics may increase seizure risk. Amongst first-generation antipsychotics, chlorpromazine, haloperidol, and loxapine are more likely to cause seizures. Among the atypical or second-generation options, seizure risk may be increased most with clozapine but has also been reported in patients taking quetiapine and olanzapine.
Incorrect Answers:
B. The FDA does not have a black box warning regarding seizures with olanzapine (Zyprexa).
C. The FDA does not have a black box warning regarding seizures with lurasidone (Latuda).
D. The FDA does not have a black box warning regarding seizures with aripiprazole (Abilify).
Vital Concept:
While all antipsychotics may lower the seizure threshold, primarily when used in larger doses or titrated too quickly, clozapine currently carries a black box warning from the FDA regarding this potential complication.
References:
A 25-year-old patient is brought to the ED by the police. They state that the patient punched someone in the face and smashed their windshield with a shovel. When discussing the events with the patient, they stated that the person they assaulted parked so close to their car that they couldn’t back out of their parking spot. Upon further questioning, the patient admits to losing control in violent acts a few times a year and that they just “lose it” when someone “pisses [them] off.” They deny symptoms of psychosis (hallucinations, delusions) or using any substances before this event occurred. They have no other medical or psychiatric history, but their family history is positive for depression and chronic alcohol use disorder. These behaviors meet the DSM-5-TR criteria for which diagnosis?
A. Intermittent explosive disorder
B. Manic episode
C. Antisocial personality disorder
D. Temporal lobe seizures
Correct Answer: A.
Intermittent explosive disorder
DSM-5-TR criteria for intermittent explosive disorder in individuals over the age of 5 include repeated eruptions of rage or aggression as indicated by either:
three or more instances within a year that involve physical attacks (of people or animals) or property damage
instances of physical or verbal outbursts of rage that happen at least twice per week over a 12-week period (these do not result in physical damage or assault)
These outbursts are not designed for some intention or purpose or planned in advance
These outbursts incur legal or financial repercussions, cause personal anguish for the patient, or lead to significant dysfunction (socially or professionally)
These outbursts are disproportionate in relation to the inciting or preceding event
These outbursts should not be better attributed to a substance, another medical condition, or another mental health condition
These outbursts should not occur in children between 6-18 as a component of an adjustment disorder.
This disorder can occur comorbid with ADHD, conduct disorder, oppositional defiant disorder, or autism spectrum disorder.
Incorrect Answers:
B. Manic episodes are characterized by increased activity, euphoria, excess spending, risk-taking, and a decreased need for sleep.
C. Antisocial personality disorder is characterized by a general lack of concern for others, accountability, or honesty.
D. Temporal lobe seizures would present with a lack of awareness, confusion, or abnormal movements.
Vital Concept:
The diagnosis of intermittent explosive disorder is based on repeated incidents of aggression and rage directed towards people, animal,s or property that lead to significant anguish or consequences.
References:
Development disorders can be treated with a range of techniques. Which of the following is a true statement about the techniques used to teach school children with developmental disorders?
A. Discrete trial training (DTT) is used with applied behavioral analysis (ABA) in school programs for children with pervasive developmental disorders.
B. ABA is based on a single set of norms and planning for a child with autism.
C. DTT is the only technique used to teach new skills in school programs using ABA.
D. Negative reinforcement is used in DTT.
Correct Answer: A.
Discrete trial training (DTT) is used with applied behavioral analysis (ABA) in school programs for children with pervasive developmental disorders.
Discrete trial training (DTT) is used in conjunction with applied behavioral analysis (ABA) in school programs for children with pervasive developmental disorders.
Incorrect Answers:
B. ABA is a systematic approach to developing a program for each child. Steps include evaluating a child’s strengths and weaknesses, identifying educational and treatment goals, and applying techniques for skill development.
C. DTT is among many techniques used to teach new skills.
D. Positive reinforcement is used in DTT.
References:
Which of the following is a role of the Psychiatric Emergency Services (PES) mobile crisis team?
A. To provide medication for faster relief
B. To evaluate patients in community who may have mental health conditions
C. To increase rates of hospitalizations for patients with psychiatric conditions
D. To drive patients with psychiatric conditions to doctor appointments
Correct Answer: B.
To evaluate patients in community who may have mental health conditions
Psychiatric Emergency Services (PES) may have a mobile crisis team whose role is to evaluate patients in the community, to diffuse a crisis before a patient is treated at the ED, and to decrease rates of hospitalization. Case workers generally can arrange for transportation on a case-by-case basis, but this is not the explicit role of PES.
Incorrect Answers:
A. To provide medication for faster relief is incorrect as the service does not provide medications in crisis but can help get clients to a safe place where medication is available.
C. To increase rates of hospitalizations for patients with psychiatric conditions is incorrect as the goal is to decrease rates of hospitalizations.
D. To drive patients with psychiatric conditions to doctor appointments is incorrect as they do not drive clients to appointments.
Vital Concept:
Psychiatric Emergency Services (PES) may have a mobile crisis team whose role is to evaluate patients in the community, to diffuse a crisis before a patient is treated at the ED, and to decrease rates of hospitalization.
References:
Mandatory outpatient treatment (outpatient commitment) use comprehensive individualized services for prolonged periods. Which of the following statements about these treatment services is true?
A. This type of treatment program is appropriate for all patients with schizophrenia.
B. This program type is appropriate for patients with frequent relapses and hospitalizations.
C. Program participants experience no change in substance use and abuse.
D. Program participants experience no significant change in violent behavior.
Correct Answer: B.
This program type is appropriate for patients with frequent relapses and hospitalizations.
This type of program is most appropriate for patients with schizophrenia who have frequent nonadherence to care and subsequent frequent relapse and hospitalization. These programs utilize comprehensive individualized services for a prolonged length of time. Key to the success of these programs is documented improvement in quality of life and other positive outcomes.
Incorrect Answers:
A. Mandatory outpatient treatment is most appropriate for schizophrenia patients with frequent nonadherence to care and subsequent frequently relapse and hospitalization; this doesn’t describe most schizophrenia patients
C. Mandatory outpatient treatment patients have been documented to show changes in quality of life and other positive outcomes, including substance use.
D. Mandatory outpatient treatment patients have been documented to show changes in quality of life and other positive outcomes, including violent behavior.
References:
The USPTF recommends Rh(D) blood typing and antibody testing at the first prenatal visit and repeated antibody testing for unsensitized Rh(D)- for women at 24 to 28 weeks gestation unless the biologic father is known to be Rh(D) negative. This is an example of which of the following?
A. Cross-sectional study
B. Primary prevention
C. Secondary prevention
D. Tertiary prevention
Correct Answer: C.
Secondary prevention
Rhesus (Rh)-D negative women who deliver an Rh(D) positive baby or who are otherwise exposed to Rh(D) positive red cells are at risk of developing anti-D antibodies. Rh(D) fetuses of these mothers are at risk of developing hemolytic disease of the fetus and newborn, which is associated with serious morbidity or mortality. Administration of a full dose of Rh(D) immunoglobulin is recommended for all unsensitized Rh(D)-negative women after repeated antibody testing at 24-28 weeks gestation. Rh(D) blood typing and antibody testing prevents maternal sensitization and improves outcomes for newborns and is a secondary prevention measure.
Primary prevention strategies are designed to prevent onset of a targeted condition. Secondary prevention measures identify and treat asymptomatic persons with certain risk factors or in whom the condition is not yet clinically apparent. Screening tests are secondary preventive measures. With early diagnosis, the natural course of a disease can be altered to maximize the well-being of a patient. Tertiary prevention manages an existing disease to restore the patient to highest function, minimize negative consequences, and prevent complications of a disease.
Incorrect Answers:
A. Observational study that analyzes data from a population, or a representative subset, at a specific point in time; doesn’t fit this.
B. Designed to prevent onset of a targeted condition; there’s no condition to prevent.
D. Manages an existing disease; but there’s no existing disease to manage.
Vital Concepts:
Rhesus (Rh)-D negative women who deliver an Rh(D) positive baby or who are otherwise exposed to Rh(D) positive red cells are at risk of developing anti-D antibodies. Rh(D) fetuses of these mothers are at risk of developing hemolytic disease of the fetus and newborn, which is associated with serious morbidity or mortality. Administration of a full dose of Rh(D) immunoglobulin is recommended for all unsensitized Rh(D)-negative women after repeated antibody testing at 24-28 weeks gestation.
References:
Stephanie is a 19-year-old woman attending a 4-year university on an academic scholarship. She works a part-time job and is maintaining a 4.0 GPA while attaining a biology degree. A recurring theme during her appointments is that her accomplishments in life have just been a fluke. She does well in school because she was lucky enough to get good teachers or studied the right content. What cognitive distortion is this patient demonstrating?
A. Perfectionism
B. Disqualifying the positive
C. Selective abstraction
D. Unrealistic expectations
Correct Answer: B.
Disqualifying the positive
Disqualifying the positive is the correct answer for this question. This patient is chalking her success up to flukes and luck not to her hard work. There is not enough information in this scenario to determine if this patient is experiencing perfectionism and unrealistic expectations. Selective abstraction does not apply in this situation.
Incorrect Answers:
A. D. There isn’t enough information to determine if the patient is experiencing these.
C. This doesn’t apply — the patient isn’t taking details out of context.
References:
Stimulants are a class of drugs that raise psychological or nervous activity levels in the body. Which of the following statements about their side effects is true?
A. When ADHD patients present with side effects, these are rarely a manifestation of a comorbid disorder.
B. Some side effects may have presented as problems prior to starting medication; document baseline symptoms prior to starting stimulants.
C. Appetite suppression occurs with stimulant use; diphenhydramine can effectively increase appetite.
D. Melatonin is ineffective in improving sleep in children with ADHD.
Correct Answer: B.
Some side effects may have presented as problems prior to starting medication; document baseline symptoms prior to starting stimulants.
Documentation of baseline problems is important prior to the start of medication therapy.
Incorrect Answers:
A. ADHD patients may have side effects from stimulants that represent a manifestation or exacerbation of a comorbid disorder or a side effect of treatment.
C. Appetite suppression does occur with stimulant use, but the antihistamine cyproheptadine is used to increase appetite. Diphenhydramine is used to treat sleep problems.
D. Melatonin is used to treat sleep problems. Other medications used include clonidine, diphenhydramine, trazodone, and mirtazapine.
References:
There are two general categories of legislation: authorization and appropriations. Which of the following is true of appropriations bills?
A. They usually originate in the Senate
B. They provide spending for the life of a bill
C. They establish laws or programs and contain recommended dollar amounts
D. They provide for spending authority for a single fiscal year
Correct Answer: D.
They provide for spending authority for a single fiscal year
There are two general categories of legislation: authorization and appropriations. Authorization bills establish laws or programs and although they recommend dollar amounts in some cases, they do not allocate the funds or guarantee funding for a program. Appropriations bills usually originate in the House of Representatives and provide spending authority for a single fiscal year, from the first of October through September 30.
Incorrect Answers:
A. Appropriations bills usually originate in the House of Representatives.
B. Appropriations bills provide for spending for a fiscal year. General appropriations bills provide budgeting authority for most federal agencies. Supplemental appropriations provide additional funding for projects as needed through the current fiscal year. Continuing appropriations extend appropriations from one fiscal year to the next.
C. Authorization bills establish laws or programs and they may contain recommended dollar amounts, which must be funded by appropriations bills.
Vital Concepts:
Appropriations bills usually originate in the House of Representatives and provide spending authority for a single fiscal year, from the first of October through September 30.
References:
A 19-year-old male presents with facial numbness and bilateral vision loss. He reports tingling, cold feet with a bluish discoloration. His symptoms most likely resulted from inhalation of which substance?
A. Elemental mercury
B. n-Hexane
C. Acrylamide
D. Carbon disulfide
Correct Answer: B.
n-Hexane
n-Hexane exposure is most commonly due to recreational inhalation of household glues. n-Hexane is metabolized to 2, 5-hexanedione, which has neurotoxic effects. These effects are potentiated by methyl ethyl ketone, which may also be contained in adhesives. Acute exposure leads to CNS depression. Repeated glue inhalation may cause “glue-sniffer neuropathy,” a progressive, symmetric, ascending, sensorimotor, peripheral axonopathy. Motor symptoms may be predominant and similar in presentation to Guillain-Barré syndrome. The neuropathy may be associated with autonomic dysfunction, including bluish discoloration and decreased temperature of the involved extremities. This neuropathy has been referred to as a “central-peripheral-distal axonopathy,” as it may also have central effects and lead to spasticity. Other symptoms may include facial numbness, maculopathy, and optic neuropathy. Coasting may occur when symptoms continue to worsen for some period of time following the cessation of exposure.
Incorrect Answers:
A. Elemental mercury contained in dental fillings and thermometers, for example, may be inhaled. The liquid form may be ingested. Inhalation may result in “erethism,” consisting of gingivitis, tremor, and behavioral changes. This patient’s presentation is not consistent with mercury poisoning.
C. Acute toxicity due to acrylamide (a chemical used in ore processing, wastewater management, gel chromatography, and certain foods) leads to encephalopathy and ataxia. Acute exposures may also lead to a delayed neuropathy. Chronic toxicity is marked by progressive, symmetric, large-fiber, axonal, sensory or sensorimotor peripheral neuropathy with minimal associated weakness. This neuropathy may also be associated with autonomic dysfunction. In contrast to n-Hexane exposure, the skin findings in acrylamide toxicity include dermatitis with erythema and exfoliation of the skin, including palmar erythema. Acrylamide toxicity may also be distinguished from n-Hexane exposure based on this patient’s facial numbness and visual symptoms, which would be unusual in acrylamide poisoning.
D. Carbon disulfide (found in perfumes, varnishes, insecticides, and other manufacturing processes) may be inhaled. Acute exposure results in mucosal irritation and burns, encephalopathy, seizures, coma, and respiratory failure. Chronic exposure may lead to behavioral changes, extrapyramidal signs, cerebellar dysfunction, and peripheral neuropathy (sensory or sensorimotor with minimal weakness). Cranial nerve dysfunction may also occur.
References:
A 28-year-old heterosexual man is admitted to the psychiatric hospital after 3 weeks of worsening bifrontal headaches, apathy, and depressed mood. He has been treated as an outpatient for 2 years since becoming HIV-positive and was diagnosed with AIDS after an intracranial toxoplasmosis infection. Although the patient has not taken antiretroviral drugs for several months, he has adhered intermittently to his antiretroviral regimen and previously developed other opportunistic infections. Which of the following statements regarding this patient’s sequelae is true?
A. Visuospatial performance, fine motor control, and coordination usually only affected in late disease.
B. Many patients develop new-onset psychosis in early stages of disease.
C. Anxiety or a sleep disorder may develop, but these are not prevalent manifestations in HIV.
D. Low CD4 count increases likelihood that psychiatric symptoms may be determined by HIV infection.
Correct Answer: D.
Low CD4 count increases likelihood that psychiatric symptoms may be determined by HIV infection.
New onset of symptoms and a low CD4 count of the likelihood that psychiatric symptoms may be due to HIV infection or another HIV-related cause rather than a new psychiatric diagnosis.
Incorrect Answers:
A. Early cognitive and motor deficits involve attention, concentration, visuospatial performance, fine motor control, coordination, and speed.
B. New-onset psychosis is uncommon and generally seen only in advanced disease stages.
C. Problems with sleep and anxiety are prevalent in the HIV-infected population.
References:
Three weeks after initiating clozapine for treatment refractory schizophrenia, a patient develops shortness of breath, fatigue, and flu-like symptoms. On exam, he is tachycardic and has a new systolic murmur. Laboratory evidence indicates elevated CPK and troponins as well as eosinophilia. Clozapine is discontinued, and the patient recovers with supportive care. What is the next course of treatment?
A. Restart clozapine titration after obtaining baseline vitals and echocardiogram.
B. Start clozapine at last dose prior to illness.
C. Do not rechallenge with clozapine in future.
D. Restart clozapine at initial dose and add anticoagulant to patient’s medication regimen.
Correct Answer: C.
Do not rechallenge with clozapine in future.
Clozapine-induced myocarditis most frequently occurs within 1 month of treatment initiation; based on case reports, the median time of treatment before diagnosis is <3 weeks. Usual symptoms include fever and flu-like symptoms, and common objective findings include peripheral eosinophilia, EKG changes, and elevated troponin and CK levels. If myocarditis is suspected, clozapine should be immediately discontinued. With a history of clozapine-induced myocarditis, rechallenge should not be attempted due to reported incidents of symptom recurrence upon restarting the medication.
Incorrect Answers:
A. B. D. Clozapine shouldn’t be challenged in cases where myocarditis is suspected, as there have been reported incidents of symptom recurrence upon restarting medication.
References:
Anxiety disorders are the most common category of psychiatric conditions in the US. What is their lifetime prevalence in the US.?
A. <5%
B. 5-15%
C. 15-25%
D. 25-35%
Correct Answer: D.
25-35%
The most common category of psychiatric problems in the United States is anxiety disorders. The annual prevalence is 19.1%, and the lifelong prevalence is 31.1%, according to the National Comorbidity Study Replication. In the US, anxiety disorders comprise almost 33% of mental health costs. Anxiety can present symptomatically in physical, cognitive, and behavioral ways. In the DSM-5-TR, OCD is now included in obsessive-compulsive and related disorders, and PTSD and acute stress disorder are included with trauma- and stressor-related disorders.
Incorrect Answers:
A. Anxiety disorders’ lifetime prevalence in the U.S. is 31.1%, not <5%.
B. Anxiety disorders’ lifetime prevalence in the U.S. is 31.1%, not 5-10%.
C. The annual prevalence of anxiety disorders falls in this range (it’s 19%). However, this question asked for anxiety disorders’ lifetime prevalence in the U.S. which is 31.1%.
Vital Concept:
The National Comorbidity Study Replication indicates a lifetime prevalence of anxiety in the US at 31.1%.
References:
Body dysmorphic disorder is often comorbid with which of the following disorders?
A. Social anxiety disorder
B. Generalized anxiety disorder
C. Bipolar disorder
D. Specific phobia
Correct Answer: A.
Social anxiety disorder
Social phobia (social anxiety disorder within the DSM-5-TR) is often seen as a comorbid state with body dysmorphic disorder. Social anxiety disorder criteria include:
Terror or significant concern about a certain environment exposes the patient to potential judgment by the public or individuals. This may be when being watched (e.g., during observations, dining in public spaces), presenting (e.g., public speaking), or socializing (e.g., talking or interacting with new people).
The patient is concerned that they will behave poorly and be judged by others (i.e., they will be rejected, mortified)
The patient reports sudden terror or significant concern every time they are exposed to the certain environment
The patient evades the certain environment
The actual risk or threat posed by the environment is insignificant in comparison to the patient’s emotional response and concern
The terror or concern is consistent for at least 6 months
The terror/concern or active evasion of the environment leads to dysfunction (academic, professional, social, or otherwise) or substantial anguish
The patient’s symptoms are not directly related to the use of a substance, medication, or pre-existing medical diagnosis or health concern. The concern is not due to a more appropriate psychiatric condition such as autism spectrum disorder, panic attacks (i.e., panic disorder), past trauma (i.e., post-traumatic stress disorder), separation from a loved one (i.e., separation anxiety disorder), a specific trigger (i.e., specific phobia), physical judgment (i.e., body dysmorphic disorder), or a recurrent thought (i.e., obsessive-compulsive disorder).
In pediatric patients, the terror/concern must be present when interacting with peers (not only grown-ups) and may present as dependence, immobility, mutism, outbursts, or fits
The terror/concern is disproportionate or unconnected in those with a physical attribute that causes them discomfort (e.g., significant scars, facial or other obvious physical difference
Incorrect Answers:
B. GAD is not often found to be comorbid with body dysmorphic disorder.
C. Bipolar disorder is not often found to be comorbid with body dysmorphic disorder.
D. Specific phobias are not often found to be comorbid with body dysmorphic disorder.
Vital Concept:
Body dysmorphic disorder is often comorbid with unipolar depression, social anxiety disorder, personality disorders, substance use disorder, or obsessive-compulsive disorder.
References:
A nurse practitioner has a patient in his practice who has a history of bipolar disorder and is treated with lithium. During a routine examination, the nurse practitioner decides to check a lithium level, and it is elevated, but the results do not return to the clinic until 2 days after the patient’s visit. The NP calls the patient with the results at the telephone number recorded on his chart. Which of the following is true?
A. The NP should leave a message on the answering machine with the abnormal test result
B. The NP can only leave a message with the patient’s spouse or other family member if the patient is not at home
C. The NP should leave a message with a contact number for the patient to call back
D. Access to psychiatric records cannot be denied to a patient
Correct Answer: C.
The NP should leave a message with a contact number for the patient to call back
HIPAA rules are designed to protect the privacy of patients and their confidential medical information. Patient consent is required before releasing patient information to third parties, except in specific circumstances (having to do with third-party payers, reportable diseases, minors). If a healthcare provider is delivering the results of a laboratory test or diagnostic study to a patient, even if the study result is normal, that information cannot be released to anyone for whom the patient has not signed a written consent for release of information. The NP or provider who is calling a patient with test results should leave only a message with name and contact number for the patient to return the call. Information cannot be given to spouse or family members without express consent by the patient. Laboratory or test results should never be left on an answering machine. With respect to psychiatric records, a psychiatric diagnosis and/or mental health records may not be released without a specific signed authorization by the patient. However, patients can be denied access to their psychotherapy records.
Incorrect Answers:
A. Message should only have name and contact number for the patient to return the call
B. Information can’t be given to a spouse or family member without the patient’s express consent
D. Patients can be denied access to the psychotherapy records
Vital Concepts:
HIPAA rules are designed to protect the privacy of patients and their confidential medical information. Patient consent is required before releasing patient information to third parties, except in specific circumstances (having to do with third-party payers, reportable diseases, minors). If a healthcare provider is delivering the results of a laboratory test or diagnostic study to a patient, even if the study result is normal, that information cannot be released to anyone for whom the patient has not signed a written consent for release of information.
References:
Gabapentin is used treat seizures and pain caused by shingles. Which of the following is true with regards to its use?
A. It interacts with anticonvulsants
B. Somnolence and dizziness are the most common side effects and may lead to discontinuation.
C. It is metabolized by the liver.
D. It inhibits cytochrome P450 enzymes
Correct Answer: B.
Somnolence and dizziness are the most common side effects and may lead to discontinuation.
It shows no inhibition of cytochrome P450 enzymes to any degree. Somnolence and dizziness may prompt discontinuation of the drug. The extent of these symptoms can be mitigated by administering a larger percentage of the drug at night. It is not metabolized by the liver but is excreted largely unchanged.
Incorrect Answers:
A. Gabapentin doesn’t interact with anticonvulsants
C. Gabapentin isn’t metabolized by the liver; it’s excreted largely unchanged
D. Gabapentin doesn’t show any inhibition of cytochrome P450 enzymes
References:
Kelly complains about her job duties at work and her boss’s expectations. She is regularly late for work, dresses inappropriately, and misses most deadlines. When she is fired, she claims it is because, “my boss is incompetent and lazy,” instead of blaming it on her own poor work performance. Which defense mechanism explains Kelly’s reaction?
A. Denial
B. Projection
C. Undoing
D. Regression
Correct Answer: B.
Projection
Kelly’s defense mechanism is projection, instead of recognizing her role in losing her job she blames her boss. Denial could be considered but does not explain Kelly blaming her boss. Undoing and regression do not apply in this situation.
Incorrect Answers:
A. This could be considered, but doesn’t explain why Kelly blames her boss for being incompetent and lazy.
C. This is engaging in contrary behavior to remove an unhealthy or otherwise threatening thought or behavior; it doesn’t apply here.
D. Kelly hasn’t shown signs of reverting to a childlike or similarly regressed state; so regression doesn’t apply here.
References:
After the introduction of barbiturates in a depressed patient who is also taking a TCA, the effect of the TCA decreases. Why does this occur?
A. Barbiturates replace TCA on its receptor.
B. Barbituates lower plasma TCA.
C. Higher levels of TCA cause negative feedback.
D. A and C
Correct Answer: B.
Barbituates lower plasma TCA.
In depressed patients, the administration of barbiturates often reduces TCA plasma levels and diminishes antidepressant effects because of their induction of liver microsomal enzymes and accelerated degradation of the TCA. Clinicians should keep this in mind when considering the adjunctive use of hypnotics in depressed patients.
Incorrect Answers:
A. Barbiturates act on GABA receptors, while the TCA acts as an SNRI.
C. The plasma concentration of the TCA decreases.
References:
An adult patient reports difficulty expressing themself due to frequent incidences of getting “stuck” on a syllable while trying to speak. The patient often feels anxious when required to give a presentation at work. This problem became apparent when they were in 2nd grade. What is the most appropriate DSM-5-TR diagnosis?
A. Specific phobia
B. Social (pragmatic) communication disorder
C. Childhood-onset fluency disorder
D. Speech sound disorder
Correct Answer: C.
Childhood-onset fluency disorder
Childhood-onset fluency disorder (stuttering) is the most appropriate diagnosis for this patient. It is a disorder where a disturbance in the normal fluency and time patterning of speech is not appropriate for the patient’s age, with frequent repetitions or prolonging of sounds. Typically develops before age 6. Speech therapy can assist patients in speaking slowly and effectively. CBT can be helpful in identifying situations that worsen stuttering and strategies to decrease stress with stuttering.
Incorrect Answers:
A. Specific phobia is diagnosed when there is fear about a specific situation/object, and that stimulus always provokes an immediate response.
B. With social (pragmatic) communication disorder, patients have problems with verbal and nonverbal communication in social situations.
D. Speech sound disorders refer to functional (e.g., articulation, phonology) or organic (e.g., dysarthria, apraxia, cleft palate, hearing impairment) speech sound disorders.
Vital Concept:
Childhood-onset fluency disorder (stuttering) is a disorder where a disturbance in the normal fluency and time patterning of speech is not appropriate for the patient’s age, with frequent repetitions or prolonging of sounds.
References:
A nurse practitioner notices that a child with a mother who smokes appears to have frequent upper respiratory infections. She then assumes all children who have mothers who smoke will develop frequent upper respiratory infections. What type of reasoning is she using?
A. Correlational
B. Inductive
C. Deductive
D. Experimental
Correct Answer: B.
Inductive
Inductive reasoning refers to developing generalizations from specific observations. Deductive reasoning is the opposite process and refers to developing specific predictions from a set of general principles.
Incorrect Answers:
A. Correlation can be determined through statistical analysis of variables, but it is not a type of reasoning.
C. In this case, a nurse practitioner who knew that children of smoking mothers have a higher rate of respiratory infections may have used deductive reasoning to predict that this child would develop a respiratory infection.
D. Experimental design is a formal process through which the outcome of an intervention can be measured.
Vital Concept:
Inductive reasoning refers to developing generalizations from specific observations. Deductive reasoning is the opposite process and refers to developing specific predictions from a set of general principles.
References:
An adult patient with catatonic schizophrenia is started on haloperidol. After the fourth day of treatment, they gradually develop restlessness, sweating, and palpitations and are unable to sit comfortably in a single place. The patient repeatedly rubs their hands and reports feeling uneasy. Based on the most likely diagnosis, which of the following is the initial treatment of choice?
A. An increase in haloperidol dosage
B. Diphenhydramine (Benadryl)
C. Flumazenil (Romazicon)
D. A normal saline bolus and IV phenylephrine
Correct Answer: B.
Diphenhydramine (Benadryl)
The patient has developed akathisia, which is a side effect of high-potency typical antipsychotics. It is an acute extrapyramidal syndrome (EPS). It is likely related to an imbalance between dopamine receptor antagonism and muscarinic receptor antagonism. It is associated with a subjective or objective feeling of restlessness, walking, inattention, and sweating. Akathisia is also seen with antidepressants and sympathomimetics. The treatment of choice is diphenhydramine (Benadryl). A beta-blocker like propranolol may be used to reduce involuntary movements but does not affect anxiety. Other treatments include benztropine, benzodiazepines, and amantadine.
Incorrect Answers:
A. An increased dose would be warranted to manage residual agitation or psychosis. Acute psychosis is unlikely due to the absence of any psychotic symptoms.
C. Flumazenil (Romazicon) is administered to reverse an acute overdose of benzodiazepines, and would not be effective in managing an acute side effect of an antipsychotic.
D. A normal saline bolus and phenylephrine would be appropriate to manage a patient presenting with refractory hypotension related to antipsychotic overdose, but this does not correlate with the clinical scenario described.
Vital Concept:
The presentation of akathisia related to typical antipsychotics is described here. The initial treatment typically consists of an antihistamine (diphenhydramine). Alternatives include the use of benztropine, benzodiazepines, propranolol, or amantadine.
References:
Which of the following is a risk factor for child/adolescent suicide?
A. Heterosexual
B. Substance abuse
C. Living with multiple family members
D. Low IQ
Correct Answer: B.
Substance abuse
Risk factors for child/adolescent suicide include substance use, exposure to violence, previous suicide attempts, immediate family member who completed suicide, previous psychiatric hospitalizations, recent losses, social isolation, bullying, access to guns, lower socioeconomic status, current psychiatric disorder, prior suicide attempts, age >16, male gender, and gay/lesbian/bisexual individuals.
Incorrect Answers:
A. Gay/lesbian/bisexual individuals have an increased risk for suicide compared to heterosexual individuals.
C. Living alone is a risk factor for suicide, not living with multiple family members.
D. There is no correlation with low IQ and suicidality in children/adolescents.
Vital Concept:
Children and adolescents that identify as gay/lesbian/bisexual, use substances and/or live alone are at increased risk for suicide.
References:
A 14-year-old girl is admitted for left-sided paralysis. During a neurological evaluation, when the physician lifts the patient’s hand and drops it onto her face, her hand falls next to her face. When examining her paralyzed leg, the physician notes pressure in the hand that is placed under the paralyzed leg when attempting a straight leg raise. Based on these findings, the patient is diagnosed with conversion disorder. When explaining the diagnosis of conversion disorder to the patient and her family, which statement is the most appropriate?
A. “Paralysis is intentional, and she is feigning symptoms.”
B. “It is unlikely that her symptoms will remit within the next year.”
C. “She should be referred to a psychiatric clinic immediately for psychotherapy.”
D. “It is important to understand that this paralysis is involuntary.”
Correct Answer: D.
“It is important to understand that this paralysis is involuntary.”
When treating conversion disorder, confrontation about the symptoms is contraindicated. Treatment recommendations include reassurance (in this case, emphasizing to the family that the symptoms are involuntary) and reasonable rehabilitation.
Incorrect Answers:
A. To be diagnosed with conversion disorder, symptoms cannot be intentionally produced or feigned.
B. Conversion disorder is typically of short duration, usually remitting spontaneously within 2 weeks in hospitalized patients. It is unlikely that her symptoms will persist for a year.
C. Psychotherapy may be useful but may also be contraindicated, particularly if the patient is resistant to psychotherapy or the symptoms worsen when treatment is initiated.
References:
Different support systems offer varying levels of support. What type of support system in the community includes organizations and agencies which provide individual access to goods and services?
A. Formal
B. Semiformal
C. Informal
D. Family
Correct Answer: B.
Semiformal
Semiformal support systems provide physical and emotional assistance. They include organizations and agencies in the community that provide goods and services.
Incorrect Answers:
A. Formal support is regulated by laws or statutes. Formal social support is provided by social workers, financial support by Social Security, and medical support provided by Medicare.
C and D. Informal support derive from the social network, includes family, and friends, but only those who actually provide assistance in some way.
Vital Concept:
Semiformal support systems provide physical and emotional assistance. They include organizations and agencies in the community that provide goods and services.
References:
Relational theory focuses on the real relationship between patient and therapist to understand and relieve conflict and social inhibition and to achieve social intimacy. Which theorist is associated with this type of therapy?
A. Sigmund Freud
B. Anna Freud
C. Jean Baker Miller
D. Carl Jung
Correct Answer: C.
Jean Baker Miller
Jean Baker Miller is associated with relational theory.
Incorrect Answers:
A. Sigmund Freud is associated with classical or structural theory.
B. Anna Freud is associated with ego psychology.
D. Carl Jung is associated with transpersonal psychology.
References:
Autism spectrum disorder has a number of essential features. Which of the following is an essential feature of autism spectrum disorder?
A. Accelerated development of communication
B. Aversion to stimuli
C. Impaired development of social interaction
D. Broad repertoire of activities and interests
Correct Answer: C.
Impaired development of social interaction
Impaired development of social interaction is an essential feature of autism spectrum disorder.
The diagnostic criteria for ASD include:
1) Current or historical struggles engaging with others across several domains:
building, sustaining, and comprehending relationships
challenges with the back-and-forth in a group setting, talking and interacting
body language, reading facial expressions
2) Actions, thoughts, and attention that are limited and very consistent, sometimes referred to as restricted repetitive behaviors (RRBs). At least two of the following (previously or currently):
curiosity and attention to a limited number of topics
patterns of speech and movement that are unchanging
significantly increased or decreased response or attention to sights/sounds/smells/touches/feelings from their environment
very consistent patterns in daily routines
Both components 1 and 2 are required for the diagnosis of ASD, although social communication disorder may be diagnosed if no RRBs are present.
3) Signs may increase with age and elevated communication and interaction requirements but are initially evident at a very young age
4) The ability to perform successfully in crucial environments (home, school, work, friends) is significantly hindered by the condition
5) Another condition, such as developmental delay or intellectual limitation, does not provide an enhanced reason for the symptoms.
Incorrect Answers:
A. Impairment in communication is an essential feature of autism spectrum disorder; accelerated development of communication wouldn’t be an essential feature of autism spectrum disorder.
B. Aversion to stimuli isn’t an essential feature of autism spectrum disorder.
D. Repetitive patterns of activities and interests are an essential feature of autism spectrum disorder; a broad repertoire of activities and interests wouldn’t be an essential feature of autism spectrum disorder.
Vital Concept:
Autism spectrum disorder is defined by current or historical struggles engaging with others across several domains.
References:
Clozapine is an FDA-approved treatment for a specific condition. Which of the following clinical situations is it approved for?
A. Treatment-refractory bipolar disorder
B. Tardive dyskinesia
C. Leukocytosis
D. Suicidality in schizophrenia
Correct Answer: D.
Suicidality in schizophrenia
While clozapine may have some potential benefit in other situations, it is FDA-indicated for only suicidality in schizophrenia among the choices listed. Clozapine has a risk of causing leukopenia (specifically agranulocytosis). Clozapine would not be used to treat leukocytosis.
Incorrect Answers:
A, B. Clozapine isn’t FDA-indicated for these conditions.
C. Clozapine has a risk of causing leukopenia, so it wouldn’t be used to treat leukocytosis.
References:
Which of the following statements correctly explains the mechanism of increased lamotrigine levels when coadministered with valproate?
A. Valproate inhibits microsomal enzymes.
B. Valproate inhibits excretion of lamotrigine in kidneys.
C. Valproate displaces lamotrigine from protein-binding sites.
D. Valproate competes with lamotrigine for some glucuronidation sites.
Correct Answer: D.
Valproate competes with lamotrigine for some glucuronidation sites.
UDP-glucuronosyltransferase (UGT) enzymes comprise a superfamily of key proteins that catalyze the glucuronidation reactions that are needed to metabolize some drugs. Valproate inhibits UGT2B7 but does not effect UGT1A4. Lamotrigine depends on glucuronidation to be eliminated and inactivated, so anything that blocks either UGT1A4 or UGT2B7 will increase lamotrigine levels. Depakote also inhibits only UGT2B7.
Drugs that inhibit UGT1A4 include: 4-OH-tamoxifen, amitriptyline, androsterone, asenapine, chlorpromazine, clozapine, cyclobenzaprine, cyproheptadine, diphenhydramine, doxepin, imipramine, irinotecan, lamotrigine, loxapine, meperidine, nicotine, olanzapine, progestins, promethazine, and retigabine. Drugs that inhibit UGT2B7 include: almokalant, atorvastatin, buprenorphine, carvediolol, chloramphenicol, clofibric acid, codeine, cyclosporine, diclofenac, entacapone, epirubicin, febuxostat, fenofibrate, fenoprofen, fluvastatin, gemfibrozil, hydromorphone, ibuprofen, ketoprofen, lamotrigine, lorazepam, methadone, morphine, mucophenolate, nalorphine, naloxone, naltrexone, naproxen, nicotine, oxazepam (r), oxycodone, naloxone, simvastatin, tacrolimus, temazepam, valproic acid, and zidovudine.
Incorrect Answers:
A. Valproate inhibits microsomal enzymes. The correct mechanism of action is Valproate inhibits UGT2B7 that assists in lamotrigine elimination; blocking it increase lamotrigine levels.
B. Valproate inhibits excretion of lamotrigine in kidneys. The correct mechanism of action is Valproate inhibits UGT2B7 that assists in lamotrigine elimination; blocking it increase lamotrigine levels.
C. Valproate displaces lamotrigine from protein-binding sites. The correct mechanism of action is Valproate inhibits UGT2B7 that assists in lamotrigine elimination; blocking it increase lamotrigine levels.
References:
What is the estimated lifetime prevalence of schizophrenia in the general population?
A. 0.3-1%
B. 1-2%
C. 2-4%
D. 4-5%
Correct Answer: A.
0.3-1%
Schizophrenia affects less than 1% of people worldwide. It is characterized by positive symptoms (hallucinations, delusions, and disorganized speech and behavior), negative symptoms (flattened affect and alogia), and other symptoms like inattentiveness. Many patients also have a lack of insight regarding their condition. There is usually significant impairment in their social, occupational, and interpersonal lives. In order to diagnose schizophrenia, symptoms must be present for at least 6 months, including at least 1 month of positive or negative symptoms.
Incorrect Answers:
B. The estimated lifetime prevalence of schizophrenia is 0.3-1%, not 1-2%.
C. The estimated lifetime prevalence of schizophrenia is 0.3-1%, not 2-4%.
D. The estimated lifetime prevalence of schizophrenia is 0.3-1%, not 4-5%.
Vital Concept:
The DSM estimates that the lifetime prevalence of schizophrenia is 0.3-0.7%.
References:
According to the American Psychological Association (APA), integrative behavioral couple therapy (IBCT) is an empirically-proven approach in couples therapy. In this type of therapy, what does the technique of unified detachment relate to?
A. Empathic joining
B. Mindfulness
C. Emotional refusal
D. Tolerance building
Correct Answer: B.
Mindfulness
Mindfulness involves learning direct attention, increasing awareness, and promoting immersion in the present moment. Unified detachment involves examining a problem from an emotional distance with a greater objective focus.
Incorrect Answers:
A. Empathic joining is another technique used in integrative behavioral couples therapy that involves evocation of strong emotions in the dyad.
C. Emotional acceptance is promoted in integrative couples behavioral therapy through empathic joining, unified detachment, and tolerance building.
D. Tolerance building is another technique used in integrative behavioral couples therapy that involves helping dyad members to be more tolerant of upsetting behaviors.
References:
All women of childbearing age who are treated with valproate should receive a vitamin supplement. What vitamin should they receive?
A. Folate (B-9)
B. B-12
C. B-1
D. B-5
Correct Answer: A.
Folate (B-9)
All women of childbearing potential who are treated with valproate should receive concomitant folate (B-9) supplementation, regardless of whether they plan to conceive.
Incorrect Answers:
A, C, and D. These are not the vitamin supplement that women of childbearing age should receive. They should receive folate, which is vitamin B-9.
References:
A 71-year-old man with a history of depression and coronary artery disease is brought to the ED by his wife. She states that he has been confused over the past day and did not sleep last night. On physical exam, the patient appears sweaty and tremulous. His deep-tendon reflexes are 4+. Vital signs are as follows: temperature 102°F, pulse 115, respirations 18, and blood pressure 175/110. The patient’s wife reports that he takes a baby aspirin, sertraline, and simvastatin daily. They have recently tried to adopt a more organic lifestyle and are taking several herbal vitamins and supplements. Which of the following substances could explain this patient’s current condition?
A. Ginkgo biloba
B. Echinacea purpurea
C. Allium sativum
D. St. John’s wort
Correct Answer: D.
St. John’s wort
St. John’s wort is used as an herbal treatment for depression. It causes marked CYP induction and lowers the level of many common drugs, including warfarin and digoxin. It also acts as a serotonin reuptake inhibitor and should be avoided in patients taking serotonin-raising medications like MAOIs and SSRIs due to the risk of serotonin syndrome.
Incorrect Answers:
A. Ginkgo biloba has several perceived effects on blood flow and cognitive enhancement, although studies in humans have been equivocal. It should be avoided with antiplatelet and anticoagulant medications.
B. Echinacea purpurea may have anti-inflammatory and immune-enhancing properties. It may cause a flu-like syndrome but has no known drug interactions. It is generally recommended to avoid echinacea in immune-related conditions such as post-transplant immune suppression as well as autoimmune and immune-deficiency conditions (e.g. AIDS, cancer).
C. Allium sativum is garlic, which has mild cholesterol-lowering characteristics. Garlic has reported antiplatelet effects and should be used cautiously with antiplatelet and anticlotting medications.
References:
A 19-year-old patient presents with their first manic episode; they are admitted for inpatient care and started on lithium. On day 2 of their admission, the patient experiences motor excitement, mutism, and stereotypic movements. Lorazepam is trialed for several days and is ineffective. Which treatment is indicated next to treat these symptoms?
A. Start an atypical antipsychotic
B. Increase the lithium dose
C. Start electroconvulsive therapy (ECT)
D. Switch to valproate
Correct Answer: C.
Start electroconvulsive therapy (ECT)
The diagnostic criteria for catatonia in the DSM-5-TR includes at least three of the symptoms below:
catalepsy (i.e., a posture maintained due to muscular contractions, often against gravity and without intention, after being placed in that position by the examiner)
mutism (i.e., minimal or absent speech, not applicable if the patient has a history of aphasia)
posturing (i.e., a posture maintained due to muscular contractions, often against gravity and without intention, after spontaneously assuming that position)
stereotypy (i.e., recurring actions or movements that are quickly performed without purpose)
facial expressions that communicate pain or anguish
echopraxia (i.e., copying someone else’s actions)
echolalia (i.e., to copy someone else’s sounds when talking)
stupor (i.e., a lack of responsiveness)
waxy flexibility (i.e., a consistent yet minimal muscular opposition to being placed in a position by the examiner)
negativism (i.e., doing the opposite or not reacting to external directions or requests)
mannerism (i.e., strange or affected display of normal motions)
irritability not affected by the surrounding environment
There is strong evidence for treatment with a benzodiazepine and ECT. Patients generally respond well to benzodiazepines, and lorazepam has the most evidence. If a good response does not occur, ECT should be strongly considered as an effective treatment for catatonia. When the diagnosis of catatonia has been made, the use of antipsychotic and/or antidepressant medications should generally be avoided.
Incorrect Answers:
A. Antipsychotics can worsen catatonia due to parkinsonian side effects and can increase the risk of neuroleptic malignant syndrome.
B. Increasing the dose of the mood stabilizer is not as effective as ECT when there is a critical manifestation such as catatonia.
D. A change in mood-stabilizer dose or a change to another mood stabilizer is not an effective treatment for catatonia.
Vital Concept:
In patients that exhibit catatonia, first-line treatments include the use of benzodiazepines and electroconvulsive therapy (ECT).
References:
A 32-year-old male presents claiming that his father is trying to kill him by poisoning his food. The patient reports poor sleep for the past 5 weeks as well as auditory hallucinations. He has become increasingly withdrawn and suspicious over the past 2 months and is talking to himself. What is the appropriate diagnosis?
A. Schizoaffective disorder
B. Delusional disorder
C. Brief psychotic disorder
D. Schizophreniform disorder
Correct Answer: D.
Schizophreniform disorder
Schizophreniform disorder is differentiated from schizophrenia by the duration of symptoms. In schizophreniform disorder, the total duration of illness (including prodromal and residual phases) is at least 1 month and under 6 months. At follow up, 33% of patients will show full recovery, while the rest will be diagnosed with identified mood disorders or schizophrenia.
Incorrect Answers:
A. Schizoaffective disorder is diagnosed with prominent features of both schizophrenia and major mood disorders. Psychotic features occur intermittently in the absence of mood disorder.
B. Delusional disorder is characterized by the presence of 1 or more delusions in the relative absence of other symptoms of psychosis and stressors.
C. Brief psychotic disorder is mostly diagnosed with acute psychotic symptoms after any social or family stressor. Symptoms last for <1 month with full spontaneous remission.
Vital Concept:
Duration of psychotic symptoms (e.g. delusions, hallucinations, disorganized speech/behavior) is important for diagnostic clarification. In brief psychotic disorder symptoms are present for 1 day to 1 month. In schizophreniform disorder symptoms are present for 1 to 6 months. In schizophrenia symptoms are present for at least 6 months.
References:
While on call for a pediatric clinic, you receive a call from the local children’s hospital about a 2-year-old girl who was rushed to the hospital by ambulance after being hit by a car. She suffered bilateral wrist fractures, a tibia fracture, a femur fracture, and an epidural hematoma. She was intubated on the way to the hospital and admitted to the pediatric intensive care unit (PICU). Unfortunately, she dies overnight from her injuries.
Since the family is well-known for your practice, you visit the hospital to offer your condolences and support. You encourage the parents and three surviving children (ages 3, 11, and 17 years old) to follow up with you next month in the clinic. Which would be a concerning sign of complicated grief in the surviving siblings?
A. The 3-year-old cries for 10 minutes every time she is dropped off at daycare for a month after the death of her sister.
B. The 3-year-old starts having urinary accidents during the day even though she has been toilet-trained during the day for a year; these daytime accidents resolve after 3 months.
C. The 11-year-old complains of periumbilical abdominal pain that mainly occurs on weekdays; the pain is not associated with constipation or weight loss and resolves spontaneously after 1 month.
D. The 17-year-old is repeatedly caught sneaking out, drinking alcohol, and she is failing two classes.
Correct Answer: D.
The 17-year-old is repeatedly caught sneaking out, drinking alcohol, and she is failing two classes.
There are five stages of grief in the model created by Kubler-Ross: denial, anger, bargaining, depression, and acceptance. The stages of grief can occur in any order, and the duration usually varies from 6-12 months. Children may have separation anxiety, regressive behavior, somatic complaints, thoughts of death, or hallucinations of the lost loved one. Complicated grief is persistent and causes functional impairment.
Reactions to grief are influenced by developmental stages, family coping style, cultural background, and the circumstances surrounding the death. Common expressions of grief include repeated questioning, regressive behaviors, and extreme emotions like anger or fear. This adolescent is demonstrating an increase in high-risk behaviors by experimenting with alcohol. Other high-risk behaviors are drug use, delinquency, and precocious sexual activity. This alcohol use concerns complicated grief, and the family should be referred to a grief counselor.
Incorrect Answers:
A. The 3-year-old child is experiencing separation anxiety. Separation anxiety is only concerning for complicated grief if it persists for more than six months after a predictable home routine has been established following a traumatic event such as a death.
B. Daytime urinary accidents are an example of regressive behavior. This can be normal after a traumatic event if it resolves within six months.
C. School-aged children may respond to a death by reporting physical symptoms such as abdominal pain and headaches that have no organic cause. If these symptoms persist for more than 3 months, they would indicate the presence of pathological grief. Since the pain only occurs on school mornings, it is likely due to school phobia. School phobia that persists for 3 months after the sibling’s death would be considered complicated grief.
Vital Concept:
The five stages of grief include denial, anger, bargaining, depression, and acceptance. These stages may occur in any order. Sadness, shock, anxiety, and somatic complaints are also symptoms of grief and can occur in waves. If grief does not resolve within 6-12 months or causes functional impairment, it is considered complicated grief.
References:
The Lazarus Stress and Coping Theory, developed in 1984, is a way to understand and approach stress in life. Which of the following is not part of Lazarus’ conceptualization of coping?
A. Self-exploration
B. Self-instruction
C. Self-correction
D. Self-pity
Correct Answer: D.
Self-pity
Self-pity is not a central aspect of Lazarus’ conceptualization of coping. Self-exploration, self-instruction, self-correction, and self-rehearsal are part of the extensive and recursive process Lazarus notes in his “Transactional Model for Stress Management.”
Incorrect Answers:
A, B, and C. These are all part of the process Lazarus notes in his “Transactional Model for Stress Management.”
References:
Valproic acid is used to treat seizures and bipolar disorder and help prevent migraine headaches. Which of the following statements about it is true?
A. It’s much less effective than lithium in patients with mixed symptoms.
B. It is more effective than lithium in patients with prominent depressive symptoms during mania and with multiple prior mood episodes.
C. In acute mania, it is superior to haloperidol.
D. It’s superior to olanzapine in reducing acute manic symptoms and facilitating remission
Correct Answer: B.
It is more effective than lithium in patients with prominent depressive symptoms during mania and with multiple prior mood episodes.
Valproic acid is more effective than lithium in patients with prominent depressive symptoms during mania, with mixed symptoms, or with multiple prior mood episodes.
Incorrect Answers:
A. Valproic acid is much more effective than lithium in patients with mixed symptoms.
C. Valproic acid and haloperidol are equally effective in acute mania
D. Olanzapine is superior to valproic acid in reducing acute manic symptoms and facilitating remission
References:
A 32-year-old well-dressed woman presents to the emergency room at 3:00am complaining of “terrible anxiety.” She is obviously distressed and nervous and is constantly looking around and peering at the walls. The emergency room staff are exasperated with her, but she is beside herself with anxiety and cannot explain why. She will not stay in her room and is pacing the halls. The patient is diaphoretic and tachycardic and has pupils which are mildly dilated. She swears she is on no medications and takes no OTC supplements. She has never had this happen before. What is the next step in ruling out the most important possible cause of anxiety in this patient?
A. Benzodiazepine administration for delirium tremens
B. Beta-blocker or benzodiazepine for panic attack
C. Toxicology analysis for cocaine overdose
D. No acute treatment available for hypochondriasis
Correct Answer: C.
Toxicology analysis for cocaine overdose
Cocaine use is a dangerous etiology of an apparent anxiety attack, which could possibly have serious repercussions for her health. The other conditions, although scary for the patient, would not be as potentially dangerous from a medical standpoint or are much less likely in this situation.
Incorrect Answers:
A. It seems unlikely that she would be having delirium tremens, and her symptoms don’t exactly match (no “tremens” or shakes, dilated pupils).
B. A beta-blocker would be premature at this point, without determining the etiology of her anxiety. In addition there is the possibility of unopposed alpha stimulation with a combination of cocaine and beta-blockers.
References:
Many schizophrenia patients smoke. Which of the following statements about schizophrenia and smoking is true?
A. As many as 95% of all schizophrenics smoke.
B. Smoking can cause weight gain, which is another modifiable cardiac risk factor.
C. Smoking is not a modifiable cardiac risk factor.
D. Smoking appears to relieve symptoms associated with schizophrenia.
Correct Answer: D.
Smoking appears to relieve symptoms associated with schizophrenia.
Smoking cessation is important to the health of schizophrenic patients. It is unclear whether increased risk is attributable to an innate aspect of the schizophrenia or to lifestyle, poor health care in this particular population, or antipsychotic usage or other drug therapy. As many as 70-80% of all schizophrenic patients smoke. Research shows no association between smoking and antipsychotic use after controlling for schizophrenia. It is theorized that nicotine increases the release of dopamine and corrects dopamine deficiency in the prefrontal cortex, relieving negative symptoms.
Incorrect Answers:
A. 70-80%, not 95%, of schizophrenia patients smoke
B. Most people gain weight after quitting smoking; smoking doesn’t cause weight gain
C. Smoking is a modifiable cardiac risk factor; it’s a major risk factor for heart disease
Vital Concept: It is theorized that nicotine increases the release of dopamine and corrects dopamine deficiency in the prefrontal cortex, relieving negative symptoms. In other words, smoking appears to relieve negative symptoms associated with schizophrenia.
References:
Maslow’s hierarchy of needs can allow a nurse practitioner to prioritize the patient’s care needs to optimize outcomes. In Maslow’s theory, which of the following is prioritized as the “highest level of human needs”?
A. Esteem and self-esteem
B. Self-actualization
C. Love and belonging
D. Food and water
Correct Answer: B.
Self-actualization
The hierarchy of needs proposed by Abraham Maslow posits five levels, each of which must be substantially met before a person can address “higher” levels of need. The first, or basic, level of Maslow’s hierarchy of needs includes the basic elements of survival, like air, food, and water. The second level includes safety; the third level includes love and belonging. The fourth level includes esteem, including self-esteem, confidence, achievement, respect of others, and respect by others. The fifth and final level is self-actualization, which refers to the acceptance of facts, lack of prejudice, spontaneity, morality, creativity, and problem-solving.
Incorrect Answers:
A. This is level 4.
C. This is level 3.
D. This is level 1.
Vital Concepts:
The hierarchy of needs proposed by Abraham Maslow posits five levels, each of which must be substantially met before a person can address “higher” levels of need. The fifth and final level is self-actualization, which refers to the acceptance of facts, lack of prejudice, spontaneity, morality, creativity, and problem-solving.
References:
During cognitive behavior therapy, a patient states that “I was arguing with my daughter last night, and she went to her room and slammed the door. Everybody hates me, and I’m a terrible parent.” Which cognitive distortion is she displaying?
A. Selective abstraction
B. Arbitrary inference
C. Overgeneralization
D. Magnification
Correct Answer: C.
Overgeneralization
Overgeneralization involves coming to generalized conclusions based on a single experience.
Incorrect Answers:
A. Selective abstraction involves taking a small detail out of context and using it to make an entire experience negative.
B. Arbitrary inference involves coming to an incorrect conclusion based on a previous experience.
D. Magnification and minimization involve making certain events more or less significant based on a negative interpretation.
Vital Concept:
Overgeneralization can frequently affect individuals with depression or anxiety disorders. Patient will apply past experience to all future ones. For example, once a patient gives a poor speech, they think “I always mess up speeches.”
References:
A child psychiatrist cares for a 10-year-old child who is accompanied by her single father. The child’s mother passed away due to complications of childbirth. The psychiatrist and the patient’s father communicate regularly about his daughter and soon become close friends. Eventually, their relationship becomes more than a friendship, and they begin dating. How would the APA regard this relationship?
A. Ethical, but care should be taken not to breach confidentiality
B. Unethical
C. Ethical, but only if the person the physician is dating is not married
D. Ethical, but only if the father signs a non-disclosure agreement
Correct Answer: B.
Unethical
Dating or any romantic relationship between a psychiatrist and any “key third party” (which includes a parent, guardian, spouse, significant other, or surrogate) is considered unethical by the APA. Relationships of this nature between a practitioner and a person in a decision-making capacity for the patient is a clear conflict and puts the therapeutic relationship between the patient and physician at risk.
Incorrect Answers:
A, C, and D. There’s no way to make this ethical
References:
A 4-year-old child is brought to the emergency department for evaluation of wrist pain. The parent states the child is clumsy and becomes hostile when questioned further. Physical examination shows bruises over the buttocks. Which of the following is the appropriate action?
A. Discuss the suspicion with the patient’s mother
B. Call the patient’s grandparents and discuss the suspicion
C. Report the situation to Child Protective Services
D. Make a follow-up appointment for the following week
Correct Answer: C.
Report the situation to Child Protective Services
Physicians are mandatory reporters of child abuse. They are legally and ethically bound to report suspected abuse to Child Protective Services. Risk factors for abuse include young or single parents, parents with lower levels of education and unrealistic expectations for the child, or poor knowledge of child development. Red flags include no history or denial of trauma despite severe injury, implausible history for degree or type of injury, unexplained or excessive delay in seeking care, and caregiver histories that change with retelling or conflict with versions from other observers.
Any injury which is inconsistent with the history given by a caregiver should prompt suspicion of child abuse. The child should be examined thoroughly for evidence of other injuries, and a complete skeletal X-ray survey should be obtained. Other findings that should raise concern for child abuse include bruises on the buttocks, genitalia, or elsewhere on the body, lash marks, and burns from cigarettes or immersion in hot water. A retinal examination may reveal hemorrhages. X-ray studies may show spiral fractures of the long bones, subdural hematomas, and multiple bony injuries at different stages of healing. Medical providers are required by law to report cases of suspected child abuse to state authorities. If suspicion is high enough, a child may be admitted to the hospital for protection until an investigation is complete.
Incorrect Answers:
A. While discussing the concerns with the child’s mother is appropriate, calling CPS is required if there is a reasonable suspicion of abuse.
B. If there is any suspicion of child abuse, reporting to CPS is required regardless of the history provided by the caregiver or anyone involved in the child’s care.
D. This patient may be unsafe at home. Delaying evaluation by Child Protective Services increases the risk of further abuse at home and would be inappropriate.
Vital Concept:
Physicians are mandatory reporters of child abuse and are legally and ethically bound to report suspicion of abuse to Child Protective Services.
References:
What is the primary, direct cause of neuron loss in an ischemic stroke?
A. Programmed cell death
B. Interstitial fluid accumulation
C. Excessive increases in excitatory neurotransmitters
D. Interrupted blood flow
Correct Answer: C.
Excessive increases in excitatory neurotransmitters
Excitatory neurotransmitters such as glutamate control the growth of neurons and their connections via their effects on calcium flow through NMDA receptor channels. Too much calcium can arrest growth and eventually lead to the destruction of neuronal processes. High levels of glutamate produce excitotoxicity.
Incorrect Answers:
A. Programmed cell death is incorrect. In the event neuron loss in an ischemic stroke there is no programmed cell death. It involves sustained calcium entry through NMDA receptor channels activates intracellular proteases and possibly generates free radicals, ultimately leading to neuron death.
B. Interstitial fluid accumulation is incorrect. Excitotoxicity appears to play a prominent role in strokes, status epilepticus, hypoglycemia, and head trauma; these brain injuries cause neuronal depolarization, which leads to excessive electrical activity and excessive increases in glutamate release. Excitotoxicity also appears to play a role in neurodegenerative disorders like Huntington’s disease.
D. Interrupted blood flow is incorrect as it is due to the calcium flow through the NMDA receptor channels and can be desctructive on the neuronal process.
References:
A 45-year-old male is brought to ER by his son in an agitated and disoriented state after tonic-clonic convulsions. His blood pressure is 170/105, pulse is 125/min, RR is 15/min, and temperature is 99°F. Which of the following is the most likely diagnosis?
A. Alcohol intoxication
B. Cocaine intoxication
C. Cannabis intoxication
D. Alcohol withdrawal
Correct Answer: D.
Alcohol withdrawal
Alcohol withdrawal typically begins 6–8 hours after the last drink, peaks 24–28 hours after the last drink, and generally resolves within 7 days. Alcohol hallucinosis occurs in 3–10% of patients with severe alcohol withdrawal. It can present as auditory, visual, or tactile hallucinations in the presence of a clear sensorium. Delirium tremens (DT) is characterized by agitation and tremulousness, autonomic instability, fevers, auditory and visual hallucinations, and disorientation. DT usually develops 2–4 days from the person’s last drink, and the average duration is <1 week. Seizures (grand mal), another complication of alcohol withdrawal, are estimated to occur in 5–15% of patients. They usually occur in the first 24 hours from the last drink, but they can occur at any time in the first 5 days.
Incorrect Answers:
A. Alcohol intoxication is dependent on the individual’s tolerance, the amount and type of alcoholic beverage ingested, and the amount absorbed. Alcohol intoxication presents with impaired recent memory, passivity, lack of concern, and disinhibition.
B. Cocaine intoxication can resemble a manic episode with euphoria, impulsive behavior, aggression, dystonias, heightened self-esteem, mydriasis, increased vitals, hyperthermia, or cardiac conduction abnormalities.
C. Cannabis intoxication involves behavioral changes like euphoria, anxiety, isolation, impaired cognition, and somatic symptoms of dry mouth, tachycardia, and red eyes.
References:
Due to the restructuring of health care delivery and shift in the provision of nonacute care from the hospital to ambulatory care settings and the home, previous nursing research that was conducted in hospitals has changed in nature. What is now true of nursing research that was previously conducted in hospitals?
A. Now becomes applicable to all in hospital providers of care.
B. Remains relevant in the new settings, client care is unchanged.
C. Should be applied directly to client care provided in the new settings.
D. May no longer be applicable to the delivery of client care.
Correct Answer: D.
May no longer be applicable to the delivery of client care.
Such a drastic change in health care delivery may make previous studies no longer applicable.
Incorrect Answers:
A, B, and C. Health care restructuring has changed the way client care is provided. These options are not applicable.
Vital Concepts:
Such a drastic change in health care delivery may make previous studies no longer applicable.
References:
Autism spectrum disorders (ASD) can be identified in a certain percentage of the population. Which of the following is true about ASDs?
A. Medical conditions that cause ASD can be identified in <5% of cases.
B. ASD is a homogeneous disorder and due to only a few gene mutations.
C. Most cases of ASD are due to mitochondrial dysfunction.
D. FMR2 gene associated with fragile X syndrome is also associated with ASD.
Correct Answer: A.
Medical conditions that cause ASD can be identified in <5% of cases.
Medical conditions that cause ASD can be identified in <5% of cases. Medical conditions that are best established as probable causes of ASD include Rett syndrome, fragile X syndrome, tuberous sclerosis, and abnormalities of chromosome 15 involving the 15q11-13 region.
Incorrect Answers:
B. ASD is a heterogenous disorder and may be due to multiple gene changes.
C. Up to 5% (or possibly more) of ASD may be due to mitochondrial dysfunction. This is 500x higher than the general population (0.01%). A growing body of clinical, genetic, and biochemical evidence now suggests that ASD, or at least a subset of ASDs, may also be linked to impaired mitochondrial function.
D. Fragile X syndrome and tuberous sclerosis are genetic syndromes that have a high rate of comorbidity with autism spectrum disorder (ASD). The FMR1 gene associated with fragile X syndrome is associated with ASD.
References:
A recent study analyzed monozygotic and dizygotic twin pairs with at least one sibling with autism spectrum disorder (ASD) to evaluate the heritability of autism. Which of the following terms indicates the presence of the same trait in both members of a pair of twins?
A. Genetic variance
B. Cohort
C. Case-control
D. Concordance
Correct Answer: D.
Concordance
Concordance, as used in genetics, usually means the presence of the same trait in both members of a pair of twins. Twin studies can be performed with monozygotic and dizygotic twins to look for the heritability of a certain condition. They are often used in behavior genetics and include genetically informative data (e.g., pedigree, adoption, and sibling data). The studies evaluate the relative contributions of the environment and genetics on an outcome or attribute.
Twin studies are frequently performed by comparing the characteristics of identical (monozygotic) twins who were raised in different households since they share a genotype but not an environment. Comparison studies of identical twins raised together and fraternal twins raised together can also provide important information since each twin set shares the same environment, but only the identical twins have the same genotype. Fraternal twins share only half of the same genetic makeup.
Incorrect Answers:
A. Genetic variance describes the difference in an individual’s phenotype compared to a norm due to the inheritance of a specific allele.
B. Cohort study design compares groups with exposure or risk factor to groups without that exposure or risk factor. Cohorts utilize relative risk measures.
C. Case-control. Case-control study design compares people with and without disease. Case-controls utilize odds ratio measures.
Vital Concept:
Concordance, as used in genetics, usually means the presence of the same trait in both members of a pair of twins.
References:
Lithium-induced hypothyroidism is a condition that generally occurs within the first 6-18 months of lithium treatment. Which of the following statements about it is true?
A. It occurs in over 50% of patients treated with lithium
B. It occurs more commonly in men.
C. It is a contraindication to lithium treatment
D. It is associated with higher rates of depression and rapid cycling
Correct Answer: D.
It is associated with higher rates of depression and rapid cycling
After lithium-induced hypothyroidism develops, it’s associated with rapid cycling and an increased risk of developing depression
Incorrect Answers:
A. Lithium-induced hypothyroidism occurs in 5-35% of patients treated with lithium
B. Lithium-induced hypothyroidism occurs in women more often than in men
C. Lithium-induced hypothyroidism isn’t a contraindication to lithium treatment, and it can be treated with levothyroxine
References:
Posttraumatic stress disorder (PTSD) and acute stress disorder (ASD) are both reactions to stressful events. However, they are not the same disorder and need to be carefully differentiated in diagnoses. Which of the following is true about the differences between these disorders?
A. Dissociative symptoms are key to a diagnosis of PTSD.
B. Initial screening for recent or distant event exposure is necessary for diagnosis only for PTSD.
C. Re-experiencing symptoms, avoidance/numbing, and dissociative symptoms are three major clusters in clinical evaluation for ASD and PTSD.
D. Symptoms present for at least 30 days are consistent with PTSD rather than ASD.
Correct Answer: D.
Symptoms present for at least 30 days are consistent with PTSD rather than ASD.
The symptoms of ASD must persist for 3 or more days but not more than 30. By contrast, the diagnostic criteria for PTSD specify that symptoms must last a minimum of 30 days.
Incorrect Answers:
A. Dissociative symptoms may be associated with either PTSD or ASD.
B. Screening for an exposure event is a necessary first step in both ASD and PTSD.
C. The three major symptom clusters in the DSM-4 were avoidance/numbing, hyperarousal, and re-experiencing. The DSM-5 included four clusters: re-experiencing, avoidance, negative cognitions and mood, and arousal. The DSM-5-TR lists: affective/mood, intrusive, dissociative, arousal, and avoidance.
Vital Concept:
In ASD, symptoms are present for 3 or more days and may last up to a month. In PTSD, symptoms are present for at least 30 days after a traumatic experience.
References:
A nurse practitioner is reviewing a patient’s chart before performing an examination. The previous provider noted that the patient had a normal Rinne test. Which of the following cranial nerves did the provider assess?
A. CN V
B. CN VI
C. CN VII
D. CN VIII
Correct Answer: D.
CN VIII
The Rinne test evaluates conductive hearing loss. The test is performed by striking a tuning fork of 256 or 512 Hz and placing it on the mastoid process. A normal test will show air conduction time that is twice as long as bone conduction time. if there is hearing loss bone conduction is longer than air conduction. CN VIII is the auditory nerve.
Incorrect Answers:
A. CN V. The trigeminal nerve (CN V) is responsible for facial movement and sensation. To test CN V check pinprick sensation of bilateral face and have patient open jaw against resistant.
B. CN VI. The abducens nerve (CN VI) is responsible for movement of the lateral rectus eye muscle. To test CN VI test extraocular eye movements.
C. CN VII. The facial nerve (CN VII) is responsible for muscles of facial expression and taste. To test CN VII ask the patient to smile and look for any hemifacial weakness.
Vital Concepts:
CN VIII is the auditory nerve.
References:
A pediatrician provides a CRAFFT test to screen an adolescent for high-risk alcohol and drug disorders. Which kind of prevention does this represent?
A. Primary
B. Secondary
C. Tertiary
D. None of the above
Correct Answer: B.
Secondary
The CRAFFT test is a short, self-administered behavioral-health tool developed to screen adolescents for high-risk alcohol and other drug-use disorders simultaneously. Secondary prevention detects diseases early when patients are asymptomatic and address treatment to halt its progression. Examples include HIV testing, screening for nutrition and exercise, depression screening for at-risk teens, urine drug testing, or identifying prodromal schizophrenia.
Incorrect Answers:
A. Primary prevention keeps a disease from occurring by removing its causes. Primary prevention includes counseling about lifestyle changes like drinking and smoking, immunizations, and suicide prevention programs.
C. Tertiary prevention is defined as activities that prevent deterioration or reduce complications after a disease is already present. Examples include metabolic monitoring of patients on antipsychotics, assertive community treatment for people with schizophrenia, and intensive case management.
D. CRAFFT screening is a form of secondary prevention.
References:
A 44-year-old patient is admitted to the hospital with acute mania. The patient has a long history of alcohol use, with increasing use during this episode. Which of the following is true regarding bipolar disorder and alcohol use disorder?
A. The rate of alcohol use disorder is significantly higher amongst patients with bipolar disorder
B. The rate of alcohol use disorder is equivalent amongst patients with bipolar disorder
C. Alcohol use disorder is less common among patients with bipolar disorder
D. As it is sedating, alcohol use tends to improve the prognosis regarding a patient’s bipolar disorder
Correct Answer: A.
The rate of alcohol use disorder is significantly higher amongst patients with bipolar disorder
The prevalence of alcohol use disorder (AUD) in American bipolar patients is 40-60% compared to the 15% incidence in the US general population. Recommendations about treatment should be advised by an addiction specialist. Liver function tests should be monitored, as they are commonly elevated in patients with AUD. In patients with increased liver enzymes, valproate should be used cautiously. Lithium levels can reach toxic levels in patients who become dehydrated due to alcohol use.
Incorrect Answers:
B. The rate of AUD in patients with bipolar disorder is more than twice the general population.
C. The rate of AUD in patients with bipolar disorder is more than twice the general population.
D. Alcohol use has been shown to increase the number of mood episodes, mood symptoms, suicide attempts, hospitalizations, and otherwise worsen the prognosis of bipolar disorder in general.
Vital Concept:
The relationship between AUD and bipolar disorder is mutually destructive. The rate of AUD amongst bipolar patients is 40-60%, more than twice the general population.
References:
A patient tells her long-time healthcare provider that she recently signed a new will and instructed her estate to deliver a valuable coin collection to the healthcare provider after her death. Which of the following statements is correct?
A. Since the proposed gift is not monetary, the healthcare provider may express gratitude.
B. The healthcare provider must decline this valuable gift.
C. The healthcare provider can accept the gift but should provide this patient with free medical care as compensation.
D. The healthcare provider should not accept any gift of value from a patient due to the risk of noncompliance with tax laws.
Correct Answer: B.
The healthcare provider must decline this valuable gift.
A healthcare provider should consider the potential implications of accepting a gift in light of the provider-patient relationship, including any patient expectations of the healthcare provider as a result of the gift. To demonstrate favorable treatment to a patient due to a patient gift may impair the healthcare provider’s objectivity when caring for the patient and can undermine the professionalism expected of the provider. In general, a healthcare provider ethically can accept a small gift as a token of appreciation, but acceptance of a valuable gift is problematic. The promise of a bequest may cause the healthcare provider consciously or unconsciously to alter treatment decisions in the care of a patient and may tarnish the healthcare provider’s reputation by the suggestion of impropriety.
Incorrect Answers:
A. The proposed gift has a large monetary value, so the provider can’t accept it
C. This would be unethical
D. Tax law doesn’t carry risk in this case
Vital Concepts:
A healthcare provider should consider the potential implications of accepting a gift in light of the provider-patient relationship, including any patient expectations of the healthcare provider as a result of the gift.
References:
Alcoholics Anonymous (AA) is a well-established and popular alcohol cessation program that helps alcoholics achieve and maintain sobriety. Which of the following does it emphasize?
A. Enhancement of patient motivation
B. Development of patient capabilities
C. Generation of patient capabilities
D. Development of spiritual beliefs
Correct Answer: D.
Development of spiritual beliefs
Alcoholics Anonymous emphasizes the development of spirituality.
Incorrect Answers:
A. A function of dialectical behavioral therapy (DBT) is to enhance patient motivation for change.
B. A function of DBT is to develop new patient capabilities.
C. A function of DBT is to promote the generation of patient capabilities.
References:
A 45-year-old man presents to the emergency room for a “panic attack.” The patient is loud and argumentative with staff. He states that he was prescribed Xanax at 0.5mg at 3x daily by an outpatient psychiatrist but ran out this morning. He states that he has been experiencing worsening anxiety symptoms throughout the evening, prompting this visit. Because it is after hours, a message was left with his psychiatrist’s answering service.
The patient’s vital signs are significant for an elevated heart rate at 114bpm, an elevated blood pressure at 155/95 mmHg, and a low-grade temperature of 99.1°F. The patient is observed until morning, and his vital signs are closely monitored. He received a small dose of Ativan with resolution of his symptoms, was able to sleep, and did not require any additional doses of Ativan during the 6 hours he was observed. The patient remained fully alert and oriented and denied any disturbance in mood or suicidal ideation. The on-call psychiatrist who is covering the patient’s primary outpatient psychiatrist calls the ED to confirm the patient’s history and explains that the last prescription was filled a little more than 3 weeks ago. He is out of town and left his Xanax at home.
The patient has a follow-up appointment scheduled with his psychiatrist next week, but the on-call psychiatrist agrees to coordinate an appointment before the end of this week. A reasonable treatment plan for this patient includes which of the following?
A. Admit to detoxification unit
B. Admit to inpatient psychiatry unit
C. Discharge with Xanax refill
D. Discharge with enough Ativan to control symptoms until follow-up
Correct Answer: D.
Discharge with enough Ativan to control symptoms until follow-up
The physician’s response during the withdrawal phase is critical to achieving a satisfactory resolution of the physical dependency. Increasing the benzodiazepine dosage will reduce symptoms because benzodiazepines have not completely lost effectiveness and because the withdrawal syndrome is reversed; however, symptom reduction will not be as complete as during the initial therapeutic phase. The duration of the resolution phase is highly variable. Most patients will have symptom rebound lasting only a few weeks; others will have a severe, protracted abstinence syndrome lasting months to >1 year.
When withdrawing patients from benzodiazepines, use decreasing dosages of the agent of dependence or substitute a longer-acting agent, then gradually withdraw the substitute medication. Although withdrawal from high dosages of sedative-hypnotics should generally be done in a hospital, the realities of managed care often mean that many patients must be treated in part, if not exclusively, as outpatients. For patients who are withdrawing from therapeutic doses of benzodiazepines, a slow outpatient taper is a reasonable strategy and should be continued on an outpatient basis as long as the patient can tolerate withdrawal symptoms. The decision to initiate a taper or to continue treatment with benzodiazepines should be agreed upon by the patient and the primary treatment provider. In this situation, the patient’s symptoms of withdrawal are well controlled with relatively little agonist therapy; he is also actively in treatment and has close follow-up. Discharging the patient with enough benzodiazepines to manage any re-emerging symptoms before his next appointment would be a reasonable treatment plan.
Incorrect Answers:
A. Admit to detoxification unit. A common response is to declare the patient “addicted to benzos” and to refer him or her for chemical-dependency treatment; however, chemical dependency treatment is not appropriate unless the patient has a substance use disorder. Some physicians interpret escalating symptoms as evidence of the patient’s “need” for benzodiazepine treatment and reinstitute higher dosages or switch to another benzodiazepine. Discharging the patient without any benzodiazepines could be dangerous if withdrawal progresses without intervention over the next few days.
B. Admit to inpatient psychiatry unit. The patient is not a danger to themselves or others. Nor is hospitalization expected to prevent decompensation. Therefore, an admission to an inpatient unit is not appropriate.
C. Discharge with Xanax refill. The patient has follow up with their primary provider within one week and therefore a month supply of Xanax is unnecessary. He only needs a bridge of benzodiazepines until next appointment.
References:
A couple presents to the pediatrician’s office with their adopted infant for an initial visit. The baby spent the first 7 months of his life in an orphanage, and the parents are concerned the orphanage neglected him and the other children who lived there. Which of the following is an early sign of psychosocial deprivation in infants?
A. Motor delay
B. Night terrors
C. Hyperorality
D. Limb contractures
Correct Answer: A.
Motor delay
Infants deprived of affection, particularly in the first 6 months of life, are vulnerable to short-term and long-term complications, including language delay, delayed gross and fine motor development, learning disabilities, attachment disorders, and other psychiatric disorders. The length of time spent in a psychosocially deprived environment also directly affects outcomes, with worsening outcomes in children exposed to these environments for longer periods. The remaining answer choices are not as relevant as motor delay in formerly institutionalized infants.
Incorrect Answers:
B. Risk factors for night terrors include sleep deprivation, medication, stress, and intrinsic sleep disorders.
C. Hyperorality is associated with frontotemporal dementia.
D. Limb contractures are associated with cerebral palsy and muscular dystrophy in children.
Vital Concept:
Psychosocial deprivation in infants, especially in the first 6 months of life, can lead to delayed milestones in many areas of development, as well as long-term psychiatric morbidity.
References:
A number of risk factors raise the risk of suicide in children and adolescents. Which of the following is one of those risk factors?
A. Low IQ
B. Female gender
C. Heterosexual sexual orientation
D. Parental psychiatric disorder
Correct Answer: D.
Parental psychiatric disorder
History of psychiatric disorder in a parent is a risk factor for suicide attempt. The other responses are not risk factors for suicide.
Incorrect Answers:
A. Low IQ isn’t a risk factor for suicide
B. Male, rather than female, gender is a risk factor for suicide
C. Non-heterosexual sexual orientation, not heterosexual sexual orientation, is a risk factor for suicide
Vital Concept:
Risk factors include prior suicide attempts, age >16, male gender, and gay/lesbian/bisexual individuals.
References:
A 36-year-old female patient with bipolar disorder type I has been well-controlled on a stable dose of lithium for several years. In the past, she took valproic acid (gaining 40lbs) and lamotrigine, but neither worked as successfully as lithium in controlling both her depressive and manic episodes. The patient also has a history of atrial fibrillation and warfarin. In an effort to lose weight, she has started a weight loss regimen, including taking daily walks and replacing her breakfast with a health shake which contains her daily vegetable requirement and numerous health supplements. In addition to a 5lb weight loss over the past 2 weeks, she notes a decreased need for sleep and increased energy. The patient also purchased exercise equipment for her 2-bedroom apartment. Which of the following supplements may account for the change in her behavior?
A. Ginkgo biloba
B. Echinacea purpurea
C. Allium sativum
D. Ginseng
Correct Answer: D.
Ginseng
Ginseng is thought to have a large number of positive effects as well as the general ability to improve mental and physical performance. Clinical trials, however, have shown few significant and no conclusive benefits in humans. Although the mechanism has not been fully described, ginseng has been reported to interact with certain psychiatric medications (including phenelzine, lithium, and neuroleptics), causing irritability, insomnia, and mania.
Incorrect Answers:
A. Ginkgo biloba has several perceived effects on blood flow and cognitive enhancement, although studies in humans have been equivocal. It should be avoided with antiplatelet and anticoagulant medications. This patient is taking warfarin for atrial fibrillation and should avoid using ginkgo.
B. Echinacea purpurea is thought to have anti-inflammatory and immune-enhancing properties. Echinacea may cause a flu-like syndrome but has no known drug interactions. It is generally recommended to avoid echinacea in immune-related conditions such as post-transplant immune suppression and autoimmune and immune deficiency conditions (e.g. AIDS, cancer).
C. Allium sativum is garlic, which has mild cholesterol-lowering characteristics. Garlic has reported antiplatelet effects and should be used cautiously with antiplatelet and anti-clotting medications. While garlic should be avoided in patients on warfarin, it is not used as a cognitive enhancer.
References:
Which of the following statements is true regarding the heritable risk of suicidal behavior?
A. Relatives of suicide completers, not suicide attempters, have a heritable risk of both suicide attempts and completions.
B. Relatives of suicide completers and suicide attempters have a heritable risk of suicide attempts and completions.
C. Relatives of suicide completers and suicide attempters have a heritable risk of suicide attempts, not completions.
D. Relatives of suicide completers or suicide attempters do not have an increased risk of suicidal behavior compared to relatives of control subjects.
Correct Answer: B.
Relatives of suicide completers and suicide attempters have a heritable risk of suicide attempts and completions.
Considerable evidence suggests that genetic factors play a role in suicide and impulsive behavior. Many family studies have established familial clustering of both suicide attempts and completions. The relatives of both suicide completers and suicide attempters have an increased risk of suicide attempts and completions. Some studies have marked this risk as high as 4x in the relatives of suicide probands as compared to the relatives of control subjects in the community. Twin studies have estimated the heritability of reporting any suicidal ideation, suicide plans, or attempts to be about 44% and that of reporting serious suicide attempts to be 55%. Adoption studies further support the hypothesis that genetic factors contribute to suicide.
Incorrect Answers:
(A) Relatives of suicide completers, not suicide attempters, have a heritable risk of both suicide attempts and completions. Relatives of suicide completers AND suicide attempters have a heritable risk of suicide attempts and completions.
(C) Relatives of suicide completers and suicide attempters have a heritable risk of suicide attempts, not completions. Relatives of suicide completers and suicide attempters have a heritable risk of suicide attempts AND completions.
(D) Relatives of suicide completers or suicide attempters do not have an increased risk of suicidal behavior compared to relatives of control subjects. Relatives of suicide completers and suicide attempters have a heritable risk of suicide attempts and completions.
References:
How does botulinum toxin improve depression?
A. Reducing frown lines when injected into glabellar region
B. Enhancing smiling when injected into lips
C. Enhancing endorphin release when injected into buccal branch of facial nerve
D. Creating hyperactivity in left amygdala and promoting a sense of calmness and wellbeing when injected into facial nerve
Correct Answer: A.
Reducing frown lines when injected into glabellar region
Botulinum toxin injected into the glabellar region of frowning patients improved depressive symptoms, according to the Hamilton Depression Rating Scale 21, the Beck Depression Inventory, and the Patient Health Questionnaire-9. When the face cannot frown or is not able to frown as deeply, there is less activation in the left amygdala when the patient tries to mimic angry facial expressions. According to the facial-feedback hypothesis, frowning can actually make patients unhappy, and smiling can make them happy. Reducing the ability to frown creates changes in neural circuitry beyond the cosmetic and esthetic effects of botulinum toxin.
Incorrect Answers:
B. No correlation between enhancing lips and depression symptoms.
C. Botulinum toxin is not injected into the facial nerve.
D. When the face cannot frown or is not able to frown as deeply, there is LESS activation in the left amygdala when the patient tries to mimic angry facial expressions.
Vital Concept:
Botulinum toxin injected into the glabellar region of frowning patients improved depressive symptoms.
References:
Medicare Part B covers some medical services and equipment. Which of the following services is covered by Medicare Part B?
A. Eye exams and eyeglasses
B. Routine dental care
C. Prescription medications
D. Durable medical equipment and home health care
Correct Answer: D.
Durable medical equipment and home health care
Medicare Part B provides coverage for durable medical equipment. In general, Medicare Part B provides both medically necessary services/supplies (including DME) and preventative services for outpatient needs. This includes primary care appointments, preventative care, outpatient therapy, and outpatient mental health services. It does not cover inpatient services (hospital charges), medications, eye care, or dental care.
Incorrect Answers:
A. Medicare part B does not provide coverage for eye exams or eyeglasses
B. Medicare part B does not provide coverage for routine dental care
C. Medicare part D (not part B) covers prescription medications
Vital Concept:
Medicare Part B provides coverage for durable medical equipment and home health care, as both are medically necessary or preventative outpatient services/supplies.
References:
A patient has a history of drug abuse. A local law-enforcement officer arrives with proper identification and states that the police department is conducting an investigation of a possible narcotic diversion by this patient. He asks to see the patient’s medical record. What is the correct response?
A. Only turn over the patient’s medical records if you have prescribed narcotics.
B. If you suspect the patient is diverting narcotics but not using them personally, you have no obligation to give law enforcement access to the records.
C. You are legally obligated to provide access to the patient’s medical records to law-enforcement agents conducting investigations.
D. You are obligated to provide access to confidential patient information requested by law enforcement on presentation of a search warrant.
Correct Answer: D.
You are obligated to provide access to confidential patient information requested by law enforcement on presentation of a search warrant.
A healthcare provider is obligated to protect confidential patient information unless furnished with a search warrant, subpoena, or court order or if sharing the information is necessary to prevent harm to others. If a law-enforcement agent, government agent, or employer requests confidential patient information, you must refuse access under other conditions. All patient information (including genetic information, immigration status, and patient history) should be protected from third parties unless the patient authorizes their release.
Incorrect Answers:
A. B. and C. Law enforcement needs a warrant to compel release of patient records.
Vital Concepts:
If a law-enforcement agent, government agent, or employer requests confidential patient information, you must refuse access under other conditions. All patient information (including genetic information, immigration status, and patient history) should be protected from third parties unless the patient authorizes their release.
References:
Transdermal selegiline is absolutely contraindicated for certain types of patients. Which of the following types of patients can transdermal selegiline absolutely not be used in?
A. For patients who have failed multiple trials with different classes of antidepressants
B. For patients with prominent fatigue
C. For patients with cognitive deficits
D. For patients with pheochromocytoma
Correct Answer: D.
For patients with pheochromocytoma
Absolute contraindications for selegiline use include a pheochromocytoma (due to excess norepinephrine leading to a hypertensive crisis) and surgery (planned discontinuation of the patch 10 days prior to elective surgery due to the risk of severe hypotension after general anesthesia is recommended). Monoamine oxidase (MAO) is responsible for the catabolism of serotonin, norepinephrine, and dopamine. Selegiline is an irreversible MAO inhibitor and works by selectively inhibiting MAO-A and MAO-B in the CNS with preservation of the MAO-A in the gut. Its use presents little need for dietary tyramine restriction. However, the lack of clinical, peripheral tyramine effects has only been shown at doses of 6mg/24hr. Higher doses must follow a tyramine-restricted diet. The other situations are good indications for using selegiline to provide additional benefit.
Incorrect Answers:
A, B, and C. These are all good indications for using selegiline to provide additional benefit
References:
What is the strongest established risk factor for bipolar disorder?
A. Pregnancy and obstetrical complications
B. Season of birth
C. Stressful life events
D. A first-degree relative with bipolar disorder
Correct Answer: D.
A first-degree relative with bipolar disorder
There is an average 10% risk of bipolar disorder in adults who have a relative with bipolar disorder. This risk increases with the degree of kinship. Bipolar disorder is also more common in high-income countries compared to low-income. There are higher rates of bipolar I disorder in those who are widowed/separated/divorced compared to those who have never been married or are currently married. Females with bipolar disorder have an increased risk of rapid cycling and mixed states than males.
Incorrect Answers:
(A) Pregnancy and obstetrical complications are putative environmental risks that have been described for bipolar disorder.
(B) Season of birth is a putative environmental risk that has been described for bipolar disorder.
(C) Stressful life events are putative environmental risks that have been described for bipolar disorder.
Vital Concept:
The genetic link for bipolar disorder is significant, so the most established risk factor for the condition is the presence of a first-degree relative with bipolar disorder.
References:
When attempting to successfully exert influence in areas where decisions are made on future health care policies, it is important for a nurse practitioner to utilize sources of power. Which of the following describes the type of power derived from a license granted by the State Board of Nursing?
A. Expert power
B. Legitimate power
C. Power of numbers
D. Reward power
Correct Answer: B.
Legitimate power
Power refers to the potential to exert influence and it is important of human interaction. French and Raven described five categories of interpersonal power. Legitimate power is the power bestowed by the particular status or role of an individual. Licensed nurses and nurse practitioners have legitimate power through their license to practice nursing, granted by the State Board of Nursing.
Incorrect Answers:
A. Expert power refers to power derived from skill or knowledge that others lack.
C. The power of numbers refers to power derived from being part of a group with similar goals.
D. Reward power refers to the ability to give others what they want and to ask them to do things in exchange. Rewards can also be withheld as punishment.
Vital Concepts:
Power refers to the potential to exert influence and it is important of human interaction. French and Raven described five categories of interpersonal power. Legitimate power is the power bestowed by the particular status or role of an individual. Licensed nurses and nurse practitioners have legitimate power through their license to practice nursing, granted by the State Board of Nursing.
References:
Whenever 34-year-old Margaret has to give a presentation in front of a group, she becomes very nervous. She starts sweating profusely, her legs feel like “spaghetti,” and she often has immense trouble remembering what she wanted to say. The patient states that these events are infrequent. She is otherwise without anxiety symptoms and does not want to take something that would make her feel “loopy.” Vital signs are within normal limits. Medical history is significant for past alcohol abuse, from which she has been abstaining for 7 years. What is the most appropriate prescription?
A. Fluoxetine
B. Buspirone
C. Lorazepam
D. Propranolol
Correct Answer: D.
Propranolol
Beta-blockers are effective for acute treatment of performance anxiety. Several small trials have found that beta-blockers reduce performance anxiety. This patient should also administer a test dose prior to the speaking event to assess the drug’s effects.
Incorrect Answers:
A. Fluoxetine. Selective serotonin reuptake inhibitors (SSRIs) are utilized for long-term management of anxiety and require several weeks to take full effect. This patient is looking for immediate relief from performance anxiety.
B. Buspirone. Buspirone is FDA approved to treat anxiety disorders. It is utilized for long-term management of anxiety and requires several weeks to take full effect. This patient is looking for immediate relief from performance anxiety.
C. Lorazepam. Benzodiazepines (e.g. lorazepam) can be used for the acute management of anxiety. However the potential for benzodiazepine abuse is highest in persons with a history of alcohol or other substance use and is a less desirable option in this patient.
References:
Which is the least important barrier to collaborative advanced nursing practice?
A. Prescriptive authority
B. Reimbursement privileges
C. Legal scope of practice
D. Political activism
Correct Answer: D.
Political activism
Political activism is important, but it is not specific to collaborative practice.
Incorrect Answers:
A, B, and C. These are the three major issues that are central to the expansion of the nurse practitioner role.
Vital Concepts:
Political activism is important, but it is not specific to collaborative practice.
References:
Prolactin levels in women with schizophrenia should be taken into account during their treatment. Which of the following statements regarding prolactin and schizophrenia in relation to other medical complications is true?
A. Patients with preexisting osteopenia or osteoporosis with schizophrenia should use antipsychotics that raise prolactin levels.
B. Females with breast cancer with schizophrenia can take an antipsychotic that increases prolactin levels.
C. Women with menstrual or fertility problems with schizophrenia should take prolactin-elevating antipsychotics.
D. Non-prolactin-elevating antipsychotics should be used in women with menstrual and fertility problems diagnosed with schizophrenia.
Correct Answer: D.
Non-prolactin-elevating antipsychotics should be used in women with menstrual and fertility problems diagnosed with schizophrenia.
Clinicians should be certain about the severity of symptoms and whether they contributed to hyperprolactinaemia. Typical antipsychotic agents can cause an increase in prolactin, which can cause bone metabolism issues, menstrual issues, infertility issues, and breast cancer. It is important to obtain a thorough medical history before placing someone with schizophrenia on a typical antipsychotic that can potentially raise prolactin levels. Current antipsychotic therapy can be switched to prolactin-sparing agents like olanzapine, quetiapine, aripiprazole, or clozapine.
Incorrect Answers:
A. Elevated prolactin levels are associated with increased bone demineralization, osteopenia, and osteoporosis. Patients with schizophrenia with concurrent osteopenia and osteoporosis should be prescribed antipsychotics with minimal to no effects on prolactin levels.
B. Women with schizophrenia and breast cancer should avoid antipsychotics with prolactin-elevating effects. Aripiprazole may be a more appropriate antipsychotic.
C. Women with menstrual and fertility problems who are diagnosed with schizophrenia should be evaluated for prolactin secretion abnormalities such as prolactinomas, and antipsychotics that do not elevate prolactin should be used.
References:
Mirtazapine is a noradrenergic and specific serotonergic antidepressant (NaSSA). Which class of antidepressant drugs does it fall into?
A. Tetracyclic antidepressants
B. Tricyclic antidepressants
C. Monoamine oxidase inhibitors
D. Benzodiazepines
Correct Answer: A.
Tetracyclic antidepressants
Mirtazapine is a tetracyclic compound used in the treatment of depression. It is a noradrenergic and specific serotonergic antidepressant (NaSSA). Mirtazapine is unrelated to the tricyclic antidepressants (TCAs). It blocks the presynaptic alpha-2-adrenergic receptor, which results in 5-HT-1A activation and causes an increased release of dopamine.
Incorrect Answers:
B. These drugs increase norepinephrine and serotonin levels. Mirtazapine doesn’t fall into this category as it has a different MOA.
C. These drugs inhibit monoamine oxidase A and monoamine oxidase B activity. Mirtazapine doesn’t fall into this category as it has a different MOA.
D. These drugs’ core chemical structure fuses a benzene ring and a diazepine ring. Mirtazapine doesn’t fall into this category as it has a different structure.
References:
When developing a focused research question related to evidence-based practice change for patients who have diabetes risk factors, which of the following would be something a nurse practitioner should consider? (Select all that apply)
A. Age of the population of interest
B. Identification of risk factors for diabetes
C. Previous interventions
D. Cost of interventions
Correct Answers:
A. Age of the population of interest
B. Identification of risk factors for diabetes
C. Previous interventions
Using the PICO process to frame a focused research question for consideration of evidence-based change in practice, a nurse practitioner should define the characteristics of the population of interest, determining, for example, whether the population is all adults or all persons with risk factors for diabetes. Risk factors (or of a disease of interest) should also be defined. The NP should determine if the area of interest is patients with one specific risk factor or patients with all risk factors. There may be only certain risk factors applicable to the population of patients whose needs are being addressed. The NP will further need to consider the interests of the organization and patient population. Any previous interventions and/or control groups for comparison to a proposed intervention must be identified. Finally, the NP must determine the type of outcome desired. This can vary, and may refer to short-term or long-term outcomes, or to patient outcomes or provider outcomes.
Incorrect Answer:
D. Cost is not a consideration in framing a research question related to a change in evidence-based practice. Although organization priorities may be influenced by cost, this is not considered when framing a question for research into evidence on clinical practice.
Vital Concept:
Use PICO process to frame an evidence-based change in practice by identifying the population/patient/problem, intervention, comparison/control, and the outcome desired.
References:
Adjustment disorders are frequently referred to as “subthreshold” diagnoses. How does this “subthreshold” diagnosis relate to the patient’s overall health?
A. It is less likely to be noticed by a less-experienced practitioner.
B. It is less likely to be noticed by a non-specialist (e.g. the murmur only heard by a cardiologist).
C. It may be the early phase of a more serious disorder.
D. Fewer milestones and studies guide treatment.
Correct Answer: C.
It may be the early phase of a more serious disorder.
This diagnosis may indicate a major mental disorder is brewing. It is important to keep this patient under a watchful eye.
Incorrect Answers:
A. A less experienced practitioner may diagnose an adjustment disorder. There are no studies showing that senior physicians predominantly make adjustment disorder diagnoses
B. There are no studies showing that adjustment diagnoses are less likely to be noticed by a non-specialist.
D. This is not the correct interpretation of a subthreshold diagnosis.
References:
An otherwise healthy adult who has been on unemployment for several months presents with excessive worry over finding a new job. They report fatigue, difficulty concentrating, and increasingly irritable moods. The patient also reports sleep disturbance over the past 6 months. What is the most likely diagnosis?
A. Social phobia
B. Posttraumatic stress disorder
C. Generalized anxiety disorder
D. Panic disorder
Correct Answer: C.
Generalized anxiety disorder
Generalized anxiety disorder is characterized by extraordinary concern on most days for 6 months or more regarding various things (school, work, family). This is accompanied by at least three of the following: feeling tired, agitated, uneasy/fidgety, tense/clenched with poor sleep, and an inability to sustain focus. The individual must find it difficult to control the worry. The lifetime prevalence rate of GAD is 5%, and the course is fluctuating but is worse during stressful periods.
Incorrect Answers:
A. Social anxiety disorder (social phobia) includes discomfort with social events/interactions out of concern for judgment. This patient does not describe concern regarding interacting with others or being externally judged.
B. Posttraumatic stress disorder (PTSD) criteria include symptoms of intrusive and unsettling thoughts, memories, and flashbacks/nightmares related to an index traumatic experience. Symptoms must continue to cause anguish and dysfunction for 30 days or more to qualify. This clinical scenario does not include an index traumatic experience.
D. Panic disorder is characterized by repeated panic attacks, with associated concern for additional attacks in the future. This patient is not describing episodic intense panic but a generalized, consistent concern.
Vital Concept:
This patient is describing pervasive (generalized) anxiety over the past 6 or more months. The most appropriate diagnosis based on this description is generalized anxiety disorder.
References:
Which of the following statements is accurate regarding adolescence?
A. Early adolescents do not need more sleep than average adults.
B. The adolescent prefrontal cortex may not be fully formed, causing errors in tasks involving executive functions.
C. Adolescents have a well-developed capacity for abstract thinking and should be expected to make sound decisions.
D. Limit-testing and risk-taking are not normal behaviors in adolescents.
E. Late adolescents can grasp the full implications of their actions.
Correct Answer: B.
The adolescent prefrontal cortex may not be fully formed, causing errors in tasks involving executive functions.
An adolescent’s prefrontal cortex may not be fully formed. Errors in tasks involving executive functions may occur. Executive functioning tasks include controlling impulses, prioritizing, and planning. The prefrontal cortex is fully developed by the mid-20s.
Incorrect Answers:
A. Early adolescents need more sleep than average adults (average 9.5 hours a night) and also tend to wake later in the morning and go to bed later at night. Increased sleep is needed for teenagers, although many do not get it due to school and activity demands. Sleep-phase shifting also is common, creating a challenge if the teen attends a school with an early start time (e.g., before 8:00).
C. Although adolescents have the capacity for abstract thinking (formal operations), their decision-making may be faulty. They are also readily influenced by their emotions, peers, and a sense of omnipotence.
D. Limit-testing and risk-taking are a part of the adolescent process of developing one’s own boundaries and limitations. External controls such as clear parental and school rules are important supports during this phase.
E. Adolescents have not yet developed the capacity to always understand the consequences of their actions.
Vital Concept:
The adolescent prefrontal cortex may not be fully formed, possibly causing errors in tasks involving executive functions.
References:
A 15-year-old female was recently diagnosed with a terminal melanoma and begins volunteering at a hospital to work with other terminally ill children. Which defense mechanism best describes her actions?
A. Rationalization
B. Intellectualization
C. Denial
D. Altruism
Correct Answer: D.
Altruism
Altruism is the redirection of internal conflict through meeting the needs of others.
Incorrect Answers:
A. Rationalization is the use of false logic to convince oneself that no wrong has been done.
B. Intellectualization is the excessive use of thinking to avoid expressing underlying emotion.
C. Denial is the rejection of a truth.
References:
ADHD may have comorbid disorders. In such cases, which of the following is true about treatment options?
A. When used for ADHD, atomoxetine does not worsen preexisting anxiety disorders.
B. Stimulants are rarely used in children with ADHD and OCD.
C. There are no concerns regarding the use of antidepressants in children with ADHD and depression.
D. Patients with ADHD and active substance use should receive treatment for ADHD, irrespective of stabilization of substance use disorder.
Correct Answer: A.
When used for ADHD, atomoxetine does not worsen preexisting anxiety disorders.
In randomized controlled trials, atomoxetine does not worsen anxiety in patients with existing anxiety disorders or worsen tics related to Tourette’s disorder. Another option is to treat ADHD first, then add an SSRI if needed.
Incorrect Answers:
B. Stimulants are used often in children with ADHD and OCD. Stimulants are often used in conjunction with SSRIs.
C. There is a black box warning regarding the use of antidepressants and increased suicide risk in children.
D. Stabilization of substance use disorder first is preferred. In addition, the use of ADHD medications with less addiction and diversion potential such as atomoxetine and bupropion is suggested.
References:
Minuchin’s theory of family boundaries includes a concept called “enmeshment.” What aspect does this reflect a problem with?
A. Interpersonal boundaries
B. Individual boundaries
C. Generational boundaries
D. Boundary violations
Correct Answer: A.
Interpersonal boundaries
Enmeshment involves a failure to achieve appropriate interpersonal boundaries. Individuals in a family must properly differentiate from each other and achieve sufficient autonomy.
Incorrect Answers:
B. Minuchin refers to interpersonal boundaries, not individual boundaries.
C. Generational boundaries refer to the maintenance of hierarchical structure in the family through generations.
D. Boundary violations refer to the actions of psychotherapists toward a patient.
References:
One nursing model primarily addresses health promotion. Which model is it?
A. Watson’s Human Caring Theory
B. Newman’s Health Care Systems Model
C. Pender’s Health Promotion Model
D. Roy’s Adaptation Model
Correct Answer: C.
Pender’s Health Promotion Model
Pender’s Health Promotion Model is an excellent Nursing model that fits into a nurse practitioner’s scope of care.
Incorrect Answers:
A, B, and D. These models are useful because they are models for professional nursing practice.
According to Watson’s theory, “Nursing is concerned with promoting health, preventing illness, caring for the sick, and restoring health.”
The Neuman Systems Model views the client as an open system that responds to stressors in the environment.
The Roy’s Adaptation Model sees the individual as a set of interrelated systems who strives to maintain balance between various stimuli.
Vital Concepts:
Pender’s Health Promotion Model is an excellent Nursing model that fits into a nurse practitioner’s scope of care.
References:
Structure, process, and outcomes can be measured to evaluate quality in a healthcare organization. Which of the following is true of these measures?
A. Outcome measures provide the most accurate reflection of quality of care by a practitioner
B. There is no relationship between process measures and outcomes
C. Process measures cannot be easily measured or compared
D. Process measures can be based on scientific evidence
Correct Answer: D.
Process measures can be based on scientific evidence
When evaluating quality in a healthcare organization, structure, process, and outcomes can be measured. Most publicly-reported quality indicators are process measures. Outcome is used less frequently to report quality since it is more difficult to measure methodologically. Process measures can be based on scientific evidence and can also be measured and compared with relative ease.
Incorrect Answers:
A. Outcome measures are heavily confounded by patient mix. If sample sizes are small, even poor practitioners may have mortality rates that are not statistically worse than those of excellent practitioners.
B. Studies show that hospitals that perform well on process measures also have better outcomes, although the strength of the association is unclear.
C. Process measures can be easily measured or compared.
Vital Concepts:
When evaluating quality in a healthcare organization, structure, process, and outcomes can be measured. Outcome is used less frequently to report quality since it is more difficult to measure methodologically. Process measures can be based on scientific evidence and can also be measured and compared with relative ease.
References:
Many medical conditions can cause psychosis. Which of the following medical disorders can cause psychosis?
A. Fibromyalgia
B. Asthma exacerbation
C. Heart disease
D. Systemic lupus erythematosus
Correct Answer: D.
Systemic lupus erythematosus
Most common neuropsychiatric manifestations in systemic lupus erythematosus (SLE) are cognitive dysfunction, lupus headache, psychosis, seizures, and cerebrovascular events. The following can cause psychiatric syndromes in SLE: direct CNS effects, drug induced side effects, physiological reaction to chronic illness or primary psychiatric illness.
Many medical conditions can cause psychosis, including multiple sclerosis, myxedema, pancreatitis, pellagra, pernicious anemia, porphyria, lupus, temporal epilepsy, thyrotoxicosis, Huntington’s disease, Lewy body dementia, dementia, delirium, Cushing’s disease, trauma, brain tumors, meningitis, encephalitis, encephalopathy, and Addison’s disease. Medications/drugs can also cause psychosis, including amphetamines, cocaine, steroids, anticholinergics, LSD in schizophrenic patients, and L-dopa.
A. Fibromyalgia. There is an association between fibromyalgia with depression and anxiety disorders, not psychosis.
B. Asthma exacerbation. Asthma itself cannot cause psychosis, but the steroids often used to treat it can.
C. Heart disease. People with heart disease are at increased risk of developing depression, not psychosis. Patients with psychotic disorders have an increased risk of heart disease due to poor self-care and medication side effects, but the heart disease does not cause the psychosis.
Vital Concept:
Neuropsychiatric manifestations in systemic lupus erythematosus (SLE) are cognitive dysfunction, lupus headache, psychosis, seizures, and cerebrovascular events.
References:
A 45-year-old man attempted suicide by sitting in an automobile with the engine running while parked in a closed garage. He was found by a neighbor and taken to the ER by ambulance. The patient is unresponsive upon arrival. During his treatment and evaluation, an MRI is obtained. Imaging will most likely demonstrate lesions in which area?
A. Caudate
B. Globus pallidus
C. Putamen
D. Thalamus
Correct Answer: B.
Globus pallidus
The patient has likely suffered carbon monoxide poisoning. Carbon monoxide is found in motor vehicle exhaust, as well as in smoke and combustion appliances (i.e. furnaces). When this gas is inhaled, it diffuses to the bloodstream and binds to hemoglobin, forming carboxyhemoglobin. Carbon monoxide binds to hemoglobin with greater affinity than oxygen. As a result, oxygen is displaced, and the patient becomes hypoxic. Signs of acute toxicity include headache, neuropsychological changes (e.g. confusion, memory loss), seizures, impaired consciousness or coma, and death. Additional signs include cherry-red skin discoloration, cyanosis, dyspnea, and chest pain. Treatment is with hyperbaric oxygen. MRI findings include diffusion-weighted imaging lesions of the globus pallidus and deep cerebral white matter. Less commonly, lesions may be seen in the putamen, caudate, thalamus, hippocampus, substantia nigra, neocortex, and cerebellum.
Incorrect Answers:
A. Caudate is incorrect. Both putamen and caudate deal with structure in the basal ganglia and involves motor and non-motor function. It is involved in Parkinson based disorders.
C. Putamen is incorrect. Both putamen and caudate deal with structure in the basal ganglia and involves motor and non-motor function. It is involved in Parkinson based disorders.
D. Thalamus is incorrect. It depends on the location pf the lesion but lesions are commonly visible in vascular disease, inflammatory diseases, and trauma.
References:
On administration of a drug that causes CYP2D6 inhibition, which side effect of methylphenidate will increase?
A. Nervousness
B. Anorexia
C. Elevated blood pressure
D. Insomnia
E. None of the Above
Correct Answer: E.
None of the Above
Methylphenidate does not have any significant metabolism in humans via the CYP2D6 enzyme. This is likely because the majority of methylphenidate metabolism occurs at first pass and outside of the liver.
Incorrect Answers:
A. B. C. and D. Because methylphenidate doesn’t have any significant metabolism in humans via the CYP2D6 enzyme, administration of a drug that causes CYP2D6 enzyme inhibition won’t affect its side effects.
References:
Acute schizophrenia episodes (acute agitation) require emergency management. Which of the following statements about acute schizophrenia episode management and risks is true?
A. Droperidol can be safely used in patients with QTc prolongation
B. Oral-concentrate antipsychotics cannot be used in the acute setting
C. If physical restraints are indicated, the preferred positioning of a patient in restraints is the prone position
D. If a patient refuses oral medication, most states allow the administration of parenteral medication despite patient’s objection if there is a risk to themself or others
Correct Answer: D.
If a patient refuses oral medication, most states allow the administration of parenteral medication despite patient’s objection if there is a risk to themself or others
Many states permit parental medication over the patient’s objection if there are risks to self or others.
Incorrect Answers:
A. Droperidol has the potential for QTc prolongation and arrhythmia. It has a black box warning regarding this adverse effect. It should be avoided in patients with known prolonged QTc intervals or who are already taking other agents that prolong the interval.
B. Rapidly dissolving antipsychotics include olanzapine and risperidone; antipsychotics in oral concentrate form include haloperidol and risperidone.
C. The prone restraint position has resulted in sudden, unexpected deaths (possibly due to asphyxia) and is not recommended.
Vital Concept:
A patient who is in an acute psychotic episode lacks capacity in their current mental state. They should be treated in their medical best interest.
References:
A patient who is acutely intoxicated on PCP (phenylcyclohexyl piperidine) is admitted to a hospital. What is the rationale behind avoiding the use of restraints in this patient?
A. Use would violate patient’s rights.
B. Incidence of rhabdomyolysis in patients with PCP intoxication and physical restraints is much higher than baseline but can occur with PCP alone.
C. Restraints must be ordered by a medical professional when patient is in a medical facility.
D. Restraints may exacerbate possible underlying medical conditions.
Correct Answer: B.
Incidence of rhabdomyolysis in patients with PCP intoxication and physical restraints is much higher than baseline but can occur with PCP alone.
PCP intoxication is usually a clinical diagnosis heralded by fluctuating behavior, motor disturbance, nystagmus, and autonomic stimulation. Autonomic stimulation results in hypertension, tachypnea, tachycardia, flushing, diaphoresis, salivation, and hyperthermia. Severely intoxicated patients may develop metabolic acidosis and rhabdomyolysis.
Incorrect Answers:
A. Use of restraints in the short-term may be justified if for the patient’s safety.
C. While a patient is in a psychiatric facility, most states require a physician to order restraints.
D. The primary concern in this patient is in an acute episode of PCP intoxication.
Vital Concept:
Patients intoxicated with PCP are at higher risk of rhabdomyolysis when in restraints.
References:
A researcher is examining a population to find information about the development of depression and anxiety. He is studying several variables in the population over time. In the study, there is a subgroup of shift-workers and the researcher has been tracking their mental health for several years. Which of the following describes his research?
A. Case study
B. Retrospective study
C. Cohort study
D. Deductive reasoning
Correct Answer: C.
Cohort study
A cohort study examines a group of individuals with one or more common characteristics, such as people who work in shifts. Another example of a cohort is the group of nurses studied in the Framingham study, whose health habits have been studied for many years.
Incorrect Answers:
A. A case study is a study of one person that is intensive and in-depth.
B. A retrospective study is one that examines a population by looking backwards at different parameters, which may be done by chart review or recall.
D. Deductive reasoning refers to the use of broad generalizations to figure out the solution to a problem.
Vital Concepts:
A cohort study examines a group of individuals with one or more common characteristics, such as people who work in shifts.
References:
The PMH APRN is assessing a patient in the emergency room with a Urine Drug Screen (UDS) that is positive for methamphetamine. Which of the following assessment findings is likely to be noted in the chart on examination?
A. Xerosis
B. Xerostomia
C. Icterus
D. Urticaria
Correct Answer: B.
Xerostomia
Xerostomia is dry mouth caused by reduction or absent flow of saliva. Methamphetamine use has resulted in an increased incidence of “meth mouth” which is severe tooth decay caused by methamphetamine-induced xerostomia.
Incorrect Answers:
A. Xerosis is dry skin most common in the winter and exacerbated by dry heat. Xerosis is not a side effect of methamphetamine use.
C. Icterus is also known as jaundice, a yellowish pigmentation of the skin and is associated with high blood bilirubin level, not methamphetamine use.
D. Urticaria, also known as hives, is a vascular reaction of the skin marked by the transient appearance of smooth, slightly elevated papules or plaques (wheals) that are erythematous and that are often attended by sever pruritus. Urticaria is not associated with methamphetamine use.
References:
ADHD’s differential diagnosis involves ruling out a number of other possible causes of symptoms. Which of the following is NOT part of the differential diagnosis of ADHD?
A. Lead poisoning
B. Migraine headaches
C. Sleep deprivation
D. Bipolar disorder
Correct Answer: B.
Migraine headaches
Migraine headaches are not a differential diagnosis for ADHD.
Incorrect Answers
A. Lead poisoning symptoms can mimic ADHD symptoms and must be ruled out as a differential diagnosis of ADHD.
C. Symptoms of sleep disorders, such as obstructive sleep apnea, may mimic ADHD symptoms.
D. Patients with bipolar disorder may exhibit symptoms similar to ADHD.
References:
Which part of the brain controls the respiratory functions of the body?
A. Cerebral cortex
B. Medulla
C. Pons
D. Cerebellum
Correct Answer: B.
Medulla
The medulla is located in the brainstem. It controls respiration/ventilation by chemoreceptors that detect changes in the acidity of the blood and increase or decrease the respiratory rate accordingly.
Incorrect Answers:
A. Cerebral cortex. The cerebral cortex contains sensory and motor functions.
C. Pons. The pons relays signals between the cerebellum and the cerebrum. Cranial nerves V-VIII originate in the pons.
D. Cerebellum. The cerebellum coordinates body movements and balance and coordination.
References:
Which of the following statements about bipolar disorder is true?
A. First episode in women with bipolar I is usually manic.
B. Bipolar disorder patients are 15 times more likely to commit suicide than the general population
C. Bipolar disorder is more common in lower income countries
D. Rapid cycling is more common in men than women.
Correct Answer: B.
Bipolar disorder patients are 15 times more likely to commit suicide than the general population
Patients with bipolar disorder are 10-30x more likely to commit suicide than the general population. Bipolar disorder likely accounts for 3-14% of all suicide deaths. Suicidality is commonly seen in these patients while they are in a depressed or mixed condition. Up to 4-19% of patients with bipolar disorder end their life by suicide, and 20-60% attempt suicide at some point in their life.
Incorrect Answers:
A. Women with bipolar I usually demonstrate an initial depressive episode.
C. Bipolar disorder is more common in higher-income countries
D. Rapid cycling is more common in women.
Vital Concept:
The increased risk of suicide in patients with bipolar disorder is significant.
References:
A 68-year-old female with metastatic breast cancer is hospitalized with pneumonia. She is in respiratory failure and will require intubation, but she has a living will made after her last round of chemotherapy. Her living will states she does not want extraordinary measures taken to prolong her life under any circumstances. Her family decides that she must be depressed and insists that she should be treated aggressively with intubation and ventilation if required. Which of the following principles does the family’s actions illustrate?
A. Autonomy
B. Beneficence
C. Non-maleficence
D. Paternalism
Correct Answer: D.
Paternalism
Paternalism refers to making a decision for a patient that someone “believes” is in the patient’s best interests, despite expressed wishes to the contrary, or the opinions or wishes of the patient are ignored or minimized. Patients who are competent adults have the right to make their own healthcare choices, including refusal or denial of treatment.
Incorrect Answers:
A. The right to make one’s own medical decisions is based upon the concept of patient autonomy.
B. Beneficence refers to doing good.
C. Non-maleficence refers to doing no harm. In this case, intubating the patient against her wishes would be maleficence.
Vital Concepts:
Paternalism refers to making a decision for a patient that someone “believes” is in the patient’s best interests, despite expressed wishes to the contrary, or the opinions or wishes of the patient are ignored or minimized.
References:
As a nurse practitioner in an impoverished rural area, you frequently encounter a female client in a situation of domestic violence with few community options for referral. Participating in community education forums and fund raising for a safe house is an example of applying the ethical principle of:
A. Autonomy
B. Nonmaleficence
C. Justice
D. Veracity
Correct Answer: C.
Justice
Lobbying for underserved clients is an example of justice, which is the duty to treat all clients fairly, without regard to age, socioeconomic status, or other variables.
Incorrect Answers:
A. Autonomy is the client’s right to self-determination without outside control.
B. Nonmaleficence is the duty to prevent or avoid doing harm, whether intentional or unintentional.
D. Veracity is the duty to tell the truth.
Vital Concepts:
Lobbying for underserved clients is an example of justice, which is the duty to treat all clients fairly, without regard to age, socioeconomic status, or other variables.
References:
A young female is brought to the ED after being found in a coma, next to an empty bottle of Vicodin. She is unresponsive. Administration of which substance would be most effective in the management of this patient?
A. Naloxone
B. Flumazenil
C. Naltrexone
D. Phentolamine
Correct Answer: A.
Naloxone
Naloxone is included as a part of emergency overdose response kits distributed to heroin and other opioid drug users. This has been shown to reduce rates of fatal overdose. Naltrexone and buprenorphine are used primarily in the management of alcohol use disorder and opioid dependence.
Incorrect Answers:
B. This is a competitive benzodiazepine antagonist used to treat drowsiness after surgery or drug overdose; it doesn’t reverse the effect of opioids.
C, D. These are used primarily to manage alcohol use disorder and opioid dependence
References:
Which selective serotonin reuptake inhibitor (SSRI) may be associated with a small increase in risk of fetal heart defects in the first-trimester?
A. Sertraline (Zoloft)
B. Citalopram (Celexa)
C. Paroxetine (Paxil)
D. Fluoxetine (Prozac)
Correct Answer: C.
Paroxetine (Paxil)
Paroxetine (Paxil) is a selective serotonin reuptake inhibitor (SSRI) that may be is associated with small increased risk of fetal heart defects during the first-trimester use. However, the results are inconsistent. Only pregnancy complication associated with paroxetine use is a mild increase in postpartum hemorrhage.
Incorrect Answers:
A. Sertraline (Zoloft) is considered an option during pregnancy and appears to have little to no risk associated with birth defects. Associated with a mild increase in postpartum hemorrhage.
B. Citalopram (Celexa) is considered an option during pregnancy and appears to have little to no risk associated with birth defects. Associated with a mild increase in postpartum hemorrhage.
D. Fluoxetine (Prozac) is considered an option during pregnancy and appears to have little to no risk associated with birth defects. Associated with a mild increase in postpartum hemorrhage.
Vital Concept:
Most SSRIs are safe to use during pregnancy. Paroxetine has conflicting reports on an association with use during the first trimester and a small increase in risk of fetal heart defects.
References:
The fetal malformation rate of fetuses exposed to SSRI medications has been tracked by researchers. What is the rate of formation among fetuses exposed to SSRI medications?
A. 0.6%
B. 1%
C. 2%
D. 2.6%
Correct Answer: D.
2.6%
The overall fetal malformation rate with SSRIs is 2.6%. This rate is consistent with reports in the general population.
Incorrect Answers:
A, B, and C. These rates are all too low — the actual rate is 2.6%.
References:
After he was hospitalized for a procedure, a patient with Medicare Part A coverage needs extensive physical therapy and many medications to continue outpatient treatment. A nurse practitioner recommends Medicare Part D. Which of the following is true?
A. It will cover durable medical equipment
B. It will cover outpatient visits to urgent care centers
C. It will cover flu shots and other vaccines
D. It will cover certain prescription medications
Correct Answer: D.
It will cover certain prescription medications
Medicare Part D, known as the Medicare prescription drug benefit, is only available to individuals who are enrolled in or eligible for Medicare Part A and/or Part B. All prescription drug plans have a list of preferred drugs known as the formulary. If a non-formulary drug is used, it may not be covered and the patient will be responsible for “out of pocket” costs. Durable medical equipment, outpatient visits for medical care, and flu shots are covered under Medicare Part B.
Incorrect Answers:
A. B. and C. These are all covered under Medicare Part B
Vital Concepts:
Medicare Part D, known as the Medicare prescription drug benefit, is only available to individuals who are enrolled in or eligible for Medicare Part A and/or Part B. All prescription drug plans have a list of preferred drugs known as the formulary.
References:
Researchers are studying the effects of barometric pressure on residents of a community who have been diagnosed with hypertension. They choose 100 subjects and ask them to record their blood pressure and local barometric pressure each morning. Which of the following best describe the type of study the researchers are performing?
A. Experimental
B. Case-control
C. Correlational
D. Retrospective
Correct Answer: C.
Correlational
Correlational research is used when researchers are studying the effect of a potential cause that cannot be manipulated. In this example, the researchers cannot manipulate the barometric pressure, but they hope to find a correlation between barometric pressure and blood pressure in patients with hypertension.
Incorrect Answers:
A. An experimental study is a study of the effects of a treatment or intervention on a population compared to a “control” population that does not receive the same treatment. Study selection includes groups of people matched for important characteristics, usually randomly assigned to treatment or control groups.
B. A case-control study is a retrospective study that looks at people with a condition (cases) and another group of people without the condition (controls).
D. A retrospective study looks backward at variables and their effect on an outcome that is already determined.
Vital Concepts:
Correlational research is used when researchers are studying the effect of a potential cause that cannot be manipulated.
References:
HIPAA requires health care providers to preserve patients’ confidentiality in numerous ways. Which is an example of HIPAA compliance?
A. The nurse calls a patient by first and last name from the lobby.
B. The provider leaves a voicemail for the son of her elderly patient indicating the results of her labs.
C. The sign-in sheet at the front window is blacked out after each sign-in to prevent visibility by other patients.
D. The receptionist of a busy medical office phones to schedule an appointment for a patient, using his full name and diagnosis while sitting at the front sign-in window.
Correct Answer: C.
The sign-in sheet at the front window is blacked out after each sign-in to prevent visibility by other patients.
This is not a violation of HIPAA.
Incorrect Answers:
A. Only the first name should be used.
B. Unless the son has power of attorney or is her designated legal care provider, this is a breach of HIPAA. Also, leaving actual results on voicemail may be consider a breach, unless the patient has specifically approved the leaving of results. The voicemail should just ask the patient or designated person to return the call to receive the results.
D. The receptionist cannot be certain that her conversation is not being overheard by those waiting in the waiting area. This type of call should be made away from any public areas.
Vital Concept:
HIPPA was created to “improve the portability and accountability of health insurance coverage” for employees between jobs. Once it was established, the Privacy Rule was enacted for Protected Health Information (PHI) giving patients permission the right to privacy of their health information and the ability to withhold it in certain circumstances. Under HIPPA, patients have a right to privacy of their PHI.
References:
What is the primary function of neuronal cells?
A. Hold neurons in place
B. Secrete cerebrospinal fluid
C. Insulate neurons
D. Receive and transmit information
Correct Answer: D.
Receive and transmit information
Brain cells are divided into neurons and glial cells. Neurons are made up of a cell body, an axon, dendrites, and synaptic specializations; their primary function is to receive, process, and transmit information. Neurons are capable of transmitting information because they are both electrically and chemically excitable.
Incorrect Answers:
A. Hold neurons in place is incorrect as this is not the function of neuronal cells. Ion channels embedded in the cell’s plasma membrane can respond to voltage (voltage-gated channels), neurotransmitter binding (ligand-gated channels), or pressure or stretch (mechanically gated channels). Glial cells serve a multitude of functions.
B. Secrete cerebrospinal fluid is incorrect as this is not the function of neuronal cells. Ependymal cells make up the walls of the ventricles, secrete cerebrospinal fluid (CSF) and help to circulate CSF.
C. Insulate neurons is incorrect as this is not the function of neuronal cells. Astrocytes provide the scaffolding of the brain, form the blood-brain barrier, guide neuronal migration during development, remove neurotransmitters from within the synaptic cleft (preventing toxic buildup), regulate the external chemical environment, and are capable of cell-cell signaling. Oligodendrocytes produce the myelin sheath that speeds conduction of action potentials along axons in the CNS. Schwann cells, like oligodendrocytes, produce myelin in the peripheral nervous system. Microglia are the macrophages of the brain. When activated by injury, they remove pathogens or dead neurons.
References:
Individuals with poor coping skills demonstrate a number of characteristic behaviors. Which of the following is a characteristic of those with poor coping skills?
A. They tend to be more proactive than usual.
B. They are inclined to use excessive denial and elaborate rationalization.
C. They won’t show unexpected compliance.
D. They allow significant room for tolerance.
Correct Answer: B.
They are inclined to use excessive denial and elaborate rationalization.
Individuals with poor coping skills are inclined to use excessive denial and elaborate rationalization. Along with this, they’re unable to focus on salient problems.
Incorrect Answers:
A. Individuals with poor coping skills tend to be more passive, not proactive, than usual.
C. Individuals with poor coping skills may show unexpected compliance.
D. Individuals with poor coping skills allow little room for tolerance.
References:
A 52-year-old man has been contemplating suicide for several months since learning of his terminal pancreatic cancer. Which of the following medication classes is considered a first-line treatment for his underlying problem?
A. Benzodiazepines
B. Monoamine oxidase inhibitors (MAOIs)
C. Narcotics
D. Selective serotonin reuptake inhibitors (SSRIs)
Correct Answer: D.
Selective serotonin reuptake inhibitors (SSRIs)
The more seriously ill a person becomes, the more likely he or she is to develop major depression. At Memorial Sloan Kettering Cancer Center, Breitbart and Holland compared terminally ill patients with cancer and those with acquired immunodeficiency syndrome (AIDS) with suicidal ideation to similar patients without suicidal ideation. The primary difference was the presence of depression in patients with suicidal thoughts. Aggressive treatment of depression is a cornerstone of care, as it dramatically decreases suffering and improves quality of life. The antidepressant TCA, SSRI, and MAOI drugs are all equally effective in alleviating depressive symptoms. However, SSRIs are considered the first-line therapy for depression because of their significantly better tolerability and side effect profile compared to MAOIs and TCAs.
Incorrect Answers:
A. Benzodiazepines. Benzodiazepines are not used for alleviating depressive symptoms.
B. Monoamine oxidase inhibitors (MAOIs). SSRIs have significantly better tolerability and side effect profile then MAOIs and therefore are preferred.
C. Narcotics. Narcotics are not utilized to treat depression.
References:
Mirtazapine is used to treat major depressive disorder. Which receptor does it exert its effect through?
A. Postsynaptic alpha-1 adrenergic receptor antagonist
B. Postsynaptic alpha-2 adrenergic receptor agonist
C. Presynaptic alpha-2 adrenergic receptor antagonist
D. Nonselective alpha-2 adrenergic receptor agonist
Correct Answer: C.
Presynaptic alpha-2 adrenergic receptor antagonist
Mirtazapine (Remeron) is a serotonin, norepinephrine receptor antagonist and alpha-2 adrenergic antagonist. Blocks alpha-2 adrenergic presynaptic inhibitory autoreceptors and heteroreceptors increasing norepinephrine and serotonin neurotransmission. Mirtazapine also increases serotonin at the 5HT-1A receptor. It accomplishes this action with a unique mechanism where it blocks the 5HT-2A, 5HT-2C and 5HT-3 receptors resulting in serotonin being directed towards the 5HT-1A receptor, the primary serotonin receptor effecting mood.
Activation of 5HT-2A receptor in other antidepressants (e.g. SSRIs) causes diminished libido and inhibiting orgasm/ejaculation. Since mirtazapine blocks the 5HT-2A receptor there is less sexual side effects compared to other antidepressants. Side effects like weight gain and sedation are caused by mirtazapine’s potent histamine (H1) receptor antagonism. Lower doses are more sedating because higher antihistaminergic effect compared to higher doses where there is increased noradrenergic effect. If prescribing this medication with intentions of improving patients’ appetite and/or sleep prescribe at lower doses.
Incorrect Answers:
A. Prazosin is a postsynaptic alpha-1 adrenergic receptor antagonist. It is only FDA approved for hypertension but is used off-label for PTSD associated nightmares.
B. Guanfacine is a postsynaptic alpha-2 adrenergic receptor agonist. It is FDA approved for ADHD (in children only) and for hypertension. Also used off-label in opioid detoxification, tics, and autism spectrum disorder behavioral disturbances.
D. Clonidine is a nonselective alpha-2 adrenergic receptor agonist. It is FDA approved to treat ADHD and hypertension. Also used off-label in anxiety disorders, PTSD and substance use withdrawal.
References: